NY motivation

Pataasin ang iyong marka sa homework at exams ngayon gamit ang Quizwiz!

Important cause of hypocalcemia, particularly in alcoholics. Causes decreased release of PTH and PTH resistance.

*Hypomagnesemia*

Preeclamptic pt treatment

*Induction of labor*/delivery = definitive treatment

Dementia + aggressive behavior if other nonpharm ineffective --> treatment =

*aripiprazole/abilify* - D2 partial agonist - Augmentation of major depression therapy

Pt is most likely experiencing peripheral edema caused by what class of antiHT drugs?

*dihydropyridine* CCBs: amlodipine/norvasc nifedipine nicardipine isradipine felodipine nisoldipine - more selective for vasodilation

Pts with severe acute pancreatitis receive what to prevent infection complications, such as infected necrosis?

*enteral nutrition*

The fastest way to lower increased intracranial pressure (ICP) is

*hyperventilation* which lowers PaCO2

Nonsevere salmonella treatment

*no treatment* - severe = levofloxacin 500 mg once daily for 7-10 days

The first step in treating diverticulitis complicated by abscess formation (> 3 cm) is....

CT-guided drainage

SSRI with longest half-life

fluoxetine

Acanthosis nigricans association in older individuals

gastrointestinal malignancy

Bariatric surgery requires

life-long supplements

Treatment of Group A Streptococcus pharyngitis:

- Penicillin - Allergy: Erythromycin

Characterized by elevated aminotransferases, encephalopathy, and INR >1.5.

Acute liver failure - *Acetaminophen toxicity* = common cause - hepatic encephalopathy = confusion, somnolence, *flapping tremor* consistent with asterixis

Preferred treatment for adult pts with ADHD with ho substance use disorder to avoid risks of drug misuse and addiction.

Atomoxetine - nonstimulant - NE reuptake inhibitor - not addictive - other nonstimulant options with efficacy for adult ADHD = bupropion, tricyclic antidepressants

Opioid-induced hyperalgesia in cancer pt = paradoxical increase in sensitivity to pain despite increase in opioid dosage. Management?

Attempt reduction in morphine dosage --> eliminate --> start another opioid

Acute parotitis via dehydration. Treatment?

Augmentin: Amoxicillin/Clavulanate - there may be drainage on presentation

Characterized by hypercalcemia, renal insufficiency, metabolic alkalosis.

Milk-alkali syndrome

May be harmful in pts with low left ventricular ejection fractions

Nondihydropyridine CCBs - *diltiazem*, verapamil

Influenza ok for pt with allergy to eggs:

Recombinant trivalent influenza vaccine - formulated without using eggs

Whiplash is often the inciting injury. Symptoms develop months to years later. Impaired strength and pain/temp sensation in upper extremities.

Syringomyelia - 3-4% of pts with spinal cord injuries develop post-traumatic syringomyelia - enlargement of central canal of spinal cord via CSF retention - MRI = definitive diagnosis

primary treatment for ADHD in pts younger than 6 yo

behavioral therapy - 6 yo and older = start with meds

First-line for acute pyelonephritis.

fluoroquinolones - cipro

MRSA bacteremia - uncomplicated. Next step ....

followup 2-4 days after initial sed and as needed thereafter

Obstructive sleep apnea (OSA) causes transient periods of hypoxemia. The kidney responds by ...

increasing *erythropoietin* = erythrocytosis --> polycythemia

primary prevention

individuals at risk of condition --> prevent this - childhood vaccination, water fluor, antismoking, safe sex

MDD is often a recurrent illness. Pts with >/= lifetime depressive episodes, severe episodes (suicide attempts), or episodes lasting >/= 2 years are candidates for...

lifelong antidepressant treatment - ex. continuing venlafaxine indefinitely

PCOS may present with irregular menses, hirsutism, and elevated serum testosterone. Treatment involves...

wt loss + *oral contraceptives* containing estrogen and progesterone --> to regulate menstrual cycles

Hyperplastic <10 mm in size in rectum or colon --> only finding =

colonoscopy every 10 yrs

Treatment for COPD with increased risk for bruising...

corticosteroids

Child is presenting with a delayed Type IV hypersensitivity reaction likely secondary to poison ivy exposure, which usually presents when?

*24-48 hours after contact*

SSRI with bad withdrawal

*paroxetine*

HPV PCR is only recommended for screening in what pts?

*women 30 years of age and older*

The treatment of choice for heavy bleeding associated with anovulatory cycles is

*OCPs*

Best diagnostic test for vit D deficiency?

25-hydroxyvitamin D

Recognize *acute inferior STEMI*, in which immediate interventions are aimed at achieving what goals would most improve pt's long-term prognosis?

*Restore coronary blood flow* acute inferior STEMI - via abrupt right coronary artery occlusion - *prompt restoration of coronary blood flow with primary percutaneous intervention (PCI) or fibrinolysis limits myocardial damage & improves cardiovascular and overall long-term mortality* Current guidelines - recommend use of primary PCI for any pts with acute STEMI within 90 minutes of first med contact in PCI-capable hospital - within 120 minutes for pts who require transport to PCI-capable hospital from another hospital

The primary role of this muscle is the first 15 degrees of arm abduction.

*Supraspinatus* muscle - most commonly injured rotator cuff muscle - pain anteriorly and superiorly --> corresponds to its position at the glenohumeral joint - the most common etiology of injury is insidious motion, such as might occur while tilling a garden.

A 34-year-old woman comes to your office with complaints of ear pain that is aggravated with chewing. On exam, you note a "knocking" sound when the jaw is opened. Which of the following is commonly identified in the history of these patients?

*bruxism* Temporomandibular joint disorder (TMD), - commonly associated with bruxism (teeth grinding during sleep). - presents with jaw pain that is worsened with jaw use, such as chewing - this pain can be referred to the ear, which may be the primary complaint of the patient, not jaw pain

Effective initial therapy for fibromyalgia

Amitriptyline

Pt who is hypotensive with abdominal pain and has CT scan showing enlarged aortic silhouette has diagnosis of ruptured abdominal aortic aneurysm. This pt needs...

*exploratory abdominal surgery* / immediate surgery

Recognize *cellulitis*

- skin infection of deep dermis & subcutaneous fat - usually caused by beta hemolytic Strep (nonpurulent) or Staph aureus (purulent) - slowly spreading rash that is warm, tender, and erythematous with *flat, distinct borders* - labs = leukocytosis with predominance of neutrophils

asymptomatic microscopic hematuria

>/= 3 RBCs/hpf

primitive brain area involved in OCD

basal ganglia (& amygdala)

Acute angle glaucoma --> immediate referral via what symptom?

reduction of visual acuity

Should be suspected in pts presenting with sudden onset of severe epigastric pain that spreads over entire abdomen.

Chemical peritonitis due to *perforated peptic ulcer* - upright chest and/or abdominal radiographs = free ar under diaphragm

acute cognitive chest from baseline/delirium. Diagnostic test?

Confusion Assessment Method (MAD)

Most important risk factor for being victim of domestic abuse

Gender - only consistent risk factor

Contraception for organ transplant recipient female.

IUD - increased fertility post transplant - avoids interactions with meds - no risk of infection

Leading cause of human death in world as a whole?

Ischemic heart disease

Causes anterior leg pain. Casual runners. Diffuse area of tenderness. More common in overwt than underwt individuals.

Medial tibial stress syndrome / shin splints

Narcolepsy treatment, along with cataplexy.

Methylphenidate/Ritalin

Pleuritic chest pain + pericardial friction rub + diffuse ST-segment elevations on ECG <4 days following MI.

Peri-infarction pericarditis / *Acute pericarditis*

Recognize *hiatal hernia*

Rugae of the stomach can be seen in the herniated contents.

*Toxin-mediated neurotransmission blockage.* Presynaptic neuromuscular junction disorder caused by Clostridium tetani toxin, which blocks release of inhibitor NTs glycine and GABA across synaptic cleft, leading to fever, painful muscle spasms, and trismus.

Tetanus - usually only in those who are unvaccinated or incompletely immunized

Recognize *X-linked dominant* mode of inheritance

X-linked dominant - in the first generation of offspring, a male passes the X chromosome to all female, offspring so all females are affected

When you see the classic peaked T-waves, this is an indication to first administer

calcium gluconate to stabilize the cardiac cell membranes, followed by insulin and glucose, followed by kayexalate.

AAP: children 9-11 yo should be screened for

depression - beginning at 11 yo *dyslipidemia*

All RA pts should receive ...

disease modifying antirheumatic drugs (DMARDs) as early as possible in dz course - *METHOTREXATE* = initial DMARD of choice in most pts with RA - NSAIDS or glucocorticoids should be used for initial temporary symptomatic relief while awaiting response to DMARD therapy

Serotonin syndrome. Slow horizontal eye movement helpful in diagnosis. Treatment?

drug of choice = IV benzo (diazepam, lorazepam)

Acute laryngitis treatment

symptomatic treatment only

Management of myasthenic crisis with respiratory failure consists of endotracheal intubation followed by treatment with

therapeutic *plasmapheresis* (or IV immunoglobulins) & corticosteroids

Specificity of test = proportion of ppl ....

without the condition who test negative

Elementary iron supplements begin at

*1 mo old in exclusively breastfed infants* - until 12 mos old, unless multiple blood transfusions

Reduces risk of coughing and increases sleep of children and parents.

*Honey* - no in kids in less than 12 mos

What can improve iron absorption?

*Vitamin C* - or high in meat protein

Umbilical cord vessels

2 arteries, 1 vein - single artery = renal abnormalities too

The term obese is not used until what age?

2 yo

gram-positive anaerobic rod that presents as an indurated mass that progresses into multiple abscesses with sinus tracts draining a thick yellow exudate containing sulfur granules.

Actinomyces - The treatment of choice is penicillin and incision and drainage of the abscess.

Presents with a bulging of eardrum, inflammation, fever, and otalgia. There is fluid in the middle ear space.

Acute otitis media

Interstitial lung dz common offenders (drugs)

amiodarone, *nitrofurantoin* - can induce pneumonitis

Best initial test for male hypogonadism

Early morning total serum testosterone

Pt with cancer + DVT of leg or PE = cancerous associated thrombosis --> treatment?

LMWH = *Enoxaparin/Lovenox*

Nonspecific peripheral neuropathy + mildly elevated protein and increased ESR

MGUS --> *serum protein electrophoresis*

Characterized by contralateral motor and/or somatosensory deficits (more pronounced in face or upper limb than lower limb). Homonymous hemianopia or quadrantanopia.

Middle cerebral artery (MCA) stroke - dominant/left lobe involved = aphasia - nondominant/right lobe involved = hemineglect or anosognosia (lack of awareness regarding one's illness)

Hallucinogenic drug that characteristically causes nystagmus. Can also cause dissociative feelings, psychotic and violent behavior, severe HT, and hyperthermia.

Phencyclidine (PCP)

Most common source of symptomatic pulmonary embolism.

Proximal deep leg veins (*femoral vein*)

Useful for depressive symptoms in children and adolescents

Prozac/Fluoxetine

Recognize ROC curve, in which top right of curve would = high sensitivity, high specificity

Receiver Operating Curve (ROC) - demonstrates trade-off b/t test's sensitivity and specificity at various cutoff points - changing cutoff point to increase true-positive rate (directly proportionate to sensitivity) will increase false-positive rate too (inversely proportionate to specificity)

Screening for diabetes is recommended in pts with...

sustained BP >135/80 mmHg - may be considered in all pts *over age 45*, as well as those with additional risk factors for diabetes - screening options = fasting plasma glucose, 2-hr oral glucose tolerance test, hemoglobin A1c

NNT defined as...

# of people that need to receive treatment to prevent 1 additional adverse event = inverse of absolute risk reduction (ARR) Ex. Pts with diabetes who had UA/NSTEMI has 16.3% incidence of composite events with clopidogrel + aspirin and 12.3% with Efrenzia + aspirin ARR = 0.163 - 0.123 = 0.04 NNT = 1/ARR = 1/0.04 = 25 Compared to clopidogrel, Efrenzia would need to be given to at least 25 pts with diabetes undergoing percutaneous coronary intervention for UA/NSTEMI in order to prevent 1 additional composite event

This patient has bacterial meningitis caused by Neisseria meningitidis, a Gram-negative diplococci. Family members and close contacts should be treated with

* prophylactic rifampin* Chemoprophylaxis is indicated in patients who have had close contact with a person with confirmed meningococcal infection, such as a daycare worker caring for a child with meningococcal meningitis or family in the same household. Healthcare workers do not need chemoprophylaxis unless they have had direct exposure to respiratory secretions. *Rifampin is the preferred agent, but ciprofloxacin can be used in those who cannot tolerate rifampin, such as those on warfarin or other medications impacted by the CYP450 pathway*

43 yo man with 2 yr ho T2DM comes to doc for routine preventive exam. He has no complaints. His meds include metformin and aspirin. Pt works as computer programmer and has sedentary lifestyle. He drinks 1 or 2 cans of beer on weekends and has 20-pck yr smoking hx. His BP is 153/94 mmHg and his pulse is 82/min. Pt's BMI is 32.5 kg/^2. Remainder of PE unremarkable. Lab studies show HbA1c of 7.6%. At previous visit 3 mos ago, pt's BP was 149/92 mmHg. What intervention would be most effective in controlling his BP?

*10% weight loss* Weight loss = most effective nonpharm measure to decrease BP in overwt individuals - other effective measures = *DASH diet*, low sodium, moderation of alcohol, moderate exercise, smoking cessation

In pts with traumatic spinal cord injuries, what can assess for urinary retention and prevent acute bladder distention and damage?

*Bladder catheterization* / urinary catheter

Most common source of liver metastases.

*Colorectal cancer* - blood from colon moves through portal circulation directly to liver - *Colonoscopy = most appropriate next diagnostic step = both localizes tumor and provides tissue diagnosis*

Best evidence supporting its use for acute low back pain without radicular symptoms?

*Cyclobenzaprine* / Flexeril - NSAIDs, tylenol, *muscle relaxants*

45 yo man comes to office with mo of intermittent bloody diarrhea, abdominal pain, and 9-lb unintentional wt loss. He has had no dizziness or chest pain. He was diagnosed with HIV 2 yrs ago after several yrs of IV heroin use. He is noncompliant with antiretroviral treatment but no longer uses illicit drugs. He is currently engaged to HIV-negative woman and denies same-sex sexual partners. He has never traveled abroad. Temp is 100.4 F, BP is 114/68 mmHg, and pulse is 96/min. Mucous membranes moist, and moderate cervical LAD present. Abdominal exam reveals mild tenderness in left lower quadrant. Lab results normal except for CD4 count of 28/mm^3 and potassium of 3.2 mEq/L. Most likely cause of this pt's diarrhea?

*Cytomegalovirus* - fatigue, low-grade fever, wt loss, frequent, low-volume stools that are often bloody - common in pts with AIDS who have very low CD4 counts (<50/mm^3) - diagnostic test of choice = colonoscopy with biopsy = eosinophilic intranuclear and basophilic intracytoplasmic inclusions - active? --> treat with gancyclovir - ocular exam required to rule out concurrent retinitis

DVT potentially in pt with dyspnea and chest pain + low score of pretest probability --> next step ?

*D-dimer level* - low probability scenario

School nurse discovers head lice on fourth-grade student. When should student be permitted to return to class?

*Immediately* - requires head to head contact - should remain in class but avoid close contact - treat with pediculicide

30 yo female and husband can't conceive. Husband has normal semen analysis. Normal menstrual cycle. Next step?

*Luteal-phase progesterone level* - ovulation can be confirmed by prog level >5 ng/mL on day 21 of cycle

Most common form of nephrotic syndrome in pts with Hodgkin lymphoma.

*Minimal change dz* - most common form of nephrotic syndrome associated with carcinoma

Most frequent location of ectopic foci that causes atrial fibrillation

*Pulmonary veins*

Systemic sclerosis can cause what changes in the esophagus?

*Smooth muscle atrophy and fibrosis* --> atrophy and fibrosis of SM in lower esophagus --> decreased peristalsis and decreased tone in lower esophageal sphincter - typical symptoms = heartburn & dysphagia

Pts with HIV are at increased risk of community-acquired pneumonia. What is the most commonly implicated organism?

*Streptococcus pneumoniae* - "rusty sputum" = classic for pneumococcal pneumonia but may not always be present - diagnosis confirmed on chest x-ray with: lobar, interstitial, or cavitary infiltrate on chest imaging

77 yo male with postherpetic neuralgia in chest wall distribution. Most likely to be effective in diminishing discomfort?

*Topical lidocaine (Xylocaine) patches*

Maneuver used in Impaction of Shoulder = push fetal head back into the vaginal canal for emergent C-section.

*Zavanelli maneuver* Other management of shoulder impaction - delivery of POSTERIOR shoulder - rotation of shoulder 180 degrees

screening for chronic hep B infection NOT recommended in ...

*all newborns*

The mature ego defenses include

*altruism, humor, sublimation, and suppression*

Randomization used to control for...

*confounders* - during design stage of study - helps to control for known, unknown, and difficult-to-measure confounders

nasopharyngeal carcinoma associated with

*epstein-barr virus infection* - most commonly in those from Asia (southern China) and part of Africa and Middle East - nasal congestion + epistaxis + headaches + CN palsies + otitis media - early spread to cervical lymph node common

Strongest indication for thrombolytic therapy in PE...

*hypotension* - systolic BP 90 mmHg or diastolic <60 mmHg for 15 minutes or longer --> thrombolysis for those not at high risk for bleeding

PTSD nightmares treated with

*prazosin* - reduced sympathetic outflow in brain

Considered in pts with rotator cuff tendinitis

*subacromial space:* corticosteroid injection

Toe fracture + nondisplaced lesser toe fractures + *25% or greater involvement of joint surface* -->

*surgical fixation*

Indications for an IVC filter:

1. Contraindication to anticoagulation 2. Complications associated with anticoagulation 3. *Failure of anticoagulation*

Goal BP in pts age 60 and older with no comorbidities

150/90 mmHg

Child should use 2-word phrases by....

2 1/2 years old - refer to local early developmental intervention program if not

BPH treatment if PSA enlargement or PSA level >1.5 ng/mL

5 alpha reductase inhibitor = finasteride/proscar

Upper GI bleed: transfusion of RBCs at what level?

7.0 g/dL - known cardiovascular dz = 8.0 g/dL as cutoff

ADPKD + HT. First-line treatment?

ACEIs - renal protective

Uncommon inflammatory disorder. Recurrent high fevers, arthritis/arthralgias. Salmon-colored macular or maculopapular rash. ESR may be elevated.

Adult Still disease

Circadian rhythm disorder. Inability to stay awake in evening (usually after 7:00 PM). Makes social functioning difficult. Frequently have early-morning insomnia due to early bedtime.

Advanced sleep phase syndrome

Usually seen in pts age >50 yo . Presents with progressive and bilateral loss of central vision. Navigational vision preserved.

Age-related *macular degeneration*

Typically develops within 12-24 hours and lasts up to 48 hours after last drink of alcohol. Sensorium is intact and vitals signs are usually relatively stable.

Alcoholic hallucinations

Women with diarrhea-predominant IBS. Tried dicyclomide which didn't work. Treatment?

Amitriptyline - anticholinergic effects

A 5-year-old girl presents to the emergency room due to difficulty walking. According to her parents, she has been complaining of pain in her right leg for the last few days. Her neighbor's cat bit her last week and her parents attributed the pain to her healing bite. At the time of the bite they cleaned the wound and irrigated it with sterile saline water from a first aid kit. She has no past medical history and has never been hospitalized. On physical examination her temperature is 102.2°F (39°C), blood pressure is 118/78 mmHg, pulse is 90/min, respirations are 21/min, and pulse oximetry is 99% on room air. The open wound remains present on the thigh with surrounding erythema and edema. MRI is consistent with osteomyelitis. Which of the following is the most appropriate next step in management?

Ampicillin and sulbactam This young girl with a recent history of a cat bite and difficulty walking has likely Pasturella multocida osteomyelitis and should undergo empiric IV antibiotic therapy with ampicillin-sulbactam, with tapering of antibiotic regimen based on biopsy/culture results. Pasturella multocida is a gram-negative rod that is associated with cat and dog bites. Infection can lead to cellulitis and/or osteomyelitis. Treatment of minor infection is with amoxicillin or penicillin and clavulanate; however, patients suspected of having osteomyelitis should be treated empirically with IV antibiotics (e.g., ampicillin and sulbactam) and taken for biopsy/culture to identify the pathogen. Antibiotics can then be narrowed accordingly.

Potential bioterrorism agents that requires treatment with 60 days of continuous abx

Anthrax - oral fluoros and doxycycline - incubation period = 60 days

Recognize aspergilloma of *chronic pulmonary aspergillosis* "54 yo man comes to office due to fatigue, productive cough with blood-tinged sputum, and 15.4-lb wt loss over past few mos. He has no fever, chest pain, or night sweats. Ten yrs ago, pt emigrated from Vietnam, where he received treatment for pulmonary TB. He has no other med hx. He doesn't use tobacco or illicit drugs but drinks alcohol occasionally. Temp is 98 F, BP is 120/80 mmHg, & pulse is 82/min. Lung auscultation reveals right-sided rhonchi and crackles. Creatinine is 1.1 mg/dL and urinalysis is normal. CT scan of chest is shown...

Aspergillus - ubiquitous fungus - conidia inhaled into lung --> convert to potentially pathogenic hyphae - immunocompetent + ho pulm dz (cavitary TB) --> chronic pulm aspergillosis (CPA) at sites of lung damage Diagnosis via... - >3 mos of symptoms = fever, wt loss, fatigue, cough, hemoptysis, and/or dyspnea - cavitary lesions containing debris, fluid, aspergilloma (fungus ball) - positive Aspergillus IgG serology - therapy = itraconazole, voriconazole, surgery, bronchial artery embolization

Infection disease caused by aspiration of infected oropharyngeal secretions.

Aspiration pneumonia

Acute lung injury secondary to chemical burn from aspirated sterile gastric contents.

Aspiration pneumonitis

Recognize basal cell carcinoma, which *appears pearly and papular with overlying telangiectasia*

Basal cell carcinoma (BCC) - usually a pearly, papular lesion with overlying telangiectasia. - most common skin cancer in humans. - most commonly presents on the face (specifically, the nose) - more common in men - more common in older adults. - *Sun exposure is the most important risk factor.* - treatment for BCC is Mohs micrographic surgery for large, high-risk tumors and surgical excision, electrodesiccation and curettage, or cryotherapy for smaller, low-risk tumors. *Image shows the "peripheral palisading" of basaloid cells within the dermis typical of basal cell carcinoma.*

A 27-year-old woman presents to the emergency department with a migraine headache. She has had several migraines in the past but has never been put on prophylaxis for them. She complains of photophobia, aura, and severe nausea. She reports that she vomited several times at home since the migraine started two hours ago. Her past medical history is significant for hypothyroidism and depression. Her home medications include levothyroxine and sertraline. On physical exam, she is lying on the hospital bed with her hand over her eyes and appears to be in moderate distress. She is given an abortive medication as well as several doses of an antiemetic. An hour later, the migraine has broken but the patient is complaining of diplopia. Her temperature is 100.3°F (37.9°C), blood pressure is 152/94 mmHg, pulse is 107/min, and respirations are 20/min. On repeat exam, the patient's neck is flexed backwards with her jaw clenched. Her eyes deviate upwards on convergence testing. Her reflexes are 2+ throughout. Laboratory testing is performed and reveals the following: Serum: Na+: 142 mEq/L K+: 3.4 mEq/L Cl-: 102 mEq/L HCO3-: 25 mEq/L Urea nitrogen: 12 mg/dL Creatinine: 0.9 mg/dL An electrocardiogram (ECG) is performed and can be seen in Figure A. Which of the following is the most appropriate next step in management?

Benztropine This patient presents with migraine, receives an abortive medication and antiemetic, and then has manifestations of an oculogyric crisis. The most appropriate next step of acute dystonia is benztropine, an anticholinergic medication. In this case the patient's adverse event is the key to understanding the pharmacology underlying her symptoms. Her symptoms of neck flexion, clenched jaw, and upward deviation of the eye on convergence are classic for an oculogyric crisis, one of the manifestations of acute dystonia. Others include spasmodic torticollis, in which the neck is involuntarily turned to one side, and oromandibular dystonia. Acute dystonia is one type of the extrapyramidal symptoms (EPS) caused by dopamine antagonism. In this patient's case, the most likely culprit is metoclopramide, and the most appropriate management is discontinuation of the offending drug and administration of benztropine or another anticholinergic medication. Figure demonstrates an electrocardiogram with a prolonged QT interval (~590 ms).

Recognize cutaneous larva migrans, which can be caused by *walking barefoot on a sandy beach*

CLM - creepin cutaneous eruption - via dog (Ancylostoma caninum) or cat (A braziliense) hookworm larvae - acquired from walking barefoot on contaminated sand (beaches) or soil - humans = incidental hosts --> larvae unable to penetrate derma basement membrane = cutaneous manifestation - usually start with pruritic, erythematous papule at site of larvae penetration --> intensely pruritic, serpiginous, reddish-brown cutaneous tracks - cases resolve spontaneously within few wks, but antihemintics (ivermectin) can be given

Contraindicated in pregnant pts with postpartum hemorrhage

Carboprost/Hemabate

Leading cause of death in pts with RA

Cardiovascular dz

Low DLCO with normal spirometry

Chronic pulonary emboli - disrupts gas transfer in lungs without restriction or obstruction

Most commonly due to diverticular disease and presents with pneumaturia, fecaluria, or findings consistent with UTI. Abdominal CT scan with oral or rectal (not IV) contrast can confirm diagnosis by showing contrast material in bladder with thickened walls.

Colovesical fistula

Immature defense mechanism in which the individual displaces negative feelings associated with person or situation onto a "safer," more acceptable object or situation.

Displacement

Presents with contralateral sensory loss (pain, vibration, agraphesthesia, astereognosis) and contralateral inferior homonymous quadrantanopsia due to superior optic radiations inv't.

Dominant parietal lobe

Trastuzumab is used for treatment of pts with human epidermal growth receptor 2-positive breast carcinoma. What should be assessed before pts treated with this?

Echocardiography at baseline and regular intervals - increased risk of cardiotoxicity

Protozoan that can cause colitis or extraintestinal (*liver*, pleura, brain) illness in pts who live in or travel to developing countries. Amoebic liver abscess characterized by RUQ pain, fever, and (usually) a *single subcapsular cyst in right lobe of liver*

Entamoeba histolytica

Presents with worsening focal back pain, bilateral LE weakness, sensory loss, gait ataxia. Exam shows focal point tenderness in spine, *exaggerated deep-tendon reflexes in legs, upgoing plantar reflexes*

Epidural *spinal cord compression* (ESCC) - can be via mets - thoracic spine most frequently affected (60%), followed by lumbar spine (30%)

Recognize where *false negatives* would be highest *(C)*

False negative - pts who have negative result despite actually having dz - *cut-off level of diagnostic test raised --> FN also increases*

Methotrexate / *antimetabolite agent therapy* is the initial disease-modifying antirheumatic drug of choice in pts with RA. Common side effects of MTX include...

GI symptoms *Oral ulcers* or stomatitis Rash Alopecia *Hepatotoxicity* Pulmonary toxicity Bone marrow suppression - folic acid supplementation has been shown to reduce incidence of adverse effects of MTX without loss of efficacy

gout can be caused by what drug

HCTZ - make sure you don't exacerbate already existing problem

Associated with extrahepatic manifestations, including fatigue, arthralgias, and porphyria cutanea tarda. PCT presents with fragile, photosensitive skin that develops vesicles and bullae with trauma or sun exposure.

HCV - all pts with PCT should be screened for HCV

Individuals with ho high-risk sexual intercourse (unprotected or men who have sex with men) should be screened for...

HIV & hepatitis B

4th gen HIV Ab/Ag test is negative and normal creatinine. What additional test must be documented before offering preexposure prophylaxis for HIV with emtricitabine/tenofovir (Truvada)?

Hepatitis B surface Ab & antigen - tenofovir can be toxic to kidneys - also active against hepatitis B infection --> use of truvada with active HBV and stops using PrEP --> liver damage via reactivated HBV

Cornerstone of therapy for renal stone dz

Hydration ~ *fluid intake greater than 2 L/day* - detailed metabolic evaluation not needed when pt presents with first renal stone - stones <5mm in diameter typically pass spontaneously with conservative management

Autoimmune disorder presenting with increased platelet destruction and inhibition of megakaryocyte platelet production due to IgG autoAbs against platelet membrane glycoproteins. *Diagnosis of exclusion*.

Immune thrombocytopenia = *immune destruction of platelets* - treatment = systemic glucocorticoids for pts with severe thrombocytopenia (platelets <30,000/uL) or significant bleeding

Recognize pneumoperitoneum, in which the next step after initial medical management (IV fluids, broad-spectrum abx, IV PPI) is *urgent exploratory laparotomy*

Intraperitoneal free air = pneumoperitoneum - besen seen b/t liver & diaphragm - ho NSAIDs & several-day ho episodic postprandial epigastric pain followed by acute-onset, severe constant pain --> perforated peptic ulcer

Unilateral foot drop is characterized by "steppage" gait: exaggerated hip & knee flexion while walking. Common causes include:

L5 radiculopathy & compression peroneal neuropathy / *common peroneal neuropathy* - L5 radiculopathy may also have weak foot inversion & plantar flexion, while these will be normal in peroneal neuropathy - foot drop is due to weakness in foot dorsiflexion Peroneal neuropathy - due to compression of nerve at lateral aspect of fibula (via prolonged crossing of legs or squatting) - associated with paresthesias & sensory loss over dorsum of foot but normal foot inversion & plantar flexion - diagnosis can be confirmed with electromyography & nerve conduction studies

Typically occurs via posterior inferior cerebellar artery occlusion. Pts develop: loss of pain/temp sensation sense over ipsilateral face and contralateral body (via spinal trigeminal and spinothalamic tract injury), ipsilateral bulbar muscle weakness (nucleus ambiguus), vertigo/nystagmus (vestibular nuclei), and Horner syndrome (ascending sympathetic fibers).

Lateral medullary infarction

Primary hyperparathyroidism + pituitary tumors + GI/pancreatic endocrine tumors (gastrinomas). Common complications include symptomatic hypercalcemia & recurrent peptic ulcer.

MEN1

Vaccine that increases child's risk of febrile seizure for up to 2 wks after admin

MMR

15 yo male presents to ED after suffering lateral dislocation of his patella. Best method for reducing this dislocation?

Medially directed pressure on patella while extending leg - causes patella to slip back into place --> immobilize knee

Typically present with ipsilateral oculomotor nerve palsy, ataxia (via damage of superior cerebellar peduncle), and contralateral hemiparesis (cerebral peduncle).

Midbrain strokes

The patient's presentation is most likely consistent with reactive arthritis, given onset of asymmetric arthritis and extra-articular manifestations (e.g. mouth ulcers) in the setting of a recent genitourinary infection. The first line therapy for reactive arthritis are

NSAIDs

What NSAID would you recommend as being LEAST likely to be associated with increased risk of MI

Naproxen/Naprosyn - preferred over other nSAIDs in pts with underlying coronary artery dz risk factors - the menstrual cramp med

Potentially life-threatening condition that can occur after admin of antipsychotic meds. Symptoms include high fevers, lead-pipe rigidity, altered mental status, and autonomic instability. CK level & WBC count may be elevated.

Neuroleptic malignant syndrome

Characterized by hepatomegaly with mild elevations in liver transaminases and alkaline phosphatase in absence of other causes of secondary hepatic fat accumulation (alcohol). Most common in pts wiht obesity and diabetes.

Nonalcoholic fatty liver dz - management = weight loss & control of metabolic risk factors

Lesion of what lobe of the brain causes anosognosia (denial of one's disabilities) and contralateral apraxia (inability to carry out learned purposeful movements)?

Nondominant parietal lobe lesion

Recognize tracing of placental abruption, in which *emergent cesarean section* is indicated

Placental abruption - clinical deterioration of mother and fetus (hemodynamic instability and sinusoidal fetal heart rate pattern, respectively) Risk factors for development of placental abruption - history of prior abruption, pregnancy induced hypertension, eclampsia, abdominal trauma, tobacco use, cocaine use, PPROM, multifetal gestation, advanced maternal age, polyhydramnios, and intrauterine infection. *Figure shows a fetal heart tracing demonstrating a sinusoidal fetal heart rate pattern.*

Pts who receive solid organ transplant are at risk for opportunistic infections, most notably....

Pneumocystis pneumonia and *CMV* - absence of prophylaxis --> pts who present with systemic illness involving multiple organ systems (pneumonitis, hepatitis, gastroenteritis) --> should be tested for CMV viremia

27 yo man brought to ED with difficulty walking. He noticed leg weakness several days ago, and now he is barely able to walk. He also complains of foot tingling. 2 wks ago, he had URI that resolved on its own. ROS is negative for fever, headache, neck or back pain, or other symptoms. Vitals show tachycardia and orthostatic hypotension. PE shows bilateral LE muscle weakness and absent knee and ankle reflexes. Sensation intact. CSF analysis of pt would most likely show what? - Protein = - WBC count = - RBC count = - Glucose =

Protein = increased WBC count = normal RBC count = normal Glucose = normal Guillain-Barre syndrome: CSF - high protein with normal WBC count = albuminocytologic dissociation - CSF protein may be elevated due to increased permeability of blood-nerve-barrier Gold standard treatment = IV immunoglobulin & plasmapheresis

Coin rubbing practiced where?

Southeast Asia - red marks via rubbing skin with coin - trying to rub sickness out of your body

Rate of cervical dilation

Stage 1: should last < 20 hours in a primipara, and 14 hours in a multipara Stage 1 active phase: should be > 1.5 cm/hour in a multipara and > 1.2 cm/hour in a primipara Stage 2: < 2 hours in a primipara, 1 hour in a multipara Stage 3: < 30 minutes

The patient's presentation is consistent with acute otitis media. What are the most common causes?

Streptococcus pneumoniae, Haemophilus influenzae, and *Moraxella catarrhalis*

Characterized by ignoring left side of space. Shaves only right side of face, combs right side of hair, ignores subject located in left side of space.

Stroke of *right parietal cortex* for ex / Hemi-neglect syndrome - involves right / non-dominant parietal lobe

Acute compartment syndrome diagnostic test

TIssue Pressure Studies - can be done after badly bruised muscle

Multi-systemic illness + arthralgias + wt loss + fever + diarrhea + abdominal pain. PAS-positive material in lamina propria of SI.

Whipple's dz

Terminally ill cancer pts who receive palliative chemotherapy...

are less likely to die at home

Performance-related anxiety classified as performance-only social anxiety disorder. Pharm treatments include...

as-needed *beta-blockers* or benzos

Monitoring what is recommended in testosterone replacement?

erythrocytosis --> *hematocrit*

Antipsychotic meds exert their effects through dopamine antagonism. The blocking of dopamine results in hyperprolactinemia, which can lead to

galactorrhea, amenorrhea, and *infertility* - second-gen antipsychotic *risperidone* = most likely to increase prolactin

Individuals who use injection drugs, have high-risk needlestick exposure, or received blood transfusions before 1992 should be screened for...

hepatitis C

Osteoporosis in men can be caused by what in pt with: normal ESR, blood chemistry results.

hypo*testosterone*

Legionella pneumophila is an intracellular, gram-negative organism commonly spread by aerosols or droplets from contaminated water supplies. Hx of recent travel is common. Extrapulm symptoms such as confusion, ataxia, or diarrhea are suggestive of Legionnaires' dz. Treatment options include

macrolides or fluoroquinolones (*levofloxacin*)

hyperthyroidism drug with side effects of: *agranulocytosis* within first 60 days of starting. serum sickness, *cholestatic jaundice*, loss of taste, alopecia, nephrotic syndrome, hypoglycemia

methimazole

Polycythemia vera is a myeloproliferative disorder characterized by erythrocytosis. It often presents with HT, transient vision disturbances, aquagenic pruritus, or thrombosis. Facial plethora and splenomegaly are common on PE. Treatment is primarily....

serial *phlebotomy* - bone marrow suppression drugs/hydroxyurea may sometimes be added if there is high risk of thrombosis

Retropharyngeal abscess presents with neck pain, odynophagia, and fever following penetrating trauma to posterior pharynx. Infection within retropharyngeal space can drain into ...

superior mediastinum --> extension through alar fascia into "danger space" --> transmits infection in posterior mediastinum = *acute necrotizing mediastinitis*

secondary prevention

those with asymptomatic disease - routine Pap, HT, DM, hyperlipidemia

First-line treatment for tinea corporis

topical anti-fungal such as *terbinafine* Tinea corporis - causative fungal organisms include Microsporum, Trichophyton, and Epidermophyton species - risk factors for infection include diabetes, peripheral vascular disease, immunocompromised state, and chronic irritation of the skin (commonly experienced by athletes and laborers) - If this fails, then systemic griseofulvin can be used.

Man younger than 40 yo + hematospermia + sexually active -->

urine probe for chlamydia and gonorrhea

In case of amputation injury, amputated parts should be....

wrapped in saline-moistened gauze, sealed in plastic bag, and placed on ice --> bring to ED with pt

62 yo man + DM + underwent angioplasty with placement of drug-eluting stent for left main CAD and acute coronary syndrome. PT not considered high risk for bleeding and you initiate dual antiplatelet therapy with aspirin and clopidogrel. How long should continue dual therapy?

*12 months* - acute coro. syndrome not at high risk for bleeding (DM, ho CV event)

A 38-year-old man is undergoing a pre-employment health evaluation to work as a facilities maintenance technician at a school. He is healthy, has no known medical conditions, and is not an IV drug user. Part of the evaluation involves a tuberculosis PPD skin test. Which of the following is the smallest induration diameter possible for it to be considered a positive result?

*16 mm* Because this patient does not meet any specific criteria for smaller minimum values of induration (see Incorrect Answers), 15 mm is the cutoff. PPD remains the primary investigative tool for latent tuberculosis (TB) infection (LTBI). - Placement involves a 0.5 mL injection intradermally on the volar surface of the arm. - The transverse induration is measured at 48-72 hours. - Of note BCG vaccinations render a patient's PPD positive for a least one year. - Patients with a negative reaction with negative controls (typically Candidia) implies anergy from immunosuppression, old age, or malnutrition, and this does not rule out TB. According to the CDC, there are 2 options for treatment of LTBI with INH: a 9-month regimen and 6-month regimen.

4-year-old right-handed male is brought to the emergency department by his mother after cutting his right hand on a piece of broken glass. There is a 3cm curvilinear laceration over the thenar eminence and base of the first digit. His vital signs are as follows: T99.1, HR 112, BP 123/67, RR18, Weight 23kg. You decide to proceed with primary closure, but first need to administer a local anesthetic. What anesthetic agent would you choose and what is the maximum amount that can be safely administered in order to safely provide local pain relief for the repair?

*160 mg of lidocaine with epinephrine* The maximum dose of lidocaine that can be safely administered to this patient is 4.5mg/kg, or 7mg/kg if delivered with epinephrine. - The only answer choice that corresponds to maximum dosing in this schema is 160mg of lidocaine with epinephrine. Lidocaine - local anesthetic and antiarrhythmic drug - used topically for pruritis, burning and pain of the skin or as a local anesthetic for minor surgery, as in this case - functions by blocking voltage-gated sodium channels preventing pain Lidocaine overdose - can be a result of excessive administration, accidental oral ingestion, accidental intravenous injection, or prolonged use of subcutaneous infiltration anesthesia which can lead to severe toxicity or death - Lidocaine and its two major metabolites may be quantified in blood, plasma or serum to confirm the diagnosis in potential poisoning victims - physicians treating lacerations in the hand or digits may easily dilute the adrenaline concentration to one half or one fourth by adding plain lidocaine --> this practice enables the tissue ischemia to wear off more rapidly and still allows for optimal exposure of the wound Plain lidocaine, without epinephrine, acts as a vasodilator and increases bleeding. To reiterate, no modern study has ever demonstrated that the injection of lidocaine with epinephrine has resulted in the necrosis of a finger, toe, ear, nose, or penis. *Note: tumescent anesthesia refers to local anesthetic agent injected in-solution with a large volume of tumenscent fluid, which is meant to provide not only local pain relief but also tugor to a tissue area of interest (such as when harvesting split-thickness skin grafts). Similarly, large volume fluid injection can be used to bluntly dissect tissue planes and ease the removal of fat in liposuction. Illustration shows the spectrum of side effects from lidocaine toxicity as a function of blood concentration.

A 28-year-old male has a family history of colon cancer. His father was diagnosed with colorectal adenocarcinoma at age 55. At what age should this patient begin colonoscopy screening?

*40* Patients with a first-degree relative diagnosed with colorectal cancer or advanced adenoma before age 60 should undergo colonoscopy beginning at age 40, or 10 years before the family member's diagnosis, *whichever comes first*. - Patients with 2 or more first-degree relatives with colorectal cancer or advanced adenoma at any age should also begin colonoscopy at age 40. - In an otherwise healthy patient with no family history of colon cancer, the patient should undergo colonoscopy beginning at age 50.

A 17-year-old female is brought into the emergency department following a motor vehicle accident. At the scene of the accident, the patient was opening her eyes spontaneously, making random and exclamatory remarks that seemed out of context to witnesses, and she was localizing to painful stimuli. On arrival to the trauma bay, the patient would open her eyes only to painful stimuli, was making incomprehensible sounds, and displayed flexor posturing to painful stimuli. What is the patient's Glasgow Coma Scale (GCS) upon arrival to the emergency department?

*7* The patient in the question stem had a GCS 12 at the scene of the accident and a GCS 7 in the trauma bay. The difference in the GCS sum, therefore, is 5. At this point, the patient should be considered critically ill and intubated immediately for airway protection. The patient received: 2 points for eye response 2 points for verbal response 3 points for motor response for a total of 7

Cohort study was conducted to assess relationship between high saturated fat consumption and occurrence of colorectal carcinoma among women. Group of women aged 40-65 was selected. Baseline saturated fat consumption was calculated using food questionnaire, and cohort was followed for seven years for dev't of colon cancer. Study showed that women with high baseline saturated fat consumption have four times the risk of colorectal cancer in 7-year period, compared to women with low fat consumption (RR = 4.0, 95% CI = 1.5 - 6.5). According to study results, what percent of colorectal carcinoma in women with high fat consumption could be attributed to diet?

*75%* Attributable risk percent (ARP) / etiologic fraction - impact of risk factor being studied - excess risk in population that can be explained by exposure to particular risk factor = (risk in exposed - risk in unexposed) / risk in exposed = (RR - 1) / RR = (4 - 1) / 4 = 0.75 = 75% --> 75% of colorectal carcinoma in high consumption group was attributable to high saturated intake

Pts with UC are at increased risk for colorectal cancer. Screening colonoscopy with mucosal sampling should be offered to pts with UC beginning....

*8 yrs after initial diagnosis --> repeated every 1-2 yrs thereafter*

A 18-year-old female presents to her primary care physician for a pre-sports physical. She has no prior medical history except that her mother states she has recently lost weight and that she has become concerned about her weight. On physical exam she is 5'6" and weighs 102 lbs. A serum electrolyte panel is drawn, which reveals a calcium level of 8.5 mg/dL. Serum albumin is 3.0 g/dL. What is this patient's corrected serum calcium level?

*9.3* Serum calcium level is correct by the following equation: *measured total calcium + 0.8 (4.0 g/dL - measured serum albumin level)* In this case, despite her measured calcium being in the low range, she has a normal corrected calcium secondary to her low albumin levels, most likely a result of her poor nutritional status.

A 25-week pregnant woman comes to your office for a regular prenatal visit. She had no history of diabetes prior to this pregnancy. Among other tests, you perform a fasting glucose test for which she has exceeded the upper limit of normal. What is the fasting blood sugar cutoff required to warrant further testing for gestational diabetes?

*95 mg/dL* = cutoff to warrant further testing The diagnostic threshold for an elevated fasting blood sugar (FBS) in pregnancy is > 95 mg/dL. Gestational diabetes mellitus (GDM) - insulin resistance condition that arises during pregnancy as a result of the anti-insulin effects of human placental lactogen, a placental product. While a FBS > 95 alone is NOT enough evidence to diagnose GDM, it is concerning if positive and is one of the criterion for a positive 3 hour oral glucose tolerance test (OGTT). The other time/level criterion are 1 hour > 180, 2 hours > 155, and 3 hours > 140. A positive test fulfills 2 of the 4 time/level criterion.

DM can be diagnosed by a single hemoglobin A1c (HbA1C) of

*>6.5%* Type 2 diabetes - insulin insensitivity in peripheral organs, which results in an increased insulin requirement. - HbA1c correlates with the mean glucose from the previous 8-12 weeks (approximate lifespan of a red blood cell) - The currently recommended goal for management is < 7%

The classic unhappy triad involves the

*ACL, MCL and medial meniscus* - This is the result of valgus stress on the knee joint. - That being said it is not uncommon to have injury to the lateral meniscus as well, this is likely related to the stress of the femur hitting against the tibia at the time of injury. - There is controversy as to which occurs more frequently the medial or lateral meniscus. - There was a trial in the 90s that showed that lateral meniscus injury may actually occur more frequently.

Pathophysiologic mechanism of Acute GVHD

*Activation of DONOR T lymphocytes* Acute GVHD - common after bone marrow transplant - target organs = skin, intestine, liver - recognition of host major and minor HLA-antigens by donor T-cells & consequent cell-mediated immune response

Plaque psoriasis resistant to high-potency topical corticosteroids. Treatment?

*Add topical calcipotriene (Dovonex, Sorilux)* - combo of topical corticosteroid and topical calcipotriene

A 36-year-old G1P0 presents to labor and delivery triage with contractions. Vaginal exam shows 5 cm cervical dilation at 0 station with membranes intact. The estimated fetal weight of the baby is 3600 g. Three hours later she is at 0 station and 7 cm dilated. An intrauterine pressure catheter (IUPC) is placed and demonstrates regular low intensity contractions. What is the next best step in the management of this patient?

*Administer IV oxytocin* This patient is experiencing prolongation of stage 1 active phase of labor. - This woman's cervix is dilating at a rate less than 1.2 cm/hour for a primipara, which is indicative of prolongation of labor. - The best step at this stage is to administer oxytocin given this woman's weak contractions as measured by the IUPC. The most common cause of protracted or arrested dilation (stage 1) in the active phase of labor is hypotonic contractions. - Additional causes include inadequate size and shape of the pelvis, excessive fetal size, and abnormal fetal position. - Protracted dilation and labor arrest may result from hypotonic contractions that are very painful and appear like normal labor. - They are only referred to as hypotonic as they are unable to propel the baby forward, not because they are less intense and/or painful. - Treatment is induction with oxytocin with or without amniotomy. *This is Stage 1, active phase of labor, and thus dilation should be occurring at 1.2 cm/hour or greater for this primigravida. For this reason, and given the hypotonic contractions, the best next step in management would be oxytocin. This is indeed labor, it is just progressing slowly. Hypotonic contractions can still be very uncomfortable/painful, perhaps as painful as regular contractions.*

26 yo man comes to doc with sudden onset of redness, watery discharge, and itching of both eyes. His vision is slightly distorted from excess tearing but is not blurred. He otherwise feels fine. Pt's past med hx is significant for obesity, asthma, sleep apnea, and chronic back pain. On eye exam, there is bilateral injection with granular appearance of conjunctiva. Mild eyelid swelling and clear discharge noted. Pupils equally round and reactive to light. Lungs have few scattered wheezes, & mild heart has faint systolic murmur at base. Most likely diagnosis?

*Allergic conjunctivitis* - acute HS rxn - *episodic itching, hyperemia, tearing, edema of conjunctiva & eyelids* - mild photophobia & dry-eye sensation - usually subsides in 24 hrs

44 yo obese women undergoes open cholecystectomy for complicated acute cholecystitis. She is receiving IV hydration, hydromorphone, abx. On second postop day, her temp is 98.2 F, BP is 110/80 mmHg, pulse is 92/min, and respirations are 12/min. O2 sat is 89% on RA. Her arterial blood gas results are: Arterial pH 7.24 PaO2 62 mmHg PaCO2 59 mmHg HCO3 28 mEq/L Most likely cause of hypoxemia in this pt?

*Alveolar hypoventilation* Respiratory acidosis via arterial blood gas --> *High PaCO2 & low PaO2 suggestive of alveolar hypoventilation*; although, high PaCO2 alone, in range of 50-80 mmHg, is sufficiency enough for diagnosis Hypoventilation associated with normal A-a gradient & resp acidosis

Most common form of drug-induced chronic renal failure

*Analgesic nephropathy* - papillary necrosis & chronic tubulointerstitial nephritis = most common pathologies seen - pts also more likely to develop premature aging, atherosclerotic vascular dz, urinary tract cancer

On a routine check-up, a 37-year-old man tells his primary care physician that he has recently noticed bright red blood on the toilet paper when he wipes. He denies any fatigue, decreased exercise tolerance, abdominal pain, or maroon-colored or black, tarry stools. He has no family history of colon cancer. He has never had a colonoscopy. On physical exam, his temperature is 37 C, heart rate 70/min, and blood pressure 120/75 mmHg. He does not have conjunctival pallor. There are no abnormalities on cardiac, pulmonary, and abdominal exams. Rectal exam is negative for bright red blood or occult blood. What is the best next step in the work-up of this patient?

*Anoscopy* *Bright red blood on the toilet paper and no risk factors for colon cancer should be evaluated for hemorrhoids using anoscopy.* Hemorrhoids - vascular structures in the anal canal. - Normally, they help with stool control, but may become swollen or inflamed.

Characterized by painful, red eye with tearing and decreased visual acuity. Exam findings include ciliary flush, pupillary constriction, hazy "flare" in aqueous humor, and hypopyon.

*Anterior uveitis* - most often idiopathic or traumatic but can be associated with systemic inflammatory diseases, such as infections, sarcoidosis, spondyloarthritis, and IBD - uvea = layer b/t cornea/sclera and retina - anterior uvea = iris & ciliary body

Systemic sclerosis is characterized by CT thickening via fibroblast dysfunction. Cutaneous features include thickening of skin with dermal sclerosis and vascular dysfunction (Raynaud phenomenon). Extradermal manifestations include esophageal dysmotility, interstitial lung dz, and HT. What is a specific serologic marked and is associated with extensive dz?

*Anti-topoisomerase I* (anti-Scl-70)

Ex. of secondary prevention

*BP screening at local church, Mammograms* 2° (Secondary prevention) - early detection of disease to either prevent or decrease morbidity from disease before onset of symptoms - e.g., colonoscopy

Patient has bipolar disorder, for which lithium is first line therapy. What should be checked before initiating lithium

*Baseline thyroid stimulating hormone (TSH) and basic metabolic panel (BMP)* - because of the risk of hypothyroidism and nephrotoxicity - lithium decreases the sensitivity of calcium to the parathyroid gland --> eventually calcium hypercalcemia, hypocalciuria, and elevated PTH

Child + multiple exposures on areas not covered by skin (arms and legs) over last several weeks + spreads gradually + excoriated erythematous papules on both forearms and both lower legs that drain small amount of serous fluid + itchy. Problem?

*Bed bugs* - Skin rash caused by bite of Cimex lectularis or "bedbugs" - Bugs spread on clothing, bedding, mattresses, and laundry - hide during the day - feed during the night - "Breakfast, lunch,and dinner" linear pattern - Skin inflammation often result of allergic reaction to bedbugs

47 yo man comes to ED after episode of coffee ground emesis. He has had upper abdominal discomfort for several mos, which he describes as "burning and fullness" that is relieved by food. Over last wk, pt has had black, tarry stools associated with weakness & fatigue. His med hx is unremarkable. He takes no prescription or OTC meds. Pt smokes pck of cigz a day and occasionally drinks alcohol. His BP is 124/82 mmHg while supine and 110/70 mmHg while standing. Pulse is 102/min. On exam, conjunctivae and palmar creases appear pale. Abdominal exam is unremarkable. Most likely increased in this pt?

*Blood urea nitrogen/creatinine ratio* abdominal pain + hematemesis + melena = upper GI bleeding via peptic ulcer - tachycardia = mild volume depletion Upper (not lower) GI bleeding - elevated BUN & BUN/creatinine ratio - increased urea production from intestinal breakdown of hgb & increased urea reabsorption in proximal tubule via associated hypovolemia

34 yo male involved in high-speed highway motor vehicle collision. He is intubated by rescue workers at accident scene. In ED, pt has decreased breath sounds on right side, normal breath sounds on left, and hypotension. Right-sided chest tube placed. PE reveals multiple bruises over entire chest wall as well as subcutaneous emphysema. Few hours later, his chest x-ray shows accumulation of air in pleural space as well as pneumomediastinum. Most likely diagnosis?

*Bronchial rupture* Pt's chest x-ray = persistent pneumothorax despite chest tube placement & pneumomediastinum + subcutaneous emphysema (palpable crepitus below skin) on PE --> rare = *tracheobronchial perforation secondary to blunt thoracic trauma* - right main bronchus = most commonly injured in these cases - diagnosis can be confirmed with high-resolution CT scanning, bronchoscopy, or surgical exploration - operative repair indicated

46 yo man comes to office with 2 mo ho depressed mood, low energy, and probz concentrating at work. Pt has gained 10 lb and is disgusted with himself for not exercising. He sleeps 14 hrs a day and has difficulty getting out of bed. He becomes tearful when describing how he loves his children but no longer feels happy to see them. Pt denies any suicidal ideation, intent, or plan, but acknowledges that he contemplated suicide 4 yrs ago during his divorce proceedings. At that time, he didn't seek treatment but felt better over course of subsequent year. Pt's PE and lab studies are within normal limits. He is diagnosed with major depressive disorder and started on escitalopram. After 6 wks, there is no improvement and pt is switched to sertraline, which is titrated up to max dose. On follow-up visit 2 mos later, he says that his depressive symptoms are only minimally improved. Decision made to discontinue sertraline and start new med. What meds would be most appropriate for this pt?

*Bupropion* - NE dop reuptake inhibitor - does not cause weight gain - no sexual side effects - good for person with hypersomnia and weight gain This pt - has failed 2 trials of antidepressants from same class (SSRI) --> different class needed

A 28-year-old male presents to a urologist upon referral from a fertility medicine specialist who evaluated the patient and his wife. The patient was told that he had a low sperm count with 'poor quality' sperm. Examination reveals a scrotum that, on the right side only, is enlarged and feels like a 'bag of worms' on palpation. Additionally, the examination reveals right testicular atrophy. When the patient lies supine, there is no change in the appearance or size of the scrotum. An ultrasound and color Doppler study of the patient's right testicle is shown in Figure A. Which of the following is the best next step in the management or evaluation of this patient's presentation?

*CT abdomen* *Right-sided varicocele, bilateral varicocele, and failure of a varicocele to disappear upon lying supine are signs suggestive of inferior vena cava (IVC) obstruction and warrant further investigation with a CT scan of the abdomen* Varicocele - dilation of the pampiniform venous plexus and spermatic veins - over 90% of varicoceles are left-sided, likely secondary to the comparable increased length of the left testicular vein and associated increased venous pressures - *In contrast, the right testicular vein is shorter, making varicocele less likely due to lower venous pressures; therefore, when a right-sided varicocele is identified, suspicion is raised for a force restricting return blood flow from the testicle to the IVC. Potential causes may include an IVC thrombus or abdominal mass compressing the IVC* *Figure shows an ultrasound and color Doppler study of a patient with varicocele; note the numerous anechoic tubes with bidirectional flow. Illustration A depicts the anatomy of the left and right testicular veins. Note that the left testicular vein drains into the left renal vein; however, the right testicular vein drains directly into the IVC*

50 yo man comes to doc due to 2 day ho constipation and inability to pass flatus. For past 3 days he has been having intermittent, but worsening RLQ pain. He has vomited several times today & feels nauseated. Exam shows distended abdomen with tenderness in RLQ; there is no rebound, there are no masses or hernias, and bowel sounds are absent. Rectal exam shows enlarged prostate. Upright x-ray of abdomen shows gas distributed throughout small and large bowel and some fluid levels. After nasogastric tube placement and hydration, his temp is 97.7 F, BP is 140/80 mmHg, pulse is 57/min, & respirations are 12/min. Lab results: RBCs 4.5 million/mm^3 WBCs 7,400 u/L Sodium 140 mEq/L Potassium 3.5 mEq/L Chloride 100 mEq/L BUN 15 mg/dL Creatinine 1 mg/dL Urine pH 5.5 Urine sediment: WBC 2/hpf and RBCs 15/hpf, and needle-crystals. Most appropriate next step in management?

*CT of abdomen* Ileus via vagal reaction by ureteral colic - needle-shaped crystals on urinalysis indicate uric acid stones - uric acid stones = radiolucent = have to be evaluated by CT of abdomen or IV pyelography - ileus will resolve once ureterolithiasis treated - stones <1 cm may pass spontaneously with hydration & analgesia; otherwise sx needed

Evaluation of blunt genitourinary trauma in hemodynamically stable pts with evidence of hematuria.

*CT scan abdomen and pelvis*, urinalysis - hemodynamically unstable pts + evidence of renal trauma = IV pyelography prior to surgical evaluation Blunt genitourinary trauma (BGT) - costovertebral pain + hematuria - rarely life-threatening, unless kidneys and vasculature are involved

Psoas abscess commonly presents subacutely with fever and lower abdominal pain or *flank pain radiating to the groin.* "Psoas sign," abdominal pain with hip extension, can often be detected on exam. What is required to confirm the diagnosis?

*CT scan of abdomen and pelvis* - drainage with abx = mainstay of treatment

Pancreatic cancer should be suspected in pts with ho chronic pancreatitis who develop abdominal pain and weight loss. Pts with jaundice should undergo abdominal ultrasound to rule out cancer in head of pancreas. Pts without jaundice are likely to have cancer in body and tail of pancreas and should undergo....

*CT scan of abdomen with contrast* - helps exclude other conditions

31 yo woman comes to office due to intermittent episodes of double vision. Pt first noticed symptoms during dinner 2 mos ago. She had no associated eye pain or redness. Her vision was normal next morning, so she didn't seek med attention. Pt reports occasional headaches and takes tylenol as needed. She doesn't use tobacco, alcohol, or illicit drugs. She is a software engineer and spends 6-8 hours daily in front of the computer screen. Vitals normal. Pupils 3 mm, round, and reactive to light. Pt has no ptosis, and ocular movements are normal. She is found to have Abs directed against nicotinic receptors on motor endplates. Most appropriate next step in management of this pt?

*CT scan of chest* Myasthenia gravis - pts with established diagnosis --> chest imaging = CT or MRI --> evaluate for thymoma as thymectomy can lead to long-term improvement

68 yo man brought to ED for acute right-sided hemiplegia, headache, and impaired consciousness. His symptoms started an hour ago and are progressively getting worse. He has no prior ho transient ischemic attack. His med probz include obesity, gout, BPH, benign essential tremor, HT, and hypercholesterolemia. Pt's meds include allopurinol, tamsulosin, propranolol, amlodipine, and simvastatin. He has smoked pck of cigz daily for 15 yrs. Neuro exam shows right-sided weakness and hemi-sensory loss. There is carotid bruit on his left side. Most appropriate next step in management?

*CT scan of head without contrast* Stroke via spontaneous intracerebral hemorrhage - classically present with neurologic deficits that gradually worsen over minutes to hours - can be associated with symptoms of elevated intracranial pressure (headache, vomiting, altered mental status) - should initially receive head CT without contrast to rule out hemorrhage

This patient likely has sarcoidosis based on the generalized malaise, signs of uveitis, erythema nodosum, and bilateral hilar adenopathy. What portends worse prognosis?

*Cardiac signs (myocardial infarction, cardiomyopathy, and heart failure)*, along with lung parenchymal and neural involvement Protective factors include - sudden onset of symptoms - erythema nodosum - isolated hilar adenopathy on chest radiography - absence of respiratory symptoms

23 yo man comes to doc for frequent, foul-smelling, bulky stools and wt loss over last 6 mos. He has lost about 10 lb over past 3 mos. He also has poor energy and occasional joint pains. Pt has no previous med probz & takes no meds. He doesn't use tobacco, alcohol, or illicit drugs. Vitals are normal. Exam shows mild pallor. No hepatosplenomegaly or lymphadenopathy. Hemoglobin is 10.2 g/dL and serum ferritin is 10 ng/mL. Immunoglobulin A anti-tissue transglutaminase Ab screening is negative. Small-bowel biopsy shows villous atrophy. Most likely cause of pt's symptoms?

*Celiac disease* - may pts with biopsy-confirmed celiac dz will have negative results on IgA testing via associated *IgA deficiency* = common in Celiac dz - If IgA serology is negative but suspicion for celiac dz is high, total IgA should be measured (or IgG-based serologic testing should be done)

52 yo woman complains of difficulty gripping her cup of coffee in morn. She likes to solve crossword puzzles but is unable to hold pen properly due to stiffness in her rt hand. Her symptoms gradually improve during the day and she is "fully functional" by late afternoon. Exam shows nontender nodules near elbows over back of her forearm. ESR is 45 mm/hr. What would most likely be involved during course of this pt's dz?

*Cervical spine* RA - morning stiffness + metacarpophalangeal joint inv't + nontender nodules on elbow + high ESR - inflammatory - women mostly - peak age = 50-75 - reduced grip strength can occur occasionally = sensitive sign of early dz Rheumatoid nodules - 30-40% of pts - firm and nontender subQ nodules close to pressure points (elbows) - most commonly affects cervical spine = spinal cord compression, cervical spine subluxation --> hyperreflexia, Babinski testing

A 28-year-old male is rushed to the emergency department after getting into a late night altercation in a local bar. During the altercation he was stabbed with a knife in his left upper extremity just proximal to his antecubital fossa. When he arrives at the emergency department, the knife is still embedded in his distal humerus. What is the most appropriate and diagnostic step in management of ruling out vascular injury in this individual?

*Check distal pulses by doppler ultrasound* In penetrating wounds to an extremity, the presence of distal pulses does not rule out vascular injury. Perform doppler studies to assess vascular integrity. Penetrating trauma to the extremities - Vascular injuries must be ruled out by doppler studies, and not merely by palpating distal pulses. Illustration is a clinical photograph showing a doppler ultrasound of the radial artery (left) in addition to the normal triphasic flow pattern (right).

60 yo man comes to doc due to nausea & abdominal pain. He had coronary angiogram and stent placement for chest pain 5 days ago and was discharged home day after procedure. His other med probz include HT, hypercholesterolemia, PAD, & T2DM. PE shows painless, purple mottling of skin of both feet. Lab results: Hgb 13.5 g/dL Leukocytes 10,000/uL with 12% eosinophils BUN 46 mg/dL Serum creatinine 3 mg/dL Serum C3 level Low Urinalysis - pH Normal - Protein 1+ - Leukocyte esterase Negative - Nitrates Negative - WBCs 5-10/hpf - RBCs 1-2/hpf Most likely cause of this pt's findings?

*Cholesterol emboli* Systemic atheroembolism via disruption of atherosclerotic aortic plaques / cholesterol crystal embolism - livedo reticularis = reticular, erythematous or purple discoloration of skin that blanches when pressure applied - labs = *eosinophilia*, eosinophiluria, hypocomplementemia - hollenhorst plaques after recent arteriography or cardiac cath

Symptomatic arterial vascular dz with intermittent claudication. Stopped smoking and started walking. Treatment?

*Cilostazol/Pletal* phosphodiesterase III inhibitor ↑ cAMP in platelets inhibits aggregation direct arterial vasodilation Clinical use: intermittent claudication prevention of stroke and TIA with aspirin angina

30 yo female with ho prolonged QT syndrome presents with severe acute bacterial sinusitis. What abx should be avoided?

*Clarithromycin* - K current alteration - delays repolarization Prolonged QT --> torsades

Often added in empiric antibiotic treatment for suspected necrotizing soft tissue infections due to its antitoxin effects

*Clindamycin* - suppresses bacterial toxin synthesis

Should be suspected in individual with weight loss, behavioral changes, and erythema of turbinates and nasal septum.

*Cocaine abuse* - increased energy - decreased appetite & need for sleep - mood changes = euphoria, irritability - other psych complications = anxiety, panic attacks, *grandiosity, psychosis (delusions, hallucinations)*

Best for determining incidence of disease. Comparing incidence of disease in 2 populations, with and without given risk factors, allows for calculation of relative risk.

*Cohort* Ex. Town of residence is risk factor of interest and leukemia is disease of interest. Residents of town A and town B could be followed for long period of time and incidence of leukemia determined for each site. Comparing incidences between 2 towns would allow for determination of relative risk for developing leukemia in town A compared to town B.

Patient's age and vascular intranasal mass is concerning for juvenile nasopharyngeal angiofibroma (JNA). What is performed to arrive to a diagnosis?

*Computerized tomography (CT) scan of the head* JNA - benign, vascular, and hormonally sensitive tumor that primarily occurs in adolescent men CT scan with contrast enhancement of the lesion - can show a non-encapsulated and vascular enhancing soft tissue mass that often widens the pterygomaxillary fissure - may also show bony erosions of affected adjacent structures

Case-control study conducted to assess relationship between alcohol consumption and breast cancer. First, investigators interviewed pts with breast cancer. They then selected neighbors of pts with same age and race, and used them as controls. Such study design helps to control what problems?

*Confounding* Matching - efficient method to control confounding - used in case-control studies - initial step = selection of matching variables = should always be potential confounders (age, race) --> case and controls then selected based on matching variables This scenario: neighborhood "controls" matched to "cases" by age and race

23 yo man comes to ED with severe abdominal pain and fatigue. He has been nauseated and vomiting for several hours and is unable to eat. He took few tablets of ibuprofen but had no pain relief. Pt has ho anemia and "cola-colored urine" that has not been further evaluated as he tries "to avoid going to the doc." ROS otherwise negative and he has no known med probz. He emigrated from Scotland during childhood but has not traveled outside the country since. Pt's temp is 101.8 F, BP is 120/80 mmHg, pulse is 102/min, and respirations are 16/min. Exam shows tenderness in RUQ of abdomen exacerbated by deep inspiration. Splenomegaly and jaundice present. No peritoneal signs. Rest of exam normal. Lab results: CBC: Leukocytes 13,800/mm^3 Hgb 10 g/dL MCV 88 um^3 Mean corpuscular hemoglobin concentration 46% Reticulocytes 12% Platelets 350,000/mm^3 Liver function studies: Total bilirubin 2.3 mg/dL Direct bilirubin 0.5 mg/dL AST 32 U/L ALT 34 U/L Lipase 12 U/L Hematology: Blood type A+ Serum LDH 1000 mg/dL Coombs test Negative Most likely primary cause of this pt's anemia?

*Congenital red blood cell defect* Hereditary spherocytosis - inherited def of ankyrin & spectrin - *increased mean corpuscular hemoglobin concentration*, hemolytic anemia, jaundice, splenomegaly - *Northern European descent* - hyperbilirubinemia --> jaundice, dark urine, pigment (calcium bilirubinate) gallstones - this pt = acute cholecystitis (fever, RUQ pain, positive Murphy's sign, leukocytosis) via pigment gallstones --> would probz need cholecystectomy + folate supplementation + splenctomy

A 67-year-old male with a history of prior myocardial infarction and catheterization-confirmed coronary heart disease presents to his cardiologist. He notes that he has had increased chest pain on exertion for the past 2 months. He also reports two episodes of chest pain at rest in the past month. His current medications include carvedilol daily and nitroglycerin as needed. The cardiologist orders a cardiac stress test to be performed as an outpatient. Which of the following advice should the patient be given regarding preparation for the test?

*Continue to take carvedilol and nitroglycerin as instructed* Patients with known coronary artery disease (CAD) should not withhold antianginal medications (carvedilol and nitroglycerin) prior to a cardiac stress test. *Management of antianginal medications prior to cardiac stress testing depends on whether a patient has a previous diagnosis of known CAD.* - *Patients without known CAD should withhold all antianginal medications, including calcium channel blockers, beta blockers, and nitrates, for 24-48 hours prior to the test.* - In contrast, patients with a known history of CAD should continue to take antianginal medications, as the test is used to assess the efficacy of the medication with exertion.

28 yo kindergarten teacher complains of gritty sensation and discharge from right eye for 3 days. She also has rhinorrhea, mild sore throat, and low-grade fever of 100.3 F. Symptoms began week after classes started in fall. Her past med hx significant for genital herpes and endometriosis. Eye exam shows mild injection and granular appearance of tarsal conjunctivitis of right eye with profuse watery discharge. Nasal mucosa normal, and pharynx has mild erythema without exudates. Most appropriate next step in management of this pt?

*Cool, moist compresses* conjunctival inflammation + watery discharge in setting of upper resp illness = viral conjunctivitis / pink eye - mostly adenovirus - common in small children, caregivers in late summer and early fall

47 yo diabetic woman comes to doc due to recent onset of tremors. She has undergone combined pancreatic and kidney transplant secondary to end stage renal dz and diabetes. She takes multiple meds, including immunosuppressants. Her temp is 97 F, BP is 152/90 mmHg, pulse is 78/min, & respirations are 16/min. Exam shows gum hypertrophy. Lab studies show: Hb 13 g/dL WBC 8,000/cmm Serum Na 135 mEq/L Serum K 5.3 mEq/L BUN 26 mg/dL Serum Creatinine 1.7 mg/dL What immunosuppressants is most likely responsible for her presentation?

*Cyclosporine* - inhibits transcription of interleukin-2, mainly T-helper lymphocytes - nephrotoxicity = most common and serious - HT (treat with CCBs) - Neurotoxicity - Glucose intolerance - Infection - Malignancy - *Gingival hypertrophy, Hirsutism* - GI manifestations - *Tremor*

After monitoring pt for six hrs, physician decides to do bedside assessment of brain death. What can be observed in a pt with brain death?

*Deep tendon reflexes* Brain death - irreversible cessation of brain activities - evaluating cortical and brain stem functions - providing irreversibility of brain activity loss - absent cortical and brain stem functions - *spinal cord may be functioning --> deep tendon reflexes may be present*

Characterized by >/= 1 persistent delusions and no other prominent psychotic symptoms. Functioning not markedly impaired.

*Delusional disorder*

CT scan = numerous minute punctate hemorrhages with blurring of grey-white interface.

*Diffuse axonal injury* - most significant cause of morbidity in pts with traumatic brain injuries - frequently due to traumatic deceleration injury - can result in vegetative state - sudden acceleration-deceleration impact --> rotational forces --> affect brain areas where density difference is max = mostly gray white matter junction - pt loses consciousness instantaneously and later develops persistent vegetative state

Uncoordinated contractions of esophagus. *Chest pain and dysphagia.*

*Diffuse esophageal spasm* - esophageal manometry = multiple simultaneous contractions of middle & lower esophagus - esophagram = "corckscrew" pattern - first-line treatment = CCBs

Leads to increased excretion of water and electrolytes by kidneys, resulting in dehydration, wt loss, orthostatic hypotension, hyponatremia, and hypokalemia. Urinary sodium and potassium will be elevated.

*Diuretic abuse* - pts sometimes abuse diuretics to induce wt loss

56 yo right-handed man comes to doc with right-sided weakness and speech difficulty. He speaks in short sentences with considerable effort. He fully follows written and verbal commands but has difficulty writing and repeating. What lesion location is most likely responsible for observed findings?

*Dominant frontal lobe* Ischemic stroke affecting dominant frontal lobe (left) in left middle cerebral artery (MCA) territory - 95% of right-handed and up to 70% of left-handed ppl are left-hemisphere dominant for verbal & written language - lesion of posterior inferior frontal gyrus (Broca's area) --> can comprehend and follow commands but are unable to verbalize or writer properly (expressive aphasia)

Any patient with a suspected transient ischemic attack (TIA) should undergo urgent evaluation due to the high risk of a subsequent stroke. Work-up should include ...

*ECG, non-contrast head CT, carotid doppler, and an echocardiogram*

Elevated BMP higher than expected in what pt characteristics?

*ELevated creatinine* - low GFR = low albumin = higher BMP

Should be obtained in all pts with syncope due to suspected structural heart disease

*Echocardiogram* Aortic stenosis - exertional symptoms - delayed & diminished carotid pulses = pulsus parvus et tardus - single & soft S2, audible S4 - harsh ejection (crescendo-decrescendo) systolic murmur in second right intercostal space with radiation to carotids

Prospective cohort study was conducted to assess effects of oral contraceptive (OC) use on incidence of breast cancer. Crude analysis of study suggests association between OC use and breast cancer (relative risk [RR] = 1.40, 95% confidence interval [95% CI]: 1.01, 1.96). Further analysis shows that in women with fam hx of breast cancer, OC use increases risk of breast cancer (RR = 3.33, 95% CI: 2.51, 4.41). In women without fam hx of breast cancer, significantly smaller effect observed (RR = 1.27, 95% CI: 1.01, 1.60). Results of this study are best explained by what?

*Effect modification* / Interaction - when extraneous variable/modifier changes direction or strength of association between risk factor & dz - modifier is associated with dz but not the risk factor - can sometimes be confused with confounding - measures of association calculated in each of the strata are significantly different (in strength or direction) from one another - stratification can make effects of modifier more apparent

Cervical lymphadenitits in little girl. Systemic symptoms. Unilateral, skin erythema, node 2-3 cm in size. Treatment?

*Empiric abx with observation for 4 wks* - next step after = ultrasound

postcholecystectomy syndrome is persistent abdominal pain or dyspepsia either postoperatively (early) or months to years (late) after cholecystectomy. Etiologies include biliary (retained common bile duct, cystic duct stone) or extra-biliary (pancreatitis, peptic ulcer dz) causes. After US of abdomen reveals mild dilation of common bile duct, and pancreas is visualized and appears normal, what is the best next step in management of this pt?

*Endoscopic retrograde cholangiopancreatography* Abdominal imaging (US) followed by direct visualization (endoscopic retrograde cholangiopancreatography, magnetic resonance cholangiopancreatography)

Preferred as nutrition for pts with severe pancreatitis who have been on prolonged bowel rest. Associated with lower complication rates and shorter hospitalizations.

*Enteral nutrition*

Common cause of cyclic breast pain in women of reproductive age. Classic findings are diffusely nodular breasts with nonfocal tenderness and no nipple discharge or lymphadenopathy.

*Fibrocystic breast changes* - pts can be offered NSAIDs and/or oral contraceptives for symptomatic relief

A 48-year-old female presents with a chief complaint of increasing fatigue over the last 3 months. Her medical history is significant for recurrent urinary tract infections for which she takes prophylactic trimethoprim-sulfamethoxazole three times per week. Preliminary work-up reveals a hemoglobin of 8.7 g/dL, hematocrit of 26.1%, and MCV of 106 fL. Further investigation reveals normal values for serum ferritin, transferrin, TIBC, and methylmalonic acid. Serum homocysteine is elevated to 48 umol/L (upper limit of normal 15 umol/L). Which of the following is the most likely cause of this patient's anemia and associated symptoms?

*Folic acid deficiency* Megaloblastic anemia due to folic acid deficiency - methymalonic acid is normal in folic acid deficiency but elevated in vitamin B12 deficiency - can be seen in alcoholics with poor dietary intake, pregnant women who have an increased folate requirement, hemolytic anemia, or in the context of certain chronic medications such as methotrexate or trimethoprim-sulfamethoxazole Both folic acid and cobalamin deficiency are characterized by macrocytic anemia with elevated homocysteine Hypothyroidism can also contribute to folate deficiency; a TSH level should be included in work-up of a megaloblastic anemia. Treatment for folate deficiency is oral folic acid supplementation.

A 74-year-old female with a history of lung adenocarcinoma status post lobectomy, chronic obstructive pulmonary disease, congestive heart failure, and diabetic nephropathy presents to clinic complaining of hearing loss. Over the last week, she has noticed that she has had difficulty hearing the telephone or the television. When sitting in a quiet room, she also has noticed a high-pitched ringing in her ears. She denies any vertigo or disequilibrium. Further review reveals ongoing dyspnea on exertion and worsening cough productive of whitish sputum for the last month. The patient was recently discharged from the hospital for a congestive heart failure exacerbation. She lives alone and keeps track of all her medications, but admits that sometimes she gets confused. She has a 20 pack-year tobacco history. Her home medications include aspirin, lisinopril, furosemide, short-acting insulin, and a long-acting ß-agonist inhaler. She does not use supplemental oxygen. Two weeks ago she completed a course of salvage chemotherapy with docetaxel and cisplastin. In the office, her temperature is 37.4°C (99.3°F), blood pressure is 150/90 mmHg, pulse is 84/min, respirations are 18/min, and oxygen saturation is 94% on room air. Her tympanic membranes are clear and intact with no signs of trauma or impaction. Auditory testing reveals bilateral hearing impairment to a whispered voice. The Weber test is non-lateralizing. Rinne test is unrevealing. Hemoglobin: 11.8 g/dL Leukocyte count: 9,400/mm^3 Platelet count: 450,000/mm^3 Serum (Present visit): Na+: 134 mEq/L K+: 3.8 mEq/L Cl-: 95 mEq/L HCO3-: 30 mEq/L BUN: 45 mg/dL Creatinine: 2.1 mg/dL Serum (1 month ago): Na+: 135 mEq/L K+: 4.6 mEq/L Cl-: 102 mEq/L HCO3-: 24 mEq/L BUN: 22 mg/dL Creatinine: 1.2 mg/dL On follow up visit two weeks later, the patient's hearing has significantly improved. Which of the following is the most likely cause of her initial hearing loss?

*Furosemide* Bilateral-hearing loss via furosemide-induced ototoxicity in setting of acute kidney failure - associated with rapid IV admin, renal failure, or concurrent use of other ototoxins Sudden onset bilateral sensorineural hearing loss is often medication induced. Common causes include loop diuretics (furosemide), platinum containing drugs (cisplatin, carboplatin), aminoglycosides, and aspirin.

Causes hemolytic anemia due to oxidative injury to RBCs. Hemolytic episodes can be ppted by infection or meds (especially sulfa drugs, antimalarials, nitrofurantoin).

*G6PD*

Infant with FTT or SID + recurrent pneumonia + low growth curve + fam ho SID + normocytic anemia. Diagnosis?

*GERD*

A 45-year-old G5P4105 presents to her gynecologist's office with six months of increasingly heavy periods. She now soaks one super absorbent tampon every two hours for five days each cycle. Her cycles have also become irregular, with intermenstrual bleeding for the last two months. She now experiences significant dysmenorrhea, requiring 400 mg ibuprofen every four hours for the majority of each menses. In addition, she reports new onset mild dyspareunia with intercourse and a "heavy feeling" in her pelvis. She has also noticed increased urinary frequency but denies bowel changes. The patient has a past medical history of obesity and type II diabetes on metformin. Her last child was born four years ago, and she has had five spontaneous vaginal deliveries. At this office visit, temperature is 98.5°F (36.9°C), blood pressure is 137/84 mmHg, pulse is 87/min, and respirations are 14/min. Which of the following physical exam findings is most likely to be present in this patient?

*Globular 10-week sized uterus* Adenomyosis - causing her dysmenorrhea, menorrhagia, dyspareunia, and urinary symptoms - due to overgrowth of the endometrium into the myometrium, adenomyosis also often presents with a globular enlarged uterus usually not exceeding 12 weeks in size = only condition that could explain all of her complaints - appropriate demographic - the *classic patient with adenomyosis is a multiparous women in her 40s.* - diagnosis can be made by MRI or pelvic ultrasound, which can show irregularity of the myometrium, loss of a clear endomyometrial border, and thickening of the junctional zone greater than 12 mm - junctional zone is the area of myometrium that forms a border with the endometrium - pathology from hysterectomy specimen is technically the gold standard but is not required for diagnosis - hysterectomy is the most effective treatment, though levonorgestrel-releasing IUDs and GnRH analogs have also been shown to be helpful

These are used for phalangeal and metacarpal fractures. These extend from proximal forearm (not over elbow) to beyond the distal interphalangeal join and immobilizes the index and long fingers, OR the ring and little fingers

*Gutter splint*

36 yo man comes to doc with 1 mo ho diarrhea, abdominal distention, and flatulence. Over this period, he has lost 15 lb & developed night sweats and occasional arthralgias. No ho recent travel. On physical exam, his temp is 99.5 F, BP is 112/74 mmHg, pulse is 104/min, and respirations are 16/min. PE shows enlarged, nontender cervical and inguinal lymph nodes. Small ulcer seen on buccal mucosa. He has no skin rashes or joint effusions. Abdomen soft & nontender. Most appropriate next step immediate management of this pt?

*HIV testing* Acute HIV infection - symptoms within first 6 mos of infection - mononucleosis-type syndrome = fever, night sweats, arthralgias, LAD = 2-4 wks after infection - painful mucocutaneous ulcerations, skin rash, prolonged diarrhea - headache and dry cough nonspecific + GI symptoms + arthralgias + night sweats + wt loss + LAD = differential of: acute HIV, IBD, CT dz, lymphoma, Whipple's

38 yo man presents to office for health maintenance visit 4 wks in advance of a one mo trip to Egypt, where he plans to visit major cities and landmarks. His past med hx is significant for HT, well-controlled on HCTZ. He doesn't smoke & drinks alcohol only occasionally. This will be his first international trip. His vitals are all within normal limits and his PE is unremarkable. Vaccination against what would be of greatest benefit to this pt?

*Hepatitis A* North Africa CDC recommendations: *hepatitis A, hepatitis B, typhoid vaccines, polio booster* - most common vaccine-preventable ID = hepatitis A - low-risk zone for hep A (most Asian & African countries = high risk zones) - single dose of hep A vaccine = adequate protection for young immunocompetent adult - second dose should be administered for long-term immunity

Should be suspected in pts with muscle and joint aches, nausea, diarrhea, abdominal cramping, and pupillary dilation. These subjective symptoms are often severe but generally not life-threatening.

*Heroin withdrawal*

85 yo man presents with rash over his forehead, tip of nose and left eye. He also complains of pain & decreased vision. He has had fever, malaise, and burning sensation around his left eye for past 5 days. His BP is 140/90 mmHg, pulse is 92/min, respirations are 14/min, & temp is 101 F. PE reveals vesicular rash on periorbital region & lid margins. Left eye is red, with chemosis of conjunctiva. Dendriform ulcers seen on cornea. Most likely diagnosis?

*Herpes zoster OPHTHALMICUS* - via varicella-zoster virus - elder, immunosuppressed more often - travels via ophthalmic branch to forehead and eye - symptoms = fever, malaise & burning, itching sensation in periorbital region - conjunctivitis & dendriform corneal ulcers - treatment started within 72 hrs after eruption with high dose acyclovir --> reduces complications

Increases risk of stroke more than any other risk factor including hypercholesterolemia, DM, smoking, and sedentary lifestyle.

*Hypertension* - strongest association with both ischemic and hemorrhagic stroke - via elevated shearing force on intracerebral vascular endothelium -- accelerates atherosclerotic process & promotes thrombi formation

A 20-year-old girl is scheduled for breast augmentation. She has no past medical history and is not currently on any medications. Prior to endotracheal intubation, the patient's skin is marked by a provider wearing gloves using proper sterile protocol. Within seconds, the patient starts to hyperventilate and progresses to have inspiratory stridor. The patient has increased work of breathing and develops oxygen desaturation and hypotension. Visualization of the vocal cords is difficult. What is the most appropriate next step in management?

*IM epinephrine* Anaphylactic shock secondary to latex allergy - The best initial step in management is rapid administration of IM epinephrine. Anaphylaxis - most severe form of allergy and involves a systemic reaction mediated by IgE (type I hypersensitivity reaction) that may be life-threatening - can be difficult to diagnose because manifestations may vary from patient to patient, but symptoms may include: skin/mucosa (~90%, hives, itching, flushing, periorbital edema, tongue swelling), respiratory (~70%, nasal, dysphonia, stridor, dyspnea, wheeze), gastrointestinal (<50%, nausea, vomiting, diarrhea, pain) and cardiovascular (<50%, syncope, dizziness, tachycardia, hypotension) - may progress to shock and respiratory failure if not promptly identified and treated - *Treatment begins with intramuscular epinephrine and should be instituted as soon as the condition is recognized and it may be repeated every 20 to 30 minutes* - If the patient is in shock or fails to respond, IV epinephrine should be given - If the patient develops oral or facial swelling, he or she should be intubated.

A 69-year-old male presents with severe left-lower quadrant (LLQ) pain. Vitals are remarkable for a temperature of 38.3 C. Labs are remarkable for a WBC count of 15,000. An abdominal CT scan shows a localized abscess of 2.5 cm near an inflamed diverticulum. What is the next step in the management of this patient?

*IV abx* Complicated diverticulitis with abscess formation - IV antibiotics are the recommended initial treatment for abscesses < 3 cm. - With increasing severity, the patient may become febrile with a leukocytosis. - An abscess > 3cm is managed surgically with drainage, usually by interventional radiology.

Severe cases of PID merit treatment with ...

*IV cefoxitin and IV doxycycline* - patient should be transitioned to oral treatment 24 to 48 hours after clinical improvement - tubo-ovarian abscesses merit at least 24 hours of inpatient treatment, and should be monitored closely for medication failure and a need for surgical management Figure demonstrates a left-sided tubo-ovarian abscess pushing the uterus rightward, as indicated by the red arrow.

Patient is presenting with signs and symptoms of a pulmonary embolism (PE) and needs intervention immediately. With a history of recent hemorrhagic stroke, the best next step in management is placement of an ...

*IVC filter* In the setting of an EKG demonstrating sinus tachycardia (the most common finding on EKG in PE), a normal chest radiograph, and a positive D-dimer and lower extremity ultrasound, the most likely diagnosis is PE - Given this patient's history of recent hemorrhagic stroke, anticoagulation is contraindicated; thus, the best next step in management is placement of an IVC filter It is true that placing an IVC filter will do nothing to help the current clot resolve. However, if the patient is seemingly stable (oxygenating well, well perfused, no signs of RV failure) then it is likely that this episode will resolve on its own eventually, and you can monitor the patient without giving heparin. The IVC filter will prevent more clots from entering the pulmonary circulation which could potentially make the patient unstable which would require anticoagulation which would put them at risk given their history of hemorrhagic stroke (in the case they have other clots as well on ultrasound). In such a patient as this who is stable, we can keep them stable with the filter without anticoagulating them and potentially causing another stroke!

A 78-year-old woman presents to the emergency department complaining of generalized pain and edema in her left leg. She denies chest pain, shortness of breath, and hemoptysis. She takes 2.5 mg warfarin daily for atrial fibrillation and her INR is 2.6. Lower extremity ultrasound reveals the presence of a deep venous thrombosis in the left popliteal vein. Which of the following should be ordered next in the treatment of this patient?

*IVC filter* This patient is suffering from a deep vein thrombosis (DVT). The fact that the patient is already on a therapeutic dose of anticoagulant makes inferior vena cava (IVC) filter placement necessary. - The patient is taking warfarin and has reached a therapeutic level of anticoagulation (INR 2.0 - 3.0). - *The appropriate action is to have an IVC filter placed as she has failed anticoagulation*.

Resting ankle-brachial index of 1.50 indicates what?

*Incompressible vessels in LE* 1.0 - 1.4 = normal 0.9 - 0.99 = borderline *>1.4 = incompressible vessels = not reliable = older, diabetes*

44 yo man comes to ED for exertional dyspnea, abdominal distention, and abdominal discomfort. He has ho alcohol abuse, PUD, and acute pancreatitis. PE shows distended abdomen with shifting dullness and 1+ ankle edema. DIagnostic paracentesis performed and shows: Albumin 2.5 g/dL Neutrophils 200/uL LDH 40 units/L Glucose 80 mg/dL Amylase 40 units/L Blood studies: Albumin 3.8 g/dL Potassium 3.2 mEq/L Sodium 130 mEq/L Glucose 120 mg/dL AST 50 units/L ALT 40 units/L LDH 100 units/L Amylase 80 units/L Creatinine 1.2 mg/dL Most likely cause of this pt's ascites?

*Increased capillary hydrostatic pressure* Ascites SAAG - serum albumin concentration - peritoneal fluid albumin = 3.8 - 2.5 = 1.3 g/dL >1.1 --> portal hypertension = cirrhosis = increased hydrostatic pressure within hepatic capillary beds

26 yo female + 4 mo ho continuous right-sided headache. Headache associated with tearing and nasal congestion only on right, and has not responded to OTC analgesics. Pt went to ED few nights ago bc of pain, and CT of head at visit was normal. On exam you note conjunctival injection on right. Treatment now?

*Indomethacin* Cluster headache = unilateral - tearing - *chronic paroxysmal hemicrania = variant more common in women* = indomethacin-responsive headache

Effective and rapid method used to abort an acute cluster headache without major side effects.

*Inhaled oxygen* / 100% oxygen by facemask Cluster headache - acute, severe periorbital pain + autonomic manifestations (ipsilateral miosis, lacrimation) but no vision changes - subcutaneous sumatriptan can also be used as long as there are no contraindications - often being during sleep, peak rapidly, last appx 90 minutes, and occur up to 8 times daily for 6-8 weeks, followed by period of remission

74 yo woman brought to office by her daughter due to change in behavior. Over past 3 mos, pt has become more withdrawn. SHe no longer watches her fave TV show and seems less interested in her fam. Daughter has noticed increasing forgetfulness over past 3 yrs but assumed it "comes with aging." Over past yr, pt has stopped doing daily crossword, had difficulty remembering to eat and clean herself, and occasionally failed to recognize her grandchildren. Montreal Cognitive Assessment 22/30 (normal >26). Serum TSH and vit B12 normal. Most appropriate next step in management of this pt?

*Initiate rivastigmine* Cholinesterase inhibitors - donepezil, rivastigmine, galantamine

Idiopathic, chronic condition characterized by bladder pain that is worsened by filling and relieved by voiding. Dyspareunia, urinary frequency, and urgency can also be present.

*Interstitial cystitis (painful bladder syndrome)* - clinical diagnosis - treatment = palliative; trigger avoidance; amitriptyline, analgesics for pain flares

Systemic sclerosis in final stages --> develop what lung dz?

*Interstitial lung dz* - restrictive - and pulmonary artery HT

A 45-year-old male was a driver in a single vehicle roll-over MVC and brought to the emergency department by EMS. On presentation, he does not open his eyes, he withdraws to pain, and he makes incomprehensible sounds. Vital signs are as follows: HR 110, BP 115/65, RR 23, and SpO2 92% on room air. He has obvious signs of trauma to the chest and abdomen, with a positive seatbelt sign and bruising on the upper abdomen. His abdomen is distended and markedly tender to palpation. He also has an obvious open deformity of the left femur. Radial and dorsalis pedis pulses are present bilaterally. What is the first action to be taken in the management of this patient?

*Intubate* Glascow Coma Scale = less than 8 = risk of loss of airway

Often seen in PTSD when the patient describes the traumatic event without any show of emotion

*Isolation* of effect

Most common type of testicular sex cord stromal tumors. Estrogen production markedly increased in tumorous growth with secondary inhibition of LH and FSH levels. Endocrine manifestations include gynecomastia, however in prepubertal cases, precocious puberty is common.

*Leydig cell tumor* = 20-60 yo - testicular swelling - gynecomastia = first presenting symptom often - can produce androgens & estrogen -> decreased LH & FSH - golden brown and homogenous cut surface - cells = crystalloids of Reinke in cytoplasm

Vector for Bartonella quintana

*Lice* - homeless people - trench fever - influenza-like syndrome with relapsing fever - WWI when soldiers in trenches infected

Kava is a popular herbal supplement used for the treatment of mild-to-moderate generalized anxiety disorder (GAD). Kava has been shown to cause

*Liver failure* - hepatitis, cirrhosis, and liver failure Kava - popular herbal supplement that originated in the Pacific Islands and has purported benefits in treating anxiety, insomnia, depression, stress, and menopausal symptoms - roots of the kava plant have been ground up and mixed with water to form a potable liquid - Kavalactones, the active ingredient in kava supplements, interacts with numerous liver enzymes and also may cause direct hepatotoxicity - FDA recommends that patients with pre-existing liver disease or patients who are taking medications that are predominantly metabolized by the liver, should consult their physician before initiating kava supplementation due to the risk of hepatotoxicity - Kava down-regulates liver enzymes that metabolize many common medications, leading to potential hepatotoxic drug interactions

24 yo woman comes to office due to headaches that she has had every few mos for past 6-7 years. She describes these as left-sided and throbbing; they are associated with mild nausea, last for 12-18 hours, and improve after she rests in a dark and quiet room or takes ibuprofen. For past mo, she has been waking up almost daily with bifrontal headaches often accompanied by nausea and vomiting. She also has occasional blurry vision. Pt attributes these headaches to increased stress at work after being promoted to supervisor position. She has had no fevers, focal weakness, or numbness. She has no other med hx and doesn't use tobacco, alcohol, or recreational drugs. Her mom and sis have migraine headaches. Temp is 98.6 F, BP is 120/80 mmHg, and pulse is 72/min. Bilateral pupils equal & reactive to light. She has normal exam of cranial nerves, motor strength, sensation, and deep tendon reflexes. Most appropriate next step in management of this pt?

*MRI of the brain* This pt = 2 different types of headache 1= migraine 2. new type that is diff't in character (headache on awakening) - concerning symptoms = nausea, vomiting, blurry vision - new headache + symptoms = concerning for elevated intracranial pressure (from mass effect) --> evaluate early with MRI of brain to determine cause

Group of investigators is planning a study to evaluate relationship between serum fibrinogen levels and incidence of acute coronary syndrome. They assume that serum fibrinogen levels are normally distributed in population of interest. What statement is most consistent with this assumption?

*Mean is equal to median* Normal distribution: Central tendency mean = median = mode

Information derived from which of the following provides the best evidence when selecting a specific treatment plan for a pt?

*Meta-analysis* - strongest evidence for treatment screening or prevention strategy - pooling of multiple studies examining given dz or exposure - depends on original study type - larger study size - can resolve conflicts in literature - unable to eliminate limiting factors in original studies

A 24-year-old male presents to the clinic complaining of pain on urination. On exam, watery, non-purulent discharge is noted at the urethra without evidence of sores or ulcers on the penis. The patient is accompanied by his girlfriend who is not your patient, but complains of a, "nasty greenish," discharge from her vagina and asks if it could be related. Which of the following is the most appropriate next step in management?

*Metronidazole for the patient and his girlfriend* Nongonococcal urethritis refractory to azithromycin treatment. Trichomonas vaginalis infection should be suspected, and the appropriate treatment is metronidazole. Urethritis in men - presents with dysuria and discharge with neutrophils on urethral swab. - Trichomonas vaginalis is a common cause of urethritis in men that is not covered by azithromycin. Therefore, in men with nongonococcal urethritis refractory to azithromycin treatment, metronidazole therapy should be initiated. - Repeat sexual contact after initial treatment could result in a similar presentation and thus should also be taken into consideration. - Typically trichomonas is asymptomatic in men, however his female partner has the classic, frothy-green discharge seen in trichomoniasis which should point the diagnosis towards trichomonas instead of gonorrhea or chlamydia. - Typically the workup could include a wet mount demonstrating the classic trichomonads however with strong clinical suspicion treatment can be started.

68 yo man brought into ED due to confusion that started this morn. He lives with his daughter, who says that pt has had 3 days of fever and cough productive of yellow sputum. He has ho CAD and has smoked pck of cigz daily for 40 yrs. Temp is 103 F, BP is 110/70 mmHg, pulse is 110/min, respirations are 24/min, and pulse oximetry is 92% on RA. Chest ausc reveals crackles and reduced breath sounds in right lower chest. Lab results: CBC Hgb 11.8 g/dL Hct 36% Leukocytes 13,000/mm^3 Serum chemistry Sodium 132 mEq/K Potassium 3.8 mEq/L BUN 24 mg/dL Creatinine 1.1 mg/dL Chest x-ray reveals alveolar infiltrations in right lower lobe. Most appropriate step in management?

*Moxifloxacin and admit to hospital* CAP - CURB-65 - hospitalized on med floor = fluoroquinolone (moxi) or beta-lactam + macrolide (ceftriaxone + azithromycin)

This patient presents with menorrhagia and pelvic pain of two years duration and ultrasound findings consistent with uterine fibroids (leiomyomas). What is the preferred management for patients who wish to remain fertile following treatment?

*Myomectomy* - surgical treatment option for women who have not completed childbearing, or those who want to keep their uterus - there are many ways the myomectomy can be performed - there can be an abdominal approach, a laparoscopic approach, and a hysteroscopic approach

34 yo woman comes to doc with several-mo ho chest pain. Pain is left-sided, does not change with deep inspiration, and typically lasts several hours. She currently has the pain; it is unrelated to physical activity but worsens with emotional stress. Pt has no cough, syncope, or SOB. She has no significant fam hx and doesn't use tobacco, alcohol, or illicit drugs. She takes no meds and has no drug allergies. Her BP is 110/70 mmHg and pulse is 78/min. Heart sound are normal. LUngs clear to auscultation. ECG shows normal sinus rhythm with no significant abnormalities. Best next step in management of this pt?

*No further testing for coronary artery disease* Pretest probability of CAD - based on age, gender, cardiac risk factors, chest pain character - *low-risk pts = men age <40 and women age <50 with atypical chest pain and no significant cardiac risk factors (non-smoker, no fam hx of premature CAD)* - positive stress test in pts at low risk for CAD = likely to be false positive --> can lead to further unnecessary testing for procedures This pt: chest pain for several mos that is unrelated to activity + young + no risk factors for dev't of premature CAD + normal ECG --> low pretest probability of CAD --> additional testing for CAD not advised

C diff treatment

*ORAL* vanc

45 yo man comes to doc after experiencing left-sided chest pain for 1 day. Chest pain is non-radiating, constant, and increases with left hand movement. Pt has no nausea, dyspnea, or diaphoresis. He has 25 pck yr smoking hx. Vitals within normal limits. On exam, pain is reproduced with palpation of left-sided chest wall muscles. ECG shows normal sinus rhythm of 90/min, QRS complex duration of 80 msec (normal <120 msec), and PR interval of 280 msec (normal <200 msec). There are no significant ST segment or T-wave abnormalities. ECG shown (flip). Best approach to this pt's ECG findings?

*Observation* ECG: normal sinus rhythm + *first-degree AV block* - via delayed impulse transmission at # of possible locations from atria to ventricles - can occur in atria, AV node, bundle of His, or infra-Hisian conduction system - *most first-degree AV blocks with normal QRS duration occur via conduction delay in AV node & require no further eval unless there are significant associated bradycardic symptoms (eg syncope)* - *this pt: chest pain worse with movement & reproducible on palpation = more consistent with musculoskeletal rather than cardiac cause = can be observed*

Most likely to be associated with resistant HT in adults?

*Obstructive sleep apnea*

A 33-year-old female presents to clinic with a sore throat and subjective fevers. She has a history of Graves' disease and is receiving treatment for it, though cannot recall the name of the medication she takes. Vital signs are T 38C, HR 90, RR 14, and O2 saturation 99% on room air. Physical examination reveals rhinorrhea and moderate oropharyngeal erythema without exudates. Lungs are clear bilaterally. What is the next step in management?

*Obtain a CBC* Graves' disease + likely on methimazole or PTU. - A potential side effect of this medication is agranulocytosis and thus a CBC is the next step in management.

Most appropriate next step after diagnosis of infective endocarditis...

*Obtain bacterial blood cultures* Obtain serial blood cultures (min of 3 from separate venipuncture sites over specified period *prior to initiating abx therapy* to improve microbiological yield and ensure causative organism is identified)

46 yo man comes to doc for a routine health maintenance visit. He has no med probz. He smokes pck of cigz daily and doesn't drink alcohol. His mom has HT, & his dad died of prostate cancer at age 70. Pt's BP is 128/76 mmHg & pulse is 80/min. His BMI is 29 kg/m^2. Lab results: Total cholesterol 262 mg/dL HDL 50 mg/dL LDL 162 mg/dL Triglycerides 150 mg/dL Fasting glucose 100 mg/dL His calculated 10-risk of CV dz is 9%. Most appropriate next step in next step in management of this pt's dyslipidemia?

*Oral rosuvastatin* Several risk factors for ASCVD: - smoking, hyperlipidemia, age, male gender - should be counseled regarding smoking cessation a + dietary & exercise changes His 10-yr estimated ASCVD - risk of 9% - high enough to justify statin therapy in addition to lifestyle modification Estimated 10-yr risk > 7.5% in pts 40-75 yo - moderate-intensity = atorvastatin 10-20 mg, rosuvastatin 20-40 mg daily high-intensity = atorvastatin 40-80 mg, rosuvastatin 20-40 mg daily

21 yo female asks about Pap testing recommendations...

*Pap testing without HPV co-testing now and in 3 yrs if results are normal* - no HPV co-testing if under 30* --> will probz be positive, which they can clear = unnecessary

A 69-year-old man with a history of alcoholism presents to the emergency room with abdominal pain and altered mental status. He still drinks and is a Child-Pugh Class B cirrhotic. He is accompanied by his wife, who states that he was acting normally this morning but became confused as the day progressed. His vital signs are heart rate 95 beats per minute, respiratory rate 14 breaths per minute, blood pressure 130/85 mmHg, and temperature 38.1 degrees Celsius. On physical exam, he is alert and oriented to person only. He groans when you press on his abdomen, and he has shifting dullness. What is the next best step in management?

*Paracentesis* Spontaneous bacterial peritonitis - *The next step in management is paracentesis for gram stain, culture, and cell count* *Cirrhotic patients with ascites who develop a fever or a change in mental status should be worked up for spontaneous bacterial peritonitis (SBP)* - In SBP, the ascitic fluid becomes infected - The condition is associated with a high mortality rate (20-30%) as well as a high recurrence rate (up to 70% in the first year) - Common pathogens are E. coli, Klebsiella, and Streptococcus pneumoniae - Patients present clinically with abdominal pain, fever, vomiting, and rebound tenderness and may progress to sepsis. - *Diagnosis is made via paracentesis which reveals WBCs, PMNs, and positive gram stain or culture growth* - *Greater than 250 PMNs per mm3 meets criteria for SBP*

Seborrheic dermatitis is associated with

*Parkinson's disease* HIV Seborrheic dermatitis common chronic disorder in which greasy scales overlying erythematous patches or plaques are present on scalp, face, or external auditory canal - less commonly, it presents on the central chest, axilla, groin, submammary folds, and gluteal cleft - Although it is associated with Parkinson's disease and HIV, most patients with seborrheic dermatitis have no underlying disease - Pityrosporum ovale infection may be underlying the development of the disease; as such, topical antifungals are used as part of treatment.

9 yo female presents with 4 wk ho rt knee pain with activity. No hx of trauma or rcent illness. Your ecam reveals lateral patellar tracking with extension of knee. Most likely diagnosis?

*Patellofemoral pain syndrome* - isolated tenderness with palpation at medial and lateral aspect of knee - positive grind test

A 38-year-old female presents to the emergency department with difficulty eating. She reports that over the past week she has had increasing difficulty chewing her food and progressive pain in the left side of her jaw. This morning, she was unable to close her mouth for several minutes after taking a bite of her breakfast. Two months ago, the patient had a root canal of one of her left molars. Her past medical history is significant for hyperlipidemia, mild intermittent asthma, and type II diabetes mellitus. She has never smoked and has 10-15 alcoholic drinks per week. She works as an aide at a nursing home. Her temperature is 100.8°F (38.2°C), blood pressure is 133/74 mmHg, pulse is 105/min, and respirations are 14/min. On physical exam, there is a bluish hue to the skin on the lower left side of the patient's face. A 3x4 cm non-tender mass can be palpated inferior to the angle of the left mandible. Thick exudate is draining from an opening in the skin. Gram stain of the exudate can be seen (flip for image). Which of the following is the treatment of choice for this patient?

*Penicillin* Fever, trismus, and cutaneous drainage of sulfur granules in the setting of a recent dental procedure, which points to a diagnosis of *cervicofacial actinomycosis.* - The treatment of choice is penicillin. Actinomyces - gram-positive, filamentous rod that causes a cervicofacial infection after direct inoculation during a dental procedure or trauma - typically presents in an indolent fashion over a time course of weeks to months in which a hard, indurated mass evolves into multiple abscesses that drain through sinus tracts through the skin - exudate is classically thick with small yellow "sulfur" granules, although it may also be serosanguinous - since the infection spreads by direct extension, patients may complain of pain when the infection infiltrates or compresses nearby structures. Trismus may occur for the same reasons. (Figure demonstrates the histologic appearance of the "sulfur granules" found in the draining exudate in actinomycosis.*

A 84-year-old female nursing home resident is admitted to the MICU following a severe episode of urosepsis. Several days into a course of appropriate antibiotic therapy, her white blood cell count begins to rise. As part of your workup, you perform a right upper quadrant ultrasound which shows a distended gall bladder with a thickened wall and pericholecystic fluid. What is the most appropriate next step in the management of this patient?

*Percutaneous cholecystostomy* Acalculous cholecystitis (ACC) - After the diagnosis is made with a right upper quadrant ultrasound, emergent percutaneous cholecystostomy should be performed. - most commonly in severely and chronically ill patients, resulting from an ascending infection of the gall bladder in absence of a gall stone.

46 yo man undergoes elective inguinal hernia repair. After induction of general anesthesia, he is noted to be pale & tachycardic. Pt's med hx significant for frequent headaches, HT, and anxiety disorder. His outpt meds include lisinopril, alprazolam, and as-needed naproxen. Pt regularly drinks 6-pck of beer on weekends but doesn't use tobacco or illicit drugs. Fam hx unremarkable. His temp is 98.4 F, BP is now 250/140 mmHg (it was 144/90 mmHg before induction), and pulse is 125/min. ECG shows sinus tachycardia with no ischemic changes. Most likely diagnosis?

*Pheochromocytoma* - HT + frequent headaches - HT + pallor + sinus tachycardia - *concern for occult pheochromocytoma with catecholamine surge due to anesthesia* - In MEN IIa & IIb

Most common cause of abnormal hemostasis in pts with chronic renal failure. PT, PTT, platelet count normal. BT prolonged.

*Platelet dysfunction* - DDAVP = treatment of choice --> increases release of factor VIII:von Willebrand factor multimers from endothelial storage sites - platelet transfusion NOT indicated bc transfused platelets quickly become inactive

45 yo Asian-American female comes to doc due to bloating, flatulence, abdominal cramps and explosive watery diarrhea. These symptoms occur after ingesting dairy products. She has not had any wt loss. She has not had bone pain or easy bruising. PE shows abdominal distention and borborygmi. You decide to investigate pt further. What test results is most likely to be observed?

*Positive hydrogen breath test* Lactose intolerance - most commonly in Asian-Americans - osmotic diarrhea - positive hydrogen breath test = rise in measured breath hydrogen level after ingestion of lactose = bacterial carb metabolism - *positive hydrogen breath test, positive stool test for reducing substances, low stool pH and increased stool osmotic gap* - no steatorrhea

A 32-year-old woman with Graves' disease is undergoing treatment with radioactive iodine. Her initial presentation consisted of symptoms of sweating, weight-loss, and intermittent palpitations along with a physical examination significant for mild-to-moderate exophthalmos. After completing one week of radioactive iodine therapy, she reports worsening of her proptosis, with increased pain and worsened periorbital edema. Which of the following could have prevented the worsening of this patient's exophthalmos?

*Pre-treatment with prednisone prior to initiating radioiodine therapy* Graves' ophthalmopathy was worsened by the initiation of radioactive iodine. - *Pretreatment with glucocorticoids, such as prednisone, may prevent this adverse effect.* - due to the release of excess thyroid hormone during the destruction of thyroid cells by the radioactive iodine - Administration of radioactive iodine may also precipitate a thyroid storm through an identical mechanism - *Preventive administration of steroids for several months (2-3) followed by a brief taper prior to initiating radioiodine therapy is recommended for patients with mild, moderate, or progressive ophthalmopathy.*

54 yo man evaluated for erectile dysfunction. Pt has been having probz attaining and maintaining erections during sexual activity for past 6 mos. Prior to dev't of this prob, he had active & satisfactory sex life. Pt continues to have normal early-morning penile erections. Med xh notable for HT, mixed hyperlipidemia, GERD, and lumbar stenosis with chronic right L4 radiculopathy. His current meds include ibuprofen, ranitidine, lisinopril, and atorvastatin. Pt smokes pck of ciz daily & drinks alcohol only on social occasions. BP is 144/78 mmHg and pulse is 80/min. BMI 35 kg/m^2. PE shows normal body hair distribution and testicular size with no gynecomastia. Most likely cause of this pt's erectile dysfunction?

*Psychosocial factors* - sudden-onset - situational - persistence of nonsexual nocturnal erections - normal nocturnal erections indicate intact vascular and nerve function in penis = usually absent in pts with organic causes

57 yo man presents to ED complaining of dyspnea, cough, pleuritic chest pain. Chest radiograph shows large left-sided pleural effusion. Thoracentesis shows pleural fluid protein to serum protein of 0.7 and pleural fluid LDH to serum LDH ratio of 0.8. Cause of pleural effusion?

*Pulmonary embolism* Exudative - pleural fluid protein to serum protein ratio >0.5 - pleural fluid LDH to serum LDH ratio >0.6 Exudative - malignancy, TB, bacterial infection, empyema, viral infections, PE with infarct, RA, SLE, pancreatitis, post-MI, PE, trauma

36 yo woman evaluated in clinic due to left hand clumsiness. Pt works as office secretary and has been having difficulty typing. Six mos ago, she had episode of vertigo and difficulty walking that improved after several weeks. Pt also has ho bipolar disorder and takes valproic acid. On PE, she is asked to close her eyes and stretch out her arms with the palms facing up. Several seconds later, her left arm involuntarily drifts downward and palms turn toward the floor. What best explains the findings observed in this pt?

*Pyramidal tract lesion* young woman + neuro deficits disseminated in space & time (hand clumsiness, ataxia, vertigo) = *MS* Pronator drift - specific & sensitive for *UMN or pyramidal /corticospinal tract* disease - affected arm drifts downward and palm turns/pronates toward floor

A 45-year-old female presents to general medical clinic with a chief complaint of a growth on her neck. Physical exam reveals proptosis, pretibial myxedema, and a diffuse goiter. If the patient's symptoms are due to thyroid-stimulating TSH receptor antibodies, what treatment would provide the best long term outcome for her symptoms?

*Radioactive iodine* The therapy for Graves' disease with the best long-term outcome is radioactive iodine therapy. - Contraindications are pregnancy and severe ophthalmopathy. Graves' disease/diffuse toxic goiter - most common cause of hyperthyroidism, responsible for approximately 80% of all cases - autoimmune disorder caused by the production of thyroid-stimulating antibody that bind to the TSH receptors on thyroid cells causing hormone synthesis

A 95-year-old male visits your urology clinic five days after right hip replacement complicated by postoperative urinary retention. The patient has a history of hypertension, type II diabetes mellitus, stroke, dyslipidemia, a past myocardial infarction and severe osteoarthritis in his right hip. He is not compliant with his medications and his multiple comorbidities that are poorly managed. His older brother, age 99, was diagnosed with adenocarcinoma of the prostate one year ago. In the hospital, the patient's urinary retention was treated with foley catheterization, and upon discharge he had no further urinary symptoms. At clinic, the patient's serum-specific prostate specific antigen (PSA) is 6.0 ng/mL (normal is < 4 ng/mL). Digital rectal examination (DRE) demonstrates a nontender prostate several rock hard nodules. Which is the appropriate next step in management?

*Reassurance* A transient elevation in PSA will often decrease after a period of 2-6 weeks once the underlying cause has resolved. This elderly patient should not be screened for prostate cancer given his age and multiple comorbidities. PSA - serine protease that liquefies semen after ejaculation - has a diagnostic role in prostate cancer and is also used to measure response to cancer treatment - also commonly elevated in benign prostatic hyperplasia, which is common among elderly males. Transient PSA elevations - can occur due to urinary retention, prostatic inflammation, as well as cystoscopy PSA screening and DRE are no longer recommended as routine screening for prostate cancer. In particular in this elderly patient with multiple comorbidities it is of little value to the patient to pursue a workup for prostate cancer. - Exposing him to multiple tests and potential biopsies and ultimately a procedure that may have no impact to his quality of life will only bring him harm. The key to this question is knowing the guidelines for pursuing and diagnosing prostate cancer. - It is important to know that DRE and PSA levels are no longer recommended as routine screening for prostate cancer. - In particular it is not useful to pursue elevations in elderly patients with other multiple comorbid conditions because even if they do have prostate cancer, by the time it manifests symptomatically it is likely that one of their other comorbid conditions would have caused their mortality. - In this patient in particular there is a reason for the PSA to be elevated (urinary retention, catheterization) even further supporting the idea that this patient needs only reassurance. - *In a younger patient with an elevated PSA and a hard nodular prostate the best answer could be ultrasound and trans rectal biopsy however in this case this is unnecessary.* PSA should never have been performed in the first place. - What did DRE reveal? What symptoms does this man have? - In general, DRE and PSA should not be performed except in vague cases of "shared decision making." - The threshold of 4 ng/mL is used as a cutoff, but it has a 70% false positive rate. At this point you can discuss this with the patient and see if they want to undergo a biopsy, but you should explain to them what PSA is, how it could lead to more harm than if nothing was done, and how it can be elevated in many benign processes that would not be treated anyways.

73 yo man comes to office due to 3 mos of progressive urinary urgency, hesitancy, nocturia, and weak urinary stream. He has no fever, abdominal pain, hematuria, malaise, or wt loss. His only med is lisinopril for essential HT. Pt has no ho DM or ischemic heart dz. He doesn't use tobacco, alcohol, or illicit drugs. Digital rectal exam reveals smooth, firm and enlarged prostate without induration or asymmetry. Neuro exam normal. Urinalysis shows no proteinuria or hematuria. Neuro exam normal. Urinalysis shows no proteinuria or hematuria. Pt's serum creatinine is 2.1 mg/dL, which is higher that his baseline creatinine of 2.1 mg/dL 4 mos ago. PSA normal. Most appropriate next step in eval of this pt's AKI?

*Renal ultrasound* lower urinary tract symptoms + smooth, enlarged prostate on exam = BPH --> initial eval of pts with LUTS = urinalysis & PSA --> assess risk of prostate cancer pt has evidence of AKI via interval rise in serum creatinine - can be elevated in bilateral obstruction, such as in pts with severe bladder outlet obstruction in BPH - *pts with acutely elevated creatinine require imaging (preferably renal ultrasound) to assess for hydronephrosis & exclude other cause of obstruction* - placement of urinary cath in pts with hydronephrosis can provide quick relief of obstruction

A 62-year-old female presents to general medical clinic for health maintenance. She is due for a colonoscopy but before she schedules it, she would like to have a full exam. She has no complaints and no significant past medical history. She has been in good health for most of her life. Vital signs are stable. Her physical examination is benign. Routine labs reveal a calcium of 11.2 mg/dL. What is the next step in management?

*Reorder serum calcium* Asymptomatic hypercalcemia - *first test should be to confirm hypercalcemia with a second serum calcium* - If this test returns positive, then a PTH level is the next step in management - If the hypercalcemia were severe or there were symptoms in this patient the management would be: 1. IV fluids, 2. Loop diuretics (furosemide) 3. Calcitonin then 4. Bisphosphonates (long term management). Often times only IV fluids and a loop diuretic are needed.

A hospitalized 70-year-old male with a history of alcoholic cirrhosis is evaluated for an episode of hematemesis. Two days ago, he presented to the hospital for black, tarry stools and hypotension. The patient was resuscitated and transfused with two units of blood. Esophagogastroduodenoscopy (EGD) demonstrated esophageal varices with no evidence of recent bleeding, and colonoscopy revealed old blood with no active bleeding source. While under observation in the hospital, he vomited copious blood with a coffee grounds appearance. On exam, temperature is 37.1 degrees Celsius, blood pressure is 100/65 mmHg, and pulse is 105/min. Hemoglobin is 10.1 g/dL and INR is 1.3. Following intravenous fluid resuscitation and nasogastric tube placement, which of the following is the next appropriate step in management of this patient?

*Repeat esophagogastroduodenoscopy (EGD)* Acute variceal bleeding secondary to portal hypertension in alcoholic cirrhosis - Although the patient's previous EGD did not demonstrate evidence of recent bleeding, repeat EGD is appropriate following adequate fluid resuscitation and nasogastric tube placement. Repeat EGD - excellent diagnostic and therapeutic potential - likely to help localize the source of the upper GI bleed, and, if the bleeding is from esophageal varices, EGD can provide sclerotherapy or banding as treatment options. If EGD fails to control bleeding, more aggressive options may be used, including surgical shunts or TIPS.

Patient's clinical presentation is consistent with schizophrenia. What is helpful for patients who have difficulty with medication adherence?

*Risperidone* - available in a long-acting, injectable form, which is helpful for patients who have difficulty with medication adherence Antipsychotics - cornerstone of treatment for schizophrenia - frank psychosis often leads to medication non-adherence secondary to issues of disorganization, paranoia, or lack of insight into the illness *Three antipsychotics (risperidone, haloperidol, and fluphenazine) are available in a long-acting, injectable form, that can be given once every two weeks to one month. These medications may be appropriate for patients with proven issues with adherence.*

Secondary spontaneous pneumothorax should be suspected in pts with underlying lung dz presenting with rapid worsening of respiratory symptoms. What is the most common cause in pts with COPD.

*Rupture of alveolar blebs* COPD + acute-onset SOB + hypoxia + unilaterally decreased breath sounds = secondary spontaneous pneumothorax - pts with COPD, CF - chest pain, hyperresonance on percussion - cigarette smoking = increased risk of pneumothorax - chronic destruction of alveolar sacs --> formation of large alveolar blebs --> can eventually rupture and leak into pleural space - diagnosis via chest x-ray = visceral pleural line beyond which no pulmonary markings are apparent

Patient's presentation is consistent with tinea versicolor, a superficial fungal infection. What are the most appropriate treatment?

*Selenium sulfide* or topical antifungals (2% ketoconazole cream)

Saline-responsive metabolic alkalosis + volume depletion + low serum Cl- + decreased urine chloride, hypokalemia.

*Self-induced vomiting* Saline-responsive metabolic alkalosis - low urinary chloride excretion - volume contraction - corrects with saline infusion alone

Defined as presence of a middle ear effusion without signs of an active infection. Exam commonly reveals a dull tympanic membrane that is hypomobile on pneumatic otoscopy.

*Serous otitis media* - most common middle ear pathology in pts with acquired immunodeficiency syndrome - via auditory tube dysfunction arising from *HIV* LAD or obstructing lymphomas - *conductive hearing loss* = most common symptom experienced

50 yo man comes to doc complaining of feeling fatigued and weak for a year. He becomes easily tired in day and also finds it difficult to remember things. He is currently unemployed and lives on Social Security benefits. Pt says that he drinks 2-3 beers a day but has been charged multiple times with driving under influence. His parents died of old age. He lives with 4 friends. His vitals are within normal limits Lab results: WBCs 5,500/uL Hgb 7 g/dL Hct 22% Platelets 196,000/uL RBCs 1.7 million MCV 114 fL Mean corpuscular hemoglobin 36 pg/cell Reticulocytes 0.4% Peripheral smear shows anisocytosis, poikilocytosis, and basophilic stippling. Best next step in management of this pt

*Serum B12 and folate levels* Macrocytic anemia: Megaloblastic anemia - MCV>110 fL + low retic count + macroovalocyte RBCs + hypersegmented neutrophils (mean lobe count >5) + anisocytosis + poikilocytosis - *alcoholism (this pt) can also cause basophilic stippling*

45 yo woman comes to office due to progressive fatigue, myalgias, and muscle weakness in both LEs for past month. She has slight difficulty getting up from chair, and has weakness and cramping in legs after walking short distance. Pt's other med probz include HT, which is treated with lisinopril. She doesn't use tobacco, alcohol, or illicit drugs. BP is 140/80 mmHg, pulse is 64/min, & respirations are 14/min. Exam shows mildly decreased strength in proximal muscles of LEs with sluggish ankle jerks bilaterally. Lab tests show normal ESR and elevated serum creatinine kinase level. Serum sodium, glucose, potassium, and creatinine are normal. Most appropriate next step in management of this pt?

*Serum TSH & free T4 tests* myalgias & proximal muscle weakness + elevated serum CK + relatively young healthy woman with fatigue and delayed deep tendon reflexes = *hypothyroid myopathy*

Copper T 380A intrauterine device (ParaGard) would be preferred over levonorgestrel-releasing device (Mirena) in pt with ho the following?

*Severe cirrhosis* - cirrhosis or liver cancer = NO leven.-releasing IUD --> avoid hormonal contraceptive use (breast cancer another contraindication)

43 yo woman comes to urgent care center due to 3 day ho back pain. She works in a home improvement store and was lifting bag of potting soil when she felt sudden "grabbing" pain in her lower back. No radiation of pain, no LE weakness, and no associated urinary symptoms. Pt's med hx is notable for PCOS, for which she takes oral contraceptives. She doesn't use tobacco, alcohol, or illicit drugs. Vitals normal. PE shows tenderness in right lumbar paraspinal muscles and slight reduction in lumbar lordosis. ROM in lumbar spine normal. Gait, LE strength, and deep tendon reflexes are also normal. With pt supine on exam table, her legs are lifted sequentially to 70 degrees with knee held str8 and ankle dorsiflexed; this causes feeling of tightness in hamstring muscles but no radicular radiation of pain. Most appropriate next step in management of this pt's symptoms?

*Short course of naproxen* Acute low back pain + lumbar strain - <4-6 wks - uncomplicated - advise to continue moderate activity - *nonopioid analgesics = NSAIDs, acetaminophen* = initial management

70 yo woman brought to ED by her daughter due to 2 days of fever and cough and a day of excessive somnolence. Pt has ho HT & OA. She doesn't use tobacco or alcohol. She lives independently in senior residential facility and ambulates with assistance of a cane. Temp is 102.6 F, BP is 110/76 mmHg, pulse is 110/min, & respirations are 24/min. No pharyngeal erythema or cervical lymphadenopathy is present. Lung auscultation reveals crackles in right lower lung field. Lab results: Leukocytes 13,800/mm^3 BUN 22 mg/dL Creatinine 1.2 mg/dL Chest x-ray reveals right lower lobe infiltrate. What pathogens is most likely cause of this pt's condition?

*Streptococcus pneumoniae* Elderly pt + fever + cough + tachypnea + leukocytosis + chest-ray: lobar infiltrate = *pneumoniae* *Community-acquired* - nonhospitalized setting = *assisted-living facility / nonhospital setting* - most common cause of CAP = s pneumoniae - most pts remain symptomatic --> clear inoculation within 4-6 wks - those with aggressive serotypes or certain underlying risk factors (age >65 yo, immunosuppression) = more likely to develop invasive dz (pneumonia, meningitis, endocarditis)

65 yo man brought to ED after having seizure. Past med hx significant for non-small cell lung cancer diagnosed 2 yrs ago and treated surgically. He continues to work as college professor and usually walks 1-2 miles daily. MRI of brain shows solitary cortical mass (2 cm - 3.5 cm) in right hemisphere at grey-white matter junction that is most consistent with metastasis. CT scan of chest and abdomen shows no evidence of recurrent malignancy. Pt is admitted for further management and started on phenytoin. Most appropriate next step in management of this pt?

*Surgical resection of mass* Metastatic brain tumors - most common type of intracranial brain tumors - may be solitary or multiple - usually seen at grey-white matter junction - frequently seen in pts with non-small cell lung cancer (NSCLC) Favorable outcome after diagnosis - age <65 yo - good functional performance status - stable extracranial dz Single brain met - surgical resection --> followed by sterotactic radiosurgery (SRS) or whole brain radiation therapy (WBRT) to tumor bed

Group of investigators conducted randomized placebo-controlled clinical trial to assess effect of a new aldosterone receptor antagonist on progression of chronic heart failure. Primary outcome was all-cause mortality. Decrease in all-cause mortality in treatment group was reported, with relative risk of 0.71 (p = 0.001). What statement is the best interpretation of the reported association?

*There is a 0.1% probability of observing the given relative risk (or more extreme) by chance alone assuming no differences in mortality* RR = point estimate of association but doesn't account for random error p-value used to strengthen results of study - probability of observing given (or more extreme) result by chance alone, assuming null hypothesis true

Benefit of using probiotics in children?

*They prevent abx-associated diarrhea* - saccharomyces boulardii - NNT = 10 - breast-fed infants = decreased crying time - prevention of eczema and URI

29 yo woman comes to office due to frequent episodes of headache. She has moderate to severe, left-sided, throbbing pain associated with nausea & occasional vomiting. Her headache is often preceded by tingling sensation in right hand that gradually involves right arm & face and is followed by numbness lasting <1 hr. She feels irritable while experiencing headaches. Pain typically improves following several hours of rest in dark and quiet room. Pt has had these episodes every 1-2 mos for past 5 yrs. She has taken as-needed sumatriptan with some relief, but symptoms have caused her to miss several days of work. She has no other med hx and doesn't use tobacco, alcohol, or recreational drugs. There is fam hx of depression. Vitals are within normal limits and PE is normal. Most likely to be helpful in preventing this pt's headaches?

*Topiramate* Migraine prophylaxis - topiramate - beta blockers/propranolol - amitriptyline

A 40-year-old African American female presents to the emergency room complaining of right-sided, colicky flank pain. Past medical history includes systemic lupus erythematosus, and she has been hospitalized twice for nephrolithiasis in the past year. Serum laboratory values are notable for the following: Sodium: 138 mEq/L Potassium: 2.9 mEq/L Bicarbonate: 15 mEq/L Chloride: 112 mEq/L You suspect the patient's current episode is related to a chronic condition. Which of the following is the most likely diagnosis?

*Type 1 RTA* - *non-anion gap hyperchloremic metabolic acidosis and recurrent nephrolithiasis consistent with type 1 RTA* - associated with lupus and other autoimmune disorders such as Sjogren's syndrom - inability to secrete H+ at the distal tubule leads to an inability to acidify the urine and the increased excretion of the positive ions calcium and phosphate - increased excretion of calcium and phosphate into alkaline urine in turns leads to nephrolithiasis - hypokalemia is also present.

Bimodal distribution with second peak at age 50-80. Abdominal pain + bloody diarrhea + tenesmus + fecal incontinence. Endoscopy demonstrates continuous inv't with erythematous and friable mucosa and ulcers.

*UC* - most common extraintestinal manifestations = arthritis, uveitis, episcleritis, erythema nodosum, primary sclerosing cholangitis

What vitamin supplement is recommended by USPSTF to help prevent falls in elderly pt living at home?

*Vitamin D* - mechanism not clearly understood - ages 65 & older

Most beneficial lifestyle intervention for obese patients

*Weight loss* - While the majority of hypertensive patients will require pharmacotherapy to meet their blood pressure goals, lifestyle modification is the first step in treatment - *Weight loss is the most effective lifestyle change - every 10 kg reduction in weight can decrease systolic blood pressure by 5-20 mmHg* - For all patients, the goal BMI is 18.5-24.9 kg/m^2 - Regular aerobic exercise and dietary sodium restriction also help lower blood pressure, but less so than weight loss. - There is strong evidence to support treating hypertensive persons aged 60 years or older to a BP goal of less than 150/90 mmHg and hypertensive persons 30-59 years to a goal of less than 140/90 mmHg

A 31-year-old female with a past medical history of follicular thyroid cancer treated two years prior with a total thyroidectomy followed by radioactive iodine ablation presents to her gynecologist with questions relating to pregnancy. She and her partner are thinking about getting pregnant, but she is concerned about her thyroid replacement hormone, specifically the complications of being hypothyroid during pregnancy. She wants to know how her levothyroxine dosing should be handled prior to conception and during her pregnancy. Which of the following would be an appropriate response to this patient's concerns?

*Your current dose will only need to be increased during pregnancy, not before.* In hypothyroidism during pregnancy, the levothyroxine dose will only need to be increased during pregnancy, not before. - The levothyroxine dose needs to be increased during pregnancy to maintain a euthyroid state as a result of the increase of thyroglobulin binding hormone in pregnancy which decreases the amount of circulating free T3/T4. Thyroid disease, if untreated during pregnancy, increases the risk of miscarriage, placental abruption, hypertensive disorders, and growth restriction.

Risk factors for trace mineral deficiency include malabsorption, *bowel resection*, poor nutritional intake, and dependence on parenteral nutrition. Clinical manifestations of what mineral includes hypogonadism, impaired taste, impaired wound healing, alopecia, and skin rash with perioral inv't?

*Zinc* - mostly absorbed in duodenum & jejunum

Patient in requires analgesic therapy but is on lithium. Any drug that decreases his glomerular filtration rate could potentially cause lithium toxicity (tremor, ataxia, dysarthria, nystagmus, renal impairment, confusion, and convulsions) and should be avoided if possible. What is the only safe choice.

*acetaminophen*

Treatment for Henoch-Schonlein purpura (HSP)

*acetaminophen* Management approach: as the disease resolves spontaneously over time, medical therapy is typically unnecessary, unless there is persistent nephropathy Conservative - supportive care - pain management Medical: angiotensin-converting enzyme (ACE) inhibitor - for patients with persistent proteinuria corticosteroids - for patients with persistent proteinuria despite ACE-inhibitors - not recommended for prevention of renal disease

postop delirium. what's important?

*adequate pain control*, reorientation

Recognize inverse psoriasis, which is treated with...

*corticosteroid cream* flexural/inverse psoriasis - can be found in other places too

Treatment for trochanteric bursitis.

*corticosteroid injection* Treatment: Conservative - physical therapy - rest and ice Medical - nonsteroidal anti-inflammatory drugs (NSAIDs) - injected corticosteroids = management of pain that is refractory to NSAIDs

Based on clinical exam findings and direct immunofluorescence (IF), this patient is suffering from pemphigus vulgaris, which is a disease that results from autoimmune antibodies against

*desmoglein 3* - component of desmosomes

Epidural hematoma is most common in children/adolescents and occurs as result of traumatic head injury, which leads to tearing of middle meningeal artery and bleeding between dura mater and skull. CT scan of head typically reveals hyperdense biconvex lesion that does not cross suture lines. Symptomatic pts require....

*emergent neurosurgical hematoma evacuation*

A 17-year-old female presents to her pediatrician due to lack of menstruation. She states that she developed breasts 4 years ago but has not experienced menses yet. The patient denies abdominal pain and has no past medical history. Her mother underwent menarche at age 13. The patient is a volleyball player at school, is single, and has never attempted intercourse. At this visit, her temperature is 98.3°F (36.8°C), blood pressure is 110/76 mmHg, pulse is 72/min, and respirations are 14/min. She is 5 feet 7 inches tall and weighs 116 pounds (BMI 18.2 kg/m^2). Exam shows Tanner IV breasts, Tanner I pubic hair, and minimal axillary hair. External genitalia are normal, but the vagina is a 5-centimeter blind pouch. Genetic testing is performed. Which of the following is the best next step in management?

*gonadectomy* Primary amenorrhea in the setting of normal breast development and minimal pubic hair, as well as a blind pouch vagina, most likely due to *complete androgen insensitivity syndrome (CAIS)* - *She likely has intraabdominal testes, and gonadectomy after puberty is indicated to reduce the risk of testicular malignancy* CAIS - mutation in the androgen receptor gene on the X chromosome in a 46,XY patient, making the body unable to respond to circulating androgens - in adolescence, primary amenorrhea results from the lack of Mullerian structures (uterus, upper third of vagina) - because her androgens are aromatized peripherally into estrogen, she appears phenotypically female with normal breast development but has a blind pouch vagina and lack of adrenarche (due to the lack of androgen response) - she is also tall and lanky phenotypically, which is classic for CAIS. - Her diagnosis can be made clinically, but laboratory testing would show serum testosterone levels within or above the normal male adult range. - Since the risk of testicular cancer before puberty is so low, it is considered safe to wait until after puberty is completed to perform gonadectomy. *Illustration shows a sagittal MRI of a patient with CAIS. Note the blind pouch vagina (circled in red) between the bladder and rectum.*

An 11-year-old girl presents to the ER complaining of a "terrible" headache. She states that it began 8 hours ago and describes it as a throbbing pain in the front of her head. She also has been nauseous all day and vomited once (nonbloody, nonbilious). Light hurts her eyes. Her mother gave her acetaminophen when the pain first started, but it didn't help. What is the most appropriate next step in treatment?

*ibuprofen* A girl with a multi-hour long throbbing frontal headache associated with nausea, vomiting, and photophobia most likely has a *migraine*. - *Acetaminophen and NSAIDS are the first line treatment for migraines in children and adolescents. Since acetaminophen has already been tried, ibuprofen should be used next.* American Academy of Neurology - recommends both acetaminophen and NSAIDS for children aged 6 and older - for adolescents aged 12 and older, nasal sumatriptan seems to be effective, despite a lack of FDA approval.

Polycythemia vera is a myeloproliferative disorder marked by erythrocytosis. It is a form of primary polycythemia, almost always caused by mutation of JAK2 in myeloid precursor cell. Erythropoietin levels tend to be...

*low* - unlike most causes of secondary polycythemia

Pts with severe liver cirrhosis have increase nitric oxide generation in the splanchnic circulation secondary to portal HT. This is thought to cause systemic vasodilation, which reduces peripheral vascular resistance and BP, causing...

*renal hypoperfusion* - hepatorenal syndrome in pts with severe liver cirrhosis secondary to systemic and renal hypoperfusion - acute renal failure = creatinine > 1.5 mg/dL with very low urine sodium level, typically <10 mEq/L, and absence of blood, casts, or protein in urine - renal function doesn't improve with IV fluid resuscitation

The Transurethral resection of the prostate (TURP) procedure, as treatment for BPH, commonly causes what side effect?

*retrograde ejaculation* TURP - can lead to damage of the preprostatic (internal) sphincter system. - The innervation is a combination of somatic and autonomic innervation. - Of greatest importance is the pudendal nerve (S2,3,4) which can become damaged. Whether the damage is to the nerve or to the muscle itself is another question.

Severe sepsis implies

*sepsis accompanied by signs of end-organ damage, such as acidosis, low urine output, hypoxia, or altered mental status* Severe sepsis - can cause hypotension resulting in tissue hypoxia, which leads to anaerobic respiration and the build-up of lactic acid in the blood - As a result, the arterial blood gas shows an anion-gap metabolic acidosis (decreased pH with a primary decrease in bicarbonate)

This patient is intoxicated with MDMA 3,4-(methylenedioxy-N-methylamphetamine), resulting in dangerous behavior. Both MDMA intoxication and withdrawal are treated with

*supportive care* MDMA/ecstasy - is the active ingredient in ecstasy. - Intoxication produces symptoms similar to amphetamine intoxication, although with noted strong feelings of empathy. - Neither intoxication nor withdrawal from MDMA is life-threatening, and supportive care is the treatment of choice in both states. - Because MDMA can cause hallucinations, impaired judgement, and impulsivity, intoxicated patients should be watched closely in order to mitigate the risk of self-harm. "club drugs," including MDMA, ketamine, rohypnol, and gamma-hydroxybutyrate. - Treatment for intoxication with all four drugs is supportive care, including management of hypertension, hyperthermia, and rhabdomyolysis.

The farther the value of RR from 1 ....

*the stronger the association* Ex. RR for bronchogenic cancer in smokers is >2 --> strong association between smoking (risk factor/exposure) and bronchogenic cancer (outcome) RR - used as measure of association in cohort studies - ratio of risk in exposed group to that in unexposed group - RR>1 = positive association between risk factor and outcome - RR<1 = negative association between risk factor and outcome

Elderly patient with isolated systolic hypertension. Treatment options include

*thiazide diuretic*, ACE inhibitor, or long-acting calcium channel blocker (CCB) Isolated systolic hypertension may be made based on 2-3 separate measurements of elevated systolic blood pressure (> 140) and normal diastolic blood pressure (< 90) - has a distinct pathophysiology that results from decreased elasticity or hardening in the arterial walls - strongly associated with aging. - *Pharmacologic treatment should be initiated to lower the blood pressure to a goal BP < 150/90 mmHg*

At less than 20 weeks gestation, minimal vaginal bleeding and a closed cervix in the setting of a normal fetal ultrasound is consistent with a

*threatened abortion*

Used for scaphoid fractures as well as extra-articular fractures of the thumb metacarpal. This splint also starts at the radial aspect of the forearm (not over the elbow), and wrapps the thumb up to the distal interphalangeal joint.

*thumb spica splint*

A 26-year-old female visits your office concerned about a mass in her left breast. The patient explains that her mother was diagnosed with breast cancer at the age of 60 and her aunt was diagnosed with breast cancer at age 65. The patient's last menstrual period was one week ago. Physical examination reveals a 1 cm, painless, firm, solitary mass in the lateral upper quadrant of her left breast. Which of the following is an appropriate next step in the patient's management

*ultrasound* Fibroadenoma - benign, firm tumor of fibrous stromal tissue common in young women - *Ultrasound is preferred to mammography as an initial screening test in young women (<30 years) because the increased density of breast tissue in this population* - As the incidence of carcinoma is low in young women, negative ultrasound findings have a high negative predictive value

Patient most likely has acute post-streptococcal glomerulonephritis (PSGN). The next step in the management of any patient with suspected renal pathology should be

*urinalysis* - cheap, fast, and informative test - detects urine leukocytes, nitrites, urobiliniogen, protein, pH, blood, specific gravity, ketones, bilirubin, and glucose In this case of PSGN, we would expect to see increased RBCs, WBCs +/- elevated protein.

The triad of symmetric, bilateral thigh and buttock claudication, impotence, and symmetric atrophy of the lower extremities is likely due to

*vasculature*: chronic ischemia from aortoiliac occlusion / Leriche syndrome - vascular cause of lower extremity pain - 15-20% of adults older than 70 years have PAD - An ABI of less than 0.9 is associated with a 2-4 fold increase in relative risk for cardiovascular events and all-cause mortality

Recognize *psoriatic arthritis*

- morning stiffness - dactylitis (sausage digit / swollen finger) - all fingers swollen - many of the nails show pitting and onycholysis / separation of nail bed - well-demarcated red plaques with silvery scaling on dorsum of each hand - current treatment = NSAIDs, methotrexate, anti-tumor necrosis factor agents

Meds that have been shown to improve long-term survival in pts with left ventricular systolic dysfunction

ACEIs ARBs BBs MRAs - *eplerenone*, spironolactone AA pts = hydralazine and nitrates

Fatigue, prolonged bleeding, easy bruising. Pancytopenia, coagulopathy, elevated lactate dehydrogenase. All of this strongly suggests...

Acute promyelocytic leukemia (APML) = *atypical promyelocytes in bone marrow* = form of AML - life-threatening coagulopathy via DIC (prolonged PT/active PTT, *hypofibrinogenemia*)

Pts with Neisseria gonorrhoeae are at high risk of simultaneous coinfections with several other sexually transmitted pathogens, including...

Chlamydia trachomatis HIV Treponema pallidum (*syphilis*) Hepatitis B - pts should be screened for these infections in addition to receiving appropriate abx

Results in diastolic heart dysfunction. Pts present with signs of decreased CO and venous overload. Common etiologies in US = viruses, cardiac sx, chest radiation, idiopathic causes. *TB* = most common cause in developing countries and endemic areas such as Africa, India, and *China*

Constrictive pericarditis - sharp x and y descents characteristically seen on central venous tracing

Transient ischemic attack (TIA) secondary to carotid stenosis Diagnostic test: Tx:

Diagnostic test: -Ultrasound of carotid Tx: -If >70% stenosis: carotidendarterectomy -If <70% stenosis: daily aspirin

Recognize *acute hemorrhagic stroke*

Initial work-up of a patient with concern for stroke is a non-contrast head CT in order to rule out a hemorrhage before consideration of tPA, as distinction between ischemic and hemorrhagic strokes can be difficult based on symptoms alone. If hemorrhage is identified, tPA is contraindicated. If an ischemic stroke is suspected clinically and CT is negative for evidence of a hemorrhagic stroke, the recommended treatment is to give IV tPA if the presentation is within 3-4.5 hours. *Figure A shows a non-contrast CT of a hemorrhagic stroke. Note the hyper-dense blood. An infarct would be hypo-dense.

Acute chest syndrome + pneumonia + SC dz --> what abx?

Macrolides/*Azithromycin* for Mycoplasma and Chlamydophilia + ceftriaxone

Characterized by fever, pharyngeal pain, and earache. Exam findings include trismus, muffled voice, and deviation of uvula.

Peritonsillar abscess - treatment = needle aspiration or incision and drainage + abx

Technically a sodium channel blocker. It primarily blocks late inward sodium current and thus can reduce both early and late afterdepolarizations. It has been shown in multiple clinical trials to be beneficial in refractory chronic stable angina.

Ranolazine

FVC and FEV1/FVC both less than lower limit of normal as defined by Third Nat'l Health and Nutrition Exam Survey. Type of lung dz?

Restrictive

Compulsive behavior (ex. consuming same meal daily), hyperorality (ex. increased cigarette consumption), apathy, lack of insight, executive dysfunction (ex. difficulty with tasks that require planning) are consistent with...

behavioral variant of *frontotemporal dementia* - disinhibition = another common symptom of FTD - age 60 = onset - AD - 50-10 year survival rate - accumulation of tau protein inclusions = Pick bodies in hippocampi, temporal lobes, frontal lobes - SSRIs or trazodone = some benefit in management of neuropsych symptoms

Alcohol withdrawal --> treat with

benzos (chlordiazepoxide)

Pneumocystis pneumonia is a common opportunistic infection in pts on immunosuppressant med after solid organ transplant. Diagnosis requires identification of P jirovecii organisms in respiratory secretions using microscopy with specialized stains. Samples are obtained using induced sputum, or, if this is unrevealing, what can be done?

bronchoscopy with *bronchoalveolar lavage* Post-transplant pt with PCP = acute resp failure (tachypnea, hypoxia), dry cough, fever - LDH elevated = chest x-ray = bilateral, diffuse interstitial infiltrates

Female with UTI + prior UTI hx. Calls office with symptoms. Tx...

can call in her prescription by phone = *prescribe nitrofurantoin (macrobid, macrodantin)* - or: TMP-SMX, fosfomycin

Delusions, tactile hallucinations (bugs crawling under skin), aggressive behavior, severe insomnia, poor dentition and skin sores are suggestive of...

chronic *methamphetamine* abuse - highly addictive - very potent CNS stimulant - "meth/crystal meth/ice/glass"- wt loss, psychotic symptoms, excoriations due to *skin pickings* - meth mouth = brown discoloration, tooth decay, cracked teeth via severe clenching - elevated BP & pulse, hyperthermia, sweating, pupillary dilation - visual & tactile hallucinations tend to be more common in substance-induced psychotic disorders - long-term management = CBT to prevent relapse & antipsychotic meds

Can affect hemoglobin A1c levels

chronic hemolytic anemia - shortens RBC survival

hereditary hemochromatosis should be considered in pts who have elevated liver enzymes, DM, and skin hyperpigmentation. It can progress to ...

cirrhosis = increased risk for *hepatocellular carcinoma* - restrictive or dilated cardiomyopathy - increased susceptibility to Listeria, Vibrio vulnificus, Yersinia enterocolitica

Prosthetic joint infections can be acquired by perioperative contamination or by extension from overlying wound infection. Infections due to virulent organisms (Staph aureus) present within 3 mos with acute pain, fever, and local signs of infection. Infections that have a delayed onset and present with chronic pain, implant loosening, gait impairment, or sinus tract formation include

coagulase-negative staphylococci / *staphylococcus epidermidis* 3-12 mos - coag-negative staph - propionibacterium - enterococci

septic olecranon bursitis via minor trauma to elbow. nontender boggy mass over olecranon. swelling & erythema, warmth pain. Treatment if NOT septic/aseptic?

compression dressing - ice, avoidance of activities

USPSTF screening for abnormal glucose levels in adults 40-70 yo and who are overwt or obese + impaired fasting glucose = screening:

confirm normal blood glucose = *repeat fasting plasma glucose in 1-2 wks*

Patient's clinical presentation is consistent with premature ejaculation, with can be treated with

couple therapies, squeeze-and-go techniques, and *selective serotonin uptake inhibitors (SSRIs),* such as sertraline Premature ejaculation - characterized by ejaculation just before or just following vaginal or anal penetration, usually as a result of underlying anxiety - some authorities define "premature" to be within 1 minute of the start of intercourse; others reduce the threshold to 15 seconds - current evidence shows that the average time from penetration to ejaculation is roughly 6.5 minutes in men aged 18-30 years - *SSRIs (and other antidepressants) are the mainstay of medication treatment, and a delay in ejaculation time can be expected roughly 1 week after the start of the medication*

Diagnosis of Sjogren syndrome requires....

evidence of dry mouth and eyes (positive Schiermer test result for decreased lacrimation) with either histological evidence of lymphocytic infiltration of salivary glands or serum autoantibodies against *SSA (Ro) and/or SSB (La)*

Symptoms of multiple sclerosis may worsen during...

exposure to high temperatures - ex. moving to Arizona in June = *Uhthoff phenomenon* - when MS suspected, *MRI* should be performed to support diagnosis

Preferred narcot for pts with end-stage renal dz

fentanyl - secreted by liver = safe use in pts with renal failure

Pts with typical gastroesophageal reflux symptoms + alarm symptoms (dysphagia, odynophagia, weight loss, anemia, GI bleeding, recurrent vomiting) or men >50 with chronic (>5 yrs) symptoms and cancer risk factors (tobacco use) require...

gastrointestinal endoscopy - all other pts can receive empiric trial of PPI therapy & further eval if refractory to therapy

The clinical presentation of legionella can be very similar to other organisms, but may also have

gastrointestinal symptoms, hyponatremia, and *LFT abnormalities* Legionella - along with mycoplasma and chlamydia, legionella forms the third most common pathogen implicated in atypical pneumonias - common in transplant patients, patients with renal failure, COPD, and smokers - urinary antigen assay is very sensitive and the antigen may persist in the urine for weeks even after treatment has begun - patients may have LFT abnormalities and GI symptoms - treatment should include a quinolone or macrolide, such as erythromycin - transmission occurs through aerosolization of a contaminated water source

dull or sharp groin pain. radiates to lateral hip, anterior thigh, or buttocks. pain usually has insidious onset - occasionally begins acutely after traumatic event. 1/2 = mechanical symptoms = catching or mechanical clicking. FADIR and FABER test are effective in detecting intraarticular pathology.

hip labral tear - magnetic resonance arthrography = diagnostic test of choice

Treatment of rhabdomyolysis with

isotonic saline - to prevent and treat AKI

40 - 75 yo with diabetes mellitus --> treatment with cholesterol med?

moderate-intensity statin - LDL of 70-189

Treatment for renal artery stenosis with good control

monitor creatinine

45 yo woman brought to doc after she was found to have nausea, fever, & upper abdominal discomfort. Her med probz include severe depression, migraine, and RA. Pt describe severe tinnitus and vertigo. She admits that she overdosed on one of her meds several hours ago. Her temp is 100.5 F, BP is 120/76 mmHg, pulse is 115/min, & respirations are 24/min. PE otherwise unremarkable. What acid-base findings is most likely to be present on arterial blood gas analysis? pH = PaCO2 = HCO3 - =

pH = 7.39 (near normal-normal) PaCO2 = 20 HCO3- = 12 Aspirin toxicity: - low PaCO2 via primary resp alkalosis & resp compensation for metabolic acidosis - low HCO3- via primary metabolic acidosis & metabolic compensation for resp alkalosis - *arterial pH normal range as 2 primary acid-base disturbances shift pH in opposite directions*

Nonpurulent cellulitis in COPD pt. Treatment?

penicillin, *ceftriaxone*, cefazolin, clindamycin - most common cause = beta-hemolytic strep

spiritual assessment in pt can be performed by

physician - Spiritual History Tool or HOPE questions - FICA Tool

Patient presents with anti-glomerular basement membrane (anti-GBM, formerly Goodpasture's) syndrome and emergent treatment with what should be initiated?

plasmapheresis + steroids + cyclophosphamide

Lead is primarily absorbed via...

respiratory tract - may otherwise be absorbed through skin or GI tract - 99% bound to erythrocytes --> can *disrupt heme synthesis* = microcytic anemia

Lifestyle modifications that are recommended to prevent recurrent gout attacks

*alcohol cessation* and weight loss - meds for lowering serum urate indicated for pts with repeated attacks of gouty arthritis or complicated dz (tophi, uric acid kidney stones)

The patient's presentation is most consistent with acute otitis media. Given the age and fever for 2 days, antibiotic treatment is indicated. First-line treatment is

*amoxicillin* Most common cause of otitis media are the penicillinase-sensitive organisms - amoxicillin fails after 48-72 hours, then consider amox/clav as it will work against beta lactamase producing organisms. = the efficacy of this is reducing antibiotic use and risk for antibiotic resistance along the way.

Patient presents with acute cholangitis and broad spectrum antibiotics that cover common bile pathogens (gram negative bacteria) should be initiated. Therefore, the patient should be started on

*ampicillin and gentamicin* Acute cholangitis - exhibits Charcot's triad of right upper quadrant pain, jaundice, fever, and chills which is present in up to 70% of patients. - sometimes also is in shock and has altered mental status. - Together with Charcot's triad, these findings form Reynold's Pentad which is seen in a smaller percentage of patients with acute cholangitis. - *Acute cholangitis in this case should be treated with supportive care and broad-spectrum antibiotics such as ampicillin and gentamicin with or without metronidazole as well as stent replacement via ERCP.* - Most patients present with obstructive jaundice caused by compression of the bile duct in the head of the pancreas. - Epigastric or back pain, vague abdominal symptoms, and weight loss also are characteristic of pancreatic cancer. - The majority of patients with acute cholangitis respond to antibiotic therapy, but endoscopic biliary drainage is ultimately required to treat the underlying obstruction.

Patient presents with agitation and signs of acute mania. Initial management calls for treatment with

*antipsychotic* Manic episode - period of seven or more days of elated or irritable mood, where the mood is not caused by drugs/medication or a medical illness (e.g., hyperthyroidism) - causing obvious difficulties at work or in social relationships and activities - requires admission to hospital to protect the person or others - person is suffering psychosis *Antipsychotics such as haloperidol are acute in onset and are the agent of choice in initial management of extreme agitation due to mania.*

Recognize patient with hyperkalemia with significant EKG changes, who should receive...

*beta 2 adrenergic agonist* - will decrease extracellular potassium - albuterol - used to shift potassium intracellularly by activating the Na-K ATPase pump - a fast-acting but temporary solution - the total body potassium needs to be decreased by limiting potassium intake and using a potassium-binding agent or diuretic Hyperkalemia - defined as serum potassium > 5.0 mEq/L - caused by increased release of potassium from cells or by decreased renal excretion of potassium - usually asymptomatic, but can cause muscle weakness leading to flaccid paralysis Figure shows an ECG with the classic findings of hyperkalemia - peaked T-waves and widened QRS morphology

Medications that may exacerbate psoriasis include

*beta blockers* (such as propranolol) NSAIDs ACE-inhibitors anti-malarial drugs lithium

Acute urinary retention (AUR) is common in elderly men, especially in setting of underlying BPH. Risk of AUR is further increased during postop period. Diagnosis is made using...

*bladder ultrasound* - diagnosis of AUR confirmed by demonstrating *>300 mL of urine* Elderly male pt with agitation, tachycardia, lower abdominal/suprapubic tenderness days after surgical repair --> AUR - likely that this pt has also recently taken meds (anesthetics, opioids, anticholinergics) that are common precipitants of AUR - treatment = insertion of Foley cath

A depressed patient exhibiting signs or symptoms of sexual dysfunction (whether medication induced or pre-existing condition) should be started on

*bupropion* / Wellbutrin - works by blocking the reuptake of norepinephrine and dopamine - adverse effect profile of this medication includes seizures and psychosis at high doses, and increased anxiety in some patients - contraindicated in patients with seizures, anorexia or bulimia (because of higher risk for seizures), and those currently taking a monoamine oxidase inhibitor - can be appropriately used as a pharmacologic intervention for diabetic neuropathy or post-herpetic neuralgia - may also be beneficial in patients with neuropathic pain - alternative use of bupropion for female hypoactive sexual desire disorder; this increased level of arousal was only temporary and was not able to improve the feeling of partnership in sexuality

Septic shock can be distinguished from other types of shock because

*cardiac index remains high*, while systemic vascular resistance and PCWP are low - Mortality rates are 25% to 30% for severe sepsis and 40% to 70% for septic shock

A 27-year-old male is brought to the ED following a motorcycle accident. The trauma team is activated. He is noted to have no eye-opening even in the presence of painful stimuli. He flexes his extremities to painful stimuli and is producing no verbal sounds. What is this patient's Glasgow Coma Scale (GCS)?

*5* The patient in this vignette has suffered a traumatic brain injury. The correct GCS would be 5, with the lowest scores (1) for both verbal response and eye opening and a score of 3 for motor response. 1. No eye opening = 1 2. No verbal response = 1 3. Flexion to pain = 3 This gives a total score of 5 for this patient rather than 3. Regardless, management would likely not change for this patient with scores of 3, 4, or 5.

Mechanism of buspirone

*5-HT1A partial agonist (vs. antagonist)* - It is an anxiolytic and can be used in cases of anxiety - It has relatively few side-effects and is rather safe in patients - does not cause dependence, sedation, or anorgasmia as other medications can, so it is desirable by most patients Clinical use - anxiolytic - generalized anxiety disorder - social phobia unlike other classes of anxiolytics - no sedation, addiction, or tolerance - no interaction with alcohol - can be used safely with other sedatives - no withdrawal

A1c associated with lower mortality in Type 2 Diabetic

*7-8%* - >9% = greater mortality

Malignant melanoma --> pathological confirmation via...

*excisional biopsy with 1- to 3-mm border around lesion* - narrow margin

Be overly suspicious for intraocular foreign body in pts with high-velocity injuries (drilling, grinding, etc). If initial pen light exam doesn't reveal any conjunctival and corneal abrasions or foreign bodies, proceed with...

*fluorescein exam*

Pts who receive solid organ transplantation require high-dose immunosuppressant med to prevent organ rejection. This creates immunocompromised state with high risk of opportunistic infections, most notably...

PCP and CMV --> oral *trimethoprim-sulfamethoxazole* (TMP-SMX) needed = also efficacy against most strains of Listeria monocytogenes & Toxoplasma gondii --> TMP-SMX can be safely discontinued (usually 6-12 mos after transplant)

Thiamine deficiency can cause Wernicke encephalopathy, which is characterized by encephalopathy, oculomotor dysfunction, and gait ataxia. This is generally seen in malnourished pts (*anorexia*, chronic alcohol use) and may be induced iatrogenically by...

admin of glucose without thiamine - bc the body's requirement for thiamine (cofactor) increases with high metabolic rate or glucose intake, admin of glucose *before* thiamine can induce or worsen condition --> coma or death

"turtle sign"

*fetal head retracts back into introitus during delivery* Impaction of shoulder - delivery of POSTERIOR shoulder - rotation of shoulder 180 degree - zavanelli maneuver = pusing fetal head back into vaginal canal --> emergent C-section

Following a MI, ventricular remodeling occurs and gradually causes dilatation of left ventricle with thinning of ventricular walls. This can result in CHF. Ventricular remodeling occurs in weeks to mos following MI; therefore it can be not seen on echo. What drug limits ventricular remodeling?

ACEI (ex. *enalapril*) - should be initiated within 24 hs of MI in all pts without contraindication

HIV pts with odynophagia and visible oral thrush likely have candidal esophagitis and should receive 1-2 wks of empiric oral fluconazole therapy. If symptoms persist or no thrush is apparent, endoscopy with biopsy should be performed. Herpes simplex esophagitis is characterized by small, well-circumscribed round, ovoid ulcers and intranuclear inclusions; it is treated with acyclovir. *Cytomegalovirus esophagitis is characterized by large linear ulcers and intranuclear and intracytoplasmic inclusions; it is treated with*

IV *ganciclovir* CMV esophagitis - focal substernal burning pain + odynophagia + large linear ulcerations (shallow & in distal esophagus) on endoscopy

Pt with known adrenal insufficiency secondary to hypopituitarism who is undergoing period of stress such as surgery should be given

IV corticosteroids - 50 mg q8h

First-line options for bipolar disorder maintenance treatment.

Lithium & *valproate* - lithium excreted unchanged in kidneys & should generally be avoided in pts with renal dysfunction due to risk of systemic toxicity & long-term nephrotoxic effects

Antibiotic therapy in pts with infective endocarditis should be tailored to specific causative microorganisms as soon as they are identified. Most viridans group streptococci are highly susceptible to penicillin and should be treated with...

IV aqueous penicillin G or *IV ceftriaxone* for 4 wks

Pts with symptomatic sinus bradycardia should be treated initially with...

IV atropine

Treatment for hepatitis C in pregnancy

- no good treatment options for Hepatitis C for the mother during pregnancy - mother will have to wait until after delivery to initiate antiviral therapy - Mothers who are HCV-positive should also be screened for hepatitis B - Mother should also be further counseled against alcohol consumption during pregnancy - Women who are HCV-positive should be counseled that there is no good evidence to support the use of cesarean delivery for reducing vertical transmission of HCV - breastfeeding has not been associated with an increased risk of HCV infection in newborns

Occurs following fracture of large, marrow-containing bones (femur, pelvis). Pts classically have triad of respiratory distress, neurologic dysfunction, petechial rash.

Fat embolism syndrome / *fat globules entering bloodstream*

Bipolar disorder is a highly recurrent illness that requires maintenance pharmacotherapy. In pts not adequately controlled with monotherapy, what is the treatment?

combination of *lithium* or valproate and second-generation antipsychotic (*quetiapine*) = 1st-line

Omeprazole --> risk of

hip fracture

Necessary prior to starting PrEP to avoid inadvertent treatment of HIV infection with emtricitabine/tenofovir (Truvada)

negative HIV Ab test - don't want to develop resistance

IUD inserted within 7 days since menstrual started ... contraception?

no backup contraception - inserted more than 7 days after menstrual bleedings starts --> contraception for next 7 days

Pt is most likely experiencing symptoms of a pulmonary embolism (PE). Most patients presenting with PE will have what findings on chest x-ray?

normal

Hypovolemic hyponatremia: renin = aldosterone = antidiuretic hormone =

renin = high aldosterone = high antidiuretic hormone = high Hypovolemic hyponatremia - via nonosmotic stimulation of ADH secretion in response to hypovolemia, hypotension, decreased renal perfusion (via ang II)

The correlation coefficient (*r*) shows...

strength and direction (positive, *negative / -r*) of linear association between 2 variables - does not necessarily imply causality Ex. negative correlation between HDL2 and carotid intima-mediate thickness (given r = -0.25 < 0), when there was increase in one of the variables (HDL2), there was decrease in other variable (carotid intima-media thickness)

HIV pts who should receive varicella vaccine if never received it before/have low titers...

those with CD4 counts >200/mm^3 who have low titers

This patient presents with mild, noninflammatory acne that has not responded to salicylic acid, and topical retinoids have caused excessive drying. The next best step in management is

topical benzoyl peroxide and clindamycin

29 yo male presents to your office with 2 wk ho anal pain and bright red blood on his stool with bowel movements. He says he typically has bowel movements every 3-5 days and his stool is usually hard. He has not had any purulent drainage or perianal masses. Most likely diagnosis?

*Anal fissure* - posterior midline fissure = pain during and after defection via passage of hard stool --> stretched = bleeding

Pseudotumor cerebri typically presents in overweight females with headache, double vision, papilledema, and cerebral edema on MRI; risk factor for pseudotumor cerebri is =

*hypervitaminosis A*

Middle-aged adult, superficial unilateral hip pain that is exacerbated by external pressure to upper lateral thigh (*as when lying on affected side in bed*) suggests...

*trochanteric bursitis* - inflammation of bursa surrounding insertion of gluteus medius into femur's greater trochanter - excessive frictional forces secondary to overuse, trauma, joint crystals, infection - hip pain when pressure applied (when sleeping) & when external rotation or resisted abduction

First-line treatments for HT

ACEIs ARBs CCBs *Thiazide diuretics* - decreased mortality rate

Greatest risk in completing homicide.

Access to firearms

Usually caused by abx (penicillins, cephalosporins, *trimethoprim*, rifampin), NSAIDs, and diuretics. Pts present with fever, *maculopapular rash, & renal failure. Urinalysis reveals WBC casts and, less frequently, eosinophils.

Drug-induced *interstitial nephritis*

Recognize retinal detachment, which is associated with *"Curtain drawn down" vision loss*

Episodes of flashers and floaters and a fundoscopic exam positive for retinal "crinkling," suggesting the diagnosis of retinal detachment - separation of the layers of the retina - complain of photopsia (flashes of light) and showers of floaters (spots in the visual field), which are both associated with posterior vitreous detachment - may then progress to the most classic description of retinal detachment, which is "curtain coming down" vision loss Fundoscopic examination - crinkling and/or grey retina and changes in vessel direction - surgical emergency, treated by either laser therapy or cryotherapy to create permanent adhesions

Most commonly inherited hypercoagulable disorder in Caucasian population. Leads to increased risk of thrombosis. Testing should be considered for unprovoked first-time thrombus in young (age <45) pts or those with unusual site of thrombus.

Factor V Leiden / *Activated protein C resistance* - SOB + leg swelling + sinus tachycardia + elevated D-dimer = PE

Recognize *senile purpura*

Given a patient's history of similar lesions, recent sun-exposure, and otherwise healthy appearance, senile purpura is the most likely diagnosis. Senile purpura - occurs in >10% of individuals over the age of 50 - dark purple, irregularly shaped patches that are approximately 1-4 cm in diameter with well demarcated margins - typically are found on the extensor surface of forearms and the dorsal surface of hands - as people age there is a loss of elastic fibers in perivascular connective tissue that makes blood vessels fragile --> minor trauma and photodamage can lead to superficial vessel damage - can take up to 3 weeks to resolve and typically do not undergo the characteristic color changes as seen with normal bruising

midwest (Iowa) + chest radiograph: BB-sized calcifications in miliary pattern. PPD skin test negative.

histoplasmosis

Patient has cutaneous malignant melanoma, for which treatment is

wide surgical excision with *2.0 cm* margins and sentinel lymph node biopsy - with the possibility of subsequent radical axillary lymph node dissection Malignant melanoma - high metastatic potential and mortality rate - depth of the lesion is the single most important prognostic factor for survival = used to determine the appropriate surgical margin for excision In situ lesions require 5mm margins Lesions with depth less than or equal to 2mm require 1cm margins Lesions with depth >2.0mm necessitate 2cm margins

patient has signs and symptoms of hyperthyroidism. Low uptake of radioactive iodine in conjunction with a low TSH and thyroglobulin are characteristic of...

* excessive doses of levothyroxine / *factitious thyrotoxicosis*

235 o 250 people with dz has positive test and 600 of 680 people without disease had negative test. Specificity?

*600/680 = 88%* true negative / true negative + false positive

Levels diagnostic for prolactinoma

*>200 ng/mL* Prolactin-producing lactotroph adenomas / prolactinomas - most common - hypogonadism with low testosterone levels - low-normal LH - TSH may be normal or low

17 yo boy comes to doc complaining of worsening acne. He has scattered papules and pustules with mild-to-moderate redness at forehead, nose, and chin. His acne has been present for 2 yrs. Lesions are not painful, but boy is concerned as symptoms have progressively worsened and his peers at school are starting to make fun of him. He currently uses topical tretinoin and washes his face twice daily with benzoyl peroxide. His med hx is otherwise negative. On exam, he has moderate pustular acne with erythema but no scarring. Best next step in management of this pt?

*Add topical erythromycin* Initial management for inflammatory acne = topical retinoids & benzoyl peroxide --> *most pts with moderate or moderate-to-severe acne require topical abx (erythromycin, clindamycin)*

Prevention of preeclampsia in pregnant women at risk of preeclampsia

*Aspirin* Prophylaxis: - low dose aspirin in high risk patients can reduce pre-eclampsia by 24% - decreases TXA2 while maintaining vascular wall prostacyclin - decreases ATII

Pregnant woman + nausea --> first-line treatment if wants to avoid sedation?

*B6/pyridoxine*

What test has higher sensitivity (>97%) in pts with primary syphilis?

*Fluorescent treponemal antibody absorption* / Treponemal tests

Man with positive hepatitis C Ab test. What test would be needed?

*Hepatitis C RNA* - current infection confirmation

PANDAS

*Pediatric Autoimmune Neuropsychiatric Disorders Associated with Streptococcal Infections* - Obsessive compulsive disorder (OCD) and/or tic disorders suddenly appear following a strep infection (such as strep throat or scarlet fever) - The symptoms of OCD or tic symptoms suddenly become worse following a strep infection. - The symptoms are usually dramatic, happen "overnight and out of the blue," and can include motor and/or vocal tics, obsessions, and/or compulsions. - In addition to these symptoms, children may also become moody or irritable, experience anxiety attacks, or show concerns about separating from parents or loved ones.

34 yo man evaluated for malaise and palpitations over last 4 wks. He also reports low-grade fevers. He denies any sick contacts or recent travel. Pt has ho T1DM and strong fam hx of colon cancer. 2/6 holosystolic murmur heard at apex. Echocardiography shows 5-mm mobile mass attached to anterior leaflet of mitral valve and mild to moderate mitral regurgitation. Blood cultures grow Eikenella corrodens. Most likely predisposing condition to this pt's current condition?

*Periodontal infection* Infective endocarditis (IE) of mitral valve via infection with Eikenella corrodens - belongs to HACEK group - minority of cases of IE - gram negative anaerobe - common constituent of normal human oral flora - usually seen in setting of poor dentition and/or periodontal infection, along with dental procedures that involve manipulation of gingival or oral mucosa

38 yo woman comes to ED due to 2 hrs of left leg weakness and numbness. Over last 3 mos, pt has had several similar episodes in which her left foot tingles and becomes numb and her entire left foot becomes limp, causing her to trip. Symptoms resolve spontaneously within few hours. Her right leg has never displayed similar symptoms. Pt has no significant med hx but does have fam hx of multiple sclerosis. She doesn't use tobacco, alcohol, or illicit drugs Pt has experienced major psychosocial stress after losing her job 6 mos ago, but this has tempered somewhat since she joined meditation group. Vitals within normal limits. PE shows diminished pinprick sensation over dorsum of left foot & weakness of left big toe on extension. Pt able to walk on her left toes but not on her left heel. Most likely cause of this pt's current symptoms?

*Peripheral nerve compression* Common fibular neuropathy: Common fibular nerve/common peroneal nerve - travels near fibular head - susceptible to compressive injuries via leg immobilization (cast, bedrest), prolonged leg crossing, protracted squatting - unilateral foot drop - numbness/tingling over dorsal foot & lateral shin - impaired ankle dorsiflexion / walking on heels / great toe extension - preserved plantar flexion /walking on toes & reflexes - diagnosis via electromyography & nerve conduction studies

40 yo man comes to office due to 2 day ho retrosternal chest pain, pain with swallowing, and epigastric burning. Pain is severe & pt is afraid to swallow food. He has never had similar symptoms before. Pt has no associated SOB, vomiting, melena, or blood in stool. Past med hx notable for nonischemic cardiomyopathy, for which he takes furosemide, carvedilol, spironolactone, lisinopril, and potassium chloride. Temp is 99 F, BP is 110/65 mmHg, and pulse is 67/min. Exam shows soft holosystolic murmur at cardiac apex, similar to that in prior exams. Rest of exam shows no abnormalities. Pt's ECG shows sinus rhythm with no acute ischemic changes. Endoscopy reveals circumferential deep ulceration with relatively normal surrounding mucosa at middle third of esophagus. Most likely diagnosis?

*Pill-induced esophagitis* - abrupt-onset retrosternal pain + severe odynophagia - via direct effect of certain meds on esophageal mucosa = via acid effect (tetracyclines), osmotic tissue injury (potassium chloride), disruption of normal gastroesophageal protection (NSAIDs) - most common in mid-esophagus due to compression by aortic arch or enlarged left atrium

A 27-year-old graduate student returns from the Democratic Republic of the Congo, where he spent the last several months studying the mating habits of a subspecies of gorilla. For about a week prior to his return, he reports experiencing intermittent fevers and headaches that now appear to be worsening in severity. On physical exam, the patient is presently afebrile. His lungs are clear to auscultation. His cardiac exam is unremarkable. He has an enlarged liver on palpation, but his abdomen is soft and non-distended. Given his recent travel, he is referred to an infectious disease specialist who orders imaging studies of the brain (flip for image). Which of the following pathogens is most likely responsible for this patient's symptoms?

*Plasmodium falciparum* Malaria + cerebral involvement = *Plasmodium falciparum* - Cyclical fevers and headaches are characteristic of malarial disease. - The diagnosis of malaria is made via blood smear, which in this case shows Plasmodium falciparum. Brain imaging of a patient with acute cerebral malaria.

63 yo man comes to ED due to 2 day ho right knee pain & swelling. Pain is severe enough to interfere with walking. Pt doesn't recall any significant trauma to knee. Med hx notable for T2DM, HT, hypercholesterolemia, and mild COPD. Pt has 40 pck yr smoking hx and drinks 3-5 alcoholic beverages a night. He is industrial concrete finisher and frequently works extended hours when on job. Temp is 99 F, BP is 115/75 mmHg, and pulse is 92/min. On exam, there is mild swelling anterior to patella, along with faint erythema and sharp tenderness. Passive ROM at knee is normal. Most likely cause of this pt's acute symptoms?

*Prepatellar bursitis* / Housemaid's knee - acute pain & localized tenderness - occupations requiring repetitive kneeling = concrete work, carpet laying, plumbing - very commonly due to S aureus - diagnosis confirmed with aspiration of bursal fluid for cell count & Gram stain - pts treated with drainage and systemic abx

A 71-year-old Caucasian male is found to have a small crusting, ulcerated nodule on his neck. A biopsy is consistent with squamous cell carcinoma and there is no evidence of regional lymph node metastasis. If the patient refuses surgery at this time, which of the following is the most appropriate next step?

*Radiation* Squamous cell carcinoma - generally treated with surgery - radiation is an alternative treatment. - common in the elderly - arises from epidermal cells undergoing keratinization - sunlight exposure is the most important risk factor, along with chronic skin damage and immunosuppressive therapy - actinic keratosis is a pre-malignant skin lesion that results from sun exposure and may progress to SCC - SCC has an excellent prognosis (95% cure rate) in lesions without lymph node involvement that are completely surgically excised

In clinical trials, this is said to be successful when a similarity of baseline characteristics of pts in treatment and placebo groups is seen.

*Randomization* - purpose = to make distribution of all potential confounders even (between treatment and placebo groups) - potentially controls known, as well as unknown confounders - method of assessing adequacy of randomization = looking at distribution of baseline characteristics in both groups --> similar = can assume that randomization evenly distributed the confounders between the groups --> successful randomization

28 yo woman, gravida 1 para 1, comes for routine 4-wk postpartum visit following uncomplicated vaginal delivery. Pt describes some mild vaginal soreness & breast tenderness but has no other physical symptoms. She says, "The baby is adorable, but I worry about being a good mom." She tearfully describes feeling exhausted and unable to return to sleep after getting up in middle of night to feed baby. She says, "The baby cries all the time and nothing I do helps. I tried to breastfeed, but I gave it up bc she was fussy and I was afraid she wasn't getting enough to eat. My husband tries to be supportive and has offered help with nighttime feedings, but I am up at night anyway bc I can't sleep." Pt has little time to care for herself and has been eating poorly. She feels increasingly depressed and has little energy. Pt says, "Everyone expects me to be happy, but it has been so hard. I don't deserve to be a mother." Pt has no psychiatric hx & no thoughts of hurting herself or the baby. PE normal. Routine lab tests, including hgb & TSH levels, are within normal range. Most appropriate next step in management of this pt?

*Recommend treatment with SSRI* Postpartum depression - depressed mood + insomnia + appetite disturbance + lower energy + feelings of worthlessness - persists beyond 2 wks - functional impairment-->treatment indicated = psychotherapy, pharmacotherapy - SRRIs commonly used as first-line - in breastfeeding pts, sertraline preferred as levels in infant sera are usually undetectable

Most common correctable cause of secondary hypertension. Should be suspected in all pts with diffuse atherosclerosis and resistant HT. Presence of *systolic-diastolic abdominal bruit* has high specificity.

*Renal artery stenosis* / Renovascular HT - resistant HT = persistent HT despite use of >3 antihypertensive agents of different classes --> should be evaluated for secondary cause of HT

24 yo female with painless ulcer on labia. You suspect syphilis. RPR test negative. Best strategy for confirming or ruling out syphilis?

*Repeat RPR in 2 wks* - positive within 3 wks of primary chancre

Carbamazepine side effects in elderly

*hyponatremia* - SIADH - interferes with ability to dilute urine

A 46-year-old woman presents to her primary care provider for itching. She reports that she has always had dry skin but that the itching has gotten significantly worse over the last few years. The patient also endorses fatigue and dull abdominal pain. Her past medical history includes Hashimoto's thyroiditis, mitral valve prolapse, and osteoarthritis. She takes levothyroxine and ibuprofen for pain in her knees. The patient drinks 2-3 beers per week. She has a 10 pack-year smoking history but quit 15 years ago. She denies any family history of cancer. On physical exam, her sclera are anicteric. Her abdomen is soft and tender to palpation in the right upper quadrant. Her bowel sounds are normal and hepatomegaly is present. A right upper quadrant ultrasound shows no evidence of extrahepatic biliary dilation. Laboratory studies are performed which reveal the following: Aspartate aminotransferase (AST): 76 U/L Alanine aminotransferase (ALT): 57 U/L Alkaline phosphatase: 574 U/L Total bilirubin: 1.6 mg/dL This patient is most likely to have which of the following additional findings?

*hyperlipidemia* Fatigue, pruritus, cholestatic pattern of liver injury, and a right upper quadrant (RUQ) ultrasound without extrahepatic biliary dilation, which is consistent with a diagnosis of *primary biliary cirrhosis (PBC)* - commonly presents with hyperlipidemia and anti-mitochondrial antibodies - In this case, the patient's normal RUQ ultrasound makes intrahepatic injury more likely, as extrahepatic pathology would likely lead to dilation of the common bile duct - most common in middle-aged women, and fatigue and pruritus are frequently the presenting symptoms - also associated with other autoimmune conditions, including this patient's Hashimoto's thyroiditis - diagnosis of PBC would be confirmed with positive antimitochondrial antibodies *Illustration A demonstrates the multifocal dilation and stricturing of intra- and extrahepatic ducts found in primary sclerosing cholangitis (PSC).*

Exertional heat stroke is characterized by >104 F and CNS dysfunction. Management involves...

*immersion in ice water* - rapid cooling, preferably with ice water immersion

Polymyalgia rheumatica (PMR) affects pts age >50 yo and is characterized by pain and stiffness in neck, shoulders, pelvic girdle, along with elevated ESR. Treatment of choice for uncomplicated PMR is ...

*low-dose prednisone* = rapid relief of symptoms PMR - subacute-to-chronic (>1 mo) pain in shoulder and hip girdles - morning stiffness lasting >1 hr - frequently associated with giant cell arteritis (GCA)/temporal arteritis

Benefit of treatment of GDM is decreased risk of....

*maternal preeclampsia*, LGA, shoulder

Gout is a common complication of ...

*myeloproliferative disorders* - via excessive turnover of purines & resulting increase in uric acid production Polycythemia vera = ex - pruritus triggered by hot baths = aquagenic pruritus - headaches - hepatosplenomegaly

Patient's symptoms of lower leg claudication and exam findings are consistent with peripheral artery disease (arterial insufficiency). Patients with peripheral artery disease (PAD) are at an extremely high risk of experiencing a

*myocardial infarction*

Pt develops whistling noise during respiration following rhinoplasty --> one should suspect...

*nasal septal perforation* - likely via septal hematoma

Compression of median nerve at wrist causes carpal tunnel syndrome, characterized by pain & paresthesias in first 3 digits and radial half of fourth. Diagnosis is usually made on clinical ground, but what can be used to confirm diagnosis?

*nerve conduction studies* - will show slowing in median nerve at wrist

senile purpura usually presents with ecchymoses in elderly pts in areas exposed to repeated minor trauma (extensor surfaces of hands & forearms). It is due to...

*perivascular CT atrophy* - age-related loss of elastic fibers in perivascular CT

A 92-year-old female with CAD, hypertension, atrial fibrillation, hyperlipidemia, and type II diabetes is brought to the emergency room from her nursing home due to altered mental status. The patient had been confused and lethargic for one day prior to her arrival in the hospital. While being examined by the emergency room physician, the patient has a seizure. Physical examination reveals decreased deep tendon reflexes symmetrically. Jugular venous pressure is normal, mucous membranes are moist, and no edema is present in the patient's extremities. CT scan of the head is shown in Figure A. Laboratory studies reveal the following: Sodium: 115 mEq/L Potassium: 4.5 mEq/L Chloride: 86 mEq/L Bicarbonate: 16 mEq/L BUN: 41 mg/dL Creatinine: 2.09 mg/dL Glucose: 213 mg/dL Urine studies show: Urine sodium: <10 mEq/dL Urine osmolality: 75mOsm/kg Urine creatinine: 14 mg/dL Which of the following is the most likely cause of the patient's neurological symptoms?

*poor solute intake* Altered mental status and urine and electrolyte findings consistent with *euvolemic hypotonic hyponatremia due to poor solute intake* - Hyponatremia classically occurs in elderly patients on 'tea and toast' diets, alcoholics with 'beer potomania', and anorexic patients with very low calorie intake - Euvolemic hypotonic hyponatremia occurs because the clearance of free water is linked to excretion of solute in the urine - Urine osmolality is less than 100 mOsm/kg in these patients

Can be more activating (akathisia) and less sedating with less potential for weight gain. For this reason, it is the drug of choice when obesity and diabetes are a concern in a schizophrenic pt.

Aripiprazole

Can present similarly to other atypical community-acquired pneumonias. However, concurrent GI and CNS symptoms may suggest this.

Legionnaires' disease - *urinary Legionella antigen* - hotels & cruise ships - treatment = macrolide or fluoroquinolone

Results in triad of fever, back pain, neurological deficits

Spinal epidural abscess - minority of cases - severe focal back pain --> nerve root pain = shooting/electric --> more severe neuro manifestations = motor weakness, sensory changes, bowel/bladder dysfunction, paralysis - diagnostic test of choice = MRI of spine

Most pts on prolonged glucocorticoid therapy (especially if taking other immunosuppressant agents) should receive prophylaxis with...

TMP-SMX Risk of developing *Pneumocystis jirovecii pneumonia* - HIV or chronic glucocorticoids PCP + no HIV - acute respiratory failure = tachypnea, dyspnea, hypoxia + resp alkalosis - dry cough - fever - lactate dehydrogenase elevated - chest x-ray = bilateral, diffuse interstitial infiltrates - diagnosis confirmed via resp samples (induced sputum or bronchoalveolar lavage fluid) using microscopy with specialized stains

Frequent cause of allergic contact dermatitis. Erythematous, vesicular rash involves exposed skin & can form linear streaks where skin has brushed against plant leaves.

Toxicodendron plants = poison ivy/oak/sumac

Characterized by non-anion gap metabolic acidosis, persistent hyperkalemia, and mild-to-moderate renal insufficiency. Commonly occurs in ps with poorly controlled diabetes.

Type 4 *renal tubular acidosis* / Hyperkalemic renal tubular acidosis - normal anion gap = 8-12 mEq/L - damage to juxtaglomerular apparatus --> hyporeninemic hyperaldosteronism

Underlying pathology of Wolff-Parkinson White (WPW) Syndrome...

accessory *bundle of Kent* --> causes ventricular pre-excitation and can predispose to paroxysmal tachycardia Bundle of Kent - accessory conduction pathway found in 0.1%-0.3% of the general population - problems arise when this pathway creates an electrical circuit that bypasses the AV node Characteristic EKG changes in WPW - 'delta wave' that demonstrates ventricular pre-excitation --> delta wave makes the PR interval appear short and the QRS interval appear long Short-term treatment consists of calcium channel or beta blocks when the vagal maneuver no longer works. For long-term therapy, patient can be given atrioventricular nodal blocking agents or class IC or III antiarrhythmics. Also, catheter ablation can be used in patients who cannot tolerate the medications and experience persistent or recurrent supraventricular tachycardias, and for WPW patients this can be curative.

Osteoarthritis is a clinical diagnosis supplemented with radiographic evidence. Four of the following six clinical criteria should be met:

age >50 bony tenderness bony crepitus bony enlargement *minimal or no morning stiffness* not warm/cool joint.

Nonspherical cornea can lead to ...

astigmatism = blurry vision at distance and up close

Condition that requires use of much higher than expected dose of levothyroxine (Synthroid) to adequately treat hypothyroidism

atrophic gastric, PPI chronic use, *H pylori*

Major metformin side effect

lactic acidosis - important in setting of AKI

preseptal cellulitis is an infection of the eyelid anterior to the orbital septum. it presents with fever and leukocytosis as well as erythema and edema of the eyelid and is treated with...

oral abx (doxycycline)

Recommendation in all pts with croups

oral dexamethasone as single dose

Progressive bilaterally symmetric hearing loss with subjective tinnitus, advanced age, and absence of other neuro signs suggest

presbycusis

Pt with hyperthyroid symptoms, suppressed TSH, "hot" nodule = toxic adenoma. Untreated hyperthyroid pts are at risk for...

rapid *bone loss* - from increased osteoclastic activity in bone cells - can also put pt at increased risk for: cardiac tachyarrhythmias, including atrial fibrillation

Infantile hemangiomas after 5 mos old + residual skin changes + complete involution --> treatment?

referral for surgical excision

Best for making diagnosis of fibromyalgia?

structured symptom history

Recognize pleural effusion in active TB pt, which would be treated with *thoracentesis* "A 28-year-old woman presents to the emergency department with shortness of breath. She immigrated from Uganda two months ago. She reports cough with occasional blood-tinged sputum and night sweats for the past week. She denies any chest pain or discomfort currently. The patient denies any sexual activity or intravenous drug use. She denies any history of cancer in her family. She currently lives in an apartment with several other female migrant workers. She received a vaccination against tuberculosis when she lived in Uganda. Her temperature is 99°F (37.2°C), blood pressure is 110/70 mmHg, pulse is 100/min, respirations are 20/min, and oxygen saturation is 94% on room air. Breath sounds are decreased in the left posterior lower and middle lung fields; egophony is negative bilaterally in all lung fields. The remainder of the physical exam is unremarkable. A complete blood count and basic metabolic panel are obtained: Serum: Na+: 135 mEq/L Cl-: 100 mEq/L K+: 3.5 mEq/L HCO3-: 26 mEq/L BUN: 10 mg/dL Glucose: 105 mg/dL Creatinine: 0.6 mg/dL Leukocyte count: 20,000/mm^3 Segmented neutrophils: 69 % Bands: 10 % Eosinophils: 2 % Basophils: 1 % Lymphocytes: 25 % Monocytes: 3 % Hemoglobin: 11 g/dL Hematocrit: 33 % Platelet count: 400,000/mm^3 An electrocardiogram reveals sinus tachycardia without acute ST-T wave changes. Chest radiography is obtained in Figure A. What is the best next step in management?"

productive cough, night sweats, and unexplained pleural effusion in this otherwise healthy young immigrant from a tuberculosis-endemic country merits workup for active tuberculosis. Of the answer choices, diagnostic thoracentesis is the best next step. Pleural effusions - pathologic accumulations of fluid in the pleural space *Figure shows a left pleural effusion. Note the meniscus at the upper margin of the white opacification and the absences of the left costophrenic angle and the left heart border. Despite this, the visible portions of the mediastinum suggest a normal cardiomediastinal silhouette.*

injured to overall valgus support during overhead throwing (volleyball). pop followed by immediate pain and bruising around medial elbow.

ulnar collateral ligament injury - moving valgus stress test = 100% sensitivity

External hordeolum is an acute inflammatory disorder of the eyelash follicle or tear gland and presents as an erythematous, tender nodule at the lid margin. It is often due to infection with Staphylococcus aureus but can be sterile. Initial treatment includes...

warm compresses

Pt's presentation of hyponatremia, history of small cell lung cancer, and urine/serum osmolality values is consistent with a diagnosis of syndrome of inappropriate secretion of antidiuretic hormone (SIADH). Initial treatment of SIADH is with

water restriction

Occurs predominately in ppl aged 55-70 yo. Presents with acute onset of severe eye pain and blurred vision associated with nausea & vomiting. Exam reveals red eye with steamy cornea and moderately dilated pupil that is non reactive to light.

*Acute angle closure glaucoma* - via closure of pre-existing narrow anterior chamber angle - permanent cure = laser peripheral iridotomy

70 yo right-handed man brought to ED due to sudden onset of right-sided weakness and urinary incontinence that began about 10 hours ago. He has had T2DM for last 20 yrs and HT for last 28 hrs. On exam, there is 4/5 strength in RU extremity, 1/5 strength in right lower extremity, and Babinski sign on right side. Sensation decreased throughout right foot and leg. Visual fields are full with no deficits. Most likely diagnosis of this pt?

*Anterior cerebral artery stroke* (ACA) stroke - contralateral motor and/or sensory deficits = more pronounced in lower limb than upper limb - urinary incontinence (via damage to cortical micturition centers of mesial frontal lobe), gait dyspraxia, primitive reflexes (grasp, sucking), abulia, emotional disturbances

72 yo man comes to ED due to severe left LE pain. He first noticed tingling sensation in his leg several hours ago and it has since become increasingly painful. Med hx significant for HT, T2DM, paroxysmal atrial fibrillation, and OA. Pt has 45 pck yr smoking hx. His BP is 142/80 mmHg & pulse is 112/min & irregular. His left LE skin appears mottled and is cool to touch. Distal pulses of left leg are not palpable. Right carotid bruit. What med could have best prevented this pt's acute problem?

*Apixaban* - non-vit-K antagonist oral anticoagulant (along with dabigatran, rivaroxaban, edoxaban) - reduces risk of systemic embolization in pts at moderate to high risk of thromboembolic events acute pain + paresthesia + pallor + pulselessness = acute arterial occlusion of LLE via thromboemoblism in setting of *atrial fibrillation* - irregular pulse

33 yo woman comes to office with severe bilateral facial pain for past several days. She reports sharp, shooting pain that is confined to her cheeks and jaw, lasts several seconds, and occurs 10-20 times a day. Pain is sometimes triggered by cold breeze, brushing her teeth, or chewing. Pt has been taking ibuprofen for pain without much relief. She has never experienced this condition before, but 6 mos ago, she was treated for acute sinusitis. Pt is primary school teacher and doesn't use tobacco, alcohol, or illicit drugs. Her temp is 98 F, BP is 118/82 mmHg, pulse is 72/min, and respirations are 14/min. On PE, similar pain is elicited by lightly touching pt's cheeks. Otherwise, neurologic exam demonstrates no focal deficits. Most likely cause of this pt's symptoms?

*Demyelination of nerve nuclei* Trigeminal neuralgia: Multiple sclerosis - autoimmune demyelinating CNS disorder - *may present with trigeminal neuralgia BILATERALLY* - via demyelination of nucleus of trigeminal nerve or nerve root --> improper signaling of nerve & paroxysms of severe pain - pt's episode of right hand numbness that lasted 2 wks & spontaneously improved = first symptom of MS

Recognize chronic dacrocystitis, in which the treatment after recurrence is *referring the patient to an oculoplastic surgeon for a dacryocystorhinostomy*

Chronic dacryocystitis (infection of the lacrimal sac) - After the acute infection is treated, he should be referred to an oculoplastic surgeon for a dacryocystorhinostomy (DCR). - This is the procedure of choice for chronic dacryocystitis to prevent a relapsing infection. - Infection of the lacrimal gland caused most commonly by staph aureus but also strep pneumoniae, H. influenzae, and S. pyogenes. - Treatment is indicated to prevent the spread to the cavernous sinus resulting in thrombosis or meningitis. - A dacryocystorhinostomy is a procedure to restore the flow of tears into the nose from the lacrimal sac when the nasolacrimal duct does not function.

Acute urticaria best managed with

*H1*-histamine blockers - 2nd gen = longer duration of action, less drowsiness

Arrhythmia most specific for digitalis toxicity?

*Atrial tachycardia with AV block* Digitalis toxicity = increased ectopy & increased vagal tone

What has been shown to decrease the number of recurrences and slow the progression of disability in Relapse-Remitting Multiple Sclerosis (RRMS)?

*Interferon-beta*

Per the AAFP guidelines, what opening pressures are diagnostic of intracranial hypertension?

*opening pressures above 250 mm H2O*

What drugs can cause Idiopathic intracranial HT (IIH)

GH Tetracyclines (minocycline, doxycycline) Excessive vitamin A - its derivatives: isotretinoin, all-trans-retinoic acid

Recognize *internuclear ophthalmoplegia (INO)*, in which IV corticosteroids should be used to treat an acute exacerbation of MS

IV methyprenisolone usually

USPSTF "D" recommendation

moderate or high certainty that service has *no* net benefit or that harms outweigh benefits

warfarin-induced skin necrosis (WISN) is secondary to ...

protein C & S depletion

Contraindicated on active bone growth plates in active children

ultrasound - ex. severe's disease

Part of normal flora but may cause invasive infection in pts who have dental infections or trauma (especially in those with weakened immunity). Symptoms include chronic, slow-growing, nontender mass at or near mandible. Multiple sinus tracts to skin may drain purulent fluid with sulfur granules (discrete, yellow granules).

*Actinomyces* - *penicillin* = preferred therapy - surgery sometimes required for severe cases

Emotional or behavioral symptoms (anxiety, depression, disturbance of conduct) developing within 3 mos of identifiable stressor and lasting no longer than 6 mos once stressor ceases.

*Adjustment disorder* - impairing symptoms but don't meet criteria for another mental disorder

72 yo woman with poorly controlled T2DM presents to clinic one wk after being discharged from hospital. She had been admitted with pyelonephritis secondary to multi-drug resistant organism, and received several days of IV abx. Her serum creatinine on admission had been 2.1 mg/dL. Today it is found to be 4.9 mg/dL. Urinalysis reveals rare epithelial casts and no WBCs. FENa greater than 2%. What antibiotic did she most likely receive during her hospitalization?

*Amikacin* ARF in setting of CKD Pyelonephritis with multidrug-resistant organism, probz gram-negative rod - aminoglycoside most commonly used in this setting --> potentially nephrotoxic

Essential tremor is an active tremor that tends to worsen at end of goal-oriented actions (ex. drinking from cup). Manifestations usually arise in bilateral hands and forearms; head is also commonly involved. First-line therapy?

*Beta blockers / propranolol*

A 21-year-old female college student presents with headache, malaise, and fever of 3 hours duration. Past medical history is unremarkable and the patient does not take medications. She denies recent alcohol or drug use. Temperature is 39°C, blood pressure is 110/70 mmHg, pulse is 76/min and respirations are 17/min. Physical examination is notable for a petechial rash present in the lower extremities bilaterally. Extension of the knee when both the hip and thigh are flexed produces pain. Which of the following is the most appropriate next step in management?

*Blood cultures* Petechial rash and Kernig's sign consistent with likely *meningococcal meningitis.* - *Blood cultures should be obtained prior to initiation of antimicrobial therapy.* - common in college students and others living in close quarters, such as military recruits. - Symptoms include headache, fever, neck stiffness, photophobia, and altered mental status. - Petechial rash is present in over half of patients with meningococcal disease, though Haemophilus influenzae and pneumococcus may present with petechial rash as well. *Illustration demonstrates a Gram stain of meningococci from a culture of cerebral spinal fluid. Meningococci are Gram negative diplococci.* *Cultures from blood are quick and don't typically delay antibiotic administration (it is easier to stick for a blood culture than start an IV for antibiotics for example). CSF studies may take time to obtain, and antibiotic administration should not be delayed for lumbar puncture and CSF studies to be sent, especially if a CT scan is needed prior to LP.* *1) First LP AND THEN ABx ASAP if NO MRI to be done; 2) Abx, then MRI if MRI is to be done.* *For step exams, the technically correct answer is to first obtain cultures (CSF, blood, etc.) then prior to obtaining culture results (as this could take hours or even days) broad spectrum antibiotics are started which are then tailored down based on the culture results.*

Helps minimize pain of lidocaine (Xylocaine) injection?

*Buffering solution with sodium bicarbonate* - decreases pain associated with injection - warm to room temp - rapidly inserting needle though skin, injection slowly, steadily withdrawing needle

Should be suspected in pts with unexplained chronic abdominal pain, weight loss, and food aversion. Most cases due to atherosclerotic changes.

*Chronic mesenteric ischemia*

A 42-year-old male commercial sex worker comes to the emergency department complaining of persistent fatigue and malaise for the past three weeks. On physical exam, you observe a lethargic male with icteric sclera and hepatomegaly. AST and ALT are elevated at 600 and 750, respectively. HBV surface antigen is positive. Albumin is 3.8 g/dL and PT is 12. A liver biopsy shows significant inflammation with bridging fibrosis. What is the most appropriate treatment at this time?

*Combined lamivudine and interferon* *Combined therapy of interferon and lamivudine is warranted in patients with hepatitis B (HBV) who show signs of active disease.* - Usually HBV infection clears on its own and does not require treatment. - *Individuals who should be considered for therapy include those with elevated liver enzymes and HBV levels.* - Currently, there are seven medications licensed for treatment of hepatitis B infection in the U.S.: lamivudine, adefovir, tenofovir, telbivudine, entecavir, interferon alpha, and PEGylated interferon alpha. - The goal of therapy is thus to slow or halt the progression of fibrosis. - Serologic testing is the key to diagnosis in HBV infection - The diagnosis of acute HBV infection is based upon the detection of HBV surface antigen (HBsAg) and IgM anti-HBV core antigen (HBc) - The diagnosis of chronic HBV infection is based upon the persistence of HBsAg for more than six months

25 yo woman comes to doc complaining of transient vision loss in right eye. Six mos ago, she was told that her BP was high at local health fair. Her past med hx is otherwise unremarkable and she takes no meds. Her grandmother had stroke at age 50. Her BP is 164/103 mmHg and pulse is 77/min. Her BMI is 26 kg/m^2. Exam shows bruit below right mandibular ankle. Chest and abdominal exam unremarkable. She has 2+ LE pulses. Lab results: Serum creatinine 0.9 mg/dL Serum potassium 3.5 mEq/L Plasma renin High Plasma aldosterone conc/plasma renin activity ratio 10 Best next step in management of this pt?

*Computed tomography angiography of abdomen* - or duplex US Fibromuscular dysplasia - transient monocular vision loss = amaurosis fugax - transient vision loss + fam hx of stroke + carotid bruit - women 15-50 yo - noninflammatory and nonatherosclerotic - abnormal cell dev't in arterial wall --> vessel stenosis, aneurysm, dissection

66 yo male presents to ER with SOB. Symptoms started one wk ago with dry cough and exertional dyspnea. His past med hx includes HT & recent stenting for double-vessel coronary artery dz. He was hospitalized six mos ago for pneumoniae. He has 35 pck yr smoking hx. His temp is 98.9 F, BP is 160/90 mmHg, & HR is 90 & regular. On exam, pt is in mild respiratory distress, but he can speak in full sentences. Chest auscultation reveals decreased breath sounds at lung bases, bilateral crackles and occasional wheezes. His ABG shows: pH 7.46 pO2 73 mmhg pCO2 31 mmHg Most likely explanation for this pt's symptoms?

*Congestive heart failure* Exacerbation of CHF - pt's ho CAD = risk for HF via left ventricular dysfunction - on pulm exam --> pt's with worsening CHF = bibasilar crackles - decreased breath sounds at bases could be due to pleural effusions from CHF - wheezing can be occasionally present in HF (cardiac asthma) - blood gas = hypoxia, hypocapnia, resp alkalosis = CHF

54 yo man being evaluated for SOB & cough. His med probz include RA, HT, and ho DVT & PE. He takes multiple meds & has smoked for 30 yrs. BMI is 34 kg/m^2. Exam shows dullness to percussion over right lower lobe. Breath sounds are increased, especially during expiration, over right lung base compared to left. Cardiac exam shows regular rate & rhythm with normal S1 and S2. No murmur. Moderate peripheral edema present. Most likely cause of this pt's SOB?

*Consolidation of lung* SOB + cough + dullness to percussion with increased breath sounds over right lower lung field = lobar pneumonia --> focal lung consolidation - more rapid sound conduction = increased tactile fremitus + egophony in areas of lung consolidation - crackles often heard

76 yo man with multi-infarct dementia being evaluated for cough and low-grade fever. He was treated for pneumonia twice in last yr. For past 6 mos, he has had difficulty swallowing and occasionally regurgitates undigested food. Pt has long hx of HT and chronic atrial fibrillation. His BMI is 22 kg/m^2. His temp is 101.3 F, BP is 150/90 mmHg, pulse is 102/min, and respirations are 16/min. PE notable for foul-smelling breath and fluctuant mass in left neck. Auscultation shows crackles in right lung base. Chest x-ray shows infiltrate without cavitation in right lower lung field. Pt admitted, sputum and blood cultures sent, and abx started. Most appropriate next step in his management?

*Contrast esophagram* - will clearly show diverticulum Zenker's diverticulum - common in elderly, particularly men - posterior lower cervical esophagus near cricopharyngeus muscle - dysphagia & regurgitation - halitosis secondary to pooling of material in diverticulum - large = palpable - at risk for aspiration pneumonia

A 45-year-old female presents to the emergency room after an episode of hemoptysis. On review of systems, she has had a chronic fever, chronic sinus infections, weight loss, and urine that looked a little "bloody." What is the best treatment regimen to start in this patient?

*Corticosteroids and cyclophosphamide* Granulomatosis with polyangiitis (GPA) / Wegener's - first-line treatment option is corticosteroids and cyclophosphamide - chronic sinusitis is most classic for granulomatosis with polyangiitis

Inherited dz causing recurrent renal stone formation. Personal hx of recurrent kidney stones from childhood & positive fam hx. Urinalysis shows hexagonal crystals. Urinary cyanide-nitroprusside test used as qualitative screening procedure.

*Cystinuria* = *abnormal acid transport abnormality* - impaired transport of cystine and dibasic amino acids: ornithine, lysine, arginine by brush borders of renal tubular and intestinal epithelial cells --> decreased reabsorption (increased urinary excretion) of cystine = poorly soluble in water --> cystine stones = hard & radiolucent Cyanide-nitroprusside test - can detect elevated cystine levels --> can help confirm diagnosis - helpful to detect individuals who are homozygous for mutations = impaired amino acid transport

A 60-year-old man presents to your clinic one month following partial gastrectomy for a peptic ulcer. He complains of postprandial abdominal pain, nausea, vomiting, and diarrhea. He also complains of flushing and shortness of breath. The patient does not smoke cigarettes or consume alcohol. His medications include omeprazole and atorvastatin. Physical examination reveals increased bowel sounds and mild abdominal distention. The patient's postsurgical scar is healing well. Which of the following is the most appropriate treatment for this patient

*Decrease carbohydrate intake* Dumping syndrome - complication of both gastric bypass and sleeve gastrectomy - Early symptoms are divided into GI complaints (abdominal pain, diarrhea, borborygmi, bloating, and nausea) and vasomotor complaints (flushing, palpitation, perspiration, tachycardia, hypotension, and even syncope) and late symptoms include hypoglycemia, perspiration, hunger, fatigue, and syncope - Diagnosis is based on clinical assessment and a modified oral glucose tolerance test. - *Patients are advised to eat more frequently with smaller portions and to avoid drinking during meals. Intake of carbohydrates should be reduced.* - Acarbose is administered to patients who fail dietary management.

75 yo man comes to clinic due to 6 mo ho periodic substernal chest pressure, which he experiences when walking uphill or climbing 2 flights of stairs. His med hx is significant for hyperlipidemia and he takes atorvastatin. Pt smokes pck of cigz a day and occasionally consumes alcohol. BP is 120/78 mmHg and pulse is 75/min. PE shows no abnormalities. Resting ECG normal. Treadmill stress test shows horizontal ST-segment depression in leads II, III, and aVF at 73% of maximally predicted HR. Echocardiography demonstrates normal resting left ventricular systolic function. Pt prefers med management & is started on aspirin and as needed sublingual nitroglycerin; third med prescribed to help prevent his symptoms. Third med most likely acts through what primary mechanism?

*Decreased myocardial contractility* CAD + stable angina - via mismatch of myocardial oxygen supply & demand - *3 main med classes for prevention of stable angina symptoms = beta blockers, CCBs, long-acting nitrates* Beta-blockers - first-line - reduce myocardial oxygen demand via decrease in HR & myocardial contractility

67 yo woman brought to office by her son, who visits her infrequently. He reports that his mom has had periodic confusion, memory loss, and poor sleep and seems "kind of out of it sometimes." These symptoms have gradually worsened over past 1-2 yrs. Occasionally, pt tells her son that there are "strangers in the backyard," who are no there when he looks for them. More recently, she has begun walking more slowly and has fallen twice in the past mo without any significant injuries. Pt has ho HT & HERD as well as a remote ho postpartum psychosis. She is a nonsmoker and doesn't drinks alcohol. Pt denies any ho illicit drug use. Her meds include atenolol and omeprazole. Temp is 98 F, BP is 120/60 mmHg, pulse is 70/min, & respirations are 14/min. On PE, she has mild bilateral hand tremors and mild bilateral lower limb rigidity. She walks slowly. On cognitive exam, pt appears to be oriented to person & place, can state the days of the wk forward but doesn't cooperate with stating them backwards, and can recall 1 of 3 items in 5 minutes. Chemistries, CBC, vit B12, & thyroid function tests are normal. Serum treponemal test nonreactive. Urinalysis normal. Chest x-ray normal. MRI of brain shows mild generalized cortical atrophy. Most likely diagnosis in this pt?

*Dementia with Lewy bodies* - 1-to-2-yr ho periodic confusion + visual hallucinations + parkinsonian motor symptoms - 10-20% of dementias in US - alterations in consciousness, fluctuations in cognition, visual hallucinations, parkinsonism, relatively early compromise of executive functions - repeated falls & sleep disturbance = characteristic - Lewy bodies = eosinophilic intracytoplasmic = accumulations of alpha-synuclein protein = may be seen in neurons of substantia nigrans, locus ceruleus, dorsal raphe, substantia innominata - pharmacotherapy of DLB = cholinesterase inhibitors - if psychotic symptoms persist --> low-dose, second-generation antipsychotics via severe neuroleptic sensitivity seen in pts

Characterized by fluctuating cognitive impairment, recurrent visual hallucinations, parkinsonism.

*Dementia with Lewy bodies* - fluctuating cognition - bizarre, visual hallucinations - *parkinsonism with severe sensitivity to potent dopamine antagonists* (first-gen antipsychotics & risperidone)

72 yo man comes to ED due to 24 hr ho progressive lower abdominal discomfort and difficulty voiding. Pt has never had urinary difficulty in past. He has ho HT, ischemic stroke with mild left-sided residual weakness, & recent episodes of abdominal shingles. Pt also reports several days of non-productive cough and has been taking OTC diphenhydramine for 2 days. He is a former smoker and doesn't drink alcohol. Temp is 98 F, BP is 150/80 mmHg, & pulse is 105/min. Pt appears restless. Bilateral breath sounds normal with no added sounds. Cardiac exam reveals regular rhythm. Previous areas of shingles on abdominal wall has no active lesions but area is hyperesthetic. There is suprapubic fullness and tenderness. Mildly enlarged, nontender prostate palpated on rectal exam. Most likely cause of this pt's current condition?

*Detrusor hypocontractility* - impaired detrusor muscle contraction Acute urinary retention via adverse effect of diphenhydramine - elderly pt with difficulty voiding + palpable abdominal mass (overfilled bladder) - first-gen H1 antihistamines = diphenhydramine, chlorpheniramine, hydroxyzine --> sign anticholinergic effects = at muscarinic receptors of parasympathetic nervous system --> dryness of eyes, oral mucosa, resp passages; urinary retention

As part of prescription drug misuse monitoring, doc performs random urine drug screen that is positive for opioids and PCP. Pt insists that he is not taking PCP. What med could be causing a false-positive results on his urine drugs screen?

*Dextromethorphan* Standard urine drug screen (UDS) = amphetamines, cocaine, cannabis, opioids, PCP - usually performed by immunoassay --> can yield false-positive results when specific cross-reacting meds or drugs present *Meds that can cause false-positive PCP results: diphenhydramine, doxylamine, ketamine, tramadol, venlafaxine* - gas chromatography/mass spectrometry = highly sensitive & specific -- can be used to check accuracy of standard UDS - semi-synthetic (hydrocodone, hydromorphone, oxycodone) & synthetic (fentanyl, meperidine, methadone, tramadol) opioids --> don't trigger positive UDS

22 yo man comes to office due to left scrotal mass that he first noticed a few mos ago. Pt reports scrotal heaviness and an increase in size of mass after prolonged standing at his job as a supermarket cashier. His symptoms improve when lying down. Pt plays basketball when not working and doesn't remember any obvious scrotal trauma. He is sexually active with his gf and uses condoms consistently. PE shows irregular, soft mass in left scrotum above and separate from left testis. Mass doesn't transilluminate and increases in size during Valsalva. Most likely cause of this pt's condition?

*Dilation of pampiniform plexus* Varicocele - tortuous dilation of pampiniform plexus of veins surrounding spermatic cord & testis - 20% of postpubertal males - soft, irregular mass (bag of worms) that increases in size with standing and Valsalva - can cause elevated scrotal temps = increased risk for infertility and testicular atrophy - more common on left side - left renal vein vulnerable to compression beneath SMA (nutcracker effect) --> increased pressure in spermatic vein, incompetence of valves, retrograde blood flow, venous dilation

Lesion of what lobe of the brain can affect comprehension (receptive aphasia), ability to speak nouns (anomic aphasia), repetition (conductive aphasia) due to arcuate fasciculus inv't, and contralateral superior homonymous quadrantanopsia due to inferior optic radiations (Myer's loop) inv't.

*Dominant temporal lobe* lesion - pts usually have intact expressive speech, motor, primary sensory functions

64 yo man with long ho uncontrolled HT comes to ED with chest pain for last 12 hrs. He has never been hospitalized before. ECG shows normal sinus rhythm with ST-segment elevation in anterior leads. Coronary angiography shows complete occlusion of proximal LAD and no significant dz in other coronary arteries. No intervention performed and pt started on appropriate medical therapy. Next day he reports left leg pain. On exam, left leg is cold with mottled appearance. Minimal swelling with absence of distal pulses. Vascular surgery consulted. What should also be considered in this pt?

*Echocardiogram* Acute limb ischemia from arterial occlusion - typically via cardiac emboli, thrombosis, or trauma Large anterior STEMI = highest risk of LV thrombus & anteroapical aneurysm formation - LV EF <40% - high risk for systemic embolization (stroke, peripheral arterial occlusion) - require immediate anticoagulation & vascular surgery evaluation - *transthoracic electrocardiogram with echo contrast must be performed to screen for LV thrombus*

A 23-year-old G1P0 female at 11 weeks gestation presents to her obstetrician's office for a prenatal visit. Past medical history is unremarkable. The patient's medications include prenatal vitamins. She does not smoke and has not used alcohol since learning of her pregnancy. Ultrasonography shows a gestational sac with an absence of fetal heart tones. Pelvic examination demonstrates a cervix that is long, closed, and posterior. Which of the following is the most appropriate next step in the management of this patient?

*Elective dilation and curettage* Missed abortion - fetal demise without cervical dilation or vaginal bleeding - Ultrasonography showing an absence of fetal heart tones is diagnostic - In a hemodynamically stable woman without evidence of bleeding, treatment options include elective D&C, pharmacologic abortion (with misoprostol or mifepristone), or expectant management *Illustration A is an ultrasound of the uterus showing an empty gestational sac, indicative of intrauterine fetal demise*

60 yo man brought to ED by his daughters due to 2 day ho confusion & lethargy. According to daughter, he has had constant dry cough, fatigue, anorexia, polyuria, and constipation for several weeks. In addition, pt has los 20 lb over past 3 mos. He has no associated pain. Pt's med hx significant for HT treated with chlorthalidone and for GERD treated with OTC antacids. He smokes 2 pcks of cigz daily & consumes alcohol occasionally. BP is 130/90 mmHg & pulse is 90/min. Temporal wasting noted. Chest & abdominal exams normal. MME shows somnolence and disorientation to time. Neuro exam shows decreased deep tendon reflexes. Lab results: Serum chemistry - Sodium 140 mEq/L - Potassium 4 mEq/L - Chloride 104 mEq/L - Bicarbonate 24 mEq/L - Creatinine 1.6 mg/dL - Calcium 14.4 mg/dL - Glucose 100 mg/dL Alkaline phosphatase, serum 130 U/L Phosphorous (inorganic), serum 2.2 mg/dL Most likely cause of this pt's symptoms?

*Elevated parathyroid hormone-related protein* polyuria & neuropsych symptoms = hypercalcemia weight loss + temporal wasting + dry cough + smoking hx = *humoral hypercalcemia of malignancy (HHM)* - via squamous cell carcinoma of lung - paraneoplastic syndrome via release of parathyroid hormone-related protein (PTHrP) be malignant cells = structurally similar to parathyroid hormone --> acts on PTH-1 receptor - PTHrP --> increased bone resorption & reabsorption of calcium in distal renal tubule - increased phosphate excretion --> hypophosphatemia - severe = >14 mg/dL - rapid-onset hypercalcemia - diagnosis often suspected on clinical grounds - can be confirmed by elevated PTHrP level & suppressed PTH level

A concerned mother brings her previously healthy 5 year child to the family care physician after 3 mornings of significant swelling around the child's eyes and feet. The mother is concerned since she feels that the child has not been urinating at his normal frequency, and that his urine has been "thick and frothy." The physician requests a 24hr urine protein collection which reveals nephrotic range proteinuria. What is the next step in the management of this patient?

*Empiric steroid therapy* Minimal change disease - most common cause of nephrotic syndrome in children and is highly responsive to steroid therapy - often brought on after an infection - podocyte effacement in the glomerulus occurs causing protein and lipids to leak into the urine --> urine with a thick and frothy appearance secondary to increased protein concentration - lack of albumin in circulation results in a decrease in oncotic pressure that commonly causes swelling in the periorbital region and extremities

Pseudoachalasia, which is due to narrowing of distal esophagus secondary to causes other than denervation (esophageal cancer), can closely mimic achalasia. Clues pointing to pseudoachalasia include significant weight loss, rapid symptom onset, and presentation at age >60. Consequently, what is recommended to exclude malignancy in all pts with suspected achalasia?

*Endoscopic evaluation* - dysphagia to solid and liquids + dilated esophagus with smooth tapering of distal esophagus = either primary achalasia (loss of peristalsis in distal esophagus with lack of lower esophageal sphincter relaxation) or pseudoachalasia via esophageal cancer

Drugs that cause progressive decrease in impulse conduction with faster rates, leading to increase in QRS complex duration.

*Flecainide*, Propafenone: Class I antiarrhythmic drugs - block sodium channels - inhibit initial depolarization phase (phase 0) of action potential - used in treatment for atrial fibrillation (maintenance of sinus rhythm) - slowest rate of drug binding & dissociation from sodium channel receptor Use dependence - in pts with faster heart rates, drug has less time to dissociate form sodium channels --> higher number of blocked channels = progressive decrease in impulse conduction & widening of QRS complex - seen most frequently with class IC - mechanism behind their efficacy against supraventricular arrhythmias Class IC Na+ channel blockers *F*ries *P*lease Flecainide Propafenone - ↑ ERP in atrioventricular node but not in ventricular tissue

64 yo man brought to ED after motor vehicle collision. He was restrained driver when truck hit his car on driver's side at intersection. He was dazed for several minutes after collision and subsequently began to experience left chest & leg pain. His BP is 94/61 mmHg and pulse is 117/min. Pulse oximetry shows 96% on RA. Pt conscious and answers questions appropriately. He is wearing cervical collar. PE shows multiple facial lacerations & bruises on left chest wall. Pupils are equal and reactive to light. Heart sounds normal. Trachea midline. Bilateral breath sounds present. Abdomen diffusely tender. Obvious deformity and tenderness of his left thigh. What is the best immediate step in management of this pt?

*Focused bedside ultrasound* motor vehicle collision + diffuse abdominal tenderness = blunt abdominal trauma (BAT) = risk of intra-abdominal injury Signs of serious intra-abdominal injury - seat belt sign = ecchymosis over abdomen in pattern on seat belt; hypotension, rebound tenderness, abdominal guarding/distention, coexisting femur fracture First step after fluid resuscitation = determine if pt needs surgical management --> pts with BAT should be assessed for intraperitoneal free fluid or hemorrhage --> most commonly used approach = *Focused Assessment with Sonography for Trauma (FAST)* FAST - evaluates both abdomen and pericardium for evidence of organ injury or hemorrhage - first step in alert & hemodynamically stable (systolic BP >90) pts - can be performed rapidly at bedside

60 yo man comes to your office complaining of difficulty hearing for past few wks. He has T2DM, which is well-controlled by diet alone. His past med hx is also significant for essential HT, CHF secondary to diastolic dysfunction, and chronic renal failure. Meds include aspirin, diuretics, ACEI, and beta blocker. His pulse is 82/min, BP is 140/90 mmHg, and respirations are 14/min. Exam reveals hearing loss in both ears. What med is a potential cause of this pt's hearing probz?

*Furosemide* new-onset, bilateral hearing loss = med-induced ototoxicity - aminoglycosides - chemotherapeutics - aspirin - loop diuretics - aspiring usually causes tinnitus, but in very higher doses it can cause (6-8 grams/day) hearing loss --> this pt more likely = loop diuretics

Patient is suffering an episode of acute decompensated heart failure. The patient's history of ischemic cardiomyopathy and CHF along with a negative troponin level point toward this diagnosis. This patient is volume-overloaded and requires a diuretic to decrease preload to his heart. What will decrease preload and is the best choice?

*Furosemide* - a loop-diuretic Acute decompensated heart failure - also known as acute congestive heart failure (CHF) exacerbation - In systolic heart failure, the heart cannot expel blood sufficiently, while in diastolic heart failure the heart cannot relax and fill with blood normally. - Systolic heart failure is demonstrated by a decreased ejection fraction. - presented with acute decompensated heart failure and received 20 mg/hr IV furosemide on admission, followed by mitolazone and dobutamine on hospital day 2 in an attempt to increase diuresis. *Image shows the classic appearance of a chest radiograph of a patient with CHF exacerbation. Note the pulmonary edema and cardiomegaly typical of this disease.*

A 24-year-old male presents with 6 months of auditory hallucinations, paranoid delusions, and gross disorganization. Which of the following would favor a good prognosis for this man's condition?

*History of a ppting event* Schizophrenia associated with a better outcome if there is a history of a precipitating event. - can be a very disabling disease, often resulting in joblessness, homelessness, or suicide *Good outcomes are associated with a late and quick onset, and no family history of schizophrenia*

Pt presents with clinical presentation consistent with cellulitis with systemic manifestations. Most appropriate treatment?

*IV cefazolin* - Cellulitis: most common organisms are Staphylococcus and Streptococcus. - Either cefazolin, a 1st generation cephalosporin, or nafcillin, a penicillinase-resistant penicillin, is typically used to provide appropriate Gram-positive coverage.

The patient in this vignette most likely has necrotizing otitis externa (NOE), which is an life-threatening infection requiring admission and

*IV ciprofloxacin* - for Pseudomonas coverage.

Recognize the EKG of an alcoholic with hypokalemia, in which the treatment is...

*IV magnesium and potassium replacement*

In a large population with almost negligible growth rate and minimal migration, the incidence of T2DM is 3 cases per 1,000 individuals a year, and has been stable for last 30 years. However, prevalence of T2DM has increased progressively over same period. Most likely explanation for this trend over time?

*Improved disease management* Prevalence = Incidence x Duration of disease Rising prevalence + stable incidence of diabetes - more ppl are being documented as having diabetes although # of ppl being newly diagnosed each year remains same - stable incidence --> increasing prevalence likely via prolonged duration bc of improvements in quality of care & dz management = increased life expectancy

53 yo man comes to office bc of difficulty reading fine print over last yr. He now has to hold books, menus, and magazines at an arms length in order to read them. He has never had visual probz before. Most likely abnormal in this pt?

*Lens elasticity* Presbyopia - common age-related disorder - via loss of elasticity in lens --> prohibits accommodation of lens (required to focus on near objects) - easily improved with reading glasses

A 51-year-old African American male presents to a new primary care physician for the first time after moving to a new city. The patient has no history of medical problems. Family history is remarkable for his father's death from prostate cancer at age 70. Which of the following is true regarding prostate cancer screening?

*Many prostate cancers remain asymptomatic* - Prostate cancer screening with digital rectal exam and prostate specific antigen (PSA) have been shown to reduce mortality from prostate cancer. However, these screening methods also detect many prostate cancers that, if left alone, would cause no symptoms and progress so slowly that the patient would die of other causes. - Unfortunately, there is no way to determine which prostate cancers are aggressive and which are relatively harmless. Over-treatment and complications of treatment result. - Most men receive PSA testing not as a matter of routine but after presenting with prostate-specific symptoms. Other values such as PSA density, age-related PSA, or PSA velocity can provide further guidance as to whether a patient should undergo a prostate biopsy.

A 42-year-old male presents to the emergency department complaining of 2 days of depressed mood, muscle aches and spasms, and diarrhea. The patient reports a long history of IV drug use and says he drinks half a pint of vodka daily. Three days ago he had his wallet stolen and has since not been able to buy drugs or alcohol. Physical examination is notable for diaphoresis and dilated pupils. Temperature is 37.8 degrees Celsius. Which of the following is the most appropriate treatment for this patient?

*Methadone* - opioid agent used to stabilize patients suffering from withdrawal Individuals who abruptly withdraw from opioid use may see symptoms begin within 24 hours - Flu-like symptoms may be present, including nausea, diarrhea and stomach cramps, and patients often complain of joint and muscle aches - Though symptoms may be severe, they are not life-threatening - *First-line stabilization agent is methadone* Methadone programs have often reduced street heroin use less than expected due to suboptimal dosing and other deviations from evidence-based approaches

Pt presents with a localized rash consistent with contact dermatitis. The most common metal to cause this type IV hypersensitivity reaction is ....

*Nickel* - base metal of most costume jewelry. Contact dermatitis. - results from contact with an allergen - triggers include poison ivy, poison oak, nickel, detergents, cosmetics, rubber, and latex. - It often presents days to weeks after the initial exposure since it is a type IV hypersensitivity reaction. - On physical examination it appears as a pruritic, erythematous rash with vesicles at the site of exposure. - These rashes may become superinfected with skin flora when scratched resulting in pus-filled vesicles, thus scratching should be avoided.

Pts with cervical spine injuries require initial stabilization of cervical spine. What is preferred for establishing an airway in an apneic pt with cervical spine injury?

*Orotracheal intubation* with rapid-sequence intubation

A 3-year-old boy is brought into the pediatrics clinic for a scheduled well-child visit. According to his mother, he has had a history of "ear infections." The last infection was approximately one month ago. The child currently has no symptoms and does not complain of any ear pain. On physical exam, you note a cloudy eardrum that is hypomobile on pneumatic otoscopy. There is no erythema noted. What is the most likely diagnosis?

*Otitis media with effusion* This child is most likely presenting with otitis media with effusion, as noted by an absence of acute inflammation in the setting of a middle ear effusion. Otitis media with effusion (OME) - lack of acute inflammatory signs. - watchful waiting for three months to assess for spontaneous resolution

50 yo man comes to doc complaining of 2 mos of muscle cramps. He also complains of weakness and fatigue. He has 20 pck yr smoking hx but does not drink alcohol. Pt's past med hx and fam histories are noncontributory. His temp is 98 F, BP is 150/96 mmHg, pulse is 78/min. His BMI is 28 kg/m^2. Rest of exam within normal limits. Lab results: Plasma sodium 147 mEq/L Potassium 2.6 mEq/L Serum creatinine 0.8 mg/dL Most appropriate next step in management of this pt?

*Plasma renin activity and aldosterone concentration* Primary hyperaldosteronism - *should be suspected in any hypertensive pt presenting with hypokalemia - prone to develop diuretic-induced hypokalemia - severe hypokalemia --> muscle cramps/weakness - other findings = metabolic alkalosis and mild hypernatremia (143-147 mEq/L) - ratio of plasma aldosterone concentration to plasma renin activity preferred initial screening test

75 yo AAM comes to office for his annual check up. He is a known diabetic & hypertensive. His meds include lisinopril and atenolol. His vitals are normal. Exam of his fundus reveals cupping of optic disc. Visual field exam reveals constricted peripheral vision. Most likely diagnosis?

*Primary open angle glaucoma* - usually asymptomatic in earlier stages - more common in AAs - gradual loss of peripheral vision over years --> eventually = tunnel vision - intraocular pressure is high - cupping of optic disc with loss of peripheral vision - beta-blockers like timolol eye drops = initial management - if continuous increase in intraocular pressure --> surgical trabeculectomy

Pt taking isoniazid. Microcytic/Hypochromic anemia stimulating iron-deficiency anemia, but iron studies reveal elevated serum iron level and decreased TIBC. Next step in management?

*Pyridoxine* Sideroblastic anemia - via defective heme synthesis via pyridoxine-dependent impairment in early steps of protoporphyrin synthesis - in pts with identifiable cause of vit B6 def (alcoholism, drugs), admin of pyridoxine can easily correct prob

Should be suspected in all pts with resistant HT and diffuse atherosclerosis, asymmetric kidney size, recurrent flash pulmonary edema, or elevation in serum creatinine >30% from baseline after starting ACEI or ARB. *Presence of continuous abdominal bruit has high specificity for presence of this.*

*Renovascular HT* - resistant HT = persistent HT despite using >3 antiHT agents of different classes (one being a diuretic) --> should be evaluated for 2nd cause - onset of severe HT >55 yo - 1% of pts with mild HT - 45% of white pts with severe HT & 25-35% of pts with peripheral artery dz

80 yo woman comes to office due to progressive LE edema over last 2 mos. Her mobility is impaired due to knee OA, but recently she has become somewhat tired and SOB with daily activities. Her stomach feels full and her appetite has been decreased. Pt has had no forgetfullness or mood probz. BP is 140/75 mmHg & pulse is 70/min & regular. Jugular veins distended with pts in seated position. Abdomen distended with flank dullness on percussion. 3+ peripheral edema. Lab results: Hemoglobin 11.2 g/dL Sodium 132 mEq/L Creatinine 0.9 mg/dL Urine protein excretion 1 g/24 hr. Echo reveals left atrial enlargement, marked concentric left ventricular hypertrophy, and left ventricular ejection fraction of 70%. Most likely cause of this pt's symptoms?

*Restrictive cardiomyopathy* Amyloidosis --> fibril deposition in tissues Cardiac amyloidosis = form of restrictive cardiomyopathy - suspect in pts with CHF with echo findings of concentric LV hypertrophy & nondilated LV cavity, especially in absence of hx of HT - rt heart failure symptoms tender to predominate - LV wall thickness increases (via fibril deposition) --> LV cavity size decreases & restrictive physiology develops - atrial enlargement = common = cardiac abnormalities - important cause of HR with preserved ejection fraction - proteinuria + nephrotic syndrome + waxy skin + anemia + easy bruising with ecchymosis + hepatomegaly + GI bleeding + early satiety + subcutaneous nodules + enlarged tongue + peripheral or autonomic neuropathy

35 yo AAF comes to clinic due to exertional SOB. Pt's exercise tolerance a year ago was walking 7 blocks but now she walks less than a block before becoming SOB. She also feels lightheaded when she forces herself "to go on." Pt denies any episodes of syncope, chest pain, cough, wheezing, orthopnea, nausea, or LE edema. She has been treated for Raynaud phenomenon and fingertip ulcerations for last 2 yrs. She also suffers from severe heartburn controlled with high-dose pantoprazole. Fam hx not significant. She doesn't smoke cigz or consume alcohol. BP is 112/72 mmHg and pulse is 94/min. Pulse oximetry shows 95% on RA. BMI 23 kg/m^2. What would be most expected finding on physical exam?

*Right ventricular heave* CREST syndrome - strongly associated with limited cutaneous systemic sclerosis - AA pts most commonly affected by SS SS - vascular dysfunction = prominent component of dz - PAH common Pulmonary arterial hypertension (PAH) - intimal hyperplasia of pulmonary arteries - typically presents with progressive dyspnea - exertional syncope or presyncope via right ventricular failure & inadequate venous return to left atrium - PE = RV heave / parasternal heave = impulse palpated immediately to left of sternum = RV enlargement - loud pulmonary component of second heart sounds & signs of right heart failure (peripheral edema, hepatomegaly) may be present

Patient presents with symptoms of major depression and auditory hallucinations consistent with major depression with psychotic features. Appropriate treatment includes a

*SSRI & atypical antipsychotic* - ex. fluoxetine & risperidone Psychotic depression = antidepressant + antipsychotic Catatonic depression = antidepressant + benzodiazapene Melancholic/atypical depression = consider MAOI (test answer often), but in practice usually first an SSRI For all of the above (and all treatment resistant depression), next line is ECT

Decrease abdominal pain and improves global assessment score in those with IBS

*SSRIs*, tricyclic antidepressants - ex. fluoxetine

45 yo man comes to doc due to epigastric pain & diarrhea. Past med hx significant for PUD. He has 20-pck-yr smoking hx but doesn't use alcohol or illicit drugs. PE shows abdominal tenderness without rebound or rigidity. Endoscopy shows prominent gastric folds, 3 duodenal ulcers, & upper jejunal ulceration. Most appropriate next step in management of this pt?

*Serum gastrin concentration* - fasting serum gastrin (off PPI therapy for 1 wk) should be checked in suspected gastrinoma --> level <110 pg/mL --> rules out - level >1000 pg/mL = diagnostic - elevated --> gastric pH should also be measured (gastrin may be elevated due to failure of gastric acid secretion / achlorhydria) recurrent PUD + multiple ulcers + jejunal ulceration = gastrinoma/Zollinger-Ellison syndrome - usually sporadic but found in conjunction with MEN-1 in 20% of cases - ages 20-50 yo + dyspepsia + reflux + abdominal pain + wt loss + diarrhea + frank GI bleeding

A 26-year-old woman tests positive for chlamydia. In what clinical scenario would you be prompted to treat her with amoxicillin?

*She is pregnant* The treatment of choice for Chlamydia in pregnancy is amoxicillin or azithromycin The typical first line agents for Chlamydial infections are doxycycline and azithromycin; however, doxycycline is contraindicated in pregnancy - Therefore, amoxicillin can be used

62 yo Caucasian man presents to your office with occasional ear pain and lump in his neck. His past med hx is significant for HT treated with HCTZ & DM treated with metformin. He smokes 2 pcks of cigz per day and consumes alcohol occasionally. He is not sexually active. PE reveals hard, non-tender submandibular mass that is 3 cm in diameter. Chest exam unremarkable. Abdomen soft & non-tender. Liver span is 8 cm & spleen is not palpable His extremities have no cyanosis, clubbing, or edema. CBC within normal limits. Most likely cause of this pt's complaint?

*Squamous cell carcinoma* Hard, unilateral, non-tender lymph nodes = cancer Older pt + smoking hx + lymph nodes in submandibular or cervical region = head & neck cancer - vast majority = SCC --> should undergo prompt biopsy to further evaluate mass

A 12-year-old boy scout returns from a weekend camping trip and develops linear, erythematous plaques, with occasional vesicles on his lower legs. His pediatrician diagnoses dermatitis following contact with poison ivy. What is the underlying immunologic process causing the skin findings?

*T cell mediated response* Contact dermatitis following poison ivy exposure is the result of a delayed type IV hypersensitivity reaction. - This process is mediated by sensitized T cells, which release cytokines following exposure to an antigen. - Following a prior exposure to an antigen, CD4 lymphocytes are sensitized and the antigen is presented via type II major histocompatability complexes (MHC). - Upon a second antigen exposure, CD4 cells release IL-2 and other cytokines that leads to the activation of macrophages and the production of a local inflammatory response. - This process results in the characteristic skin findings approximately 2-3 days following exposure. - Unlike other hypersensitivity reactions, it is the only one that is cell mediated rather than antibody mediated.

A sexually active 25-year-old woman comes to your office desiring a new contraceptive. After seeing an advertisement in a magazine, she asks you about implantable contraceptives. Which of the following is a true statement regarding this form birth control?

*They have the lowest failure rate of any nonpermanent contraceptive methods* Implantable and injectable contraceptives have the lowest failure rate of any non-permanent method and can remain implanted for 3-5 years. - They lead to irregular bleeding and typically have a return to fertility by 18 months but can be difficult to remove secondary to scarring. Implantable contraceptives - attractive choice for women who want a very effective agent and/or cannot reliably take oral contraceptive pills - slow release of progestins that can thicken the cervical mucous to impair the movement of sperm

An 11-year-old boy presents to general pediatrics clinic for a regular health visit. He had been doing poorly with school. His teachers reported that his attention span was very limited and that he had trouble controlling his impulses. If the patient was started on a first line medication for his trouble at school, what would be another possible indication for this drug class?

*Treatment of narcolepsy* Being treated with methylphenidate (Ritalin) for attention-deficit hyperactivity disorder. This drug may also be used to treat narcolepsy. Attention-deficit hyperactivity disorder (ADHD) - *Treatment centers around methylphenidate (Ritalin), amphetamines (Dexedrine), or atomoxetine (an SNRI)* - *Stimulants used in treatment of ADHD may also be used for appetite control as well as treatment of narcolepsy.* Narcolepsy - *Classic compounds used to treat narcolepsy include methylphenidate, clomipramine and newer agents such as modafinil, venlafaxine and sodium oxybate.*

A 65-year-old woman with a long history of constipation presents with steady left lower quadrant abdominal pain. Physical exam reveals low grade fever, mid-abdominal distention, and lower left quadrant tenderness. Stool guaiac test is negative. An absolute neutrophilic leukocytosis and a left shift are noted on laboratory blood tests. She is placed on broad spectrum oral antibiotics, bowel rest, and pentazocine for pain. Her symptoms do not seem to improve over the next day, and the decision is made to proceed to sigmoid colectomy with primary resection and anastamosis. The procedure is uncomplicated, and she is recovering as expected until noted to have a temperature of 101 F on post-operative day 3. Which of the following is the most like reason for this patient's fever?

*UTI* *Fever on post-operative day 3 is most likely caused by a urinary tract infection following foley catheterization* 5w's of post-surgical fever Wind---pneumonia, atelectasis at 1st 24- 48 hours Water---urinary tract infection at Anytime after post op day 3 Wound---wound infections at Anytime after post op day 5 Wonderdrugs---especially anesthesia Walking---walking can help reduce deep vein thromboses and pulmonary embolus usually occurs at Day 7-10

A 68-year-old gentleman with a history of schizophrenia has been taking haloperidol for approximately four weeks. If he were to have an adverse reaction to this medicine, with what signs and symptoms would he most likely present?

*Urge to move and restlessness* Akathisia - restlessnes - onset approximately four weeks after starting the medication. Typical antipsychotic medications, such as haloperidol, have a group of common side effects known as extrapyramidal symptoms (EPS) - caused by the blockade of dopamine receptors by antipsychotic medications. - Haloperidol is the most common antipsychotic to elicit EPS, especially when used for schizophrenia. - The EPS include, acute dystonic reactions (four hours), pseudoparkinsonism (four days), akathisia (four weeks), and tardive dyskinesia (four months).

Recognize EKG with right axis deviation which would be seen with right ventricular hypertrophy in pulmonary hypertension, in which this patient presenting with symptoms suggestive of pulmonary hypertension should be treated with...

*bosentan* - reduces the growth of the pulmonary vasculature - addresses the underlying pathophysiology in pulmonary hypertension - endothelin inhibitor Pulmonary hypertension - middle-aged female with shortness of breath, weakness, fatigue, JVD, and hepatomegaly Right-sided heart strain - can occur and results in right axis deviation on EKG and increased right ventricular wall thickness on ultrasound

This patient's clinical presentation is consistent with major depression. While most selective serotonin uptake inhibitors (SSRIs) can cause sexual dysfunction, what is a norepinephrine and dopamine reuptake inhibitor that does not have this side effect?

*bupropion* -Useful in depression -No significant weight gain -Useful in patients trying to quit smoking -No sexual side-effects (unique and relevant to this case) - anorexia is a risk factor when combined with buproprion as the seizure threshhold is lowered (and this is important for boards purposes)

A 30-year-old male presents to the emergency room with a laceration over the 4th metacarpophalangeal (MCP) joint of his right hand. The wound is shown in Image A. He reports that he is a mailman, and his closed fist was bitten by a dog while he was delivering mail. The patient is allergic to penicillin. Which of the following is the most appropriate treatment for this patient?

*clindamycin & doxycycline* Amoxicillin-clavulanate is the usual first line antibiotic for patients with a human or dog bite. *In patients with a penicillin allergy, clindamycin plus doxycycline or clindamycin plus TMP-SMX is appropriate.* Clenched fist injury (CFI) - generally kept open (i.e. not sutured). - Amoxicillin-clavulanate (Augmentin) is first-line therapy for prophylaxis and treatment of human bites, dog bites, and CFIs in general - put the patient at a high risk for osteomyelitis, septic arthritis, and tenosynovitis; therefore, prophylactic antibiotics are appropriate. - Amoxicillin-clavulanate provides essential coverage against P. multocida, commonly found in the mouths of dogs. - *In patients with a penicillin allergy, choose an antibiotic with coverage against P. multocida (such as TMP-SMX, doxycycline) paired with anaerobic coverage (clindamycin or metronidazole).* Prevention and treatment of dog bites. - Appropriate treatment consists of immediate, copious irrigation, assessment for risk of tetanus and rabies, and administration of prophylactic antibiotics as discussed above. - Because infection of the joint capsule, the extensor tendon, or the deep fascial spaces may occur resulting in significant morbidity, physicians should have a high index of suspicion for these injuries and treat them aggressively with antibiotics.

A five-year-old girl comes to the emergency department crying, holding her stomach and coughing up pinkish sputum. Her father frantically reports to you that she "swallowed something" at home about two hours ago. After appropriate airway and hemodynamic stabilization, you would next perform esophagogastroduodenoscopy (EGD) if the father tells you that the patient had ingested:

*drain cleaner* Ingestion of caustic fluids (acid or alkali) such as drain cleaner may lead to esophageal damage and stricture. - Airway patency must be established first; then the extent of esophageal damage should be examined by EGD when the patient is stable, typically within 24-48 hours of injury. Injury due to ingestion of alkaline fluids such as drain/oven cleaner or perm relaxers occurs rapidly in the first minutes to hours and is characterized by liquefactive necrosis of the esophageal tissue. - Subsequently esophageal strictures form due to scarring of the affected tissue. - The patient should NEVER be given acids such as vinegar, nor should gastrointestinal lavage be performed, as both are likely to lead to further tissue damage. Illustration shows findings from endoscopy in the fundus of the stomach after caustic ingestion.

The patient in this vignette suffers from a case of intractable epistaxis. It is most likely an anterior bleed; posterior bleeds are mainly seen in patients with hypertension, history of cocaine abuse, recent nasal surgery, or nasal tumors. After nostril pinching and topical vasoconstrictor, the next step is the treatment of this patient is

*electrocautery of Kiesselbach's plexus*

This patient has a closed, minimally displaced fracture of the middle third of his left clavicle. Appropriate treatment is immobilization in a

*figure-of-eight immobilization device* Closed, minimally displaced fracture of the middle third of his left clavicle - Appropriate treatment is immobilization in a figure-of-eight immobilization device - occur as a result of either direct trauma or a fall on an outstretched arm and present with guarding of the shoulder and tenderness over the clavicle - Diagnosis is confirmed with radiographs, and the vast majority are treated with immobilization in a traditional arm sling or figure-of-eight sling (see illustration B) *Figure is an A/P radiograph demonstrating a minimally displaced fracture of the middle third of the left clavicle*

Presence of "succussion splash" can indicate...

*gastric outlet obstruction* - initial management = nasogastric suctioning to decompress stomach, IV hydration, endoscopy for definitive diagnosis Abdominal succussion splash - elicited by placing stethoscope over upper abdomen and rocking pt back and forth at hips - retained gastric material >3 hours after meal --> splash sound = presence of hollow viscus filled with fluid and gas

It can be a very perplexing case when a seemingly normal patient presents with hypokalemia that is refractory to treatment. In patients such as these, it is critical to check for and treat

*hypomagnesemia* The hypokalemia will likely not correct until the low Mg levels are treated. - This is a boards favorite for Step 2 where the next step in management would be to check or replete Mg levels, or in Step 3 where you will need to specifically order the Mg level and then treat the condition.

Obstructive sleep apnea is a condition cause by ...

*intermittent upper airway collapse* - transient obstruction of upper airway due to laxity of pharyngeal tissue = nocturnal hypoventilation - daytime sleepiness, snoring, brief choking or gagging sensation while sleeping

Pts with upper GI bleeding who have depressed level of consciousness and ongoing hematemesis should be...

*intubated* - to protect airway as part of initial stabilization & resuscitation - prompt endoscopic treatment with ligation or sclerotherapy should then be performed to stop bleeding

female pattern hairloss presents with parietal hair thinning iwth preservation of frontal hairline. treatment?

*minoxidil* - regrowth of hair in female pattern - finasteride = male-pattern hair loss treatment

What findings have same pathogenesis of spider angiomas?

*palmar erythema*, gynecomastia, testicular atrophy, decreased body hair - via hyperestrinism

This patient presents with isolated thrombocytopenia with no other clear causes for this abnormality; therefore, the most likely diagnosis is immune thrombocytopenic purpura (ITP). Given his symptoms and low platelet count, what is the best initial step in management?

*prednisone*

What is most likely to be seen with diastolic dysfunction?

*preserved ejection fraction* >50% - older population - dyspnea - non-dilated left ventricle - structural heart disease - filling problem

In a patient with atrial fibrillation or tachycardia secondary to hyperthyroidism, the initial appropriate treatment in a stable pt is...

*propranolol* Even without the tachycardia, there are no noticeable P waves and the rhythm is irregularly irregular. This is extremely suggestive of atrial fibrillation.

Recognize *new onset atrial fibrillation,*in which the most likely site of origin for ectopic foci is the

*pulmonary veins* Atrial fibrillation - most common chronic arrhythmia - supraventicular arrhythmia that causes an irregularly irregular rhythm - Some risk factors include COPD, pulmonary embolism, chest surgery, myocardial ischemia, cardiomyopathy, pericarditis, hypertension, atrial septal defects, mitral or aortic valve abnormalities, atrial myxoma, hyperthyroidism, alcohol, and sepsis - Presentation can be asymptomatic, but common symptoms include dyspnea, palpitations, chest pain, or syncope. - Hemodynamically stable patients should be rate-controlled with beta-blockers, diltiazem, or digoxin (in order of preference) - If the patient has been in atrial fibrillation for less that 48 hours, cardioversion can be attempted without anticoagulation - If the arrhythmia had occurred for more than 48 hours or the duration is not known, anticoagulation should be achieved before cardioversion, unless left atrial thrombus has been ruled out by transeosophageal echocardiogram (TEE).

Initial management of frostbite is based on...

*rapid rewarming* of affected tissues in 37-39 C (98.6 - 102.2 F) water bath

Patient most like has a threatened abortion as evidenced by the closed cervix and small amount of vaginal bleeding. The most appropriate management for threatened abortions is

*reassurance with outpatient follow-up* Threatened spontaneous abortion - defined as a normal ultrasound with minimal bleeding and NO cervical dilation - vaginal bleeding and pain in first half of pregnancy is presumed to be threatened abortion unless another diagnosis can be made

Complication of hypoparathyroidism?

*refractory heart failure* tetany seizures stridor altered mental status - via low calcium

After a diagnosis of otitis externa has been made, the most appropriate step is to ...

*remove any cerumen in the ear canal that may act as a nidus for continued bacterial growth* The most common bacterial offenders in otitis externa are (1) Pseudomonas aeruginosa and (2) Staphylococcus aureus.

Penile fracture is a urologic emergency and requires urgent operative repair. Most common cause is bending injury of the erect penis, frequently during sexual intercourse. Pts with evidence of urethral injury (blood at meatus, dysuria, urinary retention) should undergo...

*retrograde urethrogram*

A 56-year-old former intravenous drug user presents for his routine appointment at the infectious disease clinic. He was diagnosed 5 years ago with hepatitis C and is followed closely. He has had multiple liver biopsies that have yet to show cirrhotic changes. His current liver function tests (LFTs) are: AST: 25 IU/L and ALT: 40 IU/L. What is the most appropriate next step in the management of this patient?

*sofosbuvir* Chronic hepatitis C with normal liver enzymes and stable findings on biopsy - *Previously, no treatment would have been indicated due to extensive side effects; however, the development of new antiviral drugs such as sofosbuvir have shown great efficacy in clearing the virus with minimal side effects* - Hepatitis C becomes a chronic condition in 50-80% of the patients diagnosed in the acute setting. - Within the last few years the combination of sofosbuvir/simeprevir has been shown to cure >90% of patients and is becoming the standard of care. It is important to note that the board exams are currently adapting to this change so you should be familiar with both the old and the new treatment algorithms. - Development of polymerase inhibitors such as sofosbuvir in combination with protease inhibitors such as simeprevir have revolutionized treatment away from ribavirin/interferon based treatments.

A 23-year-old male presents to his primary care physician with complaints of fatigue and mild left cheek pain that started a day ago. On physical exam there is pain to palpation of the left maxilla. His temperature is 37.9 C. Which of the following is the most appropriate next step in the management of this patient?

*supportive care for 1 week and reassess* The initial management of acute sinusitis is to observe for 7 days, with antibiotics given only if there is no clinical improvement during observation or temperature exceeds 38.3 C. Rhinosinusitis - common medical condition with several subtypes: acute, subacute, recurrent acute, and chronic. Acute rhinosinusitis is further specified as bacterial or viral, of which viral is more common and is most often seen following viral upper respiratory infections. - Of note, radiographic imaging (radiograph or head CT) is not recommended in the evaluation of uncomplicated acute rhinosinusitis.

In patients with SLE, what can be used to monitor disease activity?

*the level of anti-dsDNA antibody* There is an especially strong correlation between IgG anti-dsDNA and glomerulonephritis in the context of SLE. Serum C3 and C4 levels are also used to track SLE disease activity.

CONTRAINDICATION to GLP-1 agonist / exenatide (Byetta)

*thyroid cancer* (medullary) / MEN - associated with pancreatitis in rare cases incretin mimetics (exanatide, liraglutide) - agonizes GLP-1 receptors --> decreases glucagon, increases insulin, delays gastric emptying - SC injection - adjunct to other drugs adverse rxns - mild wt loss - nausea - hypoglycemia - slight risk of pancreatitis

If a ruptured AAA is suspected and the patient is hemodynamically stable with SBP >90, what can confirm the diagnosis?

*urgent abdominal ultrasound or CT* - If a patient is hemodynamically unstable, he/she should be taken to surgery emergently USPSTF guidelines: https://www.uspreventiveservicestaskforce.org/Page/Document/UpdateSummaryFinal/abdominal-aortic-aneurysm-screening

Atypical antipsychotics act mainly on

5-H2A receptors - improves positive and negative symptoms of schizophrenia - less commonly associated with extrapyramidal symptoms than typical antipsychotics

Recognize butterfly glioma, which is *the most common malignant primary brain tumor*

Butterfly glioma with central necrosis = glioblastoma multiforme, the most common malignant primary brain tumor. Glioblastoma (a grade IV astrocytoma) - most deadly as well as the most common malignant primary brain tumor - prognosis is grave with median survival just over one year with surgical resection, chemotherapy, and radiation - most common location of these tumors is the cerebral hemispheres - due to their invasive nature, these tumors may cross the corpus callosum creating a butterfly type appearance - despite their invasive nature locally, these malignancies very rarely metastasize outside of the central nervous system. - biopsy reveals pseudopalisading, pleomorphic tumor cells with a central area of necrosis and hemorrhage - mainstay of treatment is temozolomide. - recurrences may be treated with bevacizumab (an anti-VEGF agent) - typical symptoms include persistent headaches, seizures, nausea, vomiting, neurocognitive symptoms, and personality changes

The patient is suffering from acute drug-induced pancreatitis. What is the most common cause of drug-induced pancreatitis?

Didanosine = anti-retroviral used in *HIV* Known to cause drug-induced pancreatitis: hydrocodone Premarin furosemide (commonly tested on the USMLE!) hydrochlorothiazide orthotricyclen. The drugs responsible for the most number of cases of drug-induced pancreatitis are: didanosine (HIV) (>800 cases) asparaginase (immunosuppressive) azathioprine (immunosuppressive) valproic acid (seizures, bipolar). Other commonly tested drugs that cause pancreatitis include: sulfasalazine 5-ASA pentamidine metronidazole tetracycline

This patient is most likely suffering from rhabdomyolysis following a prolonged loss of consciousness (and resultant pressure necrosis) secondary to a heroin overdose. What should be done next?

EKG - The resultant release of potassium from muscle cells may result in hyperkalemia and fatal cardiac arrhythmias, thus an EKG should be obtained immediately.

Can cause idiosyncratic liver injury with histological features similar to those seen in pts with viral hepatitis - liver biopsy: panlobular mononuclear infiltration and hepatic cell necrosis.

Hepatitis secondary to isoniazid usage

Characterized by *corneal vesicles and dendritic ulcers*

Herpes simplex keratitis - frequent cause of corneal blindness in US - pain + photophobia + blurred vision + tearing + redness - usually recurrences ppted by excessive sun exposure, outdoor occupation, fever, or immunodeficiency

Nontoxic-appearing febrile infants 29-90 days of age who have negative screening lab workup, including CBC with diff and normal urinalysis. Treatment?

Home care and reevaluation in 24 hours - can be sent home - empiric abx not necessary - any infant younger than 29 days --> sepsis workup

Recognize muscle biopsy of *polymyositis*

Polymyositis - inflammatory muscle disease of unknown etiology marked by slowly progressive proximal muscle weakness with or without muscle tenderness - confirmation of the diagnosis is made with muscle biopsy

Recognize seborrheic keratosis, in which the treatment is *no therapy*

SK - benign neoplasm that commonly appears on the extremities, face, and trunk of the elderly - presents as a flat, greasy, pigmented squamous proliferation that appears to be "stuck on" the skin -classic histological finding is keratin-filled cysts If many SK lesions appear within a short time, this is known as the Leser-Trelat sign - most commonly associated with adenocarcinomas of the gastrointestinal tract, especially gastric adenocarcinoma, but may also be associated with lung, breast, urinary tract, and lymphoid cancer

Recognize inferior wall infarction via *occlusion of right coronary artery* on EKG

ST segment elevations in inferior leads II, III, aVF with reciprocal ST depression in leads I & aVL = acute inferior wall MI ST-segment depression in leads V1 & V2 = posterior wall MI

All pregnancy women should be screened for asymptomatic bacteriuria when?

between 11 & 16 wks

Lithium treatment should not be started in patients with evidence of

renal insufficiency - increased creatinine Lithium - only mood stabilizer shown to decrease suicidality but must be managed appropriately - Every patient should have a basic metabolic panel prior to starting lithium therapy to assess baseline blood urea nitrogen and serum creatinine levels - *Any reduced renal function may result in increased serum lithium levels and could precipitate toxicity* - After commencing lithium therapy, serum levels should be routinely monitored due to lithium's narrow therapeutic index. Additionally, T-wave depression or inversion can be seen on EKG

The patient's down-and-out ocular finding, as well as nonreactive pupils suggests that compression is the cause of this

third-nerve palsy - reactive pupils suggests an ischemic etiology, such as in diabetics posterior communicating artery aneurysm - can classically cause this presentation (as well as uncal herniation) - why this is a neurosurgical emergency is due to the risk of rupture, leading to a subarachnoid hemorrhage

A 57-year-old man with a 40-pack-year history of smoking presents to general medicine clinic. He has diabetes and hypertension. He had bypass surgery for three vessel coronary artery disease. He also has a history of depression. He has been trying to quit smoking since his surgery over the past several months, but keeps relapsing. He reports that he was trying to quit using his will power. Vital signs are notable for blood pressure of 149/89. Physical examination is notable for an S4 on cardiac examination. To help him quit, you prescribe:

*A nicotine receptor antagonist that should be combined with nicotine replacement* Bupropion - antagonist of the nicotine receptor - should be combined with nicotine replacement and is a good medication for a patient who has comorbid depression - works by blocking the nicotine receptor - most efficacious when combined with nicotine replacement and is a good choice in patients with comorbid depression - In contrast, varenicline, a nicotine receptor partial agonist, should be avoided in patients with mood disorders because it can increase the risk of suicide and should not be used with nicotine replacement

A 33-year-old G2P1 presents to labor and delivery in active labor. She received all of her prenatal care at another hospital. When her chart was faxed you noticed that this current pregnancy was complicated by herpes gestationis. How does this typically present?

*A periumbilical vesicular rash* Herpes gestationis /pemphigoid gestationis - vesicular rash involving the periumbilical region. - not caused by a herpes virus but instead is an autoimmune phenomenon, similar to the bullous diseases - can present as papules, vesicles, plaques or urticaria - occurs during the second half of pregnancy, but has also been reported to erupt postpartum - benign and not associated with any adverse outcomes for the mother or fetus - Treatment is typically oral corticosteroids if the mother is seeking symptomatic relief. - presence of subepidermal vesicles on routine histologic examination and the linear deposition of C3 and IgG along the basement membrane of perilesional skin. - Detection of C3 and IgG is accomplished by immunoflourescence microscopy.

A 35-year-old G3P2 female presents to her obstetrician's office at 11 weeks gestation for prenatal genetic testing. She reports that she has one child with Down's syndrome, and she would like to know if her current gestation also has trisomy 21. Which of the following is the most important risk factor for distal limb reduction abnormalities in patients undergoing chorionic villus sampling (CVS)?

*A procedure performed at < 10 weeks gestational age* Known complication of chorionic villus sampling (CVS) is distal limb reduction abnormalities. - The gestation age of the fetus is the most important in reducing this risk, with the younger the fetus (< 10 weeks), the higher the risk. CVS - used between 10 and 12 weeks gestation in order to determine the genetic characteristics of the fetus. There are two different techniques for performing CVS. - Transvaginal approaches are more appropriate in cases where the placenta is positioned in the posterior uterus, while transabdominal approaches are used if the placenta is on the anterior uterus.

A 73-year-old female with diabetes and hypertension presents to the emergency room with one hour of slurred speech. Her symptoms resolve while she is waiting to see the physician. The emergency room physician decides to order a diffusion-weighted MRI scan. Which of the following scoring tools can be used to predict her risk of stroke?

*ABCD2* Transient ischemic attack - *the ABCD2 tool can be used to evaluate the risk of stroke at 3 and 7 days after a TIA* - The risk of stroke after a TIA is significantly increased, and the risk is highest during the days immediately following the TIA. - The ABCD2 tool is designed to predict the risk of stroke in the 3 to 90 days after a TIA. Patients receive one point each for: - age>60 - blood pressure>140/90 - clinical symptoms (one point for slurred speech or two points for unilateral weakness) - duration (one point for > 10 minutes or two points for >60 minutes) - diabetes The 2009 American Heart Association & American Stroke Association guidelines recommend neuroimaging within 24 hours of a suspected TIA, preferably with MRI with diffusion weighting, given its ability to detect strokes (and differentiate from TIAs) in the acute phase

A 34-year-old male with a history of Crohn's disease presents with recent onset periumbilical abdominal pain, weight loss, and fever. On physical exam he is noted to have a palpable abdominal mass in the right lower quadrant. What is the appropriate next step in management?

*Abdominal CT* Crohn's disease - palpable mass is concerning for an abdominal abscess and *computed tomography (CT) is the diagnostic test of choice* This patient has a history of Crohn's disease, a chronic inflammatory bowel disease (IBD) characterized by focal, asymmetric, transmural, and occasionally granulomatous inflammation of the gastrointestinal tract - *The palpable mass found on physical exam is concerning for an intra-abdominal abscess, and must be assessed* - *The test of choice is abdominal CT with intravenous and oral contrast* - *On abdominal CT, abscesses are most commonly described as a ring-enhancing fluid collections, with or without air* *Illustration is an abdominal CT scan. The large arrow is pointing out an intra-abdominal abscess*

A 34-year-old male is brought to the emergency department by fire and rescue after being involved in a motor vehicle accident. Paramedics report that the patient was driving the car and crashed into a tree. There were no passengers, and he was awake but disoriented at the scene. His temperature is 97.9°F (36.6°C), blood pressure is 131/88 mmHg, pulse is 89/min, respirations are 14/min, and SpO2 is 96% on room air. He is mumbling incoherent words and opens his eyes to voice. He has multiple lacerations on his face and arms. His cardiac exam is normal, and his lungs are clear to auscultation bilaterally. He withdraws to physical exam of the abdomen, and he is moving all extremities spontaneously. His blood alcohol level is above the legal driving limit. What is the best next step in management?

*Abdominal CT* This patient is presenting with stable vitals following a motor vehicle accident. *Since this patient is hemodynamically stable but cannot cooperate with an exam, the best next step is an abdominal CT.* Blunt mechanism of injury - *Since this patient has an unremarkable head CT and is moving all extremities spontaneously, it is most likely that his injury, if present, is localized to the abdomen.* - In blunt abdominal trauma, the next step in management depends upon whether the patient is hemodynamically stable. - With a normotensive blood pressure and pulse within the normal range, this patient would be considered hemodynamically stable enough to undergo imaging to evaluate for injury to the abdominal organs. - *Because the patient cannot cooperate with an exam due to his intoxication, the best diagnostic test is an abdominal CT.* - If the patient were alert and cooperative, the best test would be a bedside ultrasound known as the FAST exam. - If the patient had been hemodynamically unstable, the best test would be a FAST exam if the patient could cooperate, or a diagnostic peritoneal lavage if unable to cooperate.

A 28-year-old male crashed his motorcycle while traveling at 65 mph and is flown by helicopter to the nearest level 1 trauma center for management. He was intubated and 2 large bore IVs were placed with administration of 2L of lactated ringers solution while in transit. On arrival to the emergency department, his vital signs are as follows: T 37.2, HR 128, and BP 70/38. During the primary survey, no open wounds are noted; however, significant bruising is seen on the lower abdomen. Which of the following is the most appropriate technique to determine if this patient is hemorrhaging into his abdomen secondary to the blunt abdominal trauma he experienced?

*Abdominal US* This patient is suffering from hemorrhagic shock, likely secondary to intraperitoneal bleeding from blunt abdominal trauma experienced during the accident. After checking the ABC's, in a hemodynamically unstable patient, abdominal ultrasound is needed to evaluate for intraperitoneal hemorrhage. - The first step in the management of a trauma patient should be to evaluate airway, breathing, and circulation (ABCs). - In addition, intubation or other airway manipulation, placement of 2 large bore IVs, and administration of fluids and/or transfusions as needed should be pursued. - The secondary survey includes checking neurologic function, exposure through removing clothing, complete physical examination, foley placement, and imaging assessment to rule out hemorrhage in the abdomen and/or pelvis. - *If the patient is stable, a CT scan with contrast of the abdomen is warranted; however, if the patient is unstable an abdominal ultrasound is the imaging modality of choice.* - *The FAST ("Focused Assessment with Sonography in Trauma") ultrasound examination can quickly identify free fluid / bleeding in the pericardial, pleural, and/or peritoneal spaces.*

36 yo man comes to doc for routine pre-employment physical. He has no complaints except for occasional morning headaches. His dad died suddenly at age 54. Pt's BP is 175/103 mmHg in right arm & 180/105 in left, and pulse is 82/min. Lungs clear bilaterally & heart sounds normal. Bilateral, nontender, upper abdominal masses palpated on exam. His hemoglobin level is 15.2 g/dL & creatinine conc is 0.8 mg/dL. Most appropriate next step in evaluating this pt's condition?

*Abdominal US* early-onset HT + bilateral upper abdominal masses = PCK - AD = most common genetic cause of CKD - HT, hematuria, proteinuria, palpable renal masses, progressive renal insufficiency - flank pain via renal calculi, cyst rupture or hemorrhage or UTIs - extrarenal complications = cerebral aneurysms, hepatic or pancreatic cyst, cardiac valvular abnormalities = MVP, aortic regurgitation; colonic diverticula, ventral and/or inguinal hernias - diagnosis of ADPKD based mainly on abdominal US --> enlarged kidneys with numerous cysts

A 25-year-old G3P1 female presents to her obstetrician for an anatomy scan. The fetus is 22 weeks gestation, and the study results are all within normal limits. Which of the following biometric parameters are all appropriate for estimating fetal gestational age?

*Abdominal circumference, cerebellar diameter, femur length, biparietal diameter of the skull* The four classic biometric parameters of fetal growth include cerebellar diameter, abdominal circumference, femur length, and biparietal diameter of the skull - Crown-length is another appropriate measure. - Pregnant women should undergo ultrasound assessment of fetal growth parameters during the 2nd trimester - Four fetal measurements are usually used, as each measurement has a wide "normal" range. - The biparietal diameter and the abdominal circumference can be used to estimate fetal weight, although the calculation often has a large error. - Cerebellar diameter is an additional parameter that is most accurate between 14 and 20 weeks of gestation. It is not significantly altered by IUGR (thus its unique utility): https://radiopaedia.org/articles/transverse-cerebellar-diameter

34 yo man comes to ED with severe abdominal pain. He describes pain as "sharp" and "unbearable." It is located in lower left abdomen and radiates to groin. He has vomited twice since it began. Pt has no burning with urination. He is afebrile. He has difficulty lying still on exam table due to discomfort. Mucous membranes appear slightly dry. Lungs clear to auscultation. Heart sounds normal and there are no murmur or gallops. Abdomen soft and nontender to palpation. Inspection and palpation show normal genitalia. Extremities have no cyanosis, clubbing, or edema. Most appropriate next step to diagnose the pt's condition?

*Abdominal ultrasonogram* severe left lower quadrant abdominal pain radiating to groin, vomiting, unremarkable findings on abdominal exam --> *obstructive ureterolithiasis* - writhing in pain + unable to sit still in exam room - obstruction of hollow viscera --> obstruction of small intestine, colon, common bile duct, gallbladder, urinary bladder = referred pain - *diagnosis of ureteral stone via ultrasonography or noncontrast spiral CT scan of abdomen and pelvis*

A 43-year-old woman presents with right upper quadrant pain. She has always had intermittent pain in the same area after certain meals, but her symptoms have never been this persistent before. Vital signs are temperature 38.2 degrees Celcius, heart rate 97, blood pressure 137/84, respiratory rate 14, and oxygen saturation 99% on room air. Body mass index is 29.1. Physical examination reveals tenderness to palpation of the right upper quadrant, worsened when the patient is asked to take a deep breath. To confirm the diagnosis in this patient, what is the recommended imaging study, and what would be the treatment?

*Abdominal ultrasound; Antibiotics and laparoscopic cholecystectomy within 72 hours* Acute cholecystitis - most useful imaging study to confirm the diagnosis is a right upper quadrant ultrasound - treatment includes cholecystectomy within 72 hours. This patient has multiple risk factors for acute cholecystitis. She is female, in her forties, and has a history of biliary colic. Recall the F's of gallbladder disease: fair, fat, forty, fertile, and female. Ultrasound should be the first imaging modality used when suspecting cholecystitis. It is non-invasive, quick, tolerable, and highly reliable in the hands of an experienced operator. Findings include gallbladder wall thickening, stones in the gallbladder, pericholecystic fluid, and a sonographic Murphy's sign (tenderness over the gallbladder from the ultrasound transducer). Acute cholecystitis should be treated with IV antibiotics initially and cholecystectomy within 72 hrs during same hospitalization. Laparoscopic is the approach of choice.

27 yo man comes to ED due to episodic abdominal pain. Pain is concentrated in epigastrium and is gnawing in quality. It wakes him up during night and is promptly relieved by glass of water and piece of bread. Pt has no associated vomiting or diarrhea but has experienced occasional "dark stools." He has no significant past med hx and takes no meds. Fam hx significant for biliary dz in his mom. He smokes pck of cigz daily and consumes can of beer most days. Vitals within normal limits. PE shows mild epigastric discomfort on deep palpation. After diagnosis is confirmed, what is the most appropriate intervention to provide long-term relief of this pt's symptoms?

*Abx and pantoprazole* epigastric pain + intermittent melena = duodenal ulcer - pain often worse on empty stomach - possibly via alkaline fluid secretion into duodenum - majority = via H pylori or NSAIDs

A 48-year-old male presents to his primary care physician with a complaint of lower back pain that has developed over the past week. He works in construction but cannot recall a specific injury or incident that could have led to this pain. He denies any pain, weakness, or change/loss of sensation in his legs. The patient also reports no episodes of incontinence and confirms that he has not noted any changes in his bowel movements or urination. Vital signs are as follows: T 37.2 deg C, HR 74 bpm, BP 122/70 mmHg, RR 12, SpO2 100% RA. Physical examination is negative for any focal spine tenderness and demonstrates 5/5 strength and intact sensation to light touch throughout the bilateral lower extremities. Which of the following is the most appropriate next step in the management of this patient?

*Acetaminophen, continuation of activities as feasible, and limiting bending/twisting* Lower back pain of acute-onset without any associated red-flag symptoms. - *Conservative therapy for 4-6 weeks with should be attempted first before pursuing a more intensive work-up.* A musculogenic (strain) etiology is the most common cause of low back pain; many of these cases resolve completely without intervention. - Initial therapy includes NSAIDs or acetaminophen for pain relief and education to continue activity as possible and avoid bed-rest.

A 30-year-old man is brought to the emergency department by police after attempting to steal several recording devices from an electronics store. During initial police questioning, the patient was extremely agitated and insisted that an alien race had instructed him to steal the equipment in order to establish communication with their planet. In the hospital, the patient admits that he is currently hearing voices and he insists that he be allowed to leave. He is given haloperidol to treat his agitation. Several hours later, the patient experiences a side effect of the medication as seen in Figure (flip for pic) and is subsequently treated with benztropine. The patient likely suffered from which of the following antipsychotic medication side effects?

*Acute dystonia* Benztropine and diphenhydramine are used to treat acute dystonia, a known side effect of typical antipsychotics such as haloperidol. Acute dystonia - quickest onset of any of the known extrapyramidal side effects of typical antipsychotics, typically between four hours and four days - characterized by sustained muscle contractions causing twisting and repetitive movements or abnormal postures. Anticholinergics and antihistamines - commonly used in treatment. - While benztropine is used to treat all extrapyramidal side effects, acute dystonia has the quickest onset of the listed side effects and is therefore the correct answer choice given the timeframe mentioned in the question. Figure illustrates a possible position assumed with sustained muscle contraction in dystonia.

65 yo man comes to hospital with 1 day ho increasing SOB & cough. He has been using albuterol occasionally without relief of symptoms. Pt noncompliant with meds and has not seen doc in past yr. He smoked 1 pck of cigz daily for 40 yrs. Temp is 99.5 F, BP is 110/70 mmhg, pulse is 98/min, & respirations are 20/min. His pulse oximetry shows 86% on RA. Exam shows JVD, and lung auscultations reveals diffuse wheezing with no crackles. His heart sound are distant. Mild hepatomegaly present, and he has bilateral pitting edema halfway to knees. Chest x-ray reveals hyperinflated lungs & flattened diaphragm with no infiltrates. He is started on systemic steroids, bronchodilator nebulization, and furosemide. Lab studies at time of admission and 5 days later: Admission: - pH = 7.32 - PaCO2 = 65 - Bicarb = 32 - Serum sodium = 136 - Serum potassium = 4.6 - Serum chloride = 101 - BUN = 28 - Serum creatinine = 1.1 - Blood glucose = 110 Day 5 - pH = 7.33 - PaCO2 = 45 - Bicarb = 23 - Serum sodium = 139 - Serum potassium = 4.5 - Serum chloride = 98 - BUN = 60 - Serum creatinine = 2.4 - Blood glucose = 144 Most likely explanation for changes in acid-base balance and serum chemistry values b/t day 1 & 5 in this pt?

*Acute kidney injury from diuretic therapy* Extensive smoking hx + COPD exacerbation --> treated with glucocorticoids & bronchodilators --> Also: elevated JVP + hepatomegaly + peripheral edema in presence of clear lungs = *pulmonary HT & cor pulmonale, likely via COPD* *Cor pulmonale --> loop diuretics (furosemide) often used to lower right ventricular filling volumes & reduce peripheral edema --> can lead to hypovolemia, low cardiac output, renal hypoperfusion --> prerenal azotemia/AKI can develop*... - elevated creatinine - elevated BUN (BUN:creatinine ratio >20) - elevated anion gap metabolic acidosis likely via uremia

27 yo woman comes to office due to persistent anxiety. Pt is brought by her husband as she is no longer able to drive. She is afraid that she will have panic attack while driving and could faint and lose control of the car. Pt used to have panic attacks several times a week that were characterized by sudden onset of palpitations, sweating, dizziness. She started med a year ago, and panic attacks have since resolved. However, pt now has anxiety any time she has to leave the house. She has been reluctant to look for work, has stopped going to the gym, and frequently relies on her husband for transportation. Pt has no other med probz. She takes citalopram 20 mg. PE normal. Pt drinks 2 cups of coffee in morning and glass of wine at night. She doesn't use illicit drugs. ROS shows wt gain of 5 lb, which she attributes to lack of exercise. Most appropriate next step in management of this pt?

*Add cognitive-behavioral therapy* Panic disorder complicated by dev't of agoraphobia - *first-line = antidepressants (SSRIs) & CBT - citalopram effectively controls this pt's panic attacks, but she remains significantly impaired (unable to work, drive, leave home)

An overweight, 42-year-old man comes to the physician complaining that his asthma has been worsening over the past few months. He notes that this change coincided with a recent job promotion in which he works late and has very little time to unwind or do anything but go to bed after dinner. He also notes that he has a consistently sore throat and hoarseness in the morning; but, he does not notice it later in the day. Occasionally, he experiences chest pain but he does not think it is related to exertion. In addition to asthma, his medical history is significant for hypertension. He has taken the same medications for years: albuterol inhaler as needed, low-dose fluticasone inhaler daily, and lisinopril. On physical exam, there are no abnormalities noted. What is the most appropriate next step in management?

*Add omeprazole* GERD - should be started on a trial of an anti-reflux medication, such as omeprazole (a proton pump inhibitor). Clues that point toward GERD in this patient include: obesity, going to bed immediately after dinner, sore throat and laryngitis, and chest pain unrelated to exertion. - *very prevalent among asthmatics and can exacerbate airflow obstruction via microaspiration of gastroduodenal contents into the upper airway, increased vagal tone, and heightened bronchial reactivity*

A 55-year-old former longtime alcoholic presents to clinic complaining of new onset increasing abdominal girth and no other complaints. He had been a Child's Class A cirrhotic for some time. His wife, who has accompanied him on this visit, reports that his mental status is unchanged and that he is eating well and attending his Alcoholics Anonymous meetings. On physical exam, his vital signs are stable. His abdomen is distended and tense without appreciable hepatomegaly. There is a fluid wave and shifting dullness. You conduct abdominal paracentesis in the office and aspirate 3L of clear fluid. If sodium and water restriction fails to control this patient's symptoms, what would be the next step in management?

*Add spironolactone* *Management of ascites involves sodium and water restriction followed by spironolactone, loop diuretics, and frequent abdominal paracentesis.* Classification of liver disease: Child's Classification is divided into class A, B, and C Class A = patients have: no ascites, bilirubin < 2, no encephalopathy, excellent nutritional status, and albumin > 3.5 Class B, patients have: controlled ascites, bilirubins 2-2.5, minimal encephalopathy, good nutritional status, and albumin 3-3.5 Class C, patients have: uncontrolled ascites, bilirubins > 3, severe encephalopathy, poor nutritional status, and albumins < 3. These patients often require transplant.

An infant is born via spontaneous vaginal delivery to a G1P0-->1 mother without any complications. The pregnancy was notable only for limited prenatal care. Maternal Streptococcus agalactiae serology was unknown at the time of delivery. The mother's past medical history is notable only for cocaine and IV drug use; however, the patient denies any illicit substances for over a year. During delivery there was a prolonged second stage of labor complicated by uterine atony, which was managed with bimanual massage, IV fluids and oxytocin in a rural hospital in Alabama. On day 4 of life, the newborn presents with a severe bilateral mucopurulent discharge from her eyes. She is crying and inconsolable. Blood pressure is 65/40 mmHg and pulse is 118/min. Which of the following is the best next step in management for this patient?

*Administer IM ceftriaxone* Newborn is presenting with mucopurulent discharge from the eye within 5 days of birth consistent with ophthalmia neonatorum (gonococcal conjunctivitis). - The treatment of choice is IM ceftriaxone to cover N. gonorrhea. Newborns that present within 5 days of birth with a mucopurulent discharge from the eye should be considered to have bacterial conjunctivitis caused by Neisseria gonorrhea (ophthalmia neonatorum). - Typically erythromycin drops are given to newborns to prevent gonococcal conjunctivitis; however, the history in this case suggests that this routine prophylaxis did not occur. - In contrast, conjunctivitis from Chlamydia trachomatis tends to occur days to weeks later and is less purulent in nature than gonococcal conjunctivitis. - This mother's limited prenatal care predisposes her to having untreated sexually transmitted infections that could affect her newborn, especially when the mode of delivery is vaginal. - *The best treatment for gonococcal conjunctivitis is IM ceftriaxone and irrigation of the eye.*

A 61-year-old male complains of chest pain two days after undergoing revascularization of his left anterior descending artery due to acute myocardial infarction (MI). The patient complains of severe chest pain that is worse upon inspiration and is relieved by sitting up and leaning forward. EKG findings are shown in Figure A. Echocardiography shows no evidence of tamponade. Which of the following is indicated in the treatment of this patient?

*Administer aspirin* pleuritic chest pain + EKG changes = *acute pericarditis* in the setting of MI - usually self-limited - *aspirin should be administered and non-steroidal anti-inflammatory drugs (NSAIDS) may be indicated for symptomatic relief* Pericarditis - classically presents with symptoms of dyspnea, cough, fever, and pleuritic chest pain - Physical exam findings include pericardial friction rub, elevated JVP, and pulses paradoxus - *Patients are at risk for pericarditis during several days following acute MI and weeks to months later (Dressler's syndrome)* Dressler's syndrome is distinct from the acute pericarditis seen immediately following an MI as it involves an autoimmune process. *Figure shows EKG changes in acute pericarditis. Note PR depression as indicated by arrows in leads II, V4, and AVF. Other possible EKG changes include diffuse ST elevation and T wave inversions.*

A 65-year-old man presents to the emergency department with left-sided chest pain. He says the pain occurs at rest and is localized exclusively to the chest without radiation to the extremities. The patient has had chest pain with exertion for the past three years, but over the past year his symptoms have occurred following activities of daily living. The current episode provoked anxiety and led him to seek medical treatment. Past medical history includes hypertension that he treated successfully with exercise and diet modifications. While speaking with the emergency department physician, the patient reports that the pain has resolved. Electrocardiogram (EKG) shows left bundle branch block (LBBB) unchanged from a prior EKG taken 3 years ago. Serum troponin tests are normal. Which of the following is the most appropriate next step in management in this patient?

*Admit the patient and schedule an adenosine perfusion stress test* The patient has chest pain at rest with an indeterminate EKG due to LBBB and normal troponins. *The presentation suggests unstable angina and further risk stratification is warranted.* - *Low risk patients with unstable angina should be admitted to the hospital for further risk stratification, and an exercise or pharmacologic stress echo should be performed.* - Unstable angina in high risk patients, such as those with known CAD, greater than 3 CAD risk factors, or ST depressions greater than 1mm, or those patients with elevated cardiac biomarkers (NSTEMI) should be sent for angiography. - *This patient's LBBB may mask EKG signs of cardiac ischemia, and he has a history of exertional angina which limits his ability to exercise.* - *Adenosine perfusion stress tests are more sensitive than exercise stress tests in patients with significant baseline EKG abnormalities or an inability to exercise to an adequate level.* - Perfusion imaging is useful for both diagnosis and prognosis in patients with coronary artery disease. In patients unable to exercise, the coronary vasodilators dipyridamole and adenosine may be used in conjunction with thallium imaging.

16 yo girl comes to office for routine exam. She is wearing sweater despite warm weather and says that she often feels cold. She also wonders if thyroid prob could be making it difficult for her to lose wt. Pt wants to lose about 10 lb bc she believes that she is "enormous." She feels depressed bc everyone at school "looks at me funny" and "I don't fit in anywhere." For past yr, she has been on veg diet bc meat makes me feel bloated. Menarche was at age 12 and her LMP was 2 mos ago. Pt is honors student, plays basketball, and is on cross-country running team. Her fam hx is significant for Graves' dz in her mom & maternal gramz with Sjogren's syndrome and MDD. Temp is 97.2 F, BP is 88/58 mmHg, pulse is 39/min, and respirations are 16/min. BMI 15 kg/m^2. Pt is alert & cooperative but appears tired. Her skin is dry with lanugo. Lab results: Serum chemistry - Sodium 136 mEq/L - Potassium 2.9 mEq/L - Chloride 102 mEq/L - Bicarbonate 24 mEq/L - BUN 18 mg/dL - Creatinine 0.6 mg/dL - Calcium 8 mg/dL - Glucose 78 mg/dL - Magnesium 2.3 mg/dL - Phosphorous 2 mg/dL Thyroid function tests - TSH 4.5 uU/mL - Free T4 4 ug/dL - T3 65 ng/dL Urine hCG negative Most appropriate next step in management of this pt's symptoms?

*Admit to hospital* Anorexia nervosa - type A personality often - participation in activities where they is pressure to be slender Medically unstable pts - require hospitalization for acute stabilization - indications = dehydration, unstable vital signs (bradycardia <40/min, hypotension, hypothermia), cardiac arrhythmias, electrolyte disturbances (hypokalemia, hypophosphatemia), organ inv't due to malnutrition, very low wt

A 69-year-old man presents to his primary care physician for trouble sleeping. The patient states that he recently retired from working the day shift at a cemetery. When the patient retired, his goal was to finally be able to go out with his wife; however, he finds that he is unable to stay awake past 6 pm in the evening. His inability to stay awake has been straining his marriage as his wife is disappointed that they cannot do any activities in the evening together. The patient has tried drinking caffeine but finds that it does not help. The patient's wife claims that the patient seems to sleep peacefully, and the patient states he feels rested when he awakes. The patient has a past medical history of irritable bowel syndrome which is managed with fiber supplements. The patient's neurological exam is within normal limits. Which of the following is the most likely diagnosis?

*Advanced sleep phase disorder* Difficulty staying awake in the early evening = *advanced sleep phase disorder (also known as advanced sleep phase syndrome)* - can present in patients of all ages - difficulty staying awake in the early evening (5 to 8 pm) - however, will typically sleep a normal quantity of hours - will often impair social functioning which can cause distress to the patient - diagnosis of advanced sleep phase disorder can be made when these symptoms are present in addition to an absence of another organic cause (such as obstructive sleep apnea)

53 yo man comes to office with 3-day ho right leg swelling & pain. He has had no chest pain, dyspnea. Pt describes himself as healthy, and his last visit to doc was 10 yrs ago. He has had no wt loss or abdominal pain. He smokes and has 30 pck yr hx. He has active lifestyle and has had no recent travel. Pt's mom died of breast cancer and his dad has CHF. Temp is 98.6 F, BP is 140/80 mmHg, pulse is 70/min, respirations are 14/min, and O2 sat is 97% on RA. Normal vesicular breath sounds & cardiac sounds heard on chest auscultation. Abdominal exam unremarkable. There is swelling and tenderness of right leg up to mid-thigh. Results of CBC & coag studies are within normal limits. Duplex US demonstrate incompressible popliteal and femoral veins, & anticoag is started immediately. Chest x-ray unremarkable. Most appropriate in eval of this pt's current condition?

*Age appropriate cancer screening* DVT of popliteal and femoral veins - risk factors for VTE = inherited or acquired (immobilization, surgery, *malignancy*, meds) Absence of any clear provoking factors in pts with first episode of VTE = *age-appropriate cancer screening (colonoscopy)*

73 yo woman comes to office due to dry eyes. For last yr, she has had mild burning discomfort associated with sensation of dust or sand in eyes. Recently, pt has had difficulty reading small-print books due to decreased visual acuity. She uses OTC eye drops to relieve discomfort. Her med hx is notable for hypothyroidism & OA of knees, which are treated with levothyroxine and acetaminophen. She drinks glass of wine nightly and doesn't smoke. Vitals normal. On exam, pupils are symmetrically round & reactive to light. Conjunctivae normal bilaterally, and fundoscopic exam normal. Oropharyngeal exam shows dry mucosa with mild dental caries. Neuro exam unremarkable. Serum chemistry panel, CBC, and TSH normal. Serum ANA negative. Most likely cause of this pt's symptom?

*Age-related exocrine gland atrophy* older woman + dry eyes & mouth = *age-related sicca syndrome (ARSS)* / age-related dry eye syndrome - exocrine output from lacrimal and salivary glands declines with age = atrophy, fibrosis, ductal dilation of glands - decreased blink rates, oxidative damage, excessive evaporation of tears (via meibomian dysfunction) - more common in women - increased risk in pts with diabetes or thyroid disorders - pts can develop corneal epithelial erosions = impaired vision - cyclosporine drops can be used to increase tear secretion

A 43-year-old, HIV-positive male presents with two weeks of worsening right upper quadrant abdominal pain, fever of 102 F, chills, and cough. Notably, he traveled to Mexico earlier this year. CT reveals a single 4 cm mass in the right lobe of the liver with a double target sign and a trace right pleural effusion. What is the most likely diagnosis?

*Amebic abscess* Single abscess in the right lobe of the liver in a young, immunocompromised patient with recent travel to an endemic area is most concerning for amebic abscess, caused by Entamoeba histolytica - transmitted via the fecal-oral route - the second leading cause of parasitic death worldwide Those who have traveled to or lived in endemic locations are most at risk. Men are more likely than women (3:1) to have more invasive disease, including liver abscesses, as are the very young or old and those with compromised immune systems. Metronidazole is the preferred treatment. If left untreated, the abscess may rupture. An echinococcus cyst would have evidence of daughter cysts, pericysts and CALCIFICATIONS. An hepatic abscess may have an enhancing wall, evidence of air, and/or a double target sign.

A 52-year-old female with a past medical history of rheumatoid arthritis presents to her primary care physician for complaints of increased swelling in her legs. She also notes her urine to be more "frothy" than usual. On physical exam she is noted to have a blood pressure of 142/90 mmHg. At all prior visits, this patient has had normal blood pressure. A 24-hour urine collection for protein contains 3.8 g. Which of the following is most likely present in this patient's kidney?

*Amyloid deposition* Secondary nephrotic syndrome as a result of AA amyloid deposition - the most probable cause of nephrotic syndrome in patients with rheumatoid arthritis (RA) - may affect the entire body and can be of primary or secondary cause. Primary amyloidosis - result of AL amyloid deposition (derived from Ig light chain) - Classically this is seen in multiple myeloma. Secondary amyloidosis - result of AA amyloid deposition (derived from acute phase reactant of inflammation). - Classically this is seen in RA and other forms of chronic inflammation. Amyloid deposits have apple-green birefringence under polarized light after staining with Congo red.

54 yo man comes to doc complaining of morning facial puffiness and bilateral leg swelling. His other med probz include recurrent pulmonary infections due to bronchiectasis and psoriasis. His BP is 143/92 mmHg and pulse is 92/min. His BMI is 24 kg/m^2. Exam shows fourth heart sound. Hepatomegaly, palpable kidneys, and 2+ pitting edema of lower extremities to knees bilaterally are present on exam. Urinalysis shows 4+ proteinuria and normal urinary sediment. Most likely diagnosis in this pt?

*Amyloidosis* facial swelling + bilateral LE swelling + massive proteinuria = nephrotic syndrome nephrotic syndrome + palpable kidneys + hepatomegaly + ventricular hypertrophy S4) in setting of chronic inflammatory dz (recurrent pulm infections, bronchiectasis) = *secondary amyloidosis (AA)* - treatment = directed at underlying inflammatory dz - *colchicine* = both treatment & prophylaxis of AA

A 31-year-old female lawyer is walking to work in January when she steps on a patch of ice and falls forward on her outstretched right hand. On physical exam you note pain to palpation at the base of the thumb and lateral wrist. A wrist radiograph is taken which does not show any signs of fracture. Which of the following is the best next step in the management of this patient?

*Analgesia with casted immobilization for 6 weeks* Scaphoid fracture, even with a negative radiograph, which is not a sensitive test immediately after trauma - best option is a long thumb spica cast for 6 weeks. - scaphoid is the most commonly fractured carpal bone, most often as result of a fall on outstretched hand - seen mainly in young adults as in older adults the point of fracture is the distal radius (Colle's fracture) - radiograph may take up to two weeks to show a fracture - scaphoid fracture complications include avascular necrosis (AVN), as a result of the distal to proximal blood supply of the scaphoid bone

A 69-year-old man presents to your office with severe lower back pain and urinary hesitancy. A radionuclide bone scan reveals osteoblastic bony metastatic disease in the vertebral column. The most appropriate INITIAL treatment in this patient has which of the following mechanisms of action?

*Androgen antagonist* Metastatic prostate cancer - usually initially treated with androgen depletion and luteinizing hormone releasing agonists (e.g. leuprolide) as first line agents. *However, in patients with severe ureteral obstruction and painful vertebral metastases, an antiandrogen (e.g. flutamide or bicalutamide) is given prior to beginning LHRH agonist therapy.* - LHRH agonists (e.g. leuprolide) bind to LHRH receptors in the anterior pituitary and initially promote the release of LH and FSH, resulting in an undesired increase in testosterone. However, after continuous agonism, the LHRH receptors downregulate, causing a decrease in LH release and ultimately produce the desired decrease in testosterone. - *Patients presenting with painful metastases and ureteral obstruction should receive a direct anti-androgen first.*

With the summer months approaching, a mother asks her pediatrician about his recommendations for sunscreen application. Her children are of Colombian descent. Which of the following is an appropriate piece of advice?

*Apply sunscreen at least 15 minutes prior to sun exposure* According to the most recent recommendations from the American Academy of Dermatology, sunscreen should be applied at least 15 minutes prior to sun exposure. - Current recommendations indicate that people of all skin types should be using sunscreen for skin cancer protection throughout the year. - Sun should be avoided during the hours of 10 am and 4 pm, as this is the time when the UV rays are the strongest. - Extra care should be taken when near water or snow, as the sun's rays are reflected in these environments. - The most common physical sunscreens contain agents like titanium dioxide or zinc oxide which physically reflect the sun's rays conferring both UVA and UVB protection. - Chemical sunscreens absorb UV rays and dissipate the energy as light or heat.

68 yo man comes to ED due to right leg pain for past several hours. Pt was resting at home when he suddenly had severe pain in right leg; leg has since become numb. He has never had similar symptoms and has no ho trauma, fever, or chills. Pt recently suffered acute anterior wall MI results in cardiogenic shock and is currently undergoing cardiac rehab. His other med probz include HT, T2DM, hyperlipidemia. Temp is 98.1 F, BP is 120/70 mmHg, pulse is 90/min & regular, & respirations are 16/min. Cardiopulm exam unremarkable. Compared to left leg, his right leg appears pale and cool to touch. In right LE, popliteal pulse normal, but more distal pulses are not palpable. Pulses in other extremities are normal. Neuro exam shows loss of sensation over dorsum of right foot & mild weakness with dorsiflexion. Most likely cause of this pt's symptoms?

*Arterial embolism* Classic 6 Ps of *acute arterial occlusion* of rt LE = *acute limb ischemia* Sudden dev't of symptoms in previously asymptomatic pt = embolic occlusion - majority of arterial emboli originate from heart - LEs more common - potential cardiac sources of emboli = left atrium thrombus via a fib, *left ventricular thrombus following anterior MI (this pt)*, infective endocarditis (septic emboli), thrombus from prosthetic valves

A 21-year-old male is rushed to a level-one trauma center after sustaining a stab wound to the neck during a gang fight. On physical exam, his vitals are as follows: BP 90/70, HR 110, RR 21, O2 Sat 95%. He is noted to have a 3 cm laceration 1 cm inferior to the mastoid process on the right side. After placement of two large bore peripheral IV's, intravenous fluids are started. After 3 L of normal saline and 2 units of blood, his HR is 105 and BP is 100/75. Which of the following is the most appropriate next step in the management of this patient?

*Arteriogram* This patient has sustained a penetrating neck injury (PNI) to zone III (above the angle of the mandible). This should be managed with an arteriogram due to the difficulty of accessing this region surgically. 3 defined zones with PNIs: 1. above the angle of the mandible (Zone III) 2. between the angle and the cricoid cartilage (Zone II) 3. below the cricoid cartilage (Zone I) Classically Zone I and III require endoscopy and angiography as surgical exploration of these zones surgically is challenging Zone II is classically managed with a surgical exploration

60 yo man comes to doc complaining of wt loss & fatigue. ROS positive for change in bowel habits. Further eval shows presence of colon carcinoma. Pt told diagnosis and is educated about his treatment options and prognosis; he listens politely & doesn't ask any questions. He is a high school graduate and worked in a care factory until taking extended sick leave in recent wkis. Pt appears to comprehend info without difficulty. He says he doesn't want treatment or intervention of any kind. Doc also serves as PCP for pt's wife. What would be the most appropriate next step in addressing this situation?

*Ask the pt why hy doesn't want treatment or intervention of any kind* Although pt likely has capacity to refuse treatment, his reasoning should first be thoroughly explored & discusses with doc - he may be in state of disbelief or denial or afraid of pain or unfamiliar procedures - pts may also have mental health issues

A 71-year-old female presents with a transient episode of right arm and hand weakness that resolved in approximately one hour. Her past medical history is notable for hypertension, diabetes, anxiety, and dyslipidemia. Her current medications include insulin, metformin, and fluoxetine. Examination reveals a left carotid bruit. Ultrasound duplex of her carotid arteries demonstrates right and left carotid stenosis of 35% and 60%, respectively. Which of the following is the best next step in management?

*Aspirin* Transient ischemic attack and carotid artery occlusion < 70% - The best next step in management is daily aspirin. Carotid stenosis can lead to neurologic sequelae such as a transient ischemic attack. If carotid stenosis is < 70%, then the best next step in management is anti-platelet agents such as aspirin. Indications for carotid endarterectomy include symptomatic patients with 70-99% stenosis. Patient's with stenosis > 70% who can not undergo a surgical procedure can also be managed medically with anti-platelet agents.

Abrupt onset of regular tachycardia that resolves with cold-water immersion is consistent with paroxysmal supraventricular tachycardia (PSVT). Cold-water immersion relieves symptoms abruptly by directly altering....

*Atrioventricular node conductivity* Atrioventricular nodal reentrant tachycardia (AVNRT) - most common form of PSVT - caused by reentry mechanism due to presence of dual electrical pathway (flow & fast pathway) in AV node Vagal maneuvers - increased parasympathetic tone = temporary slowing of conduction in AV node & increased in AV node refractory period --> termination of AVNRT

32 yo woman comes to office due to lightheadedness. She had uncomplicated vaginal delivery 6 wks ago with minimal blood loss. Pt had mild fatigue & backache during last trimester of pregnancy, but fatigue has worsened since delivery. She has also been having intermittent generalized abdominal pain, loss of appetite, and weakness. She says her skin is tanning even though she has spent only minimal time outdoors. Yesterday, pt felt like she was "going to pass out" and experienced several episodes of lightheadedness accompanied by nausea & abdominal discomfort. She has no chest pain, SOB, or bleeding. Pt has ho hypothyroidism and takes levothyroxine. Her temp is 97 F, BP is 110/60 mmHg supine & 85/45 mmHg standing, and pulse is 104/min. Skin shows generalized hyperpigmentation. Lungs clear, and her HR is tachycardic but regular with no gallops or murmurs. Abdominal exam shows mild, diffuse tenderness without guarding & normal bowel sounds. Lab results: Hct 32% Leukocytes 9,000/mm^3 Sodium 131 mEq/L Potassium 5.6 mEq/L Bicarbonate 18 mEq/L Creatinine 1.4 mg/dL TSH 4.6 uU/mL Most likely diagnosis?

*Autoimmune adrenalitis* - loss of glucocorticoid, mineralocorticoid, and adrenal androgen secretion - severe symptoms - hypotension, hyperkalemia, hyperchloremic acidosis - frequently associated with *hyperpigmentation* via increased pituitary secretion of ACTH & melanocyte-stimulating hormone Postpartum adrenal insufficiency (AI) - may be via adrenal (primary AI) or pituitary (secondary AI) dz - *hyperpigmentation & signs of mineralocorticoid deficiency = primary*

38 yo woman comes to office with 3 wk ho wt loss, nausea, abdominal pain, postural dizziness. She traveled to Thailand 6 mos ago and has felt fatigued since then. Med hx notable for moderate persistent asthma treated with inhaled beta-2 agonist & inhaled corticosteroid. Over last 2 yrs, pt has had several asthma exacerbations requiring oral prednisone. She also has hypothyroidism treated with levothyroxine. Pt married & is a stay-at-home mom. BP is 90/60 mmHg & pulse is 96/min. Pharyngeal exam shows bilateral tonsillar enlargement. Skin exam shows increased pigmentation at palmar creases and mucous membranes as well as few patches of vitiligo. Initial lab testing shows mild hyponatremia and hyperkalemia with normal renal function. CBC normal, but differential shows moderate eosinophilia. Follow-up testing shows low 8 AM serum cortisol. Most likely cause of this pt's adrenal insufficiency?

*Autoimmune adrenalitis* Chronic primary adrenal insufficiency (PAI) - wt loss, abdominal pain, fatigue - hyperpigmentation common via cosecretion of melanocyte-stimulating hormone with ACTH (both derived from propiomelanocortin) = increased in response to cortisol deficiency - hypotension, hyponatremia, hyperkalemia - *eosinophilia, hyperplasia of lymphoid tissue (tonsils) = common but nonspecific*

24 yo woman brought to office by her roommate due to worsening paranoia, anxiety, and change in behavior. Pt's anxiety began 6 wks ago after her boss threatened to fire her for "taking too many days off work" for a flu-like illness the wk before. Her anxiety over losing her job steadily worsened, and she began having increasing difficulty sleeping. Over last few wks, pt has become convinced that her boss is following her outside of work to find reason to fire her. Last wk she bough extra deadbolt for door to prevent her boss from "breaking into house to find evidence against me." Pt's roommate recalls event earlier this wk during which he noticed that she was staring blankly and smacking her lips rhythmically for about a minute while they watched television. Pt has no med or psych history. Temp is 100.2 F, BP is 137/86 mmHg, pulse is 102/min, and respirations are 14/min. Pt recalls 1 of 3 objects in 5-minute delayed recall task. She sometimes appears distracted by internal stimuli and mumbles to herself throughout the exam. Upper and lower limb deep tendon reflexes are 2+. Rigidity present in upper extremities. Most likely diagnosis for this pt?

*Autoimmune encephalitis* psych symptoms (anxiety, psychosis, insomnia) + autonomic instability + short-term memory + rigidity + focal seizure following flu-like prodrome = *anti-NMDA receptor / anti-NMDAR encephalitis* - for of autoimmune encephalitis - multistage syndrome - median onset of age = 21 yo - women more som - association with ovarian teratoma - some cases may be complication of HSV encephalitis - can progress to mutism, hypoventilation, decreased level of consciousness - can be confirmed by presence of CSF Abs to GluN1 subunit of NMDAR - immunosuppression = cornerstone of therapy - recovery often slow

65 yo man comes to office due to recurrent falls. For last several mos, he has had sensation of imbalance that is worse at nt, but with no associated pain or motor weakness. Med hx notable for T2DM diagnosed 20 yrs ago and HT diagnosed 10 yrs ago. Pt was found to have nonproliferative diabetic retinopathy at his last ophthalmologic visit. Current meds include insulin, metformin, rosuvastatin, and ramipril. He doesn't smoke, but drinks 1 or 2 beers daily and often more on weekends. PE shows decreased proprioception and vibration sense in feet. Ankle jerk reflexes absent. Finger-to-nose and heel-to-shin tests normal,but pt sways and tends to fall when his eyes are closed. Bilateral hammer toes deformities present in feet. Remainder of exam unremarkable. What processes is predominately responsible for this pt's neuro symptoms?

*Axonopathy of large nerve fibers* - predominance of negative symptoms = numbness, loss of proprioception & vibration sense, diminished ankle reflexes - loss of position sense --> gait imbalance = worsened with eye closure bc no visual compensation for loss of proprioception & weakness of intrinsic muscles of feet = foot deformities (hammer toe deformity) DM: length-dependent axonopathy - *clinical features occur first in longest nerves (ex, feet)* - sensorimotor polyneuropathy = most common neuropathy in pts with diabetes

A 32-year-old G1P0 at 36 weeks gestation presents to your clinic with severe headache. Temperature is 37.1 degrees Celsius, pulse is 90/min, respiratory rate is 20/min, and blood pressure is 175/105 mmHg. The patient denies contractions, and her cervix is long, closed, and posterior. Urine protein is 3.5 g/24 hours. Which of the following is the most appropriate next step in management for this patient?

*BP control & seizure prophylaxis* Severe preeclampsia - Seizure prophylaxis and blood pressure control are necessary prior to delivery. Antihypertensive agents should be used to control diastolic blood pressures above 100 mmHg in patients with symptoms of severe preeclampsia, notably proteinuria, hypertension, headaches, changes in visual, and right upper quadrant pain - Acceptable antihypertensive agents for pregnant females include hydralazine, labetalol, and nifedipine - Seizure prophylaxis with magnesium should be used to prevent progression to eclampsia - A loading dose of 6g of magnesium sulfate followed by an hourly infusion of 2g per hour is used for seizure prophylaxis - Antihypertensive therapy is used to lower systolic blood pressure below 160 mmHg and diastolic blood pressure below 100 mmHg

57 yo woman comes to ED with 12 hrs of fever, chills, and severe generalized weakness. She also has discomfort in her RUQ and has had 2 episodes of vomiting. Pt underwent liver transplant for primary biliary cirrhosis 2 wks ago & currently takes prednisone, tacrolimus, and mycophenolate. Temp is 102.5 F, BP is 85/55 mmHg, and pulse is 130/min. PE reveals tenderness in her RUQ with mild guarding. Lab results: CBC Hgb 10.8 g/dL Platelets 450,000/mm^3 Leukocytes 28,000/mm^3 Serum chemistry Sodium 139 mEq/L Potassium 4.1 mEq/L Chloride 101 mEq/L Bicarb 25 mEq/L BUN 31 mg/dL Creatinine 1.3 mg/dL Calcium 9.2 mg/dL Glucose 105 mg/dL Liver function studies Total bilirubin 1.3 mg/dL Alk phos 203 U/L AST 105 U/L ALT 63 U/L Coag studies INR 1.3 Most likely cause of this pt's condition?

*Bacterial infection* Liver transplant 2 wks ago + RUQ pain + high fever + hypotension + tachycardia + leukocytosis = ongoing bacterial infection <1 mo - bacterial causes via operative complications, etc - hepatic abscess, biliary leak, wound infection 1-6 mos - opportunistic pathogens - CMS, aspergillus, mycobacterium tuberculosis - in settin go high-dose immunosuppressive meds >6 mos - community-acquired pathogens

HIV pt + vascular cutaneous lesions, systemic symptoms, sometimes organ inv't (liver, CNS, bone).

*Bartonella* henselae, quintana - can cause bacillary angiomatosis in immunocompromised - treatment = antimicrobials = doxycycline, erythromycin + antiretroviral

25 yo man presents to ER with SOB & cough productive of blood-tinged sputum for past few days. He denies associated fever, arthralgias or weight loss. He has never had these symptoms before, and is extremely concerned. He has no ho recent travel or sick contacts. He smokes half a pack of cigz daily, and has had 2 sexual partners in past 6 mos. On PE, his temp is 98.9 F, BP is 120/70 mmHg, pulse is 102/min, & respirations are 22/min. Lung auscultation reveals patchy bilateral rales. Chest x-ray demonstrates bilateral pulmonary infiltrates. His serum creatinine is 2.6 mg/dL and urinalysis shows dysmorphic red cells. Most likely cause of his current condition?

*Basement membrane antibodies* renal & pulm findings = Goodpasture's - young adult males - nephritic-range proteinuria (<1.5 g/day), acute renal failure, urinary sediment with dysmorphic red cells & red cell casts - underlying cause = Abs to alpha-3 chain of type IV collagen - diagnosis via renal biopsy = linear IgG Abs along glomerular BM

57 yo woman comes to office with intermittent headaches, shoulder & neck pain, fatigue, and insomnia. Her headaches sometimes interfere with her ability to concentrate at work. Pt's symptoms not new, but they have worsened over past 8 mos due to stress of starting new job & placing her elderly mother in an assisted living facility. Pt lies awake at night worrying about her mom, her own health probz, and finances of her 2 adult children, who are having difficulty supporting themselves. During day, she is tired due to poor sleep and worries about her job performance. Pt has ho HT, irritable bowel syndrome, and tension headaches. She drinks 1 or 2 glasses of wine before bedtime to help her relax and fall asleep. Temp is 98 F, BP is 130/80 mmHg, pulse is 88/min, & respirations are 16/min. Exam shows sweaty palms and mild diffuse abdominal tenderness; no other abnormalities noted. CBC, chemistry panel, TSH, urinalysis, ECG normal. Most appropriate next step in management of this pt?

*Begin escitalopram & recommend CBT* GAD - insomnia, fatigue, poor concentration - although excessive worry about multiple issues is key symptom, pts frequently come to PCP with somatic symptoms, especially muscular tension - *effectively treated with serotonergic antidepressant, CBT, or combo*

29 yo man comes to clinic 2 wks after ED visit for epistaxis requiring anterior nasal packing. In ED, his BP was 170/110 mmHg. He has occasional headaches and fatigue but no chest pain, palpitations, or syncope. His past med hx is unremarkable and he doesn't use tobacco, alcohol, or illicit drugs. Pt's current BP is 180/112 mmHg and pulse is 78/min and regular. Cardiac auscultation in supine position reveals no murmurs or additional sounds. Abdominal exam shows no periumbilical bruits. ECG shows normal sinus rhythm, high-voltage QRS complexes, downsloping ST-segment depression, and T wave inversion in leads V5 and V6. Lab results: Hgb 14.2 g/dL Platelets 230,000/mm3 Creatinine 1 mg/dL Best next step in eval of this pt?

*Bilateral arm and leg BP measurements* - pts with abnormal findings should undergo confirmation with echocardiogram elevated BP + LVH (high-voltage QRS, lateral ST segment depression, lateral T wave inversion) = long-standing systemic HT - essential HT in young pts --> look for secondary cause Coarctation of aorta - epistaxis, headaches, LE claudication can occur - continuous murmur usually present but may be difficult to auscultate in suping position

24 yo woman delivers healthy baby by cesarean section and is discharged on postpartum day 3. Four days later, her husband brings her to ED as he is worried that she is acting strangely. Since discharge from hospital, pt has become increasingly agitated. Initially, she asked her husband repeatedly for reassurance that the baby is normal. Subsequently, she ignored the baby completely. This morn, the husband found the baby unattended and half-submerged in the bathtub. Pt is sleeping poorly and not eating or showering. On exam she is tearful and does not acknowledge having given birth. She says, "The devil is not my baby." Pregnancy was planned and without complications. Pt has no psych hx, but there is fam hx of unspecified psychiatric illness. Disorder most commonly associated with this pt's condition?

*Bipolar disorder* Postpartum psychosis - acute onset of agitation, bizarre behavior, delusions shortly after giving birth - relatively rare - within first 2 wks after delivery - depressed and/or manic moods, severe insomnia, agitation, disorganized behavior, delusions and/or hallucinations with content frequently related to infant - medical emergency - high risk for suicide and may harm baby - often in pts with ho bipolar disorder - may be first manifestation of bipolar disorder

58 yo woman undergoing systemic chemo for advanced breast cancer with metastases to bone comes to office for routine follow-up. She has mild, vague bone pain that is relieved by occasional use of acetaminophen. PE shows well-healed mastectomy scar on right and palpable right supraclavicular lymph node. Mucous membranes moist. Lab results: Serum chemistry - Sodium 144 mEq/L - Potassium 3.8 mEq/L - Chloride 102 mEq/L - Bicarbonate 24 mEq/L - BUN 14 mg/dL - Creatinine 0.8 mg/dL - Calcium 11.4 mg/dL - Glucose 146 mg/dL Liver function studies - Albumin 3.4 g/dL Best next step in management of this pt?

*Bisphosphonate therapy* - zoledronic acid - inhibits osteoclastic activity of bone --> stabilizes destructive bony tumors - reduces risk of skeletal-related events such as pathologic fracture and malignant hypercalcemia Hypercalcemia in setting of metastatic breast cancer to bone - asymptomatic or mild hypercalcemia (calcium <12 mg/dL) does not require urgent therapy, but hypercalcemia of malignancy may worsen over time

A 65-year-old patient presents to the ED after passing a bloody stool. When would a technetium-99 labeled erythrocyte scintigraphy be the next appropriate step in the workup of this patient?

*Bleeding source not seen on colonoscopy and patient continues to bleed* Patient with continued lower GI bleed not seen with colonoscopy can be localized with a highly sensitive tagged-RBC study such as technetium-99m scan. - The management should begin with stabilization of the patient, fluid resuscitation, and a type and cross. - Once an upper GI cause is ruled-out via NGT on endoscopy, a lower source should be sought with colonoscopy. - A technetium-99m-tagged red blood cell scan can be helpful. - This involves removing a sample of the patient's blood, tagging with a radiolabeled tracer, and injecting it back into the circulation. - A nuclear imaging study can then determine where the blood is entering the GI tract. Illustration shows an example of a tagged-RBC study. The arrows indicated the location of bleed.

32 yo man comes to ED due to 5 days of intermittent high fever, chills, and drenching sweats. He also has malaise, fatigue, and dark urine. Pt has no med hx other than splenectomy after motor vehicle collision 10 yrs ago. He drinks alcohol on social occasions and doesn't use tobacco or illicit drugs. He is married and in a monogamous relationship with his wife. Pt lives on Long Island, NY and works in Manhattan as software programmer. He has not traveled outside country but likes to hike and explore the woods. He recently camped in New England and found 2 ticks on his legs when he returned home. Temp is 103 F, BP is 110/70 mmHg, and pulse is 116/min. Mild scleral icterus present. No enlarged lymph nodes or skin rash. Mildly tender liver edge palpable 3 cm below right costal margin. Lab results: CBC - Hgb 9.4 g/dL - Reticulocytes 10% - Platelets 110,000/mm^3 - Leukocytes 12,000/mm^3 Liver function tests - Total bilirubin 4.3 mg/dL - Alkaline phosphatase 150 U/L - AST 62 U/L - ALT 74 U/L Serum LDH 300 U/L Most likely to yield diagnosis in this pt?

*Blood smear exam* Babesiosis - tick-born protozoal illness endemic to *northeastern* US - via ixodes scapularis 48-72 hrs after attachment - may = coinfection with Borrelia burgdorferi, Anaplasma phagocytophilum - symptoms = flu-like = fever, chills, malaise - laves = intravascular hemolysis, thrombocytopenia - diagnosis via peripheral blood smear = Maltese cross

A 19-year-old female is brought to a psychiatry clinic by her parents stating that she is "too skinny." On exam, you note an extremely thin female, with body downy hair. Which of the following, if found during the history, physical, or subsequent labs, would result in hospitalization for this patient?

*Body weight less than 75% of average* Anorexia nervosa (AN). - *Patients with body weight less than 75% of average should be hospitalized* - excessive dieting with or without purging due to a distorted body image In someone with anorexia nervosa, there are 4 equally serious, interrelated electrolyte abnormalities: hyponatremia, hypokalemia, contraction metabolic alkalosis, and hypophosphatemia. - Generally volume depleted from excessive vomiting, generating the contraction metabolic alkalosis. - The metabolic alkalosis is maintained by volume depletion, low chloride, and low potassium levels. - Basic metabolic panel should be checked at least twice a day initially. - These can all be reversed initially with intravenous isotonic saline and potassium chloride. - The reason why all of these are interrelated and potentially deadly is that they can all contribute to cardiac arrhythmia (hyponatremia, hypokalemia, hypophosphatemia) and respiratory collapse (the response to metabolic alkalosis is respiratory acidosis, which decreases respirations).

67 yo man brought to ED after syncopal episode. He lost consciousness while shopping at mall. He has no nausea, diaphoresis, chest pain, or SOB. Pt has had 2 episodes of lightheadedness over past mo but has not sought treatment. His other med probz include long-standing HT, which is being treated with enalapril. His BP is 135/90 mmHg while supine and 130/85 mmHg while standing; pulse is 64/min. ECG shows sinus rhythm with high voltage, prolonged PR interval, prolonged QRS-complex duration, normal QTc interval, and occasional premature ventricular contractions. BMP is unremarkable. Echo shows left ventricular hypertrophy and ejection fraction of 55%. Most likely cause of this pt's syncope?

*Bradyarrhythmia* Syncope - abrupt and transient loss of consciousness with loss of postural tone, followed by spontaneous and complete recovery - usually benign & self-limiting but can be initial presenting symptom of life-threatening dz process This pt: conduction system abnormality - prolonged PR interval & intraventricular conduction delay (prolonged QRS-complex duration) on initial ECG --> suggestive of *bradyarrhythmia* or high-grade AV block AV block with associated bradyarrhythmia - can be intermittent - should be referred for additional electrophysiologic testing and eval of pacemaker implant

A 5-day-old full-term infant presents in the winter with decreased activity and a temperature of 100.5 F. His mother states that he has been fussy during breastfeeding and has some nasal discharge. His exam is notable for decreased activity, yellow sclerae, and subcostal retractions with nasal flaring. Which of the following is the most appropriate next step for the care of this infant?

*CBC, UA, urine and blood cultures, lumbar puncture* Index of suspicion for sepsis must be high in neonates because signs can be subtle and nonspecific, and there is serious risk for morbidity and mortality from this condition. - *Any ill-appearing neonate (infant in the first 28 days of life) must receive a full work up, including a CBC, UA, urine and blood cultures, and a lumbar puncture (LP)* In a neonate, a fever is any temperature greater than 100.4F. - Since neonates do not have a fully developed immune system, they may not appear as ill as an older child or adult with sepsis. In particular, jaundice between days 3-7 of life in an ill-appearing infant is very concerning for sepsis. *Normally the CBC only takes an hour or two to come back and the white blood cell count can indicate whether the etiology is infectious. Once we receive that, we may start the antibiotics because some antibiotics (some cephalosporins) can actually harm the liver of the baby and worsen the jaundice.*

62 yo woman sent to ED from her rehab facility due to sudden-onset dyspnea and nonproductive cough. Symptoms started acutely while pt was trying to get out of bed. She has no subjective fever, hemoptysis, wheezing, palpitations, leg pain, or LE edema. Pt underwent right knee replacement 2 wks ago for severe degenerative joint dz. Her other med probz include PUD from NSAIDs, HT, and T2DM. She drinks alcohol socially and doesn't use tobacco. Her temp is 100.4 F, BP is 130/80 mmHg, pulse is 110/min, & respirations are 22/min. Pulse oximetry is 91% on RA. BMI is 36 kg/m^2. Pt alert & cooperative without cyanosis or jaundice. Lungs clear to auscultation. Abdomen soft, nondistended, and nontender. Right knee without erythema, warmth, or tenderness. ECG shows sinus tachycardia. Chest x-ray normal. CBC shows leukocyte count of 6,500/mm^3. Best next step in management of this pt?

*CT angiogram of chest* - test of choice in clinically stable pts in whom PE is likely sudden-onset dyspnea + nonproductive cough + tachycardia + mild hypoxia = acute PE This pt's Wells score = 6 - absence of more likely diagnosis = 3 - tachycardia = 1.5 - immobilization via recent knee sx = 1.5 - PE more likely

A 48-year-old man presents to the emergency department with 30 minutes of severe chest pain radiating to the back. He has no history of angina or known cardiac disease. His uncle died suddenly of an unknown cause at age 50. Vitals include T 37.3 C, BP 160/60 mmHg, HR 100/min, RR 20/min. On physical exam, he is 6'6" tall and has a high-arched palate. The appearance of his chest and fingers are shown. Cardiac auscultation reveals a decrescendo diastolic murmur at the third left intercostal space. The rest of the physical exam is within normal limits. What is the next best step in the management of this patient?

*CT angiography of the chest* CTA is likely needed for surgical planning purposes in the setting of an acute dissection. *Marfan syndrome and a family history of possible sudden death from aortic disease most likely has a thoracic aortic dissection, which should be evaluated with CT angiography of the aorta.* Aortic dissection - medical emergency and can quickly lead to death, even with optimal treatment, due to decreased blood supply to other organs, cardiac failure, and sometimes rupture of the aorta - The diagnosis is made with medical imaging (computed tomography, magnetic resonance imaging or transesophageal echocardiography).

A 47-year-old male has been feeling fatigued and has gained 20 pounds during the last two months. On exam, you note central obesity with proximal muscle atrophy and weakness. Vital signs are as follows: T 98.4 F, BP 155/90 mmHg, HR 85 bpm, RR 14 rpm. BMP is as follows: Na 140, K 3.9, Cl 102, CO2 23, BUN 19, Cr 0.9, Glu 115. 24-hour urinary free cortisol level is 165 nmol/day (upper range of normal 110-138 nmol/day) and morning serum ACTH is 83 pg/mL (normal is 9 - 52 pg/mL). Two days later, he receives a high-dose 2 mg dexamethasone suppression test orally every 6 hours for 48 hours. The following morning at 9 am, his serum cortisol level is 300 nmol/L (normal is 50-200 nmol/L). At 48 hours it is 250 nmol/L. Which of the following should be the next step?

*CT of the thorax and abdomen* Ectopic ACTH production from a tumor in the abdomen or thorax - *High resolution CT will localize the tumor in the majority of cases* - fatigue, central obesity, and proximal muscle wasting with an elevated 24-hour urinary cortisol level are indicative of hypercortisolism - his elevated ACTH suggests that this is an *ACTH-dependent process* - the failure of the high-dose dexamethasone to suppress his cortisol level indicates this is an ectopic source of ACTH Diagnosis: 1. Overnight dexamethasone suppression test should => low ACTH + low cortisol in the morning (many false positives though) 2. 24 hour urinary cortisol collection (if dexamethasone test fails) - labor intensive GOLD STANDARD TEST -If abnormal test then this is Cushing's, must figure out if in pituitary, lung, or adrenals 1. Give HIGH DOSE DEXAMETHASONE SUPPRESSION TEST -Suppression = pituitary adenoma (more benign and suppressible) -Then do an MRI then surgery -No suppression = ACTH producing tumor (lung cancer) or adrenal neoplasia -Measure ACTH, high = tumor (CT of chest, usually not resectable), low = neoplasia (do imaging w/MRI or CT and find it) -Hyperpigmentation in ACTH producing tumor from Pro-opiomelanocortin and MSH over production -No hyperpigmentation in adrenal neoplasia => no ACTH

76 yo man admitted to coronary care unit after episode of substernal chest pain. His other med probz include HT, hyperlipidemia, and T2DM. He has ho diverticular bleed 2 yrs ago. After initial workup, cardiac cath performed & shows 70% left main coronary artery stenosis, 90% proximal LAD descending artery stenosis, & 80% right coronary artery stenosis. Antiplatelet agents stopped, & pt continued on heparin drip in prep for coronary artery bypass surgery the next day. 5 hrs after cath, his BP is 75/60 mmHg & pulse is 120/min & regular. He complains of some generalized weakness & back pain but denies chest pain, SOB, nausea, & abdominal discomfort. On PE, he appears diaphoretic and clammy. Neck veins flat. Heart sounds normal, & chest is clear to auscultation. Right groin arterial puncture site mildly tender, without any swelling or bruits. He receives 1000 mL of normal saline with symptomatic improvement. His BP is 96/60 mmHg and pulse 85/min. His repeat echo (ECG) is unchanged from initial ECG at presentation. Most appropriate next step in management of this pt?

*CT scan of abdomen and pelvis without contrast* Recent cardiac cath + anticoag with heparin + sudden onset of hypotension + tachycardia + flat neck veins + back pain = *retroperitoneal hematoma* - via bleeding from arterial access site (with retroperitoneal extension) - most hemorrhage or hematoma formation occurs within 12 hrs of cath - localized discomfort and/or swelling of soft tissue - above inguinal ligament --> hematoma can extend into retroperitoneal space = hemodynamic instability and ipsilateral flank or back pain

A 25-year-old male presents to a neurologist with headaches. He notes that the headaches usually occur at night, and that they are usually left-sided and centered behind his eye. The headaches last two hours at a time, and he has experienced several headaches per day for the past four days. He notes a similar experience several months ago, which resolved after a week. His neurologist prescribes a medication for headache prophylaxis. What is the most likely class of medication prescribed?

*Calcium channel blocker* Cluster headaches - can be treated with verapamil for prophylaxis - characterized by multiple headaches in a short period of time that last for more than 1.5 hours each, and often occur at night - usually unilateral and retroorbital - can be associated with an ipsilateral Horner's syndrome - more common in men than women Oxygen is the mainstay of acute treatment, and verapamil and lithium are the most well-known prophylactic treatments

55 yo man brought to ED with sudden onset of palpitations and chest tightness. His other med probz include HT, gout, and T2DM. Cardiac monitoring shows atrial fibrillation at rate of 120-140/min. His initial BP is 112/70 mmHg and oxygen sat is 92% on RA. As nurse is attempting to establish IV access, pt becomes unresponsive. There is no palpable pulse over carotids or femoral arteries, and he has agonal breathing. Cardiac monitor still shows atrial fibrillation at same rate. What is the best next step in management of this pt?

*Chest compressions* Pulseless electrical activity (PEA) - organized rhythm on cardiac monitoring without measurable BP or palpable pulse in cardiac arrest pt *Recent advanced cardiac life support guidelines recommend managing PEA with CPR & vasopressor therapy (epinephrine) to achieve adequate cerebral and coronary perfusion* - defib or synchronized cardioversion has no role in management of these pts

32 yo man comes to office due to 3 days of fever, malaise, and cough productive of clear sputum. He has had no nasal congestion, rhinorrhea, sore throat, or chest pain. Pt has mild, intermittent asthma and seasonal allergic rhinitis. He doesn't use tobacco but drinks alcohol occasionally. His 2 yo son had fever, cough, and rhinorrhea last wk. Temp is 100 F, BP is 120/80 mmHg, pulse is 92/min, respirations are 20/min, and pulse oximetry is 96% on RA. Oropharynx is normal, & palpation of neck shows no cervical LAD. Lung auscultation reveals crackles at right lung base and occasional expiratory wheezing. Heart sounds normal. Best next step in management of this pt?

*Chest x-ray* Community-acquired pneumonia - pulmonary parenchymal infection - symptoms develop acutely - fever, cough, pleuritic chest pain, dsypnea - tachycardia, tachypnea, pulm auscultation abnormalities may be present - diagnosis requires presence of lobar, interstitial, cavitary infiltrate on chest imaging = usually chest x-ray

26 yo man comes to doc with 1 wk ho dysuria and increased urinary frequency. He has had multiple sexual partners in past mo and is inconsistent with condom use. His temp is 98.9 F, BP is 110/70 mmHg, and pulse is 68/min. Mucopurulent discharge seen at urethral meatus. Urinalysis shows: Blood Negative Glucose Negative Ketones Negative Leukocyte esterase Positive Nitrites Negative WBCs 50-100/hpf RBCs 0-1/hpf Bacteria None Gram stain of discharge shows no bacteria. Culture of discharge and urine show no growth after 48 hrs of incubation. Most likely diagnosis?

*Chlamydial urethritis* - no growth on gram stain & urine culture (culture-negative urethritis) - diagnosis made via nucleic acid amplification testing of first-catch urine sample - treatment = azithromycin or doxycycline Urethritis - dysuria + pyuria (WBCs > 10/hpf) + urinary frequency + urethral discharge

A 61-year-old female presents to her gynecologist with complaints of increasing abdominal distension over the last 3 months as well as more recent onset of early satiety and constipation over the last several weeks. A pelvic exam is performed and is significant for a sizable right ovarian mass. Pelvic ultrasound is then conducted and confirms the presence of a right ovarian mass. Her physician suspects cancer and would like to pursue a work-up. Baseline serum CA-125 level, CT abdomen/pelvis, chest radiograph, mammogram, and a pap smear are obtained. Which of the following should also be included in this patient's work-up?

*Colonoscopy* Age and clinical presentation make ovarian cancer the most likely diagnosis: *Pre-operative work-up in suspected ovarian cancer should include baseline serum CA-125 level, CT abdomen/pelvis for staging, chest radiograph, mammogram, pap smear, and colonoscopy, particularly when a patient is experiencing gastrointestinal symptoms* Ovarian cancer - most deadly gynecologic cancer in the United State - second most common gynecologic cancer behind endometrial cancer Oral contraceptive use is believed to be protective.

49 yo man comes to doc with 3 wk ho fever, wt loss, and anorexia. He also reports muscle aches. He has no cough or SOB. Pt has 15 pck yr smoking hx. His temp is 102 F, BP is 120/76 mmHg, pulse is 90/min, & respirations are 16/min. Lungs clear to auscultation. Rest of PE unremarkable. Both blood cultures grow Streptococcus gallolyticus (Streptococcus bovis type 1). Echo reveals vegetations on the mitral valve. Other than antibiotic treatment, what additional step is recommended to this pt?

*Colonoscopy* Infective endocarditis of mitral valve due to infection with Streptococcus gallolyticus - S bovis biotype 1 = 1 of 4 major species that belong to group D strep - increased risk of colorectal cancer & endocarditis vs. S bovis type II --> should have colonoscopy to look for underlying occult malignancy

A 67-year-old male active smoker with a history of gout, congestive heart failure (ejection fraction 35%), and moderate COPD is hospitalized for a CHF exacerbation. On the third day of his hospitalization, the patient has much improved from a respiratory stand-point but has developed a warm, painful right knee. Of note, the patient's home allopurinol was held during his hospitalization. Which of the following joint fluid analysis results would be most consistent with a diagnosis of recurrent gout? Color: Clarity: WBC: Bacteria:

*Color: yellow; Clarity: cloudy; WBC: 20000 (70% neutrophils); Bacteria: none* The synovial aspirate from this patient with a recurrence of *gouty arthritis* is most likely to yield cloudy yellow fluid with 2000-50000 WBC (70% neutrophils), needle-shaped negatively birefringent crystals, and no bacteria, unless the joint is superinfected. Gout - common form of recurrent mono-articular inflammatory arthritis cause by the deposition of monosodium urate crystals in the joint space - Elevated urate levels can be due to decreased excretion of uric acid secondary to renal failure, hypertension, thiazide diuretics, or alcohol, or due to increased production of uric acid secondary to obesity, alcohol, hemolytic disease, or a purine rich diet. - Acute gout is treated with intra-articular steroids or NSAIDs such as indomethacin, whereas gout prevention is achieved with allopurinol, colchicine, or probenecid.

A 24-year-old G1P1001 female presents with primary complaints of facial acne and lack of menses for the past 7 months. She does not currently desire to become pregnant. Her temperature is 37.3 deg C (99.1 deg F), blood pressure is 135/76 mmHg, pulse is 74/min, respirations are 14/min, O2 Sat is 99% on room air, and BMI is 34.5. Physical exam is significant for comedonal acne across her forehead and chin as well as the presence of terminal hairs on her upper lip, chin, and chest between her breasts. TSH and prolactin levels are normal. Serum LH is elevated. Serum testosterone level is elevated and fasting blood glucose level is 135 mg/dL. On physical exam the physician observes darkening of the axilla. Which of the following pharmacologic treatment regimens would be the most appropriate therapy for this patient?

*Combined oral contraceptives & metformin* PCOS - The first step in management is exercise and weight loss, and pharmacologic treatment of PCOS includes combined oral contraceptives and metformin to improve insulin sensitivity. For this patient, the metformin is indicated for her insulin resistance. = Evidence of her insulin resistance includes her axillary skin darkening (acanthosis nigricans) and her fasting blood glucose of 135 (normal < 100). - Metformin has been shown to reduce insulin resistance in PCOS patients and subsequently prevent future development of cardiovascular disease and frank diabetes. - In addition, metformin also decreases hyperandrogenic symptoms, which would be helpful in this patient with hirsutism. - - Furthermore, she is starting a COC simultaneously, which will help prevent pregnancy in addition to its antiandrogenic benefits in this patient.

46 yo man comes to doc with exertional dyspnea and dry cough. He also has occasional episodes of suffocating nighttime cough that is only relieved by sitting up. PMHx significant for MI 6 mos ago and hypercholesterolemia. Currents meds include metoprolol, aspirin, and rosuvastatin. Pt doesn't use tobacco or illicit drugs but drinks alcohol on social occasions. His dad died of a stroke and his mom has T2DM. His BP is 150/100 mmHg and pulse is 60/min. Chest exam shows bibasilar crackles. Cardiac apex palpated in left sixth intercostal space. Bilateral pitting leg edema present. Most likely to be associated with this pt's condition?

*Constriction of efferent renal arterioles* exertional dyspnea + paroxysmal nocturnal dyspnea + pulmonary and peripheral edema + ho MI = decompensated *CHF* - decreased CO --> neurohumoral adaptations = increased sympathetic nervous tone, RAAS activation, increased secretion of ADH CHF: decreased renal perfusion --> RAAS activation --> increased ang II Ang II increase... - vasoconstriction of both afferent and efferent glomerular arterioles -- increase in renal vascular resistance and net decrease in renal blood flow - *preferential vasoconstriction of efferent renal arterioles* --> increased intraglomerular pressure

A 46-year-old female with a history of type I diabetes mellitus presents with complaints of chronic fatigue, muscle weakness, and headaches. These symptoms have been present for the past year. Her vital signs today are as follows: T 36.9 C, HR 82 bpm, BP 88/52 mmHg, RR 14 rpm, O2 sat98% on room air. Physical exam does not demonstrate any specific abnormalities. After inquiry, the patient reports that her blood pressure has always "run low." Initial work-up reveals an 8AM cortisol level of 3.6 ug/dL. Which of the following is the best next step in management of this patient?

*Cosyntropin stimulation test* Fatigue, weakness, and headaches in the setting of low blood pressures and a decreased 8 AM cortisol level is suggestive of adrenal insufficiency. - A cosyntropin stimulation test is the next diagnostic step in differentiating primary from secondary adrenal insufficiency and determining the etiology of this patient's low cortisol level. Cosyntropin stimulation test involves measuring baseline serum cortisol levels prior to the injection of cosyntropin (synthetic ACTH) --> Serum cortisol is then measured again one hour after the injection. - A sufficient increase in serum cortisol after administration of cosyntropin rules out primary adrenal insufficiency and necessitates further work-up for other etiologies. - An insufficient or absent rise in serum cortisol in response to the injection (typically, a serum cortisol level less than 18 ug/dL) is suggestive of primary adrenal insufficiency (Addison's disease). Primary adrenal insufficiency is often associated with a constellation of other autoimmune diseases such as autoimmune thyroid disease or type I diabetes mellitus.

A 35-year-old woman presents to the emergency room after a syncopal episode at her workplace. She has a history of type 1 diabetes and premature ovarian failure and is a current smoker. Over the past 6 months, she has noticed increasing fatigue and unintentional weight loss. On exam, she has a generalized bronzing of her skin. Labs reveal that she is hyponatremic and hyperkalemic. Her vitals are otherwise within normal limits. The physician suspects an autoimmune process though is uncertain. Which of the following is the most accurate confirmatory test of the most likely diagnosis?

*Cosyntropin test* Primary adrenal insufficiency (Addison's disease) - most often caused by autoimmune disease in developed countries - *most accurate test for this disease is a cosyntropin (ACTH) stimulation test or a CT scan of the adrenals* -associated with a number of nonspecific symptoms including chronic malaise, weakness, and weight loss - physical exam findings include postural hypotension and hyperpigmentation due to increased production of pro-opiomelanocortin (POMC), a prohormone that is cleaved into ACTH and melanocyte stimulating hormone - cardinal lab abnormalities are hyponatremia and hyperkalemia due to mineralocorticoid deficiency The specific and unique symptoms in this patient are these vague symptoms + skin hyperpigmentation, hyponatremia and hyperkalemia. - The syndrome that presents in this manner is primary adrenal insufficiency as the hyperpigmentation can be explained from the increased MSH release (and ACTH) and the hyperkalemia and hyponatremia suggests a deficiency of aldosterone.

22 yo woman brought to office by her mom due to recurrent syncopal episodes. First episode occurred about a year ago when her roommate committed suicide. Pt has had several similar episodes since then, often provoked by strong emotion. Episodes are preceded by lightheadedness, weakness, and blurred vision; last about 3 minutes; and end with rapid recovery of consciousness. She has had no significant injuries related to syncope except once when she had superficial bruises. Her med hx is insignificant. Pt takes no meds and doesn't use alcohol or illicit drugs. Her BP is 110/70 mmHg while supine and 108/70 mmHg while standing. PE findings within normal limits. ECG performed mo ago was normal. Best next step in management of this pt?

*Counterpressure maneuver education* Neurocardiogenic/vasovagal syncope - can be triggered by emotion or painful stimuli - frequently associated with prodromal symptoms prior to episode - recurrent syncope --> advise pts to avoid triggers & assume position with leg raising at onset of symptoms - physical counterpressure maneuvers = leg crossing with tensing of maneuvers, handgrip, tensing of arm muscles with clenched fists during prodromal phase --> improves venous return & CO --> can abort episodes

55 yo truck driver comes to office due to impaired vision over past 6 mos. He has had increasing difficulty when looking sideways and now has to completely turn his neck to look to side. He also has intermittent headaches that worsen in morning. Pt recently developed decreased libido, which he attributes to his long work hours and night shifts. He has no fever, weakness or numbness, or bladder/bowel symptoms. His BMI is 32 kg/m^2. Exam shows decreased vision in temporal fields on both sides. Neurologic exam shows 5/5 muscle strength, normal sensation, normal gait and coordination, and 2+ deep tendon reflexes. Most likely diagnosis?

*Craniopharyngioma* - sellar mass --> visual defects (bitemporal hemianopsia) + headache + symptoms of pituitary hormonal deficiency (decreased libido due to hypogonadism) - arises from Rathke's pouch - most commonly in children - 50% >20 yo, especially 55-65 yo - grow gradually over time - can compress optic chiasm --> bitemporal blindness - diagnosis confirmed with MRI or CT - treatment = surgery and/or radiotherapy

27 yo woman brought to ED by local paramedics. Pt was found unconscious at scene of house fire. On exam, she doesn't appear to have any burns. Black soot is present near pt's nares and mouth. Her capillary refill time is 4 seconds. BP is 132/90 mmHg, HR is 122/min & regular, & respirations are 24/min. Supplemental oxygen by non-rebreather mask is administered. Lab results: Arterial blood gases: pH 7.15 PaO2 114 mmHg PaCO2 33 mmHg Blood plasma, and serum Bicarb 12 mEq/L Lactic acid, venous 20 mg/dL What should be empirically treated in this pt?

*Cyanide poisoning* - can be treated with: hydroxocobalamin or sodium thiosulfate --> binds cyanide - alternative treatment = induction of methemoglobinemia with nitrites --> increases ferric iron (Fe3+) in circulating hemoglobin house fire --> most likely has smoke inhalation injury - 60-80% of deaths during fire incidents - smoke injury --> glottic edema & airway irritation via particulate matter found in smoke - hydrogen cyanide (HCN) & CO = 2 major products of combustion in closed spaces HCN - potent & fast-acting poison - *blood levels can't be measured rapidly to confirm diagnosis prior to treatment* - early acute toxicity = neuro & cardiorespiratory stim --> headache, vertigo, dizziness, hyperventilation, tachycardia, N/V - can cause anoxic brain injury --> permanent neuro deficits *Primary cause of acid-base disturbance = REDUCED OXYGEN UTILIZATION BY TISSUES* - cyanide binds ferric iron in cytochrome oxidase a3 in mitochondrial ETC --> blocks oxidative phosphorylation --> promotes anaerobic metabolic --> lactic acidosis

Recognize *dermatitis herpetiformis* "A 24-year-old woman with a medical history significant for asthma and celiac disease presents with a rash shown in Figure A. She denies any drug or alcohol use. She has been sexually active with her male partner for a year, with intermittent use of condoms. What is the best treatment for this patient's condition?"

*Dapsone* History of celiac disease --> papulovesicular rash is most likely *dermatitis herpetiformis (DH)* - This is best treated with dapsone and a gluten-free diet - relatively uncommon sequelae of celiac disease - presents with erythematous vesicles/bullae and papules that occur symmetrically and bilaterally in a grouped arrangement (hence the "herpetiform" appearance) - rash is intensely pruritic, however, excoriations and erosions are commonly seen on presentation - lesions commonly appear on the buttocks, neck/upper back, and extensor surfaces *first-line treatment for DH is dapsone and elimination of gluten from the diet* - During dapsone therapy, careful laboratory monitoring is necessary, as the drug is associated with a variety of adverse effects

A patient with chronic obstructive pulmonary disease is admitted to the hospital with cholecystitis. He undergoes cholecystectomy uneventfully and is transferred to the floor. Suddenly, he becomes weak and severely hypotensive. If the patient's symptoms are due to adrenal insufficiency from chronic steroid use, one would expect which of the following:

*Decreased ACTH* Secondary adrenal insufficiency - can be caused by chronic glucocorticoid use, which suppresses corticotropin-releasing hormone (CRH) and therefore, ACTH. - found in patients who are on long-term steroid therapy - When these patients develop an illness, undergo trauma, or have a surgical procedure, they are unable to produce adequate cortisol due to suppression of the CRH - ACTH axis by the exogenous steroids - In patients with chronic steroid use, ACTH and cortisol will be low but aldosterone will be relatively normal

A 43-year-old female presents with complaints of heartburn and a "sticking sensation in my throat when I try to swallow liquids or solid foods." Review of systems reveal that her fingers often hurt and change colors to white or blue in cold weather. Physical exam is significant for abnormally taught skin on the patient's hands, shown in Figure A. Which of the following would most likely be found on manometry in this patient?

*Decreased LES tone and aperistalsis in the distal esophagus* Scleroderma / CREST syndrome. - *Manometry in these patients commonly shows absence of peristaltic waves in the lower 1/3 of the esophagus and a significant decrease in lower esophageal sphincter (LES) tone.* - calcinosis, Raynaud's, esophageal dysmotility (secondary to LES sclerosis), sclerodactyly, and telangiectasias - form a systemic scleroderma (sclerosis). - excessive deposition of collagen and is most common in females between the ages of 30-50 - first symptom to present, followed by finger puffiness that later progresses to thickening of the skin of the fingers - young to middle-aged female with a history of Raynaud's who reports or presents with skin hardening and other signs or symptoms of systemic internal organ dysfunction

55 yo woman comes to office via 3 mos of night sweats. She has tried using a fan and sleeping without a comforter, but she continues to awaken several nights a week with bedsheets so drenched that she has to change them. During this time, she has unintentionally lost 11 lb. Pt traveled to Vietnam with her church group 6 mos ago and wonders if she may have "caught something." She took her antimalarial pills and received her pretrip vaccinations as recommended and has not had fevers. Pt doesn't use tobacco, alcohol, or illicit drugs. Temp is 99.4 F, BPis 144/78 mmHg, and pulse is 96/min. She has pale mucosa but no scleral icterus. No cervical LAD. Cardiopulm exam normal. Spleen tip palpable when she exhales completely. Lab results: Hgb 8.7 g/dL Platelets 610,000/mm^3 Leukocytes 66,800/mm^3 - Neutrophils 32% - Bands 12% - Eosinophils 3% - Basophils 5% - Monocytes 4% - Metamyelocytes 12% - Myelocytes 23% - Lymphocytes 9% What is expected in this pt?

*Decreased leukocyte alkaline phosphatase score* - bc neutrophils cytochemically and functionally abnormal in CML CML - myeloproliferative disorder - via BCR-ABL fusion gene - marked dramatic leukocytosis (often but always >100,000/mm^3), absolute basophilia, preponderance of early immature neutrophil precursors Leukocyte alkaline phosphatase score - can help differentiate CML from leukemoid reaction

64 yo man comes to office due to increasing pain in his right groin for past several mos. Pain increases with activity and is relieved with rest. It sometimes radiates to upper thigh. Pt has no ho trauma or falls. There is no associated fever, wt loss, or change in appetite. He has ho lumbar disk herniation but has no back pain currently. His other med probz include T2DM & HT. Pt is 66 in tall & weighs 210 lb, with calculated BMI of 34 kg/m^2. His vitals are otherwise normal. Exam shows pain on passive internal rotation of right hip joint. Direct pressure over groin doesn't increase pain. Reflexes are 2+ in LEs symmetrically, & there are no sensory deficits. Muscle bulk, tone, & power are normal. Dorsalis pedis & posterior tibial pulses are 2+ in both legs. Most likely cause of this pt's hip pain?

*Degenerative joint dz* slowly progressive pain relieved by rest = degenerative joint dz / *osteoarthritis of hip* - most common joint disorder - typically felt in groin, buttock, or pelvis --> can radiate to lower thigh or knee - mild pain & stiffness with prolonged rest - worst pain with activity and wt bearing - symptoms insidious - onset > 40 yo - decreased internal and external rotation

49 yo registered nurse comes to office due to sleep difficulties since receiving promotion to administrative position in hospital several wks ago. She finds her new job and its many responsibilities stressful. Her sleep schedule changed when she took the new position; she used to work the 4:00 PM - 12:00 AM shift and had no sleep difficulties. Not she has to be at the hospital by 7:30 AM. Pt goes to bed at 11:00 PM but cannot fall asleep until 2:00-3:00 AM. She then awakens by alarm at 6:15 AM to get to work on time. She is exhausted all the time and has trouble concentrating and staying awake during meetings. Pt is increasingly frustrated by her sleep probz, which have occurred in past with jobs or schooling requiring early start times. She has also been more irritable lately and feels down at times. Pt has same difficulty initiating sleep on weekends but feels better than as she is able to sleep until 11:00 AM. She doesn't use alcohol or tobacco. Most likely cause of her sleep difficulties?

*Delayed sleep phase syndrome* - lifelong pattern of sleep difficulties that worsen with early-morning start times - circadian rhythm sleep-wake disorder - sleep-onset insomnia - excessive morning sleepiness - occur when internal clock regulating sleep & wakefulness is misaligned with person's desired sleep time or social/professional schedule - "night owls"

46 yo hospitalized man evaluated for confusion. He was admitted 3 days ago after motor vehicle collision in which he was a passenger. He sustained fractures of right tibia and fibula, which were surgically reduced and fixed. Pt had small subdural hematoma without midline shift, which were managed nonoperatively. He has been receiving morphine for pain control. This morn, pt was agitated and disruptive and was talking to persons who were not there. He has no significant med hx and takes no regular meds. Temp is 99.5 F, BP is 170/100 mmHg, pulse is 120/min & regular, and respirations are 22/min. Pulse oximetry shows 96% on RA. Pt appears diaphoretic and tremulous and is trying to pull out IV lines. He is not cooperative for fundoscopic exam, but pupils are equal and reactive bilaterally. Breath sounds normal and no heart murmurs heard. Abdomen soft and nontender. Surgical incision shows no surrounding erythema. Pt moves all extremities without limitations. Most likely cause of this pt's acute confusion?

*Delirium tremens* Hospitalized for 72 hours + onset of acute change in cognition and attention + tachycardia + HT + tremulousness + diaphoresis + agitation + hallucinations = *delirium tremens* - via alcohol withdrawal Chronic alcohol use = insensitivity to GABA - may go undetected in pts who have trauma or encephalopathy Early withdrawal: 6-36 hours - tremor + anxiety + headache + normal sensorium Neuropsychiatric: 12-48 hours - generalized seizures + hallucinations DT: 48-96 hours - altered sensorium + agitation + tremor + autonomic instability Management = ruling out other delirium = nutritional and electrolyte supplementation + benzos

A 25-year-old pregnant woman of 28 weeks gestation presents with a severe persistent headache and visual changes. On physical exam her blood pressure was noted to be 175/112 and a spot urine dipstick shows 4+ protein. Which of the following interventions is most definitive treatment?

*Deliver promptly* The patient is presenting with classic signs and symptoms of severe preeclampsia with end-organ damage, which requires immediate delivery as treatment. Severe preeclampsia (BP > 160/110 and 3-4+ urine dipstick protein) with evidence of end-organ damage, including headache, epigastric pain, disseminated intravascular coagulation, oliguria, or pulmonary edema, is an obstetrical emergency that is managed by delivery regardless of gestation age. - If no end-organ damage is present, it is possible to monitor the patient in an ICU setting until 32 weeks gestation, when delivery is indicated regardless of end-organ damage.

27 yo woman comes to office requesting prescription for sleep med. Pt has been extremely upset and has had difficulty falling asleep since her bf broke off their relationship a mo ago. She says, "I don't know how I can go on without him. I lie awake at night worrying about how I'm going to handle everything. He was helping me apply for a new job, but now I worry about making a bad move and I'm not sure what to do. So I just avoided the whole issues and cancelled the interview." Lately, she has avoided socializing, as her bf was the one who initiated friendships and made their plans. Pt says that although she often has a bf, she has always struggled with her self-confidence and fears of rejection and abandonment. She appears anxious and sad but brightens easily. Most likely diagnosis in this pt?

*Dependent personality disorder* - excessive dependency on her bf - indecisiveness - fear of being alone - feelings of inadequacy

A 26-year-old first-year medical student with a medical history significant for GERD comes to your office because of frequent episodes of palpitations. The palpitations are sudden in onset and are accompanied by sweating and a sense that she is going to "pass out." The episodes typically last no more than 10 minutes, and although the patient feels as if she may syncopize, she never has. The episodes first appeared when she started medical school and have increased in frequency to the point where she is afraid to attend lectures out of fear of having an "attack." What is frequently associated with this patient's condition?

*Depression* Panic disorder - *patients with panic disorder are at an increased risk of depression and suicide* - depression and anxiety often coexist with panic disorder, especially when the disorder begins in adolescence - up to 60% of patients diagnosed with panic disorder have had one or more lifetime episodes of depression - also associated with agoraphobia, bipolar disorder, and substance abuse

A 37-year-old female presents to the general medical clinic with chest pain. She reports that the pain occurs at rest and feels as if an elephant is sitting on her chest. She has no other complaints. Her past medical history is significant for migraines but she has no history of smoking, hypertension, hyperlipidemia, or diabetes. Her vital signs are temperature 37 degrees Celsius, HR 70/minute, BP 110/80, RR 16/min, and oxygen saturation 99% on room air. Her physical examination reveals no murmurs. An EKG in the office during one of the episodes reveals the following as shown in figure A. Troponins are positive. She is admitted to hospital and undergoes emergent cardiac catheterization, where she is without obstructive coronary disease, but her symptoms can be provoked with administration of intravenous ergonovine. Which of the following treatments would be appropriate in this patient?

*Diltiazem* Presentation and provocation of symptoms with ergonovine are consistent with *variant or Prinzmetal's angina.* - *Treatment should include calcium channel blockers and/or nitrates to prevent coronary vasoconstriction.* Variant angina - caused by transient coronary vasospasm that is often accompanied by a fixed atherosclerotic lesion but may also occur in normal coronary arteries - pathophysiology of the condition is thought to be related to vascular endothelial dysfunction - *key finding on EKG is transient ST segment elevation resulting from transmural ischemia.* - *Diagnosis may be confirmed with coronary catheterization and the provocation of vasospasm with the administration of IV ergonovine.* - Beta-blockers and aspirin may worsen this type of angina, since they can cause vasoconstriction. *Figure depicts the classic ST elevation seen in Prinzmetal's angina.*

70 yo man brought to ED by his wife due to rectal bleeding. Pt initially had bowel movement consisting of large volume of bright red blood without associated abdominal pain. Since then, he has had persistent lightheadedness along with several urges to defecate in which blood is produced. Pt has no prior hx of GI bleeding. Past med hx notable for HT and chronic constipation. Currents meds include chlorthalidone and low-dose aspirin. Temp is 97.8 F, BP is 85/45 mmHg, pulse is 120/min, & respirations are 20/min. Abdomen is soft, nondistended, and nontender; bowel sound are normal. No masses or organomegaly palpable. Rectal exam shows bright red blood. Nasogastric aspiration returns nonbilious stomach contents without blood. Plain radiograph of abdomen normal. Most likely cause of this pt's bleeding?

*Diverticulosis* - large-volume rectal bleeding - most common cause of lower GI bleeding in adults - most common in sigmoid colon, but diverticular bleeding more common in right colon - typically painless - diagnosis confirmed by colonoscopy

A 10-year-old boy presents to his oncologist with a rash. Forty days prior he received a stem cell transplant for acute lymphoblastic leukemia. However, he has been unable to take his immunosuppressants consistently. On exam he has erythematous macules on his palms, soles, legs and ears (flip for image). What is the most likely underlying cause of his clinical presentation?

*Donor T cells against host tissue* Given the patient's physical exam findings and history of inconsistent medication use, the patient most likely has *acute graft versus host disease* - via activation of donor T lymphocytes against the patient's cells. GVHD - immunocompetent T cells from the donor tissue target recipient tissues following activation of host antigen presenting cells - response is mostly mediated by Th-1 cells and is accompanied by a release of interferon-gamma, interleukin-2, and tumor necrosis factor alpha (TNF-alpha) - clinically, the most commonly affected organs are the skin, gastrointestinal tract, and liver - in addition to the above skin findings, erythroderma, bullae, and skin sloughing may develop - widespread erythrodermic changes carry a poor prognosis. *Figure demonstrates the characteristic skin findings of dusky, erythematous macules on the soles, and in this this example, the legs, of an affected patient*

75 yo man comes to office due to intermittent right eye vision loss. Pt reports 3 episodes over past 2 mos in which he suddenly experienced "a curtain falling over right eye" for several mos before it spontaneously resolved. He has no eye pain or discomfort, focal weakness, numbness, or headache. His med hx is significant for HT, hyperlipidemia, and OA. He smoked cigz for 25 yrs before quitting at age 50. BP is 140/85 mmHg and pulse is 74/min. Visual acuity, pupillary reflex, and fundoscopic exams are unremarkable. Further neuro exam shows normal reflexes, motor strength, and sensation. Most likely to yield diagnosis in this pt?

*Duplex ultrasound of neck* Amaurosis fugax - painless, rapid, transient (<10 min) monocular vision loss - curtain descending over visual field - most common etiology = retinal ischemia via atherosclerotic emboli originating from ipsilateral carotid artery - *Pts with vascular risk factors (HT, hyperlipidemia, smoking) = eval with duplex US of neck*

A critically-ill 65-year-old man with coronary artery disease is hypotensive following a large anterior wall myocardial infarction. His extremities are cool to the touch. What would pulmonary artery catheterization likely show with regards to right atrial pressure (RAP), pulmonary artery wedge pressure (PAWP), cardiac output (CO), and systemic vascular resistance (SVR)?

*Elevated RAP, elevated PAWP, decreased CO, elevated SVR* Hypotension + poor perfusion following myocardial infarction is likely suffering from *cardiogenic shock* Pulmonary artery (PA) catheterization in cardiogenic shock typically reveals: elevated RAP, elevated PAWP, decreased CO, and elevated SVR PA catheterization (also called Swan Ganz catheterization) can be used to evaluate the hemodynamic status of critically ill patients. - Insertion of the PA catheter allows for direct measurement of RAP, right ventricular pressure, PAP, and indirect assessment of left atrial pressure, left ventricular end diastolic pressure, CO, and SVR. Cardiogenic shock occurs in approximately 7% of people admitted to the hospital for acute myocardial infarction. - It is usually a result of left ventricular infarction, but may also be caused by cardiac structural defects, such as mitral valve regurgitation, ventricular septal rupture, or cardiac tamponade. Inferior wall MI Best initial test: A EKG will show elevation in II, III and avF - very important! Diagnostic test of choice: Right sided EKG Treatment: Fluids! Inferior wall MI's can cause bradycardia as it sits on the diaphragm and irritates the vagus.

33 yo woman comes to office with intermittent dizziness. Pt describes sensation of severe spinning accompanied by intense nausea that lasts 1-2 hrs. She feels unsteady during these episodes and has to lie down with her eyes closed for relief. Pt has had several similar episodes during past 2 yrs and has not noted any particular factors that ppt the symptoms. She also reports hearing "mechanical humming" sound in her rt ear during these episodes, causing distortion of speech. Pt has no associated headaches, ear pain, or ear discharge. Her temp is 98 F, BP is 130/84 mmHg, & pulse is 86/min. On exam, air conductance is greater than bone conduction in both ears. When base of tuning fork is placed against her forehead, sound is heard more prominently in left ear. Most likely cause of this pt's condition?

*Elevated endolymphatic pressure* Meniere dz - disorder of inner ear - increased volume & pressure of endolymph - damage to vestibular & cochlear components of inner Triad - low-frequency tinnitus in affected ear + feeling of fullness - episodic vertigo + lightheadedness, nausea, vomiting - sensorineural hearing loss

Classical cause of fever from 48-72 hours (2-3 days) postpartum.

*Endometritis* - Risk factors for endometritis include emergent cesarian section and prolonged labor. - A longer labor provides increased time for bacteria to ascend into the uterus. - Notable physical exam findings include exquisite uterine tenderness and foul-smelling vaginal discharge. - The treatment of choice for endometritis is gentamicin and clindamycin because most cases are polymicrobial infections consisting of aerobes and anaerobes. In general you can recall postpartum fever with the following memory aid: Wind (atelectasis)- day 0 Water (UTI) - day 1-2 (note in surgery they will often make this day 3) Womb (endometritis) - day 2-3 Wound (infection) - day 4-5 Walk (septic pelvic thrombophlebitis) - day 5-6

A 38-year-old gentleman presents to the emergency room complaining of vague abdominal pain. He states that when he eats, he seems to get full a lot quicker than he used to. Upon further questioning, it is discovered that he was in a motor vehicle accident several weeks prior. He did not sustain any life threatening injuries at that time, but he states that he did experience a significant "blow to the chest" from the steering wheel. Physical examination elicited a vague epigastric mass and a normal cardiovascular assessment. Additional workup confirms the physician's initial clinical suspicion. What would be the most appropriate treatment for this individual?

*Endoscopic cystogastrostomy* Pancreatic pseudocyst as a complication of his traumatic motor vehicle accident from several weeks prior. - collections of fluid, tissue, and necrotic debris around the pancreas - often associated with chronic pancreatitis, trauma to the chest (steering wheel trauma), and even acute pancreatitis. - have an obscure presentation with vague abdominal mass findings along with abdominal pain - *includes endoscopic cystogastrostomy (endoscopic drainage through the gastric tissue) or other surgical decompression techniques*

After leaving him alone for approximately 5 minutes, a mother walks into the kitchen to find her 2-year-old child holding an empty bottle of oven cleaner (main ingredient sodium hydroxide). While he is in no apparent distress, she immediately takes him to the emergency department. He is seen within an hour of ingestion. What is the appropriate management of this patient?

*Endoscopy* Ingestion of caustic substances including household cleaners containing alkaline compounds such as sodium or potassium hydroxide, can cause severe damage to the larynx, pharynx, and esophagus. - Endoscopy should be completed to determine the extent of the damage. - To assess the extent of injury, direct visualization using a flexible endoscope should be completed. - The endoscope can be advanced beyond any sites of mild injury, but should not go beyond any circumferential grade 2 or 3 burns. - New literature recommends completing the endoscopy within a 12 to 24 hours window to allow for early assessment and treatment. Endoscopy after 48 hours is discouraged due to increased risk of perforation. Lastly, most agree that strong alkali ingestion mandates endoscopy.

32 yo man comes to ED with worsening SOB. Pt has 20 yr ho asthma and was in his usual state of health until 4 days ago, when he developed low-grade fever, rhinorrhea, and generalized body aches. 2 days later, he developed dyspnea & dry cough. Pt has no chest pain, & his as-needed albuterol didn't provide much help. Temp is 98.6 F, BP is 110/70 mmHg, pulse is 120/min, & respirations are 28/min. Pulse oximetry shows 84% on RA & 92% on 3L/min of oxygen. Pt is using accessory muscles of respiration. He is unable to lie supine. Exam shows markedly diminished breath sounds bilaterally with faint wheezes. Chest x-ray reveals no infiltrates but hyperinflated lungs. CBC shows leukocytes of 11,000/mm^3. Pt started on albuterol nebulization and inhaled ipratropium, & he is given dose of methylprednisolone. One hr later, arterial blood gas on 3 L/min of oxygen is obtained. Results: pH 7.32 PaO2 65 mmHg PaCO2 50 mmHg Best next step in management of this pt?

*Endotracheal intubation* Severe asthma exacerbation + signs of impending respiratory failure - *elevated or even normal PaCO2 = inability to meet increased resp demands* via resp muscle fatigue & impending resp failure - should receive inhaled SABA, inhaled ipratropium, systemic corticosteroids Other signs/symptoms of impending resp failure - decreased breath sounds - absent wheezing - decreased mental status - marked hypoxia with cyanosis

73 yo man comes to ED bc of 12 hr ho inability to void. He also complains of nocturia & probz with initiating micturition for past few wks. He denies fever, weakness, numbness, dysuria, or hematuria. His other med probz include HT that is well-controlled with lisinopril. He doesn't use tobacco, alcohol, or illicit drugs. His BP is 110/60 mmHg, pulse is 74/min, and respirations are 12/min. Neuro exam shows no abnormalities except absent Achilles tendon reflexes bilaterally. Straight cath of bladder produces 600 mL of urine. Further eval will most likely show what?

*Enlarged prostate* Acute urinary retention in elderly: most common cause of urinary retention in older men = bladder outlet obstruction via BPH or carcinoma of prostate - postvoid dribbling, decreased urinary stream, hesitancy, urgency, nocturia, urinary retention - rectal exam = enlarged prostate & high postvoid urinary residual volume when Foley inserted

34 yo man being evaluated for 3 mo ho chest pain. He describes frequent episodes of substernal squeezing-type chest discomfort that radiates to his neck. Episodes are not related to any particular activity, often occur at rest, and last up to 2 hours. He has no SOB, wheezing, palpitations, syncope, kor leg swelling. Pt doesn't use tobacco, alcohol, or drugs. Fam hx unremarkable. His BP is 123/72 mmHg, and pulse is 76/min. BMI 32 kg/m^2. ECG normal. Exercise stress test shows no abnormalities. Most likely cause of this pt's chest pain?

*Esophageal disease* recurrent episodes of prolonged chest pain over several mos that are not associated with activity, etc = underlying esophageal disorder GERD & esophageal motility disorders = common causes of non-cardiac chest pain - esophageal motility abnormalities --> heartburn, dysphagia, food regurgitation, and/or chest pain - esophageal origin of chest pain = prolonged pain lasting more than hour, postprandial symptoms, associated heartburn or dysphagia, relief of pain by antireflux therapy

32 yo man comes to ED with intense midline chest pain and diaphoresis 4 hrs duration. Prior to onset of pain, he develops nausea and recurrent vomiting after returning from a party. His med probz include HIV infection, med noncompliance, alcohol abuse, and alcoholic hepatitis. Pt admits to using cocaine regularly. His temp is 100 F, BP is 100/60 mmHg, pulse is 120/min, & respirations are 28/min. Exam shows injected conjunctivae and bilateral pupils. Chest x-ray demonstrates widened mediastinum and moderate left-sided pleural effusion. ECG shows sinus tachycardia. Pleural fluid found to be yellow exudate with high amylase content. Most likely diagnosis in this pt?

*Esophageal perforation* spontaneous esophageal rupture = *Boerhaave syndrome* - vomiting/retching & chest/upper abdominal pain + rapid progression to odynophagia, dyspnea, septic shock - fever, tachycardia, tachypnea, cyanosis, subcutaneous emphysema - chest x-ray = pneumomediastinum or unilateral pleural effusion (usually left) with or without pneumothorax - mediastinal widening can be seen as air and fluid accumulate in mediastinum --> inflammation (mediastinitis) - diagnosis can be confirmed by CT or contrast esophagraphy with Gastrografin = shows contrast extravasation from esophagus into surrounding areas - *pleural fluid analysis typically exudative = low pH & very high amylase (>2500 IU/L due to saliva in esophageal contents) & may contain food particles*

A 41-year-old woman presents to the emergency department with severe, sudden-onset abdominal pain. She points to the right upper quadrant of her abdomen when asked to localize the pain, but she also reports pain in her upper back. The pain began approximately 30 minutes after she had eaten lunch, and she vomited twice on her way to the hospital. Physical exam reveals an obese female with RUQ abdominal tenderness to palpation. Her vital signs are as follows: T 38.1, HR 99, BP 144/87, RR 22, O2 Sat 96% RA. An abdominal ultrasound is conducted and is shown in Figure (flip for image). Upon review of her medication list, which of the following agents could have most likely increased this patient's risk for developing her presenting condition?

*Estradiol* Acute cholecystitis - Elevated female sex hormones and hormonal replacement therapy are risk factors for cholecystitis - caused by prolonged blockage of the cystic duct (typically by a lodged gallstone) that leads to distension, inflammation, and ultimately superinfection of the gallbladder - Additional risk factors include female gender, age > 40, overweight/obese, diabetic, total parental nutrition, rapid fluctuations in body weight, and Native American ancestry - Abdominal ultrasound is the gold-standard of diagnosing cholelithiasis and cholecystitis - manifests on ultrasound as visible stones, a thickened gallbladder wall, and pericholecystic fluid

A 56-year-old male with a history of difficult-to-control hypertension presents to his physician with progressive fatigue and new onset muscle cramps. He has had no recent changes to his medication regimen, which includes hydrochlorothiazide, lisinopril and amlodipine. His physical exam is notable for a blood pressure of 170/100 but is otherwise within normal limits. A routine basic metabolic panel demonstrates the following lab values: Sodium: 147 mEq/L Potassium: 2.6 mEq/L Chloride: 102 mEq/L HCO3: 26 mEq/L BUN: 13 g/dL Creatinie: 0.7 mg/dl Glucose: 98 What is the mechanism causing the patient's elevated blood pressure?

*Excess sodium retention* Primary hyperaldosteronism likely secondary to Conn's syndrome (aldosterone-producing adrenal adenoma). - results from excess production of aldosterone - Aldosterone causes increased sodium retention and potassium excretion --> sodium retention leads to hypernatremia and increased plasma volume, as well as difficult-to-control hypertension. - Potassium excretion leads to hypokalemia, which causes muscle cramps and weakness. - The most common causes of primary hyperaldosteronism are an aldosterone-producing adrenal adenoma (Conn's syndrome) and bilateral adrenal hyperplasia.

35 yo man comes to office for follow-up of back pain. He has aching pain in left lumbar paraspinal area that is worse at end of day and relieved overnt with rest. Pain began 3 mos ago without any ppting trauma. Pt was initially treated with intermittent doses of acetaminophen and naproxen but continues to have moderate residual pain. No associated fever, wt loss, radicular pain, lower extremity weakness, or urinary symptoms. His med hx is unremarkable. Pt is employed as factory line worker, which requires him to lift 11-15 lb several times daily. On PE, vitals are normal & pt appears comfortable. Cervical and thoracic spine range of motion is normal, but flexion and extension of lumbar spine elicits pain. Str8 leg test normal. Mild tenderness in left lumbar paraspinal tissues but no midline tenderness. Upper & lower extremity strength and deep tendon reflexes normal. Best recommendation for management of this pt's pain?

*Exercise therapy* Uncomplicated chronic (>12 wks) lower back pain (LBP) - exercise has proven to be beneficial in reducing pain and improving function - often start with supervised exercise program that emphasizes strengthening of back muscles

28 yo man comes to doc for evaluation of infertility. He is healthy and has no other complaints. He eats high-protein diet & exercises daily in order to be muscular. He weighs 187 lb and is 5'8" tall. His temp is 98.9 F and BP is 130/82 mmHg. PE shows small testes. Remainder of exam shows no abnormalities. Initial lab studies show: Hgb 16 g/dL Platelets count 200,000/mm^3 Leukocyte count 4500/mm^3 Serum creatinine 1.4 mg/dL Serum LH low Serum testosterone normal Most likely cause of his infertility?

*Exogenous steroid use* Anabolic steroid / testosterone analog abuse - exogenous androgens --> inhibit GnRH release by hypothalamus --> decreased LH & FSH via pituitary gland --> decreased sperm and testosterone production by testes - *suppresses native testosterone production but is detected as testosterone by current assays, so PATIENTS CAN HAVE NORMAL SERUM TESTOSTERONE LEVELS* - labs = erythrocytosis (via increased erythropoiesis) + elevated hemoglobin, cholestasis, hepatic failure, dyslipidemia, slightly elevated creatinine (via increased muscle mass)

A college tennis player complains to her coach that she has constant shooting pain along the lateral aspect of the elbow that radiates down her forearm. Repeated forceful use of which muscle or muscle group is the most likely etiology of her symptoms?

*Extensor carpi radialis* Tennis elbow / lateral epicondylitis - This occurs as a result of repetitive, forceful use of the superficial extensor muscle groups, including the extensor carpi radialis. - inserts at the lateral epicondyle and includes the brevis and longus(ECRB and ECRL) - the common extensor tendon becomes inflammed with repeated forced extension and flexion, such as what might occur during tennis practice and tournaments - it is often associated with improper technique

Hydroxychloroquine is used for pts with active SLE. Pts treated with this should have what done periodically?

*Eye examination* Hydroxychloroquine - can cause retinal toxicity with prolonged use = 5-7 yrs after beginning therapy - have baseline ophthalmologic evaluation and periodic reassessment

A 4-year-old girl presents to the emergency department with fever and neck pain. On physical exam, cervical lymphadenopathy and a posterior pharyngeal wall mass is noted. Which of the following events was most likely part of this patient's presenting history?

*Falling on the playground approximately one week ago with a lollipop in her mouth* Retropharyngeal abscess - *caused commonly by penetrating trauma, in this case by a foreign object in the mouth.* - Infection of the retropharyngeal space is most commonly seen in children 6 months to 6 years. - The most common causative organisms include Group A strep, S. aureus, and Bacteroides. - Presenting symptoms include fever, neck pain (patients prefer not moving the neck as it is painful to do so), and drooling. - Physical exam may show cervical lymphadenopathy and a lateral neck mass. - typically present with swelling in the neck, which can result in sudden airway obstruction. - Lateral neck radiographs can be used to confirm the diagnosis.

Mechanical ventilation improves oxygenation by providing increased fraction of inspired oxygen (FiO2) and positive end-expiratory pressure (PEEP). What should be reduced as soon as possible below levels that predispose to oxygen toxicity (<60%)?

*FiO2* - PEEP may need to be increased to maintain adequate oxygenation Assist-Control (Volume-cycled or Pressure-targeted): Volume Cycled - Settings: tidal volume, flow rate, flow waveform, FiO2, PEEP, frequency - Airway pressures and auto-PEEP are dependent on respiratory system Pressure-targeted - Settings: pressure target, inspiratory time, inspiratory rise rate, FiO2, PEEP, frequency - Tidal volume and auto-PEEP are dependent upon respiratory system

19 yo woman comes to clinic due to recurrent headaches. Pt has had headaches for years but thinks they have recently gotten worse; they last several hours and remit spontaneously or after taking OTC acetaminophen. She has no nausea, vomiting, abdominal pain, sweating, or fever. Six mos ago, pt checked her BP at local pharmacy and was told it was high. She has no other med issues and takes no prescription meds. Her fam hx is significant for HT & diabetes. BP is 175/100 mmHg on right arm and 170/102 mmHg on left arm, pulse is 80/min, & respirations are 14/min. On exam, she appears comfortable and cooperative. Peripheral pulses are full & symmetric. Chest exam unremarkable. Systolic bruit heard under right ear. Abdominal exam shows no tenderness or masses. Most likely cause of this pt's HT?

*Fibromuscular dysplasia* Secondary HT due to FMD - systemic - noninflammatory - affects renal and *internal carotid arteries* --> arterial stenosis, aneurysm, or dissection - *recurrent headaches via carotid artery stenosis* or aneurysm - pulsatile tinnitus, neck pain, flank pain - transient ischemic attacks - HT --> secondary hyperaldosteronism - accompanying bruits may be found in neck and abdomen

15 yo girl brought to office by her mom, who is concerned about her moodiness and poor grades. Since start of school 3 mos ago, pt has been uncharacteristically irritable and withdrawn. She was previously active socially with several close friends and was an A student, but has now stopped attending afterschool activities, has no friends, and struggles to maintain C average. On weekends, she spends most of her time sleeping. When seen alone, pt is tearful and describes herself as a failure bc "I'm stupid and ugly." She can no longer concentrate to complete her homework or school projects and failed a recent math test. She has no suicidal ideation or med conditions. PE normal apart from wt gain of 4.4 lb since her preschool checkup 3 mos previously. On MSE, pt is fidgety, makes poor eye contact, and picks at her fingernails. Lab values including TSH are within normal range and urine drug screen is negative. In addition to psychotherapy, what would be most appropriate to treat this pt?

*Fluoxetine* Pediatric depression - irritability rather than depressed mood - treatment = psychotherapy and/or pharmacotherapy - fluoxetine = drug of choice

35 yo AAM comes to doc complaining of 2 wks of abdominal distention. He has been in drug rehab program for 2 yrs due to prior heroin abuse. Recent viral hep and HIV profiles were negative His temp is 98.7 F, BP is 145/82 mmHg, pulse is 80/min, & respirations are 14/min. Pt is obese with BMI of 40 kg/m^2. Exam shows periorbital edema, ascites, & 2+ pitting edema in both legs up to knees. Breath sounds decreased at right lung base. 24 hr urine shows protein excretion of 7.5 g. Most likely to be present on kidney biopsy?

*Focal segmental glomerulosclerosis* - volume overload (periorbital edema, peripheral edema, ascites) via hypoalbuminemia - right pleural effusion - primary renal etiology - most common cause of nephrotic syndrome in adults, especially AAs - can also be associated with HIV, heroin, obesity Nephrotic syndrome: - heavy proteinuria > 3.5 g/24 hr + hypoalbuminemia & edema - other findings = hyperlipidemia & increased lipids in urine

A 27-year-old female presents to her primary care physician with a chief complaint of pain in her lower extremity. The patient states that the pain has gradually worsened over the past month. The patient states that her pain is worsened when she is training. The patient is a business student who does not have a significant past medical history and is currently not on any medications. She admits to having unprotected sex with multiple partners and can not recall her last menses. She drinks 7 to 10 shots of liquor on the weekends and smokes marijuana occasionally. She recently joined the cross country team and has been training for an upcoming meet. Her temperature is 99.5°F (37.5°C), pulse is 88/min, blood pressure is 100/70 mmHg, respirations are 10/min, and oxygen saturation is 97% on room air. On physical exam you note a very pale young woman in no current distress. Pain is localized to the lateral aspect of the knee and is reproduced upon palpation. Physical exam of the knee, hip, and ankle is otherwise within normal limits. The patient has 1+ reflexes and 2+ strength in all extremities. A test for STI's performed one week ago came back negative for infection. Which of the following is the most likely explanation for this patient's presentation?

*Friction with the lateral femoral epicondyle* Pain over the lateral femoral epicondyle associated with athletics suggesting a diagnosis of *iliotibial band syndrome* - associated with friction from the iliotibial band with the lateral femoral epicondyle - classic presentation for iliotibial band syndrome is pain over the lateral knee in an athlete who participates in repetitive activities such as running - pain is typically exacerbated by activity and relieved by rest - pain is worsened by palpation and occurs over the lateral femoral epicondyle - theory surrounding this pathology is based on friction between the iliotibial band and the lateral femoral epicondyle which causes inflammation and bursitis Treatment is conservative and relies on rest, activity as tolerated, stretching, and NSAIDs with surgery in severe cases. Symptoms can progress to constant pain, even at rest without treatment.

62 yo woman comes to office due to 4 days of watery diarrhea and abdominal cramps. She has had 4 or 5 loose stools daily with no blood or mucus. She has ho constipation requiring frequent laxative use but last took a laxative 6 days ago. Pt also has ho HT, hyperlipidemia, GERD, and T2DM. Several benign colonic polyps were removed during a colonoscopy 2 yrs ago. She takes omeprazole daily & received oral abx or acute sinusitis 3 wks ago. Pt has not traveled recently. She smokes a pck of cigz daily. Her temp is 100.4 F. Abdomen soft with mild distention and mild diffuse tenderness. No guarding or rebound tenderness present. Stool assay for Clostridium difficile positive. In addition to recent abx use, what most likely predisposed this pt to her current condition?

*Gastric acid suppression* - alters colonic microbiome --> increases risk of C diff proliferation Other risk factors = hospitalization, advanced age C diff - gram-positive anaerobic - abx-resistant spores ingested --> converted to fully functional bacilli in colon - noninvasive but produces exotoxins (enterotoxin A & cytotoxin B) --> penetrates colonic epithelial cells = apoptosis and loss of tight junctions - watery diarrhea (>3 stools/day), abdominal cramping, low-grade fever, leukocytosis (15,000/mm^3)

A 20-year-old female presents to the hospital with severe chest pain. She states that the pain started suddenly and is retrosternal in nature. The pain began shortly after lunch and is worse with swallowing. She has no prior medical history except for a brief inpatient stay for what she describes as an "eating disorder." On exam her vitals are as follows: HR 120, RR 22, BP 145/90. She is flushed and taking deep breaths. Which of the following is the best confirmatory test for the most likely cause of this presentation?

*Gastrografin esophagram* - test used to visualize the esophagus - patient swallows gastrograffin and fluoroscopy is used to watch the dye traverse the esophagus. - Gastrograffin is used instead of barium if the test is being used to diagnose an esophageal tear. Esophageal rupture (Boerhaave's syndrome) as a result of repeated induced vomiting. - As time is essential in treating esophageal ruptures, the most appropriate step is a water-soluble (gastrografin) esophogram. - Symptoms of esophageal rupture include sudden-onset, severe, retrosternal chest pain, and difficult or painful swallowing. - While hematemesis may be present, it is more common in Mallory-Weiss tears. - Physical exam may show pleuritic chest pain, hyperventilation, and tachycardia. - Evaluation includes a chest CT (may show left-sided hydropneumothorax, pneumomediastinum, or esophageal thickening) or contrast studies (may show leakage from esophageal tear). - Of note, water soluble contrasts should be used before barium studies. Illustration displays a gastrografin study of a perforated esophagus. Note the contrast extravasation.

A 44-year-old woman with generalized anxiety disorder is admitted to the hospital with pneumonia. During the intake history and physical, she forgets to mention that she takes "several" alprazolam each day. Over the course of her hospitalization, she is at risk for which of the following?

*Generalized tonic-clonic seizures* The patient in this vignette, if not given some form of benzodiazepine during her hospitalization, is at risk for *acute withdrawal, which can result in generalized tonic-clonic seizures.* - *Abrupt cessation of short-acting benzodiazepines, in this case alprazolam, may cause delirium or seizures.* - Short-acting benzodiazepines can be used in the short-term treatment of generalized anxiety disorder (GAD) but are often continued for months to years. It is preferable to reduce the use of benzodiazepines by adding buspirone or another antidepressant. - Effective method of treating alprazolam withdrawal by using a long-acting benzodiazepine, in this case *chlordiazepoxide, to accomplish a rapid, well-tolerated withdrawal.*

29 yo woman comes to doc complaining of worsening pain in her right knee for last 3 mos. She tried ibuprofen, but ir provided little relief. She doesn't use tobacco or alcohol. Her mom suffers from RA. Pt's temp is 99 F. On exam, right knee is mildly swollen & tender with decreased ROM. X-ray shows expansile and eccentrically placed lytic area in epiphysis of distal femur. Serum chemistries and CBC are normal. Most likely diagnosis?

*Giant cell tumor* - "soap-bubble" appearance - benign - locally aggressive - young adults - pain, swelling, decreased range of joint motion at involved site - 10-35% of affected pts --> pathologic fractures due to thinning of bone cortex in wt-bearing areas - typically presents on x-ray of epiphyseal regions of long bones - most commonly distal femur and proximal tibia around knee joint - MRI = tumor containing both cystic and hemorrhagic regions - path = sheets of interspersed large osteoclast giant cells --> appear round-to-oval polyonal or elongated mononuclear cells - surgery (intralesional curettage with or without bone grafting) = first-line treatment

56 yo man develops oliguria three days after having kidney transplant. His postop course was uncomplicated. His BP is 160/100 mmHg and heart rate is 90/min. Palpation of transplant reveals mild tenderness. Lab studies show: Serum sodium 145 mEq/L Serum potassium 5.5 mEq/L Serum calcium 8.6 mg/dL Serum creatinine 3.2 mg/dL BUN 30 mg/dL His serum cyclosporine level is normal. Renal US does not detect dilatation of calyces. Biopsy of transplant shows heavy lymphocyte infiltration and vascular inv't with swelling of intima. Most appropriate next step in management?

*Give IV steroids* Renal transplant dysfunction in early post-op period = oliguria, HT, increased creatinine/BUN *Acute rejection* - 6 days - 1 yr after transplant - antidonor T-cell proliferation in recipient - frequently reversible - rapid institution of anti-rejection therapy, including high-dose IV steroids

A 24-year-old female is brought to the ER 1 hour after acute onset of double vision in the right eye. Two months ago, she had an acute episode of pain with movement of the left eye along with loss of both central vision and pupillary light reflex. These symptoms subsided spontaneously. Which of the following medications has been shown to be the most effective at decreasing the progression of the relapsing-remitting form of this disease?

*Glatiramer acetate* Relapsing-remitting multiple sclerosis (MS). - *Glatiramer acetate and interferon beta have been shown to decrease progression of this condition.* MS - most common permanently disabling disorder of the central nervous system seen in young adults Relapsing-remitting - most common of which typical symptoms include sensory, motor and coordination disturbances, optic neuritis, as well as fatigue - course of disease is highly variable - diagnosis is made clinically with two neurologic deficits separated in time and space - diagnosis and response to therapy can be assessed with MR imaging

24 yo man comes to urgent care clinic due to blood in his urine. He has 1-day ho mild midline back discomfort, associated with single episode of red urine. Hematuria subsequently resolved and his urine is now normal yellow color. Pt also has had sore throat for past 4 days, for which he took 3 tablets of OTC acetaminophen. He has had no fever, abdominal pain, or burning during urination. He has smoked 1 pck of cigz daily for 5 yrs. PE unremarkable. Urinalysis shows: Color Yellow Glucose Negative Protein 3+ Ketones Negative Leukocyte esterase Negative Nitrites Negative WBCs 2-3/hpf RBCs 50-75/hpf Most likely source of this pt's urinary complaints?

*Glomerulus* Hematuria: glomerular or non-glomerular Glomerular hematuria - usually causes microscopic hematuria - gross hematuria may be present in some cases - usually due to glomerulonephritis or BM diseases - symptoms absent or nonspecific (low back pain) - urinalysis = proteinuria + dysmorphic RBCs or RBC casts This pt.... *IgA nephropathy* - proteinuria + transient gross hematuria following acute pharyngitis - most common GN in adults - typified by hematuria starting within 5 days of upper resp or pharyngeal illness (synpharyngitic GN)

Current guidelines recommend initial metformin in T2DM without contraindications (renal insufficiency). Pts with suboptimal control on metformin require a second drug, with choice depending on factors such as pt preference, efficacy, risk of hypoglycemia, cost, wt gain or desired wt loss, presence of comorbidities, and side effects. What drugs would induce *weight loss* and have lower hypoglycemia risk?

*Glucagon-like peptide-1 (GLP-1) agonists* - exenatide, liraglutide

Gold standard for diagnosis of acute angle-closure glaucoma

*Gonioscopy* - Ocular tonometry can be helpful if urgent ophthalmologic consultation unavailable Acute angle-closure glaucoma - sudden-onset eye pain + headache + neausea - conjunctival erythema + corneal opacification + mid-dilated pupil

65 yo woman comes to ED due to 1 day of swelling of left side of her face, associated with high fevers and chills. She has no facial trauma or injury to that area. Pt was diagnosed with T2DM 3 yrs ago and achieves good glycemic control with diet, exercise, and oral hypoglycemic agents; she has no known complications. Temp is 102.6 F, BP is 125/75 mmHg, and pulse is 90/min. PE shows warm, tender, erythematous rash with raised, well-demarcated borders on left side of face, including left external ear. Ear canal has no discharge, and hearing is intact. Mild regional LAD present. Most likely causative organism of this pt's infection?

*Group A streptococcus* Erysipelas - skin infection of upper dermis & superficial lymphatic system - most commonly via group A strep - pts rapidly develop systemic symptoms = fever, chills, regional lymphadenitis, warm, tender, erythematous rash notable for raised, sharply demarcated borders - inv't of external ear bc skin lacks lower dermis level = most pts receive IV abx (ceftriaxone, cefazolin)

Acronym for an unusual multisystem disorder in women that consists of hyperandrogenism, insulin resistance, and acanthosis nigricans. The precipitating abnormality is thought to be insulin resistance, with a secondary increase in insulin levels and subsequent overproduction of androgens in the ovaries. Long periods of hyperinsulinism and, some suspect, hyperandrogenism can result in the cutaneous manifestation of acanthosis nigricans. Patients are often concerned about the physical manifestations of this disorder, including virilization and acanthosis nigricans, and may be less aware of systemic problems.

*HAIR-AN Syndrome*

A 53-year-old male presents with new-onset unilateral hearing loss. On otoscopic exam, the tympanic membrane is dull and pneumatic otoscopy reveals hypomobility. This condition is commonly seen as a sequelae of which of the following conditions?

*HIV* Serous otitis media - seen in HIV with lymphadenopathy - results from obstruction of the fluid outflow tracts (Eustacian tubes) and consequent buildup of fluid in the middle ear - of note, the fluid is not infected - *can be caused by many conditions, including lymphomas or lymphadenopathy* - on exam, the tympanic membrane is dull and hypomobile on pneumatic otoscopy (as a result of the fluid behind it) *The most common otologic problems reported in HIV-infected patients are serous otitis media and recurrent acute otitis media. These conditions frequently affect pediatric patients with HIV disease because eustachian tube dysfunction typical of this age group combined with depressed cell-mediated immunity markedly increases their susceptibility to middle ear infection*

Typically HSV-1 causes the oral form of the disease and HSV-2 causes the genital form. On college campuses, what is the most common cause of the genital form of the disease.

*HSV-1* - This could be due to many factors including how ubiquitous HSV-1 is, and oral sex.

The patient is presenting with signs and symptoms of an acute manic episode. What is used as a first-line therapy in pregnant patients with this disorder?

*Haloperidol*

A 37-year-old homeless man was brought into the emergency department by police after being found wandering the streets screaming that cockroaches are crawling all over the cars and storefronts, when no one else sees the insects. His vitals are as follows: T 99.6, HR 115, BP 178/113, RR 28, and O2 Sat 97%. He is tremulous and sweating. His blood alcohol concentration is <0.01 and his urine drug screen is negative for opiates, benzodiazepines, and barbiturates. In addition to benzodiazepines, which of the following is likely to be used as an adjunct in his treatment?

*Haloperidol, followed by thiamine then glucose* Delirium tremens from alcohol withdrawal - *Adjuncts to benzodiazepines for acute treatment of alcohol withdrawal include haloperidol for agitation and thiamine then glucose administration* - Given his blood alcohol concentration and profile of signs and symptoms, this man has likely not had a drink in several days and is suffering from delirium tremens. - Patients in alcohol withdrawal demonstrate at least two of the following symptoms: autonomic hyperactivity, tremor, nausea or vomiting, hallucinations, psychomotor agitation, anxiety, and grand mal seizures. - The majority of patients experiencing ethanol withdrawal can be managed with supportive care alone, but for severe or complicated withdrawal, pharmacologic therapy with benzodiazepines, especially diazepam and chlordiazepoxide, may be required.

A 47-year-old female with a past medical history of panic disorder presents to her outpatient psychiatrist. Although she works as a movie reviewer, she recently stopped feeling comfortable in movie theaters. She also refused to attend a symphony concert last week with her husband. Which of the following past experiences might have influenced this patient's decisions?

*Having a panic attack while on a plane* *Patients with a history of panic attacks may become agoraphobic after experiencing an attack in a place from which it would be difficult to escape, like a plane.* Panic disorder - characterized by multiple panic attacks with associated fear of having a panic attack - panic attacks occur at random times and are not triggered by a stimulus - *often develop agoraphobia after experiencing an attack in a place from which it would be difficult to escape* - Cognitive behavioral therapy, in addition to pharmacotherapy, is useful for treating both panic disorder and agoraphobia

A 68-year-old male presents to his primary care physician with complaints of heartburn, belching, and epigastric pain. He states that the symptoms are aggravated by drinking coffee or eating fatty foods and improved by taking chewable heartburn relievers. He is started on a trial of omeprazole, which relieves his discomfort for several weeks, but his symptoms return. When counseling this patient, which of the following statements must be made?

*He is at increased risk of esophageal cancer and should have further tests* Gastroesophageal reflux disease (GERD) - *His refractory symptoms put him at risk for Barrett's esophagus and consequent esophageal cancer* - caused by transient lower esophageal sphincter relaxation, which releases stomach acid into the esophagus - should have an upper endoscopy if any of the following alarm symptoms are present: dysphagia, odynophagia, weight loss, anemia, GI bleeding, recurrent vomiting; men over age 50 years of age with chronic symptoms and cancer risk factors (i.e. tobacco) also require evaluation - Endoscopy is valuable to rule out GERD complications, such as Barrett's esophagus, which can progress to adenocarcinoma. *Illustration displays a gross pathological specimen of esophageal adenocarcinoma. Note the proximity to the stomach.*

A 56-year-old female presents with several episodes in which she felt "dizzy." These were associated with unilateral "ringing" in her ears as well as a decrease in hearing acuity. Which of the following features distinguishes Menière's disease from benign paroxysmal positional vertigo (BPPV)?

*Hearing loss* Benign paroxysmal positional vertigo (BPPV) and Menière's disease are similar conditions. *However, BPPV does not result in hearing loss as it affects the semicircular canals but not the cochlea.* Menière's disease consists of four symptoms: episodic dizziness or whirling vertigo, low-frequency sensorineural hearing loss, tinnitus, and a sensation of aural fullness in the ear. These symptoms may or may not develop simultaneously. BPPV vertigo attacks are much shorter than Menière's attacks, lasting minutes to hours, while Menière's lasts hours to days.

A 41-year-old G1P0 woman arrives for an initial prenatal visit at 8 weeks gestation dated by last menstrual period. This was an unexpected pregnancy, so she is concerned about drinking her usual glass of wine every evening as well as her advanced age. Her past medical history is significant only for an unrepaired ventricular septal defect that results in cyanosis with moderate exertion. She also reports that she and the father have relatives with cystic fibrosis. What in this patient's presentation would prompt you to advise her to electively terminate the pregnancy?

*Heart condition of the patient* Cyanotic heart dz = contraindication to pregnancy Pregnancy - decreased systemic vascular resistance, increased circulatory volume, and increased venous return (due to increased metabolic demands of end organs). Pregnancy in a woman with congenital heart disease (such as unrepaired patent ductus arterioses, atrial septal defects, and VSDs) can result in Eisenmenger's syndrome; the reversal of a baseline left-to-right shunt to a right-to-left shunt. This reversal of flow can result in profound hypoxia and severe complications for both the mother and fetus. Maternal mortality under these conditions can reach 30-50%.

A 70-year-old male is brought to the emergency department after collapsing inside his home while preparing a roast beef sandwich. Patient is a current 2-pack-per-day smoker for 30 years, is notably sedentary, and has a past medical history for diabetes mellitus type II, anxiety, and an ischemic stroke 10 years ago. In the emergency department, his blood pressure is 90/60 mmHg and pulse is 125/min with a respiratory rate of 30/min. The patient is notably short of breath. Patient is currently on "some medications," but admits to not taking them. Lungs are bilaterally clear to auscultation. A chest radiograph is ordered demonstrating no abnormalities. An EKG is performed notable only for sinus tachycardia. An arterial blood gas demonstrates a pH of 7.47, a pCO2 of 28 and a pO2 of 75. In the ED the patient is started on oxygen. What is the next best step in management of this patient?

*Heparin* Pulmonary embolism with shortness of breath, bilaterally clear lungs (auscultation), and the ABG findings - Next best step in management when the diagnosis of pulmonary embolism is clear is to administer heparin - Do not wait to treat for other confirmatory studies when a case so clearly depicts a clinical symptom. - Can obstruct the pulmonary artery and cause acute right heart failure or shock, as pressures in the pulmonary artery exceed the pumping capacity of the right ventricle. 1. Tachycardia 2. Shortness of breath 3. A history that could lead to DVT's (endothelial damage from smoking, stasis from a sedentary lifestyle, a potential hypercoaguable state given the history of stroke) 4. Hyperventilation leading to a low pCO2, yet also a low O2 from the embolus 5. A normal chest radiograph (the #1 chest radiograph finding in PE is a normal study, could also find wedge shaped infarcts, or the vascular tree being affected) The next best step in management when it is clear that a patient is suffering from a PE is to begin treatment with heparin - If this case were more ambiguous and a PE was suspected but classic findings and history were not present, the next best step in management would be a confirmatory study such as a spiral CT scan, a V/Q scan, or other less sensitive tests such as a lower extremity Doppler or a D-dimer. Pulmonary embolism may cause signs of right heart strain on EKG - T-wave inversion on precordial leads, right bundle branch block, and the S1Q3T3 pattern - Neither sensitive nor specific for pulmonary embolism.

26 yo previously healthy woman brought to ED after having episode of seizure 1 hr ago. She has 2-day ho fever & headaches, for which she has been taking acetaminophen and ibuprofen without much relief. This morning, her fam found her behaving strangely. She has no fam hx of seizures. There is no recent travel hx. Her temp is 102 F, BP is 120/70 mmHg, pulse is 110/min, & respirations are 18/min. Pt is lethargic and confused. Hyperreflexia present. CBC & CT scan of head unremarkable. Her CSF study shows: Opening pressure 160 mm H2O Protein 100 mg/dL Glucose 55 mg/dL WBCs 150/mm^3 - Lymphocytes 90% - Polymorphs 10% RBCs 200/mm^3 Most likely diagnosis?

*Herpes simplex encephalitis* - most common cause of fatal sporadic encephalitis in US - acute onset (<1 wk duration) of focal neurologic findings (altered mentation, focal CN deficits, ataxia, hyperreflexia, focal seizures) - fever present in 90% of pts - behavioral changes seen = hypomania, Kluver-Bucy syndrome (hyperphagia, hypersexuality), amnesia CSF - lymphocytic peocytosis - increased # of RBCs (via hemorrhagic destruction of temporal lobes) - elevated protein levels Polymerase chain rxn analysis of HSV DNA in CSF = highly sensitive and specific = gold standard IV acyclovir = treatment of choice

This patient is suffering from a Mallory-Weiss tear, likely secondary to the forceful retching that has accompanied his gastrointestinal illness over the past several days. What abnormality is associated with this condition?

*Hiatal hernia* A Mallory-Weiss tear is associated with hiatal hernia in an estimated 40-100% of cases. - more common in patients with hiatal hernia likely due to the increased pressure that develops with retching in the hernia as compared to the remainder of the distal stomach --> leads to an increased potential for laceration of the mucosa

A 69-year-old male presents to the emergency department with shortness of breath. The patient has presented three times this past month with similar complaints. The patient sees no primary care physician and is currently not taking any medications. The patient states his shortness of breath started when he was walking from his car to a local restaurant. His temperature is 99.5°F (37.5°C), pulse is 100/min, blood pressure is 130/90 mmHg, respirations are 18/min, and oxygen saturation is 96% on room air. On physical exam you note a fatigued appearing gentleman. Cardiovascular exam reveals an additional heart sound after S2. Pulmonary exam is notable for bilateral crackles. Abdominal exam reveals an obese abdomen without pain in any of the quadrants. Lower extremity pitting edema is noted bilaterally. Which of the following sets of lab values most likely corresponds to this patient's presentation? BNP = ADH = Sodium = Potassium =

*High BNP, high ADH, low sodium, low potassium* CHF - *The most likely laboratory abnormalities are elevated brain natriuretic peptide (BNP), high anti-diuretic hormone (ADH), low sodium, and low potassium* As the ventricles dilate, they release BNP, a marker of CHF, and the kidneys tend to be underperfused leading to two key physiologic changes. - Activation of the renin-angiotensin-aldosterone system, which serves to increase perfusion to the kidneys by increasing blood pressure, resulting in sodium and fluid retention at the expense of potassium (hypokalemia) and hydrogen (metabolic alkalosis). - ADH is also increased to further increase perfusion, resulting in hemodilution and hyponatremia. Congestive heart failure (CHF) presents with crackles, JVD, ascites, edema, a S3 heart sound, fatigue, and shortness of breath, with common lab findings of elevated BNP, elevated ADH, low sodium, and low potassium. In congestive heart failure, cardiac output and systemic blood pressure are reduced, and "hypovolemic" hormones, such as renin, antidiuretic hormone (ADH), and norepinephrine, increase. This because although heart failure patients have increased plasma and extracellular fluid volumes, the body perceives volume depletion (reduced effective arterial blood volume) since the low cardiac output decreases the pressure perfusing the baroreceptors in the carotid sinus and the kidneys. ADH release directly enhances water reabsorption in the collecting tubules. In addition, both the low cardiac output and high angiotensin II levels are potent stimuli to thirst, leading to enhanced water intake. This increase in both water reabsorption and water intake (without concominant increase in Na reabsorption or Na intake) leads to a decreased Na concentration in the blood, aka hyponatremia.

A six-year-old girl presents in the pediatric emergency room after tripping and falling on the playground and striking her head against the ground. Her anxious parents deny loss of consciousness, vomiting, headache, and behavior changes. On physical exam, you note no hemotympanum, no periorbital or postauricular ecchymoses, and no CSF rhinorrhea. Which of the following additional aspects of her history would prompt you to obtain a non-contrast head CT?

*History of ateriovenous malformation* Non-contrast head CT must be performed after any head trauma in a child with an unreliable neurologic exam or increased risk of intracranial hemorrhage; the latter includes children with arteriovenous malformations (AVMs). - the risk of intracranial injury and hemorrhage with even minor trauma is increased in those with structural abnormalities such as an AVM - children with bleeding diatheses (such as hemophiliacs or those on long-term anticoagulation) must also undergo CT scan for similar reasons

A 27-year-old male is brought to the emergency department with an acute asthma exacerbation. The patient takes combination fluticasone proprionate and salmeterol for long-term management of severe asthma. His symptoms are well-controlled while on medication, but for the past few days he has not taken his medication since his prescription ran out. His pulse is 110/min and respirations are 26/min. On physical examination, he is using accessory muscles of respiration. Wheezing is present in both lung fields. Oxygen saturation is 86% on room air. The patient is started on 2L/min nasal cannula oxygen. Which of the following allergens is the most common cause of this presentation?

*House dust mite* Asthma exacerbation - most frequent allergen that causes allergic asthma exacerbations is the house dust mite - Dust mite and fungus growth can be reduced by keeping the household humidity level < 50%

A 64-year-old woman presents to the emergency room with complaints of severe, whole-body itching. She states that she first noticed her symptoms while in the bathtub at home. She has never had symptoms like this before. However, over the previous several months she has had episodes of severe joint swelling and pain in her hands as well as redness, burning pain, and swelling of her hands and feet. Her past medical history is significant for type II diabetes mellitus, hypertension, and osteoporosis for which she takes metformin, enalapril, and alendronate, respectively. In addition, she was found to have a deep vein thrombosis of her left leg three months prior to presentation. The patient's temperature is 98.6°F (37.0°C), pulse is 80/min, blood pressure is 135/85 mmHg, and respirations are 13/min. Physical exam is notable for a woman in discomfort with excoriations over the skin on her forearms. The patient's laboratory tests are shown below. Serum: Na+: 135 mEq/L Cl-: 100 mEq/L K+: 5.0 mEq/L HCO3-: 22 mEq/L BUN: 19 mg/dL Glucose: 130 mg/dL Creatinine: 1.0 mg/dL Hematocrit: 64% Leukocyte count: 19,000 cells/mm^3 with normal differential Platelet count: 900,000/mm^3 What is the best next step in treatment?

*Hydroxyurea* Polycythemia, leukocytosis, and thrombocytosis in the setting of pruritus after bathing, as well as episodes suggestive of acute gout flares = *polycythemia vera* - can be treated with hydroxyurea. - malignancy of the bone marrow that results in the overproduction of red blood cells (primarily), platelets, and white blood cells - may also have gout due to increased cell turnover leading to hyperuricemia - older patients (> 60 years old) and those with prior thrombosis should be treated with a myelosuppressive agent, most commonly hydroxyurea with or without aspirin

56 yo man admitted to hospital with severe nausea and vomiting. He has also had fatigue, poor appetite, polyuria, polydipsia, and constipation for several wks. Pt has ho HT & T2DM. His home meds include metformin, lisinopril, and HCTZ. He has 40 pck yr smoking hx and drinks alcohol occasionally. Temp is 98 F, BPis 110/70 mmHg, pulse is 102/min, & respirations are 16/min. His mucous membranes are dry. Rest of physical exam unremarkable. Lab results: Calcium 14.8 mg/dL Albumin 4 g/dL Parathyroid hormone, intact 5 pg/mL Creatinine 1.9 mg/dL Urea nitrogen 54 mg/dL Glucose 180 mg/dL 25-hydroxyvitamin D 50 ng/mL (normal, 20-60 ng/mL) 1,25-dihydroxyvitamin D 17 pg/mL (normal, 15-65 pg/mL) Most likely cause of this pt's hypercalcemia?

*Hypercalcemia of malignancy* Humoral hypercalcemia of malignancy (HHM) - most common cause of PTH-independent hypcalcemia - frequently presents with very high (>14 mg/dL), symptomatic (polyuria, constipation, nausea) calcium levels - should strongly be suspected in pt in light of his smoking hx, systemic symptoms (fatigue, poor appetite), and markedly elevated calcium with low PTH - via secretion of PTH-related protein (PTHrP) by malignant cells - associated with squamous cell (lung, head, neck), renal, bladder, breast, or ovarian carcinomas - PTHrP doesn't induce conversion of 25-hydroxyvitamin D to 1,25-dihydroxyvitamin D to same extent as PTH does = low or low-normal 1,25-dihydroxyvitamin D levels

79-year-old woman with hypertension, atrial fibrillation, hypercholesterolemia, gout, Crohn diseases, chronic renal disease, and a previous stroke presents with the onset of tinnitus and decreased hearing bilaterally. If she states that she takes only her prescribed dose, pharmacologic agents for which of the following conditions would most likely have resulted in this presentation?

*Hypertension* Drug-induced ototoxicity: Loop diuretics like furosemide would be the most likely drug, given her medical history, to result in this presentation. Loop diuretic ototoxicity - well documented and can result - reversible or irreversible hearing loss - direct inhibitors of the Na(+)-K(+)-2Cl- cotransport system, which function in the cochlea to secrete endolymph - may result from changes in ionic composition and fluid volume within the endolymph.

58 yo woman comes to ED after developing headache following gourmet meal that included heavy sauces & wine. On arrival, pt is anxious & tremulous. Her med hx include recurrent major depression with psychotic features, GAD, & OA. After multiple med trials, pt was finally stabilized on phenelzine and risperidone, which she continues to take as prescribed. Her depression has been in remission for past 5 yrs. Pt's other meds include ibuprofen, and she doesn't use illicit drugs. Most likely finding on PE?

*Hypertension* acute onset of headache following meal in pt treated with phenelzine = HT crisis Phenelzine - one of several MAOIs = older class of antidepressant drugs used primarily for treatment of refractory and atypical depression MAOIs - inhibit metabolism of monoamines epinephrine, dopamine, serotonin - typically not used as first-line therapy via drug-drug interactions and dietary restrictions involving foods containing tyramine Tyramine - may be present in aged cheeses, aged or cured meats, aged or fermented soy products, overripe fruits, some alcoholic beverages - sympathomimetic monoamine - can facilitate release of other sympathomimetic monoamines, such as adrenaline - metabolism inhibited in presence of MAOIs --> increased sympathomimetic (adrenergic) effect = severe "HT crisis" --> *presents first as headache* but can lead to inctracranial bleeding, stroke, death

A 52-year-old male is brought to the emergency room because of a severe headache and nausea. The patient states he has never seen a doctor before. Soon, he becomes uncooperative and is unable to follow commands. On physical exam, his blood pressure is 185/125, heart rate 88, and temperature 37.3 C. Papilledema is noted on ophthalmologic exam. Which of the following is the most likely diagnosis?

*Hypertensive encephalopathy* - blood pressure >180/120 and signs of cerebral edema. - defined as a blood pressure of > 180/120 with no signs of acute end-organ damage. Hypertensive emergency is same blood pressure criterion with acute end-organ damage and can be further classified into: - malignant hypertension = retinal hemorrhages, exudates and/or papilladema - *hypertensive encephalopathy = cerebral edema and non-localizing neurologic signs/symptoms*.

52 yo man with alcoholism comes to ED due to generalized weakness, anxiety, tremors. His last drink was 2 days ago. He has no significant med hx and no med follow-up. Pt smokes cigz but doesn't use illicit drugs. He takes no meds. On exam, he appears disheveled and malnourished. His initial electrolyte panel results: Sodium 132 mEq/L Potassium 2.9 mEq/L Chloride 100 mEq/L Bicarb 25 mEq/L He is treated with alcohol withdrawal and given aggressive IV potassium as well as oral potassium supplementation. Three days later, his electrolyte panel results are as follows: Sodium 135 mEq/L Potassium 3.1 mEq/L Chloride 102 mEq/L Bicarbonate 28 mEq/L What best explains why this pt's potassium level is very difficulty to correct?

*Hypomagnesemia* Chronic alcoholism --> hypomagnesemia = most common = commonly occurs together with hypokalemia = well-known cause of *refractory hypokalemia* Intercellular magnesium --> inhibits potassium secretion by renal outer medullary potassium (ROMK) channels in collectin tubules of kidney

A 61-year-old man presents to the emergency room complaining a racing heart, sweats, and diarrhea for 2 weeks. Review of systems is positive for unintentional weight loss of 10 pounds in 1 month. Serum TSH is found to be 0.02 mIU/L (normal 0.5 - 5.0 mIU/L). If the patient is treated with I-131 radioiodine therapy, which of the following is the most likely long term complication?

*Hypothyroidism* Graves' disease - *Hypothyroidism (due to radiation thyroiditis) may occur in the treatment of Graves' disease with I-131 radioiodine therapy* In Graves' disease, the entire thyroid gland is hyperfunctional - *Uptake of the radioactive isotope of iodine throughout the gland results in effective ablation but may destroy too much thyroid tissue, resulting in a hypothyroid state* - *More than 75% of patients become hypothyroid following radioactive iodine thyroid ablation* - mechanism of radioiodine therapy for hyperthyroidism: I-131 is actively transported into the thyroid follicular cells by an iodine symporter. Within the thyroid, the beta emissions of I-131 result in tissue necrosis, ablating thyroid tissue over a period of 6-18 weeks

59 yo woman hospitalized due to lower GI bleeding. Her past med hx includes HT, COPD with cor pulmonale, and diverticulosis. She takes albuterol, aspirin, furosemide, and lisinopril. On admission, her BP was 80/50 mmHg & hemoglobin was 6.5 g/dL. Her condition stabilized with fluid resuscitation, blood transfusions, and by withholding her home meds. Pt's urine output has been 300-400 mL per day for past 3 days. On 4th day of hospitalization, she is found lethargic and difficulty to arouse. Her BP is 110/70 mmHg and pulse oximetry is 93% on 6 L per minute oxygen. Lab testing: Hgb 10.2 g/dL Leukocytes 14,300/mm^3 Blood glucose 93 mg/dL BUN 62 mg/dL Creatinine 2.7 mg/dL Sodium 132 mEq/L Potassium 5 mEq/L Chloride 102 mEq/L Bicarbonate 18 mEq/L Arterial blood gases pH 7.15 PaO2 80 mmHg PaCO2 60 mmHg Most likely contributing to this pt's lethargy?

*Hypoventilation* Acidemia in setting of mixed acid-base disorder Resp acidosis with acute-on-chronic CO2 retention (Hypercabia) via hypoventilation in setting of COPD --> CO2 narcosis (PaCO2 >60 mmHg) AKI - via hypotension via GI bleed - can cause non-anion gap metabolic acidosis via impaired H+ excretion, ammonia generation, or bicarb reabsorption

A 26-year-old immigrant from Mexico presents to your clinic for a physical. He tells you that over one year ago, he noticed a lesion on his penis which went away after several weeks. A month later, he noticed a rash appearing on his palms and soles, which lasted several weeks and also resolved on its own. Physical exam is unremarkable and shows no evidence of any rash. VDRL and FTA-ABS are both positive. What is the most appropriate treatment for this patient?

*IM penicillin weekly x 3 weeks* Asymptomatic for syphilis but tests positive, indicating late latent syphilis (>12 months) - The first line treatment is IM penicillin for 3 weeks. Latent syphilis results from untreated primary and secondary syphilis - patients are typically asymptomatic but are positive for syphilis by serologic testing

32 yo man brought to ED with sudden onset of excruciating left-sided chest pain. His temp is 99 F, BP is 160/90 mmHg, pulse is 125/min & regular, and respirations are 20/min. He is very anxious, agitated, and sweating profusely. Both pupils dilated, and nasal mucosa is atrophic. Cardiac exam shows regular heart sounds with no murmurs or rubs. Lungs clear to auscultation. Initial ECG shows ST-segment depression & T-wave inversion in leads V4-V6. First set of cardiac enzymes, including CK-MB and troponin T, is normal. Chest x-ray shows no acute abnormality. Most appropriate next step in management of this pt?

*IV diazepam* psychomotor agitation + dilated pupils + atrophic nasal mucosa + HT + acute MI = *acute cocaine intoxication* - *all pts with acute cocaine toxicity and MI --> supplemental oxygen & IV benzos* --> reduce sympathetic outflow - also: aspiring, nitroglycerin, CCBs - NO beta-blockers!!!

S3 is low frequency diastolic heart sound associated with left ventricular failure. What provides symptomatic benefits to pts with decompensated heart failure?

*IV diuretics*

Pt has acute nociceptive pain due to obvious trauma and has had no relief with IV NSAID (ketorolac). Pt has recovering opioid addiction. Next management?..

*IV morphine* - initial management options for acute pain generally similar for all pts, regardless of abuse hx

Recurrent vomiting causes depletion of fluid, acid, and sodium chloride, leading to metabolic alkalosis, activation of RAAS, and increased urine potassium loss. Treatment?

*IV normal saline and potassium* - volume resuscitation with normal saline corrects contraction alkalosis - hypokalemia should be treated as well

A 46-year-old woman presents to the emergency department complaining of chest pain. She last felt well roughly two days ago when she began experiencing chest pain and low-grade fever, and since then these symptoms have persisted. She is otherwise healthy and takes no medications; she has never traveled outside of the United States. On examination, her vital signs are T 37.7, pulse 102 beats per minute, blood pressure 116/79, and O2 saturation 97% on room air. She is uncomfortable but otherwise well-appearing; cardiac exam reveals mild tachycardia, regular, with a scratching sound heard during systole. Examination is otherwise unrevealing. Which of the following is the most likely etiology for the patient's diagnosis?

*Idiopathic* Acute pericarditis - in the absence of suggestive features or pertinent past medical history, pericarditis is most commonly idiopathic. The most common etiology of pericarditis is idiopathic, followed by viral, malignancy, tuberculosis, autoimmune causes, and bacterial infection.

17 yo boy brought to ED following motor vehicle accident. He is found to have extensive cerebral hemorrhage leading to deep coma, as well as fractures of C4 vertebra, pelvis, and right femur. Following admission to the hospital, he is intubated and central lines are placed. Pt develops acute renal failure due to rhabdomyolysis, which improves rapidly with treatment. He is extubated after regaining consciousness. He also undergoes percutaneous endoscopic gastrostomy tube placement due to persistent probz with swallowing. Pt is unable to ambulate due to quadriparesis resulting from cervical cord injury. 4 wks after his initial injury, he develops nausea & polyuria. His lab results: Calcium 12.1 mg/dL Albumin 3 g/dL Creatinine 1.4 mg/dL Phosphorous 3.1 mg/dL Glucose 108 mg/dL PTH 9 pg/mL Parathyroid hormone-related protein undetectable 1,25-dihydroxyvitamin D 19 pg/mL normal: 20-60 pg/mL) Most likely cause of this pt's hypercalcemia?

*Immobilization* Quadriparesis has correct calcium of 12.9 mg/dL: corrected calcium = measured calcium + 0.8 x (4 - albumin) = 12.1 + 0.8 x (4-3) = 12.1 + 0.8 = 12.9 symptomatic hypercalcemia (nausea, polyuria) + suppressed parathyroid hormone (PTH) via *immobilization* - likely via osteoclastic bone resorption - risk increased in pts with pre-existing high rate of bone turnover (younger ppl, paget dz) - onset of hypercalcemia usually around 4 wks after immobilization, although pts with chronic renal insufficiency may develop hypercalcemia in as little as 3 days - bisphosphonates inhibit osteoclastic bone resorption = effective in treating hypercalcemia of immobilization and reducing associated bone loss

34 yo man brought to ED due to several hrs of confusion. His wife reports that he has had fever, malaise, and cough for past 2 days. Year ago, pt required prolonged hospitalization and extensive surgery for multiple gunshot wounds to abdomen. He takes no meds regularly and has no other med probz. Pt doesn't use tobacco, alcohol, or illicit drugs. He has no ho recent travel. Temp is 104.9 F, BP is 80/50 mmHg, pulse is 110/min, & respirations are 32/min. Mucous membranes are moist and no cervical LAD present. Dullness to percussion & crackles over left lower chest present. CV exam reveals normal first & second heart sounds and bounding peripheral pulses. Abdomen has several well-healed surgical scars. IV fluids & broad-spectrum abx initiated. Next day, blood cultures show gram-positive cocci. Most likely underlying mechanism leading to this pt's clinical presentation?

*Impaired Ab-facilitated phagocytosis* Splenectomy hc - during his operation for multiple gunshot wounds to abdomen - now has fever, hypotension, tachypnea, tachycardia in setting of bacteremia with gram-positive cocci = overwhelming Strep pneumoniae infection - encapsulated organisms such as S pneumoniae, H influenza, N meningitidis = polysaccharide exterior that conceals antigenic epitopes --> resists innate phagocytosis --> largely eliminated via humoral immune response with Ab-mediated phagocytosis (opsonization) & Ab-mediated complement activation - these pts should be immunized with pneumococcal, meningococcal, and H influenzae type B vaccines & take oral abx early in course of any febrile illness

68 yo woman comes to clinic with recent onset of right-sided facial droop. Her BP is 135/90 mmHg and pulse is 76/min. PE performed. What findings is most consistent with facial nerve palsy due to lesion below (downstream) of pons?

*Inability to close eye on affected side* rapid onset of unilateral upper and lower facial weakness = Bell's palsy / acute *peripheral* neuropathy of CN VII (*lesion below pons*) - affected side: inability to raise eyebrow or close eye, drooping of mouth corner (mouth drawn to unaffected side), disappearance of nasolabial fold - decreased tearing, hyperacusis and/or loss of taste sensation over anterior 2/3 of tongue It is important that pts be assessed for symmetry by raising their eyebrows via bilateral upper-motor-neuron innervation of forehead - forehead muscle sparing = suggestive of intracranial lesion --> warrants brain imaging to evaluate for ischemia or tumor

A 53-year-old woman presents to her primary care physician for a routine examination. She notes that she began having hot flashes several months prior to presentation along with occasional painful urination. She notes that her periods have become more frequent and irregular, but have become lighter overall. Which of the following is most likely to be true regarding this patient's levels of follicle stimulating hormone (FSH), luteinizing hormone (LH), and androstenedione?

*Increased FSH, increased LH, no change in androstenedione* Menopause - characterized by increased FSH and LH, with no change in androstenedione levels - depletion of ovarian follicles, which leads to decrease estrogen release by granulosa cells. Because estrogen negatively controls levels of FSH and LH, levels of both FSH and LH increase over the course of menopause. - A serum FSH >20 mIU/mL, in the context of consistent symptoms, is considered diagnostic of menopause. - *Theca cells in the ovaries continue to produce androstenedione, leading to roughly stable serum levels.* DEBATE??? Ovary and the adrenal cortex both produce androstenedione. - The central pathology in menopause is the decreased production of estrogen (and testosterone) and the increased FSH and LH that result from declining ovarian function. - After menopause, the ovary still produces some androstenedione and the adrenal glands continue to produce at the normal level as well. - Technically the androstenedione level would decrease (some sources say it could approach 50%).

32 yo man comes to ER bc of acute onset of left flank pain, hematuria, and vomiting. His pain is relieved with analgesics in ER. He has ho abdominal pain due to Crohn dz, but that pain was always in right lower quadrant and was never this severe. His temp is 98.2 F, BP is 120/65 mmHg, pulse is 110/min, & respirations are 16/min. Chest auscultation clear. Abdomen soft and mildly tender over left flank. He has no rebound or rigidity. Bowel sounds decreased. Laparotomy scar present in right lower quadrant. Most likely cause of his symptoms?

*Increased absorption of oxalate* Nephrolithiasis + Crohn dz - any other SI symptoms = fat malabsorption --> predisposes to hyperoxaluria Pts with fat malabsorption - calcium preferentially bound by fat --> leaves oxalate unbound & free to be absorbed into bloodstream - failure to adequately absorb bile salts in states of fat malabsorption --> decreased bile salt reabsorption in SI --> damage to colonic mucosa --> increased oxalate absorption

A 17-year-old girl presents to a women's health clinic requesting contraception. She is prescribed combination estrogen-progesterone oral contraceptives. The patient returns 3 weeks later stating that she has gained 10 kilograms of body weight. In addition she complains of headaches of new onset. On exam you note a pudgy, overweight and disgruntled young woman who aggressively blames you for the weight change stating that her boyfriend left her and wants to be taken off of oral contraceptives. Otherwise there are no other abnormalities. Her blood pressure is 145/90 mmHg with a pulse of 90/minute. Due to her recent stress she has began smoking. Which of the following symptoms are attributable to this patient's recent medication change?

*Increased blood pressure and headaches* While studies show that adolescents are frequently concerned that oral contraceptive pills (OCPs) will cause weight gain, there has been no evidence to demonstrate that OCPs have any effect on patients' weight. The patient's increased blood pressure and headaches however can be associated with OCP use. Side effects of oral contraceptive pills (OCPs) include: - bloating - nausea - breast tenderness - headache - abnormal bleeding

A 78 year-old woman is brought to the emergency department by her maid, who found her lethargic and less responsive than usual this morning. The maid had been on vacation for the last week and reports that the woman was alert and active previously. The patient has a past medical history of hypertension and early dementia, and her medications include metoprolol and memantine. Her temperature is 36.8 C, blood pressure 94/60 mmHg, heart rate 96/min, and respiratory rate 16/min. Her blood pressure decreases to 84/50 mmHg while standing. She is lethargic and oriented to person and place only. She has dry oral mucosa and decreased skin turgor. Her heart, lung, and abdominal exams are within normal limits and stool guaiac is negative. Initial labs are as follows: Sodium 124 mEq/L, potassium 3.4 mEq/L, chloride 95 mEq/L, bicarbonate 31 mEq/L, BUN 28 mg/dL, creatinine 1.0 mg/dL, and glucose 120 mg/dL. Which of the following best describes the levels of renin, aldosterone, and ADH in this patient's blood?

*Increased renin, increased aldosterone, increased ADH* Hyponatremia due to increased levels of renin, aldosterone, and antidiuretic hormone in response to volume depletion. - juxtaglomerular apparatus in the kidney senses decreased perfusion and increases the production of renin --> renin catalyzes the conversion of angiotensinogen to angiotensin I, which is converted by angiotensin converting enzyme (ACE) into angiotensin II --> this hormone increases production of aldosterone by the adrenal gland --> aldosterone, in turn, acts on the distal convoluted tubule of the nephron to increase the absorption of sodium and water; it also decreases the absorption of potassium - the secretion of ADH is also increased by the posterior pituitary in response to volume depletion. this hormone acts to increase the absorption of free water in the collecting duct of the nephron - the combined effect of these hormones is to reabsorb hypotonic fluid, resulting in hyponatremia. - question: patient with mild dementia on memantine. elderly patients with dementia can become volume depleted by not drinking sufficient amounts of water to replace losses especially in hot weather. - metoprolol's effect on renin: beta blockers can reduce the level of renin and aldosterone; however, we can determine the likely level of renin and aldosterone based on the physiological effects. In this case we see hypokalemia and a metabolic alkalosis, consistent with high levels of aldosterone. Given that this patient has no reason for ectopic production of aldosterone, we can also assume that the level of renin is high. Indeed the effect of beta blockers causes a short term decrease in RAAS but has been shown in some studies to have no long term effect on RAAS level.

Cortisol levels in major depressive disorder (MDD)

*Increased serum cortisol concentration* MDD - hyperactivity of hypothalamic-pituitary-adrenal axis = increased cortisol - may explain association of stress and trauma with increased risk for dev't of depression Other findings... - decreased hippocampal and frontal lobe volumes - decreased REM sleep latency and slow-wave sleep

A 45-year-old man presents to the emergency department with abdominal pain. He was found unconscious on a park bench. The patient is assessed and started on IV fluids and naloxone. His temperature is 99.5°F (37.5°C), blood pressure is 100/58 mmHg, pulse is 120/min, respirations are 17/min, and oxygen saturation is 98% on room air. He wakes up and begins complaining of abdominal pain. The patient is given ibuprofen and IV fluids. His lipase is 278 U/L. Repeat vitals demonstrate a blood pressure of 87/48 mmHg. Which of the following is the best explanation of this patient's current vitals?

*Increased vascular permeability* *Hypotension associated with pancreatitis* in the acute setting is likely from increased vascular permeability. - typically occurs secondary to vasodilation, hypovolemia, increased vascular permeability, and reduced plasma oncotic pressure. - In the acute phase, vasodilation and increased vascular permeability are more likely causes of hypotension from pancreatitis. In the chronic phase reduced plasma oncotic pressure could be a more prominent cause. Diagnosis of pancreatitis depends on 2 of the following three criteria: an appropriate clinical picture; imaging studies compatible with pancreatitis; elevation of amylase and/or lipase in the plasma. Illustration A is a gross specimen of acute pancreatitis.

42 yo man comes to hospital after day of severe abdominal pain and vomiting. He describes pain as constant and nagging, localizing to upper abdomen, and radiating to back. Pt has not been to doc in years and has no previous med conditions or surgeries. He takes no meds. He drinks large amounts of alcohol, but does not use tobacco or illicit drugs. His temp is 98 F, BP is 110/80 mmHg, pulse is 90/min, & respirations are 14/min. Pt admitted to hospital and given opioid analgesics and IV fluids. On day 2 of his hospitalization, his BP drops to 80/60 mmHg and his pulse increases to 120/min & regular. His oxygen sat is 92% on 2 L nasal canula oxygen. Exam shows normal jugular venous pressure. Bilateral crackles heard on lung auscultation. Pt's abdomen mildly distended, and there is tenderness in epigastrium. His urine output is 8 mL/hr. He has received total of 4 L IV fluids over last 24 hrs. Lab results: Hgb 14.9 g/dL Leukocytes 16,500/mm^3 Platelets 120,000/mm^3 Sodium 138 mEq/L Potassium 4.9 mEq/L Chloride 97 mEq/L Bicarbonate 21 mEq/L Glucose 210 mg/dL Creatinine 1.9 mg/dL BUN 45 mg/dL Most likely cause of this pt's hypotension?

*Increased vascular permeability* Acute severe pancreatitis - progressed to multisystem dysfunction = shock, renal failure, early resp failure - pancreatitis with failure of at least 1 organ - age > 75 + alcoholism + obesity + C-reactive protein >150 mg/dL at 48 hrs + increased BUN/Cr in first 48 hrs - intravascular volume loss secondary to local & systemic vascular endothelial injury --> vasodilation, increased vasc perm, plasma leak into retroperitoneum = systemic hypotension

64 yo man comes to doc due to SOB & abdominal distention. He was treated for Hodgkin lymphoma with radiation and chemotherapy 18 yrs ago and was told that he was cured. Pt drinks alcohol on regular basis. His temp is 98 F, BP is 120/76 mmHg, pulse is 92/min, & respirations are 20/min. Neck exam shows jugular venous pulsations 9 cm above sternal angle. Abdomen distended with positive fluid wave. Liver edge palpated 5 cm below right costal edge. Bilateral LE pitting edema. Initial lab results: Serum creatinine 0.8 mg/dL Albumin 4 g/dL Total bilirubin 1 mg/dL Prothrombin time 11 sec Most likely cause of this pt's condition

*Inelastic pericardium* - prevents venous return to right heart during inspiration --> RHF Right heart failure via *constrictive pericarditis* - potential late complication of radiation therapy - *survivors of Hodgkin lymphoma are at increased risk for cardiac dz, which can present as much as 10-20 yrs or more after mediastinal irradiation and/or anthracycline therapy* - peripheral edema + ascites + hepatic congestion with hepatomegaly --> cirrhosis (cardiac cirrhosis) - PE = JVP + prominent x & y descents & hepatojugular reflux, Kussmaul's sign (lack of decrease or increase in JVP on inspiration), pericardial knock = mid-diastolic sound

40 yo man comes to ED with 2 wks of fever, malaise, weakness, & unintional 5-lb wt loss. During last 4 wks, he developed left-sided chest & upper abdominal pain. He has asymptomatic mitral valve prolapse. His wife was treated for URI 3 wks ago. Pt works for large corporation and frequently travels to Mexico. He has 20 pck yr smoking hx. He doesn't use alcohol. His temp is 103 F, BP is 120/70 mmHg, pulse is 96/min, & respirations are 16/min. Lung exam shows decreased breath sounds in left lower lung field with dullness to percussion. Cardiac exam shows 2/6 systolic murmur at apex. Lab results are as follows: WBCs 27,000/uL - Neutrophils 60% - Bands 15% Hemoglobin 13 g/dL Platelets 250,000/uL Imaging studies reveal left-sided pleural effusion & splenomegaly with splenic fluid collection. Most likely underlying diagnosis in this pt?

*Infective endocarditis* Fevers + chills + LUQ pain + splenic fluid collection = *splenic abscess* - classic triad = *fever, leukocytosis, LUQ abdominal pain* - can also develop left-sided pleuritic chest pain + left pleural effusion + splenomegaly - risk factors = *infective endocarditis* & hematogenous spread, immunosuppression, IV drug use, trauma, hemoglobinopathies - 10-20% incidence of associated splenic abscess or infarction with left-sided endocarditis - mechanism = hematogenous seeding or septic emboli to spleen - splenic abscesses via staph, strep, salmonella - percutaneous drainage may be option for poor in surgical

65 yo man comes to clinic with pain in his right shoulder for past few wks. He works as house painter and experiences pain when he reaches for objects or lifts his arm above his head. Pt has no recent hx of trauma to shoulder, fever, chills, or wt loss. His other med probz include COPD and degenerative joint dz of hands. He has smoked 1 pck of cigz daily for 40 yrs and has recently decided to cut down. VItals within normal limits. On exam, doc flexes pt's arm while asking to relax and point thumb toward the floor. At 60 degrees of flexion, pt begins to shrug his shoulder and complain of pain. In spite of pain, his range of motion is normal. Most likely responsible for his current condition?

*Inflammation of rotator cuff tendons* Rotator cuff tendinopathy (RCT) - via repetitive activity above shoulder height (ex. painting ceilings) - most common in middle-aged and older individuals - chronic tensile loading & compression by surrounding structures --> microtears in rotator cuff tendons (especially supraspinatus), fibrosis, inflammatory calcification - in addition to rotator cuff itself, pain may also emanate from subacromial bursa & tendon of long head of biceps - flexion of abduction of humerus --> space b/t humeral head & acromion reduced = pressure on supraspinatus tendon & subacromial bursa - impingement syndrome = characteristic - impingement can be demonstrated by *Neer test*: pt's shoulder internally rotated & forearm pronated --> examiner stabilizes scapula & flexes humerus --> reproduction of pain = positive test

A 45-year-old lady presents to your primary care office complaining that she has lost 15 pounds in the last three months. She states that she is "just not hungry anymore." She is up to date on pap smears and mammograms (which were negative), but states that she has not had a colonoscopy. What is the best next step in management?

*Inquire about changes in mood* Presenting with unexplained weight loss - *The differential for this presentation is broad, but it is important to screen for major depressive disorder before ordering invasive tests or recommending treatments* Major depressive disorder - alterations in sleep, interest in activities, guilt, energy, concentration, appetite, psychomotor retardation, and suicidal ideation - often these symptoms manifest in a physical manner that does not immediately suggest a mood disorder, in this case weight loss - important to maintain a low threshold for screening patients for major depression - the USPSTF currently recommends routine screening for all adults if the necessary support staff is in place - The recommend the use of mental health questionnaires, particularly the Patient Health Questionnaire-2 (PHQ2) and the Patient Health Questionnaire-9 (PHQ9) - The PHQ2 assesses for the presence of anhedonia (loss of interest in activities) and dysphoria (depressed mood). If a patient answers yes to either of these two questions, then the PHQ9 is administered and will allow the clinician to make the diagnosis of major depression

A 77-year-old woman presents to general medical clinic with a chief complaint of a depressed mood. She reports that she has been unable to sleep regularly for ten days. During this time, she has no interest in knitting or reading, two of her favorite hobbies. She feels guilty that she has not been able to improve her mood. She denies any suicidal ideations. Which of the following is true regarding the diagnosis of major depressive disorder in this patient?

*Insufficient duration and number of symptoms* This patient only has 3 of the 5 required symptoms for diagnosis. Her symptoms of ten days are 4 days short of meeting criteria for duration. Major depressive disorder is defined as episodes characterized by at least 5 of the following 9 symptoms for 2 or more weeks, one of which must be depressed mood or anhedonia: - sleep disturbance - loss of interest in things that usually give the patient pleasure (anhedonia) - guilt or worthlessness - low energy - decreased concentration - appetite and weight changes - psychomotor retardation or agitation - suicidal ideation - depressed mood Major depressive disorder episodes often last 6-12 months.

A pharmaceutical company conducts a randomized clinical trial in an attempt to show that their new anticoagulant drug, Aclotsaban, prevents more thrombotic events following total knee arthroplasty than the current standard of care. However, a significant number of patients are lost to follow-up or fail to complete treatment according to the study arm to which they were assigned. Despite this, the results for the patients who completed the course of Aclotsaban are encouraging. Which of the following techniques is most appropriate to use in order to attempt to prove the superiority of Aclotsaban?

*Intention-to-treat analysis* - used to help preserve the benefits of randomization in superiority trials - In an ideal setting, all subjects enrolled in a research trial would complete the trial exactly as prescribed in the protocol. However, this ideal is rarely achieved, and thus trial design and data analysis must account for incomplete adherence and follow-up. - In which subject results are analyzed according to the group that they were initially assigned to (not according to adherence) is one technique used to analyze outcome data and preserve randomization - Because participants may drop out because of adverse effects or may need to modify their treatment course for other unknown reasons, intention-to-treat analysis helps to maintain randomization and gives conservative results in superiority trials

A 72-year-old male is brought to his primary care physician by his daughter for concerns of worsening memory. She states that he several months ago began having a hard time remembering what items to buy at the supermarket. Per his daughter, his memory has since worsened, and now he is often unable to recall the topic of television shows he watches or articles he reads in the paper. She also notes that he sometimes has a hard time finding words when speaking. She is especially concerned because last week he forgot he left the stove on after making dinner. Otherwise he has been his "usual self," and he has been enjoying spending time with his family and gardening. The patient reports that he has noted increased difficulty remembering certain things, but does not feel it is a major issue. Which of the following histologic brain findings is associated with this patient's condition?

*Intracellular neurofibrillary tangles consisting of hyperphosphorylated tau protein* Alzheimer's Disease (AD) - *Intracellular neurofibrillary tangles consisting of hyperphosphorylated tau protein are a histologic finding of AD* - most common cause of dementia occurring in old age - classically presents with short-term memory loss which progresses to long-term memory loss, disorientation, and behavioral and personality changes - can be confirmed through post-mortem histologic findings of extracellular Aß-amyloid plaques and intracellular neurofibrillary tangles consisting of hyperphosphorylated tau protein - these intracellular tangles correlate with the degree of dementia.

A 24-year-old female with a BMI of 36 undergoes a Caesarian section after prolonged labor led to fetal distress. On post-operative day 2, she is noted to have exquisite uterine tenderness and decreased bowel sounds. Her temperature is 38.8 degrees Celsius, heart rate is 102 beats per minute, and blood pressure is 132/78. Which of the following is TRUE regarding this patient's most likely condition?

*Intraoperative antibiotics decreases the risk of this condition.* Postpartum endometritis - *Among patients undergoing a Caesarian section, the risk of this condition is significantly decreased with administration of intraoperative antibiotics* - infection of the decidua that most commonly occurs between 24 and 72 hours postpartum - the risk of this condition is 1-3% overall, but increases significantly in patients who undergo a Caesarian section. In these patients, intraoperative antibiotics significantly decrease the risk of postpartum endometritis - Other risk factors for this condition include young maternal age, obesity, chronic conditions, and prolonged labor.

A 25-year-old G1P0 woman visits her obstetrician at 36 weeks gestation for a routine prenatal visit. Cultures of her vaginal and perianal regions were positive for Group B streptococcus. She has no known drug allergies. Which of the following management options is appropriate for this patient?

*Intrapartum* penicillin G Baby will be at risk for GBS infection at the time of delivery, due to GBS colonizing in areas such as the rectovaginal tract. - *If antibiotics are given once the infection is confirmed during GBS screening at 35-37 weeks, the infection may not be fully eradicated by the time of delivery. Therefore, the antibiotic (penicillin being the drug of choice for anaphylaxis) is given during the intrapartum period.* - Note that this is given intravenously to achieve high concentrations of the medication in the mother's serum and the amniotic fluid. For patients with a penicillin allergy: - If a woman has a history of penicillin allergy, but is NOT at high risk for anaphylaxis, she should receive cefazolin or clindamycin instead of penicillin intrapartum. About 10% of patients with a penicillin allergy will have an immediate hypersensitivity reaction to cephalosporins. - If a woman has a penicillin allergy AND is at high risk for anaphylaxis, she should receive vancomycin or clindamycin instead of penicillin intrapartum.

64 yo man comes to ED due to 2 days of fever, chills, productive cough, and left-sided pleuritic chest pain. Pt has ho HT, hyperlipidemia, and T2DM. He is former smoker and drinks alcohol occasionally. Temp is 102 F, BP is 110/60 mmHg pulse is 110/min, & respirations are 22/min. Oxygen sat is 94% on RA when lying on his right side but drops to 89% when he lies on his left. Dullness to percussion & bronchial breath sounds are present on left side. What pathophysiological mechanisms in the left lung is most responsible for drop in oxygen sat after change in position?

*Intrapulmonary shunting* fever + pleuritic chest pain + hypoxemia + dullness to percussion with bronchial breath sounds in left lung field = *acute pneumonia* - alveolar consolidation --> impaired alveolar ventilation in affected portion of lung --> hypoxemia via *right-to-left intrapulmonary shunting* of blood & extreme ventilation/perfusion mismatch (V~0)

75 yo man brought to ED with severe back pain that began 3 wks ago. Pain has been significantly worse for last 12 hrs. This morning, pt also noticed difficulty walking and urinating. He has ho advanced prostate cancer initially treated with local radiation therapy. Temp is 98.6 F, BP is 122/83 mmHg, and pulse is 104/min. Pt appears uncomfortable. He has point tenderness over midline spine near T10 and T11. Upper extremity strength and reflexes are normal. Muscle strength in LEs is 3/5 and deep-tendon reflexes are 3+ bilaterally. Bilateral plantar reflexes are upgoing. Straight urinary catheterization produces 800 mL of urine. Best initial step in management of this pt?

*Intravenous glucocorticoids* - should be given without delay in pt with suspected ESCC - decrease vasogenic edema (caused by obstructed epidural venous plexus) - helps alleviate pain - may restore neurologic function Advanced prostate cancer - subacute back pain that has now profoundly changed in severity - exam = LE motor weakness + hyperreflexia + bladder dysfunction --> *raises concern for epidural spinal cord compression (ESCC)* ESCC - thoracic & lumbosacral = most common locations - pain = usually first symptom = often present for 1-2 months before additional symptoms - motor findings (bilateral weakness) & ataxia are common as dz progresses - bowel & bladder dysfunction = late findings - once imaging confirms ESCC, neurosurg consultation typically required

Diagnosis of follicular thyroid cancer (FTC) based on limited tissue sample is not possible as cytologic findings are similar to benign follicular adenomas. However, exam of surgically excised nodule with show....

*Invasion of tumor capsule and/or blood vessels* FTC - can metastasize via hematogenous spread to distant tissues - 2nd most common thyroid epithelial malignancy (after papillary cancer) - peak incidence of 40-60 yo - firm thyroid nodule - often discovered incidentally on exam or imaging for other purposes - thyroid scintigraphy = nonmetabolically active cold nodule

A 7-year-old boy falls from a treehouse and lands on the back of his left elbow. In the emergency department, an radiograph of the arm is taken. On physical exam, there is a great deal of swelling over the distal end of the left humerus. The skin overlying the injury is intact. Steps should be taken immediately to prevent which of the following complications of this injury?

*Ischemia of hand* supracondylar humeral fracture - due to the proximity to the brachial artery, a dreaded complication of this injury is ischemia of the hand, resulting in Volkmann ischemic contracture. - most commonly seen in boys aged 5-8 years old - mechanism of injury is classically a blow to the back of the lower end of the humerus or a fall on outstretched hand - orthopaedic surgery should be consulted for displaced supracondylar fractures - is critically important to check the status of the brachial artery by monitoring the radial pulse after pre & post-reduction as the neuromuscular bundle running close to the fracture may be compromised - a compartment syndrome may result, in which symptoms include pain (out of proportion to physical findings, which is typically the first symptom), pain with passive motion of fingers or toes, pallor, paresthesia, poikilothermia, paralysis, and pulselessness

True regarding direct observational therapy (DOT) in treatment of active TB?

*It decreases drug-resistant TB* - all pts with TB - doesn't guarantee ingestion of all doses bc pts may miss appointments, may not swallow, etc - public health prevention

A 52-year-old overweight male presents to your office complaining of heartburn for 6 months. He describes burning in his chest brought on by meals that are especially fatty or spicy, associated with belching and sometimes coughing. He has a 20 pack-year smoking history and drinks 2 glasses of red wine with dinner nightly. He denies dysphagia, odynophagia, weight loss, melena, and hematemesis. Over the past month he has reduced his intake of fatty and spicy foods with some moderate relief of his symptoms, but he requests more aggressive therapy. Which of the following is the most appropriate next step in the care of this patient?

*Lansoprazole trial* - The patient's presentation is consistent with gastroesophageal reflux disease (GERD) - First line treatment of GERD is conservative therapy with diet and lifestyle modifications, second line treatment is pharmacologic therapy with a proton pump inhibitor (PPI) such as lansoprazole *GERD is initially treated with behavioral interventions such as avoidance of spicy foods, not drinking alcohol before bed, quitting smoking, eating small servings, and not lying down after eating. Along with those lifestyle changes, H2-receptor antagonists (H2RAs) or PPIs are used. PPIs have been shown to be more efficacious than H2RAs, and they are preferred in this patient. If medical treatment fails, surgical intervention is considered with Nissen fundoplication, in which the gastric fundus is secured around the lower esophageal sphincter (LES), thus reinforcing the LES.*

55 yo man comes to office due to elbow pain. Fow last 3 wks, pt has had vague, achy pain at left elbow that radiates to forearm and is worse with activity and at end of day. He has attempted treatment with OTC acetaminophen without relief. Pt works as airport baggage handler and has been seen in office previously for minor occupational injuries. His med hx is otherwise unremarkable. He smokes half-pack of cigz daily and doesn't use alcohol or illicit drugs. Vitals are normal. Inspection of left elbow shows no erythema or swelling. ROM normal. With elbow held in extension, passive flexion of wrist reproduces pt's pain. Most likely diagnosis in this pt?

*Lateral epicondylitis* / tennis elbow - subacute pain at elbow & forearm --> reproduced on stretching of wrist extensors - due to overuse of extensor muscles, primarily the *extensor carpi radialis brevis* & extensor digitorum commnis - primary path lesion = noninflammatory angiofibroblastic tendinosis at common extensor origin on lateral epicondyle of humerus - ho repetitive, forceful extension at wrist - exam = tenderness at lateral epicondyle & reproduction of pain with resisted extension or passive flexion at wrist - management = activity modification, NSAIDs, *counterforce bracing*

A 6-year-old boy presents to the emergency department with fever and neck pain. On physical exam, cervical lymphadenopathy is palpated, and a poorly-characterized fullness is noted in the pharynx. The uvula is midline, and no stridor is noted. Which of the following would be the most likely findings on an urgent imaging study performed in this case?

*Lateral neck radiograph showing widening of retropharyngeal space* Retropharyngeal abscess - posterior pharyngeal mass, cervical lympthadenopathy in the absence of stridor or uvular deviation - lateral neck radiograph would most likely show a widening of the retropharyngeal space. - infection of the retropharyngeal space - most commonly affects children 6 months to 6 years of age - causative organisms include Group A Strep, S. aureus, and Bacteroides - as a consequence of fascial planes in the neck, the retropharyngeal space is continuous with the mediastinum --> this communication results in the potential for infection of this space to spread into deeper areas (including the mediastinum) with disastrous consequences - emergent exploration and drainage is required when this diagnosis is made - typically, retropharyngeal abscesses present with swelling in the neck, which can result in sudden airway obstruction

53 yo man comes to ED with squeezing chest pain that started 2 hrs ago. He also has severe SOB that is worse when lying down. He has never had pain like this before. He has no significant past med hx and takes no meds. Routine health maintenance visit 2 wks ago was normal. His BP is 98/60 mmHg, and pulse is 110/min & regular. Chest auscultation reveals grade III/VI holosystolic murmur at cardiac apex and bibasilar crackles in lungs. ECG shows ST segment elevations in leads II, III, aVF. Most likely increased in this pt?

*Left ventricular filling pressure* sudden onset chest pain + ST segment elevation + holosytolic murmur at apex + bibasilar crackles = acute inferior MI + papillary muscle displacement --> *acute mitral regurgitation & pulmonary edema* - acute MI --> can develop acute MR 2-7 days after infarct due to papillary muscle rupture Acute MR - excessive volume of blood leading back into left atrium - during diastole = initial, rapid filling of left ventricle - excessive diastolic volume overload --> elevated left ventricular end diastolic volume (LVEDP, LV filling pressure) --> reflected back in left atrium & pulmonary circulation --> signs & symptoms of acute pulmonary edema & CHF

47 yo man comes to doc for routine health exam. He has no symptoms except for mild increase in frequency of urination. He has hx of elevated glucose levels but no other med probz and takes no meds. His fam hx is significant for T2DM. Pt doesn't use tobacco, alcohol, or illicit drugs. He works as editor for local magazine company. His temp is 98 F, BP is 132/80 mmHg, pulse is 72/min, & respirations are 16/min. BMI 30 kg/m^2. Fasting lab: Glucose 165 mg/dL Total cholesterol 210 mg/dL LDL 140 mg/dL Triglycerides 140 mg/dL HDL 35 mg/dL Hgb 7.2% Serum creatinine 1.1 mg/dL Most appropriate next step in management of this pt?

*Lifestyle modification + rosuvastatin* Frequency of urination increased + elevated fasting glucose & hgb A1C = diagnosis of new-onset T2DM - increased risk of astherosclerotic CV dz - >40 yo = benefit from lipid-lowering therapy with statins Diabetic pts 40-75 yo + estimated risk >7.5% - high-intensity statin therapy = atorvastatin 40-80 mg or rosuvastatin 20-40 mg daily 10-yr risk <7.5% = moderate-intensity therapy = atorvastatin 10-20 mg, simvastatin 20-40 mg

A 72-year-old female with a history of diabetes, asthma, and COPD is hospitalized for left leg cellulitis. She receives oral (PO) antibiotic therapy with clindamycin and begins to improve with significant reduction in the erythema and the area affected by the cellulitis. However, on day 4 of her hospitalization she complains to the team that she is having a large number of watery, foul-smelling stools. A stool toxin PCR is ordered; however, the PCR is found to be negative. What is the most appropriate next step in the management of this patient's diarrhea?

*Limited sigmoidoscopy* Patient with very high suspicion for C. difficile but whose toxin PCR comes back negative --> limited sigmoidoscopy or colonoscopy should be performed to confirm diagnosis by observing pseudomembranous colitis. C. difficile - gram positive bacterial infection causing pseudomembranous colitis, which often presents with recurrent, watery, foul smelling diarrhea - Diagnosis of C. difficile is made by identification of the C. difficile toxin by ELISA or PCR - *In cases where C. diff is highly suspected (recent antibiotic use, classic presentation) but a toxin assay is negative, a limited sigmoidoscopy or colonoscopy should be performed to confirm diagnosis by noting pseudomembranous colitis* In this patient, they are currently stable (no severe symptoms or signs of renal failure), thus immediate treatment with metronidazole is not mandatory. - Typically, you would do a stool toxin test which is more sensitive and specific than endoscopy. However, if the stool toxin test is negative, further workup to confirm the diagnosis needs to take place (making endoscopy a viable option).

50 yo man comes to ED due to sudden onset of severe, colicky pain in rt flank. He was admitted twice previously for similar symptoms. On both occasions, pt was managed conservatively and sent home. He has no other med probz & no ho urinary infections. He usually eats precooked food from local store & has soda with each meal. Pt doesn't use tobacco, alcohol, or illicit drugs. He is given IV fluids & pain meds. Lab results: Hgb 14.5 g/dL Leukocytes 13,000/mm^3; no bands Platelets 300,000/mm^3 BUN 16 mg/dL Creatinine 0.8 mg/dL CT scan of abdomen without contrast reveals renal calculi. Urinalysis shows moderate levels of blood and calcium oxalate crystals. Best recommendation for prevention of future stones in pt?

*Limited sodium intake* Most common renal stones = *calcium stones* - increased sodium intake --> enhances calcium excretion (hypercalciuria) - reabsorption of sodium & calcium coupled via complex mechanisms involving calcium-sensing receptor in thick ascending limb of loop of Henle --> pts with recurrent renal calculi = should restrict sodium intake --> if these pts continue to develop renal stones --> urine sodium levels may be checked to evaluate adherence to sodium-restricted diet Dietary recommendations for pts with renal calculi: - increased fluid intake - decreased sodium intake - normal dietary calcium intake

66 yo man comes to clinic for follow-up for HT Two mos ago, he had high BP (165/95 mHg) during routine office visit, and since then has had 2 follow-up visits, both documenting high BP. Pt states that he currently has no symptoms and has never been diagnosed with HT, but he does have T2DM & hyperlipidemia. He underwent stent placement for peripheral vascular dz 2 yrs ago. Pt is former cig smoker with 25-pck-yr hx. Currently, his BP is 162/93 mmHg, and his HR is 73/min & regular. His BMI is 31 kg/m^2, & PE is unremarkable. Serum creatinine is 0.8 mg/dL. CT angiography reveals 80% atherosclerotic narrowing of right renal artery. In addition to antihyperlipidemic therapy, what is the best next step in management of this pt?

*Lisinopril* Renal artery stenosis - common in older pts - high prevalence in those with severe HT or PAD - renovascular HT = most common correctable cause of 2ndary HT - pts with HT should be managed initially with ACEIs or ARBs - renal artery stenting or surgical revascularization reserved for pts with resistant HT or recurrent flash pulm edema and/or refractory HF due to severe HT

74 yo man comes to office with increasing SOB, especially on exertion. He also has cough with mucoid expectoration, especially in morning. He has no orthopnea, paroxysmal nocturnal dyspnea, or chest pain. Pt was hospitalized for CAP 2 yrs ago. His other med probz include glaucoma, BPH, & HT. He smoked pck of cigz for 40 yrs and quit 2 yrs ago. His temp is 98.9 F, BP is 144/96 mmHg, pulse is 82/min, & respirations are 16/min. Pulse oximetry shows oxygen sat of 89% on RA at rest. His face appears plethoric. Chest is barrel shaped. Breath sounds diminished throughout, and expiratory phase prolonged. Heart sounds distant but regular, and there are no murmurs or gallops. No peripheral edema or JVD. Lab results: CBC - Hct 56% - Platelets 240,000/mm^3 - Leukocytes 7,500/mm^3 Serum chemistry - Sodium 140 mEq/L - Potassium 4.2 mEq/L - BUN 18 mg/dL - Creatinine 1.2 mg/dL What interventions will have max impact on this pt's survival?

*Long-term supplemental oxygen at home* - prolonged survival Criteria for LTOT - resting arterial oxygen tension (PaO2 55 mmHg or pulse oxygen sat (SaO2) <88% on RA - PO2 < 59 mmHg or SaO2 <89% in pts with cor pulmonale, evidence of RHF, or Hct >55% (as in this pt) COPD - smoking = most important risk factor - chronic hypoxemia often present in advanced dz --> consequent secondary polycythemia may occur

On-call surgical resident paged to see 35-yo man who is having seizures. Pt underwent emergency cholecystectomy one day ago. Surgery was uneventful, but he has gradually become more restless since and became tremulous overnt. Pt has no prior health records at this hospital and lives alone. He reported med hx remarkable only for gastritis. Omeprazole is one of his home meds. Pt's temp is 100.4 F, BP is 145/92 mmHg, pulse is 114/min, & respirations are 18/min. Exam shows diaphoresis and tremors. His wound incisions show no evidence of inflammation, and biliary drainage appears adequate. Lab results: Serum chemistry - Sodium 137 mEq/L - Potassium 4 mEq/L - Chloride 101 mEq/L - Bicarbonate 24 mEq/L - BUN 12 mg/dL - Creatinine 1 mg/dL - Glucose 104 mg/dL CBC - Hgb 14.1 g/dL - WBCs 9,000/mm^3 Liver function studies - AST 212 U/L - ALT 224 U/L Arterial blood gases - pH 7.40 - PO2 94 mmHg - PCO2 40 mmHg Most appropriate immediate treatment for this pt?

*Lorazepam* - immediate-duration benzo - IV - preferred in hospital setting - safer in pts with liver dz - no active metabolites seizures + diaphoresis + tremulousness + elevated pulse and BP one day after hospitalization = *alcoholic withdrawal* - peaks during second day after cessation - seizures most likely to occur at 12-48 hrs - untreated --> delirium tremens (DT) 2-4 days after last drink = fatal in 5% of cases

50 yo man + 1 hr ho intense retro-orbital headache. Started while he was jogging and eased somewhat when he stopped, but has persisted along some pain in his neck. Other than BP of 165/100 mmHg, exam is unremarkable. Noncontrast CT unremarkable. Pain persisted after 2 hrs in ED. Next step?

*Lumbar puncture* - rule out xanthocromia/subarachnoid hemorrhage

65 yo man comes to office due to 4 mo ho periodic back pain radiating to buttocks and thighs. Pain is exacerbated by walking or prolonged standing, although he can tolerate bicycling without significant discomfort. Associated symptoms include occasional tingling and numbness in both LEs. Med hx notable for BPH, hypertension, and hypercholesterolemia, for which he takes appropriate medications. Pt doesn't us tobacco, alcohol, or illicit drugs. His BP is 140/80 mmHg, pulse is 76/min, and respirations are 14/min. On exam, distal pulses are full and symmetric. Neuro exam shows normal motor strength, deep tendon reflexes, and plantar reflexes in LEs bilaterally. Most likely cause of this pt's condition?

*Lumbar spinal stenosis* - pain radiating to back and thighs - via narrowing of spinal canal - mainly in degenerative arthritis = osteophyte formation affecting facet joints = spondylosis - most pts over age 60 - symptoms are *posture-dependent* - extension of lumbar spine = standing, walking upright --> further narrows spinal canal --> worsens symptoms - lumbar flexion = walking uphill, leaning on cane --> relieves pain - onset of pain with walking = neurogenic claudication = resembles symptoms seen in vascular claudication - *relieved by walking while leaning forward = shopping cart sign* - diagnosis confirmed by MRI - treatment = conservative, physical therapy & exercise

A 25-year-old man presents to his primary care physician with lower back pain. He states that he has had the pain for the past two years. The patient works as a butcher, and recently was moving heavy meat carcasses. The patient states that his pain is worse in the morning and that nothing improves it aside from swimming. The patient has a past medical history of anabolic steroid abuse, acne, hypertension and obesity. His current medications are hydrochlorothiazide, ibuprofen, topical benzoyl peroxide, and acetaminophen. On physical exam there is no tenderness upon palpation of the spine. There is limited range of motion of the spine in all 4 directions. Which of the following is most likely to confirm the most likely diagnosis in this patient?

*MRI of the sacroiliac joint* Young male presenting with lower back pain that is worse in the morning and relieved with exercise (swimming) - most accurate test (and most sensitive test) for making the diagnosis is a MRI of the sacroiliac (SI) joint *Any young, male patient that presents with lower back pain that is worse with rest, and improves with exercise suggests a diagnosis of ankylosing spondylitis* - Patients will often have a limited range of motion on physical exam - Initial tests include a radiograph of the spine and a radiograph of the sacroiliac joint - *The most accurate test for diagnosis is MRI of the SI joint which is a very sensitive test and could demonstrate changes earlier than a radiograph of the spine* NSAIDs, such as indomethacin, is considered first-line for symptomatic management of ankylosing spondylitis. This effectively manages pain and stiffness in the patient. When patients are NOT responsive to NSAIDs, then a TNF-inhibitor would be used, such as etanercept. Nonbiologic disease-modifying antirheumatic drugs such as sulfasalazine are ineffective for axial disease.

78 yo man comes to office due to disturbed sleep. He describes being unable to sleep well for past few mos and says the it is making him feel "slowed down" and without energy. Pt used to get 7-8 hours of restful sleep each night. Now his sleep is restless: he describes needing 45-60 minutes to fall asleep and frequently awakening during night On average, he sleep total of 5-6 hours each night. He often gets up at 430 AM as he is unable to return to sleep. Pt has ho HT treated with amlodipine. He doesn't drink caffeinated beverages or smoke cigz. He used to drink 2 beers with friends during his weekly gold & card games but has canceled these activities of late as he is too tired and no longer enjoys them. Pt is retired and has lived alone since his wife died a year ago. He admits to mild forgetfulness and says that his appetite and concentration "aren't what they used to be." Ht is 5 ft 11 in and weight is 205 lb. PE noncontributory apart from 12-lb wt loss since his last visit 6 mos ago. Most likely explanation for this pt's symptoms?

*Major depressive disorder* Majority of older adults with depression initially present to PCP and focus more on somatic complaints than on subjective changes in mood & interest - initial and middle insomnia + early-morning waking + loss of interest + low energy + impaired concentration + weight loss Common sleep changes in depressoin - multiple awakenings and early-morning awakening - decreased REM latency and restorative sleep

A 74-year-old male smoker with a history of type II diabetes presents to the emergency room complaining of sudden-onset, painless vision loss in his left eye. He describes the feeling as if things went black in his left eye. Vital signs are within normal limits and stable. Physical exam reveals 20/800 vision in the left eye with eccentric fixation and a normal right eye. Fundoscopic exam of the left eye reveals a cherry red spot on the macula. Which of the following is the best next step in management?

*Massage the globe and have the patient breath into a paper bag* Central retinal artery occlusion - *Emergent treatment includes ocular massage, CO2 rebreathing, and decompression of the anterior chamber* - presents acutely as painless monocular loss of vision - the retina will appear pale on fundus exam with a cherry-red macula = occurs because the loss of perfusion of the retina allows blood from the choroid to shine through *Treatment involves digital massage of the globe and CO2 rebreathing, which function to dilate retinal vessels in an attempt to pass the clot into a distal segment of the vessel. Reducing intraocular pressure has also shown a benefit.*

76 yo man with no past med hx has operation for strangulated inguinal hernia, with appx 40 cm of small bowel resected. On morning of third postoperative day, he falls while getting out of bed. Immediately after fall, pt is responsive but confused with slurred speech. He cannot explain what happened. His BP is 89/50 mmHg, pulse is 122/min, & respirations are 24/min. Exam shows decreased bibasilar lung sounds & distended neck veins. Electrocardiogram shows new-onset right bundle branch block with nonspecific ST- and T-wave changes. Immediate resuscitation with wide-open IV fluids attempted but unsuccessful. Shortly thereafter, pt's pupils start to dilate, his pulse drops to 45/min, and he becomes unresponsive. He eventually dies despite resuscitative efforts. Most likely cause of this pt's death?

*Massive pulmonary thromboembolism* Postoperative massive PE complicated by cardiogenic shock - PE complicated by hypotension and/or acute right heart strain - JVD on PE + RBBB on ECG = signs of acute right heart strain - right heart strain --> right ventricular dysfunction, decreased return to left side of heart, decreased cardiac output, left heart pump failure, bradycardia --> cardiogenic shock --> CNS effects = dilated pupils, unresponsive mental status - survival of PE = poor --> death often 1 hr after onset of symptoms - can be confirmed with CT pulmonary angiography if time permits

A 44-year-old female presents to the emergency department with jaundice and diffuse abdominal pain. She denies any previous medical problems and says she does not take any medications, drugs or supplements. On physical examination, sclera are icteric and there is tenderness to palpation over the right upper quadrant. Liver function tests (LFTs) reveal the following: AST: 1237 IU/L ALT: 822 IU/L Albumin: 2.6 g/dL Total bilirubin: 5.1 mg/dL Direct bilirubin: 3.5 mg/dL Alkaline phosphatase: 118 IU/L Several hepatitis serologies are ordered with the following results: (Hepatitis A IgM: Negative) (Hepatitis A IgG: Negative) (Hepatitis B Surface antigen: Negative) (Hepatitis B Surface IgG: Negative) (Hepatitis B Core antigen: Negative) (Hepatitis B Core IgG: Positive) (Hepatitis B E antigen: Negative) (Hepatitis B E IgG: Negative) (Serum cryoglobulins: pending) (Anti-smooth muscle antibody: pending) Regarding the patient's hepatitis B serology, which of the following is the most appropriate next step in management:

*Measure hepatitis B core IgM* The patient's serology results are notable for a *positive hepatitis B core IgG (anti-HBc IgG)* despite a negative hepatitis B surface antigen (HBsAg) and hepatitis B surface IgG (anti-HBs). Such a profile classically puts the patient in the *'window period',* although other scenarios are possible. - *The window period in hepatitis B occurs when seroconversion takes place against HBsAg.* - Although there are Anti-HBs antibodies present, they are actively bound to the HBsAg. - *A positive anti-HBc IgM would help confirm that the patient is in the window period and would suggest that a resolving hepatitis B infection is responsible for her liver injury. It is therefore the most appropriate next step in the work-up for her hepatitis B serology.* - The patient should also receive a repeat HBV panel to rule out false positive results. possible interpretations of Anti-HBc-alone: (1) recovering from acute infection; (2) distant immunity—testing does not detect very low level of anti-HBs; (3) false-positive anti-HBc; or (4) carrier with an undetectable level of HBsAg in the serum.

65 yo female admitted to hospital with increasing SOB, wt gain & LE edema. She has ho HT, nonischemic cardiomyopathy with ejection fraction of 30% and hyperlipidemia. Her home meds include oral aspirin, digoxin, furosemide, metoprolol, lisinopril, and atorvastatin. She is started on IV furosemide. On day three of hospitalization telemetry reveals six beats of wide complex ventricular tachycardia. PE now shows decreased leg edema & clear lungs. Most appropriate next step in management of this pt's tachycardia?

*Measure serum electrolytes* Recurrent VT - first thing to do after stabilizing pt = search for underlying cause - *pt most likely has electrolyte imbalance via diuretics* Furosemide - commonly causes hypokalemia and hypomagnesemia --> can lead to ventricular tachycardia - hypokalemia potentiates side effects of digoxin = arrhythmias, such as ventricular tachycardia

36 yo woman comes to doc for routine health exam. She has no complaints or previous med probz & takes no meds. She doesn't use tobacco, alcohol, or illicit drugs. Her menstrual cycle and regular. Pt is architect at local construction firm. She has had no exposure to excessive radiation in pat yr. Her fam hx is negative for thyroid probz. Her temp is 98 F, BP is 130/80 mmHg, pulse is 80/min, and respirations are 16/min. Exam of neck shows 2x2 cm, discrete, nontender, firm, and mobile nodule in left thyroid lobe without cervical LAD. Remainder of exam within normal limits. Most appropriate initial step in eval of this pt?

*Measurement of thyroid-stimulating hormone and thyroid US* Thyroid nodules --> evaluate based on cancer risk factors - *TSH levels & US of nodule should be obtained first* - suspicious US findings (hypoechoic, microcalcifications, internal vascularity) or cancer risk factors --> fine-needle aspiration (FNA) - radionuclide scan indicated for pts with low TSH - hot nodules almost always benign --> can be treated for hyperthyroidism

26 yo woman comes to ED with severe SOB. SHe has long hx of asthma with periodic exacerbations. Her home meds include inhaled albuterol and fluticasone. Pt doesn't use tobacco, alcohol, or illicit drugs. Her temp is 99 F, BP is 150/90 mmHg, pulse is 110/min, and respirations are 24/min. On exam, she has moderate respiratory distress, prolonged expiratory phase, and diffuse wheezing. CBC shows hemoglobin of 13.8 g/dL & leukocyte count of 9,200/mm^3. Chest x-ray reveals hyperinflated lungs but no infiltrates. Pt admitted and is given nebulized albuterol, IV methylprednisone, and supplemental oxygen. Next day, her respiratory status is improved. Repeats vitals show temp of 99 F, BP is 130/80 mmHg, pulse is 90/min, & respirations are 20/min. Some scattered bilateral wheezes are heard on lung auscultation. Changes in lab results: Hgb 14 g/dL MCV 95 um^3 Leukocytes 16,000/mm^3 - Neutrophils 82% - Band forms 3% - Lymphocytes 13% - Eosinophils 0% - Basophils 0% - Monocytes 2% Most likely explanation for abnormal lab findings in this pt?

*Medication effect* Pt being treated for asthma exacerbation --> leukocytosis with neutrophilic predominance = common side effect of *systemic glucocorticoids* (ex. methylprednisolone) - decrease number of circulating lymphocytes and eosinophils via combination of increased apoptosis, increased emigration into tissues, decreased production

4 yo boy brought to ED by his parents due to fever, runny nose, cough, and unusual behavior. They describe him as increasingly restless over past 4 days. Since yesterday, he has been making odd comments about his dog being in room when dog is clearly not there. Parents have treated his cough and cold symptoms with OTC meds for last 36 hrs and humidified the air in his bedroom without benefit. Temp is 101 F, BP is 110/60 mmHg, pulse is 100/min, & respirations are 26/min. Exam shows slightly erythematous oropharynx with no exudate, clear nasal discharge, clear lung fields, and normal first and second heart sounds. Abdomen is soft and nontender. Neurologic exam within normal limits. During exam, pt starts talking to something in room, saying that he can see a "bunny" sitting on the bed with him and that "the animals have followed us to the hospital." Most likely explanation for this boy's visual hallucination?

*Medication side effect* Child has been receiving med for his cough and cold for past 36 hrs --> acute onset of hallucinations via med side effect: *OTC cold preparations* containing antihistamines (diphenhydramine, doxylamine) - decrease nasal discharge - have anticholinergic properties = *confusion and hallucinations* Alpha-adrenergic agents (phenylephrine, pseudoephedrine) - constrict blood vessels, decrease nasal congestion - agitation & psychosis via sympathomimetic properties Dextromethorphan - NMDA antagonist - dissociative symptoms, hallucinations

A 19-year-old college student is brought to the emergency room directly from his student health clinic. He has been experiencing fevers, chills, and neck stiffness for 24 hours. He is now photophobic and confused. In the emergency room, he is started on resuscitative IV fluids and undergoes a lumbar puncture. Immediately following, he is started on broad-spectrum antibiotics and admitted to the medical intensive care unit. Gram stain of the cerebrospinal fluid reveals gram negative diplococci. After receiving 3 L of normal saline, the patient's mean arterial pressure remains 55 mmHg and he is started on intravenous norepinephrine. On day 2 of the patient's hospital course you note diffuse, symmetric dusky discoloration of the finger tips. Which of the following is the likely cause of this new exam finding?

*Medication side effect* Meningococcal meningitis has received a vasopressor for blood pressure support in the setting of septic shock - *A known side effect of vasopressors is digital ischemia secondary to vasospasm* Norepinephrine - raises blood pressure primarily by increasing vascular tone through alpha-adrenergic receptor activation. It activates both alpha-1 and alpha-2 adrenergic receptors - will also increase cardiac output through increased chronotropy (increase in heart rate), though this is to a lesser extent than its direct effects on vascular tone - the standard first-line pressor for central administration = potent vasoconstrictor and alpha-1 agonist - binds alpha-1 receptors (which are Gq coupled leading to increased intracellular calcium) --> causes vasoconstriction --> in areas with a lesser blood supply, such as the finger tips, digital ischemia (dusky blue discoloration, or very pale) can occur = interesting and high yield clinical correlate when repairing lacerations...

A 38-year-old man presents to his primary care provider for abdominal pain. He reports that he has had a dull, burning pain for several months that has progressively gotten worse. He also notes a weight loss of about five pounds over that time frame. The patient endorses nausea and feels that the pain is worse after meals, but he denies any vomiting or diarrhea. He has a past medical history of hypertension, and he reports that he has been under an unusual amount of stress since losing his job as a construction worker. His home medications include enalapril and daily ibuprofen, which he takes for lower back pain he developed at his job. The patient drinks 1-2 beers with dinner and has a 25-pack-year smoking history. His family history is significant for colorectal cancer in his father and leukemia in his grandmother. On physical exam, the patient is moderately tender to palpation in the epigastrium. A fecal occult test is positive for blood in the stool. Which of the following in the patient's history is most likely causing this condition?

*Medication use* Abdominal pain worse after meals and evidence of occult GI bleeding consistent with a diagnosis of *gastric ulcer.* - *The most likely etiology is the patient's chronic NSAID (ibuprofen) use.* - Postprandial abdominal pain that is dull or burning in quality is classic for the dyspepsia of peptic ulcer disease (PUD). - Gastric ulcers tend to be exacerbated by the gastric acid released upon eating, while duodenal ulcers are relieved by the secretion of bicarbonate. - Most PUD is caused by either infection with Helicobacter pylori or NSAID use. - NSAIDs can decrease production of the protective mucous coating of the stomach via inhibition of COX and prostaglandin production.

19 yo college student comes to office prior to trip to Indian with concerns for travel-related infections. He has no prior med probz and is UTD with basic immunizations. PE normal. Pt advised to consume bottled water & avoid raw foods. Hep A and typhoid vaccinations and prescriptions for traveler's diarrhea provided. Most effective measure for preventing malaria in this pt?

*Mefloquine chemoprophylaxis until 4 wks after return* - should begin >2 wks prior to travel --> continued during stay --> discontinued 4 wks after returning - neuropsych side effects = anxiety, depression, restlessness in 5% of pts --> change to alternate med India - malaria endemic country - high prevalence of both Plasmodium falciparum & P vivax - *chloroquine resistance* common --> travelers usually given chemoprophylaxis with *atovaquone-proguanil, doxycycline, or mefloquine*

A 3-year-old girl is brought to her pediatrician by her mother, who is concerned that her daughter has developed puffy eyes and swelling in her feet. Physical examination confirms bilateral lower extremity edema to the mid-calf. The patient's medical history is significant for acute hepatitis B infection at the age of 9 months that was acquired through perinatal transmission. Laboratory studies are obtained and show a serum albumin level of 2.2 g/dL and 4+ protein and no blood on urinalysis. Hepatitis B serologies are also obtained and show positive HBsAg and positive anti-HBc IgM antibodies. Which of the following is the most likely diagnosis in this patient?

*Membranous glomerulonephritis* Nephrotic syndrome: Although minimal change disease is the most common overall cause of nephrotic syndrome in children, *the combination of chronic hepatitis B infection and nephrotic syndrome in this patient makes membranous glomerulonoephritis the most likely diagnosis.* - presents similar to other nephrotic syndromes, with generalized edema noted on examination as well as hypoalbuminemia and gross proteinuria - Causes can include infections (Hepatitis B & C, syphilis, malaria), occult malignancy, lupus, certain medications (gold, penicillamine) - most commonly treated with steroids and cyclophsophamide - associated with slow recovery and relatively low rates of persistent asymptomatic proteinuria. When hepatitis B is passed from mother to infant in the peripartum period, the child has a 90% chance of becoming a chronic carrier of the disease - If the mother is infected, infants should receive hepatitis B immune globulin and vaccination within 12 hours of birth - Additionally, all infants should be vaccinated against hepatitis B prior to being discharged from the hospital.

15 yo girl brought to ED due to 18 hrs of headache and lethargy. Headache began last nt after she returned from 3 days of hiking and camping with her friends. Pt had fever and vomiting overnt, and this morn her fam had difficulty waking her up. She has no significant med hx. Temp is 103.3 F, BP is 90/60 mmHg, pulse is 120/min, and respirations are 22/min. Pt obtunded and responds only to sleep, painful stimulus. PE shows resistance to passive neck flexion. Several petechiae on LEs. CSF results: Glucose 20 mg/dL Protein 475 mg/dL Leukocytes 2000/mm^3 - Neutrophils 90% Most likely cause of pt's condition?

*Meningococcal infection* Neisseria meningitidis - gram-negative diplococcus --> life-threatening meningitis, especially in young children and adolescents - 12% of community acquired bacterial meningitis, trailing S pneumoniae - sudden symptoms - rapid, severe progression - begins with fever, headache, vomiting, severe myalgias - 12-15 hrs later --> nuchal rigidity, altered mental status, petechial rash - shock shortly thereafter - mortality rate >15% CSF - low glucose (<45) - high protein (>500) - neutrophilic leukocytosis (>1000/mm^3) - treatment = ceftriaxone & vanc

58 yo man brought to ED after witnessed tonic-clonic seizure. Pt was at work when he suddenly collapsed & convulsed for appx a minute. His coworker says that the pt was confused immediately afterward; however, he is now awake & cooperative. He has been having headaches for past several wks and has never before had a seizure. He has no prior med probz and takes no meds. Pt is a former smoker with a 45-pck year hx. He has lived most of his life in Texas and has never traveled outside of the country. Temp is 98.1 F, BP is 122/70 mmHg, and pulse is 77/min. Cardiopulm auscultation is normal, & no masses are present on abdominal exam. CNs intact, & there is no muscle weakness or sensory loss. MRI of brain reveals several discrete, circumscribed lesions at junction of gray & white matter with surrounding edema. Rapid HIV testing negative. Most likely cause of this pt's seizure?

*Metastatic lung cancer* Tobacco use + multiple brain lesions = metastatic lung cancer - most common malignancy to spread to brain - neoplastic cells travel through vasculature --> lodge in small-caliber vessels at gray and white matter junction - multiple lesions typically form = mass effect from tumor growth & edema MRI of brain with contrast - multiple well-circumscribed lesions with vasogenic edema at gray and white matter junction

A 63-year-old woman with a history of poorly-controlled diabetes mellitus presents to your office to review labs and her blood glucose record. Her hemoglobin A1C is 9.4%. In addition, her blood glucose records demonstrate poor control with numerous spikes and lows throughout the day. Of note, it takes > 1 hr after meals for her blood glucose to rise. She is also complaining of a decreased appetite, nausea, reflux, and early satiety. What is the most appropriate treatment for her current symptoms?

*Metoclopramide* Diabetic gastroparesis - appropriate therapy is a prokinetic such as metoclopramide. - caused by autonomic neuropathy, which may occur with poorly controlled type 1 or type 2 diabetes mellitus - common symptoms include nausea, vomiting, abdominal pain, early satiety, and poor glucose control - diagnosis is made on the basis of a gastric emptying scan - *treatment includes a low fiber and low residue diet as well as prokinetic like metoclopramide* Metoclopramide - dopaminergic receptor antagonist - mixed 5-HT3 receptor agonist/antagonist - both anti-emetic and prokinetic properties - severe case of gastroparesis may require a jejunostomy tube *With regards to diabetic gastroparesis, the acute problem for this patient is the delayed gastric emptying and inability to have proper GI motility. Therefore, even though the underlying etiology of this problem is chronic nerve damage due to poorly controlled diabetes, the important issue to resolve is the gastroparesis. Insulin control would not resolve these GI symptoms and may even cause hypoglycemic episodes given the lack of nutrient delivery to the intestinal tract*

A 78-year-old man presents to the emergency room with a three day history of palpitations. He denies dizziness, dyspnea, and chest pain, and says he sleeps comfortably on one pillow each night. His history is significant for diabetes mellitus type 2, hyperlipidemia, and hypertension. Medications include glyburide, lisinopril, and hydrochlorothiazide. The patient has a ten pack-year history of smoking but quit 15 years ago. He does not drink alcohol or use drugs. His heart rate is 115/min and his blood pressure is 145/95 mmHg. Physical examination demonstrates no evidence of heart failure. Electrocardiogram shows atrial fibrillation with rapid ventricular response. Which of the following is an appropriate therapy for this patient:

*Metoprolol and warfarin* Medical treatment for atrial fibrillation in stable patients centers around rate control and anticoagulation. This patient's CHADS2 score is 3 (age, hypertension, and diabetes), so in addition to rate control with a non-dihydropyridine calcium channel blocker or beta-blocker, warfarin is indicated for anticoagulation. The score dictates whether a patient should be treated with aspirin or warfarin anticoagulation therapy. One point is assigned for each of the following: - congestive heart failure, blood pressure consistently above 140/90 mmHg, age greater than or equal to 75 years, and diabetes mellitus. Two points are added for prior stroke or TIA or thromboembolism. Patients with a score of zero can safely be treated with aspirin. If CHADS2 is two or greater, the patient is high risk for embolic complications and warfarin is indicated. *In general for boards exams the guidelines to know are a score of:* - *0 to 1: Aspirin* - *2+: Warfarin or novel anticoagulants*

You are performing the newborn exam of a male infant born following a pregnancy complicated by oligohydramnios secondary to a posterior urethral valve. Which of the following would you expect to find on physical exam?

*Midline mass* Posterior urethral valves can lead to urethral obstruction, resulting in an inability to pass urine and a distended bladder, which is appreciated on physical exam as a midline mass. - congenital defect in which an obstructing membrane develops in the male urethra - membrane results in obstruction of the urinary system, a distended bladder, and subsequent midline mass in newborn male children - can be associated with oligohydramnios as well as as lung hypoplasia - most common cause of bladder outlet obstruction in newborn males

A 63-year-old patient with a history of bipolar I disorder is admitted to an inpatient psychiatric ward after a suicide attempt. She reports that she has attempted suicide 7 times, usually overdosing on whatever pills she has "lying around the house." During which of the following mood states is a patient with bipolar I disorder most likely to attempt suicide?

*Mixed episode* Patients experiencing a mixed episode have the highest incidence of suicide attempts. - Among the different mood states, mixed episodes have the highest rate of concurrent suicide attempts. - It is thought that patients experiencing manic episodes do not have the desire to commit suicide, and patients experiencing depressive episodes do not have the volition to carry out an attempt. - *In contrast, patients experiencing a mixed episode may have both the desire to commit suicide and the volition to carry out a plan.*

65 yo woman comes to office due to mo of back pain. Pain is primarily in her lumbar and thoracic spine and is partially relieved with acetaminophen. It is worse with activity and has limited her ability to take restorative yoga classes. Pt has ho HT, T2DM, & osteopenia. She takes OTC calcium and vit D supplements in addition to prescription meds. Pt had her yearly PE 4 mos ago; physical and lab exams were normal at that time. Temp is 97.8 F, BP is 148/82 mmHg, and pulse is 94/min. Pt is thin but appears well. Mucous membranes moist. No LAD in cervical or supraclavicular chains. Cardiopulm and abdominal exams normal. No focal tenderness over spinel. Muscle strength 5/5 in all 4 extremities. Lab results: CBC - Hgb 10.2 g/dL - Platelets 220,000/mm^3 - Leukocytes 8,800/mm^3 Serum chemistry - Sodium 138 mEq/L - Potassium 4.2 mEq/L - Chloride 102 mEq/L - Bicarb 26 mEq/L - BUN 30 mg/dL - Creatinine 2.5 mg/dL - Calcium 10.9 mg/dL - Glucose 118 mg/dL Urine dipstick negative and urine sediment bland except for few granular casts. Most likely cause of this pt's current condition?

*Monoclonal protein* Multiple myeloma - plasma cell neoplasm - constitutional symptoms + bone pain - renal insufficiency + bland urinalysis that may show evidence of granular casts - monoclonal protein can damage glomeruli --> nephrotic syndrome

42 yo woman reports bloating with mild, diffuse abdominal discomfort 4 days after undergoing elective cholecystectomy. She has not passed gas since surgery. Perioperatively, she received abx, morphine for pain, and metoclopramide for nausea. Med hx significant for HT, DM, and hyperlipidemia. BP is 132/90 mmHg, and pulse is 76/min. BMI is 33 kg/m^2. PE shows distended, tympanic abdomen with decreased bowel sounds. There is mild, diffuse tenderness but no rebound or guarding. Remainder of exam shows no abnormalities. Most likely contributing to this pt's current condition?

*Morphine* Ileus - functional defect in bowel motility - N/V, abdominal distention, failure to pass flatus or stool (obstipation), hypoactive bowel sounds Some degree of ileus occurs following most abdominal procedures; however, *persistence of signs and symptoms (>3-5 days postoperatively) = prolonged/pathologic postoperative ileus (PPI)* Techniques to prevent PPI - epidural anesthesia - minimally invasive surgery - judicious perioperative use of IV fluids (to minimize GI edema)

60 yo man comes to doc complaining of fatigue & muscle weakness in his extremities. He has also lost 15 lb during last 3 mos. He has no other med conditions. Pt smokes 2 pcks of cigz daily and consumes alcohol occasionally. His muscle strength is 3/5 in proximal muscle groups symmetrically. His reflexes are 2+ bilaterally. No sensory abnormality present. There are erythematous to violaceous papules involving dorsum of fingers. Chest x-ray reveals ill-defined mass in right lower lobe. His muscle weakness is most likely due to lesion involving what?

*Muscle fibers* smoking hx + weight loss + right lower lobe mass = lung cancer - can develop *paraneoplastic syndrome* via tumor producing hormones and/or cytokines Paraneoplastic syndromes affecting peripheral nerve and/or muscle - MG, LE, dermatomyositis Dermatomyositis - symmetrical prox muscle weakness + erythematous rash on dorsum of fingers and upper eyelids

62 yo man undergoes partial pancreatectomy for exocrine pancreatic cancer. Surgery is complicated by excessive peripancreatic bleeding requiring careful surgical hemostasis. Pt receives packed RBC transfusion during sx as well as IV normal saline. He is transferred to surgical ICU for close monitoring and is successfully extubated. 12 hrs later, nurse reports decreased oxygen sat. Pt has received multiple doses of morphine for pain control since surgery. Temp is 99.5 F, BP is 80/40 mmHg, pulse is 112/min, and respirations are 28/min. Pt's pulse oximetry shows 87% on 4 L nasal cannula oxygen. Exam reveals bilateral basal crackles. Abdomen mildly distended & tender, and bowel sounds are decreased. No rebound tenderness or rigidity. Pulmonary artery catheter readings show cardiac index of 2 L/min/m^2 (normal, 2.8-4.2) ad pulmonary capillary wedge pressure of 20 mmHg (normal, 6-15). Most likely etiology of this pt's current condition?

*Myocardial infarction* Cardiogenic shock via acute MI: *Peroperative MI* - common in pts undergoing noncardiac injury - intraoperative hemorrhage requiring blood transfusion --> increases risk (likely via reduced oxygen deliver to myocardium) - often *lack chest pain* possibly via receipt of postoperative pain control (morphine) - left ventricular infarction --> impaired contractility & decrease in left ventricular stroke volume - Cardiac index (CI) = measure of cardiac output (SV x HR) adjusted per body surface area = low bc tachycardia cannot make up for decrease in stroke volume - increased pressure in left ventricle transmitted back to left atrium and lungs --> pulmonary capillary wedge pressure (PCWL) (estimated left atrial pressure) elevated - dyspnea & hypoxemia via pulmonary edema - lung exam = bilateral crackles - decreased CO --> hypotension --> peripheral vasoconstriction & increased systemic vascular resistance - low tissue perfusion --> tissues extract more oxygen from blood --> decreased mixed venous oxygen sat

42 yo man comes to office for follow-up. He has ho GERD, HT, and alcohol use disorder. Last yr, pt was hospitalized for acute gastritis. During hospitalization, he went into alcohol withdrawal that was treated with chlordiazepoxide. Pt was abstinent for 2 wks following discharge but then started drinking 2 or 3 beers several times a week. Over past several mos, he has been drinking 6-10 beers daily. Pt says, "I want to cut down, but the cravings are too strong." He fears he will lose his job and fam if he continues to drink. He has had hx of alcohol abuse since age 14 and has smoked 1.5 packs of cigz daily for past 12 yrs. Pt doesn't use illicit drugs. There is fam hx of alcohol abuse in his mom & heroin abuse in his bro. Vitals & PE are within normal limits. In addition to psychosocial interventions, what is the most appropriate pharmacotherapy for this pt's alcohol us disorder?

*Naltrexone* - decreases alcohol craving - reduces heavy drinking (>5 drinks in men; >4 in women) - increases days of abstinence - can be initiated while pt is still drinking - contraindications: pts taking opioids, acute hepatitis, liver failure Pt desires to reduce his alcohol intake but is unable to control his strong cravings & is drinking increasing amounts - first-line = naltrexone = mu-opioid receptor antagonist & acamprosate = glutamate modulator

59 yo woman comes to office due to upper resp symptoms that have been present for 3 days. She arrives an hour late for her appt and lies about reason, blaming it on emergency at work. She is angry that the doc refused to prescribed abx for her over the phone and paces at the reception desk, insisting that the office staff work her into the schedule because "I'm on the verge of closing a very imp't business deal and must be seen immediately." She mentions that her previous doc was "the best" as he always made time for her. During visit, pt expresses irritation at having to take time out of her busy day to come to the app't. She then proceeds to talk at length about how depressed she feels bc her adult children no longer want to see her. She is upset that her daughter recently canceled plans to visit in order to help friend who was in car accident. What is the best explanation for this pt's behavior?

*Narcissistic personality disorder* - pervasive grandiosity + need for admiration + sense of entitlement + lack of empathy - can be charming and successful - often arrogant and boastful - difficulty sustaining relationships bc unwilling to recognize or identify with feelings and needs of others - medical setting = demanding and feel entitled to special treatment

22 yo college student comes to student health service due to sleep probz for past 3 mos. Pt says, "School has become more stressful, and i have been feeling more fatigued since my gf broke off our relationship last mo." He is very sleepy during day and also feels embarrassed in front of other students as he often falls asleep during classes. Pt goes to bed at 1 AM and awakens at 8 AM. After classes, he avoids socializing and frequently returns straight to his dorm room to nap. Pt also describes "weird things happening when I fall asleep, like hearing whispering voices and seeing colorful animals." He describes another unusual experience last wk when he suddenly felt weak in the face and knees following a phone call with his ex-gf. Pt drinks beer socially but doesn't use illicit drugs. He has ho depression as teenager that responded to psychotherapy. Ht is 5 ft 7 in tall & wt is 185 lb; BMI is 29 kg/m^2. PE normal. Most likely diagnosis?

*Narcolepsy* - excessive daytime sleepiness - hallucinations while falling asleep - cataplexy = transient loss of muscle tone in response to intense emotion - intrusions of REM phenomena during sleep-wake transitions = hypnagogic (falling asleep) and hypnopompic (awakening) hallucinations and sleep paralysis (inability to move immediately after awakening) - associated with low CSF or orexin-A/hypocretin-1 - onset in early 20s/adolescence - diagnostic eval via polysomnography to rule out other sleep disorders & multiple sleep latency test = decreased sleep latency and sleep-onset REM periods

32 yo woman complains of "nagging" dry cough over last 8 wks. Cough present during day & awakens her at night. No associated SOB, chest pain, or wheezing. Pt's past med hx significant for chronic rhinorrhea and occasional itching skin rash. She takes no meds. Chest x-ray shows no abnormalities. One wk of treatment with chlorpheniramine significantly improves her symptoms. Decrease in what is most likely responsible for her symptoms relief?

*Nasal secretions* 3 most common causes of chronic cough (>8 wks) - upper-airway cough syndrome / postnasal drip - asthma - GERD Upper-airway cough syndrome - rhinosinus conditions: allergic, perennial nonallergic, vasomotor rhinitis; acute nasopharyngitis; sinusitis This pt: most likely via postnasal drip associated with allergic rhinitis - *diagnosis confirmed by improvement of nasal discharge and cough with use of first-gen antihistamines* Chlorpheniramine - specific H1 antihistaminic receptor blocker - decreases allergic response

A 57-year-old immigrant from Nigeria presents to the emergency department for sudden, severe pain and swelling in her lower extremity. She was at a rehabilitation hospital when her symptoms became apparent. The patient has a past medical history of obesity, diabetes, bipolar disorder, and tonic-clonic seizures. Her current medications include metformin, insulin, lisinopril, and valproic acid. The patient is a prominent IV drug and alcohol user who has presented to the ED many times for intoxication. On physical exam you note anasarca and asymmetric lower extremity swelling. Based on the results of a doppler ultrasound of her swollen lower extremity, heparin is started. The patient is then transferred to the general medicine floor for continued management. Laboratory studies are shown below. Serum: Na+: 137 mEq/L K+: 5.5 mEq/L Cl-: 100 mEq/L HCO3-: 24 mEq/L Urea nitrogen: 22 mg/dL Ca2+: 5.7 mg/dL Creatinine: 1.7 mg/dL Glucose: 70 mg/dL What is the most likely diagnosis?

*Nephrotic syndrome* Asymptomatic hypocalcemia and deep venous thrombosis (DVT) --> nephrotic syndrome. - when large amounts of protein are lost in the urine --> can lead to hyperlipidemia (due to loss of lipoproteins), hypercoagulable state (due to loss of antithrombin III and protein C/S), and hypoalbuminemia - most serum calcium is bound to albumin, so when albumin is lost in the urine, it can lead to profound hypocalcemia - however, the ionized or free portion of calcium remains the same; therefore, these patients do not exhibit symptoms of hypocalcemia. Nephrotic syndrome can present with a hypercoagulable state due to loss of antithrombin III and protein C/S, electrolyte abnormalities, and profound hypocalcemia from loss of albumin in the urine.

51 yo man complains of difficulty walking and mild right-sided foot pain for past several weeks. Pt's med hx significant for T1DM, HT, and hypercholesterolemia. PE shows significantly deformed right foot and mildly deformed left foot. X-rays of right foot suggest effusions in several of the tarsometatarsal joints, large osteophytes, and several extra-articular bone fragments. Most likely cause of this pt's complaints?

*Nerve damage* Charcot joint / neurogenic arthropathy - via diabetes - decreased proprioception, pain, temperature perception - can be via diabetes, peripheral nerve damage, syringomyelia, spinal cord injury, vit B12 def, tabes dorsalis - normal neurologic input lost --> pts unknowingly traumatize their wt-bearing joints = secondary degenerative joint dz, joint deformation, functional limitation - x-rays = loss of cartilage, osteophyte dev't, loose bodies - associated pain typically mild - management = treating underlying dz + mechanical devices (special shoes) to assist in wt bearing and decrease further trauma

58 yo man comes to doc with one-yr ho diarrhea. Stools are watery and accompanied by abdominal cramps. He has no fever, blood from rectum, or foul-smelling stools. He has also experienced frequent episodes of dizziness, flushing, wheezing, and feeling of warmth. Pt has taken herbal meds, which failed to relieve his symptoms. He is depressed about this illness and feels hopeless about diagnosis and treatment. He appears ill. Cardiac auscultation shows 2/6 systolic murmur over left lower sternal border that increases with inspiration. Abdominal exam shows hepatomegaly 3 cm below right costal margin and no abdominal tenderness. Lab results as follows: Hgb 14 g/dL MCV 90 gL Platelets 200,000/uL Leukocytes 6,100/uL Alk phos 400 U/L AST 101 U/L ALT 99 U/L Pt at risk of developing deficiency of what vitamins or minerals?

*Niacin* episodic flushing + wheezing + diarrhea + valvular heart dz with tricuspid regurgitation = carcinoid syndrome - slow-growing - most commonly in distal SI, proximal colon, lung - secrete histamine, serotonin, vasoactive intestinal peptide --> metabolized in liver - often metastasize to liver --> unable to metabolize = carcinoid syndrome - *serotonin synthesized in carcinoid cells from tryptophan = also used in production of niacin and nicotinic acid --> advanced dz = increased tryptophan conversion to serotonin and its metabolite 5-hydroxyindoleacetic acid --> may result in tryptophan and niacin deficiency --> pellagra = diarrhea, dermatitis, glossitis, angular stomatitis, dementia*

A women who is 20-weeks pregnant presents to your clinic with mild dysuria and increased urinary frequency. She is afebrile. Physical exam is negative for costovertebral angle tenderness. What is the best agent to treat this condition?

*Nitrofurantoin* Acute cystitis - *The drug of choice to treat cystitis in pregnancy is nitrofurantoin or first generation cephalosporins* - Urinary tract infections (UTIs) are common during pregnancy; the most common causative organism being Escherichia coli (over 90%) - *All pregnant women should be screened during prenatal visits for bacteriuria and if present, undergo treatment with nitrofurantoin or cephalexin.* - In pregnancy, asymptomatic bacteriuria can result in cystitis or ascend to pyelonephritis (ruled out in this case by negative costovertebral angle tenderness).

27 yo man comes to doc with complaints of cough, chest discomfort, and dyspnea on exertion. He has lost 10 lb over past 2 mos. Pt has 10 pck yr smoking hx. He drinks average of 2 beers each weekend. He does not use illegal drugs and has not had multiple sexual partners. PE unremarkable. Chest x-ray reveals large anterior mediastinal mass. Subsequent blood work shows elevated levels of beta-human chorionic gonadotropin and alpha fetoprotein. Most likely diagnosis?

*Nonseminomatous germ cell tumors* - Large anterior mediastinal mass + elevated beta-hCG + AFP - Primary mediastinal germ cell tumors occur mainly in young males = locally invasive *beta-hCG elevated in both seminomatous and nonseminomatous germ cell tumors, but nonseminomatous germ cell tumors typically also produce AFP* - diagnosis can be confirmed by biopsy - testicular US should be performed to exclude small primary tumor - almost all germ cell tumors in anterior mediastinum are primary rather than mestastatic

34 yo woman comes to doc with daily crampy abdominal pain for last 2 yrs. Pain sometimes occurs after meals but is not always preceded by eating. It is often accompanied by passage of small, loose stools & mucus, after which she feels significantly better. Pt reports that these symptoms interfere with her daily activities, including work. She has no fever, weight loss, or blood in her stools. She takes OTC meds for gas and constipation. Her mom died of colon cancer at age 65. Most likely diagnosis on further eval of this pt?

*Normal colonic mucosa* Irritable bowel syndrome - function disorder of GI tract - most common GI diagnosis in North America - 10-15% = prevalence - young women - chronic, crampy abdominal pain - nonspecific symptoms = reflux, dysphagia, early satiety, chest pain - symptoms of ROME III criteria? & no alarm features = don't require extensive workup for diagnosis

70 yo man brought to ED after his daughter found him lethargic and confused in his home. He was sick with a "cold" for the past 2 days. Pt lives alone & ambulates with walker after suffering stroke 5 yrs ago. His daughter lives nearby and helps him with his activities. On arrival, his temp is 100.4 F, BP is 90/65 mmHg, pulse is 112/min, and respirations are 24/min. Mucous membranes are dry. His airway is maintained, & oxygen is administered. IV access secured, & blood and urine samples drawn. Lab results: Sodium 134 mEq/L Potassium 5.9 mEq/L Chloride 101 mEq/L Bicarb 22 mEq/L BUN 110 mg/dL Glucose 1000 mg/dL Serum calcium 9.2 mg/dL Amylase 100 U/L AST 15 U/L ALT 17 U/L Arterial blood gases pH 7.40 PaCO2 38 mmHg PaO2 90 mmHg Best initial treatment for this pt?

*Normal saline* severe hyperglycemia + acute mental status changes = *HHS* - effective plasma osmolality = (2xplasma sodium [mEq/L]) + (plasma glucose [mg/dL] + 18) = severely elevated = >320 mOsm/kg - induces osmotic diuresis --> deficit of 8-10 liters in total body water --> *fluid replacement = most important initial step in management* - *normal saline is recommended for volume resuscitation in first few hours of treatment, regardless of sodium levels* --> subsequent change to 0.45% if corrected sodium (measured sodium in mEq/L + [2 mEq/L for each 100 mg/dL that glucose is above 100 mg/dL) is normal or high on repeat lab testing

his patient presents in hemorrhagic shock secondary to ruptured hepatic adenoma. What is an important risk factor associated with a hepatic adenoma?

*OCP usage* hepatic adenoma - benign tumor that can cause right upper quadrant pain - risk factors such as anabolic steroid use or OCP use can predispose individuals to hepatic adenomas - in some rare cases the adenoma can rupture, leading to hemorrhage - these patient's can have unstable vitals, tachycardia, and hypotension Based on the presentation of a woman with right upper quadrant pain, the best initial diagnosis to have in mind is that of acute cholecystitis, as this is a very common diagnosis. *However, in the face of unstable vitals, right upper quadrant pain and the CT scan, it is clear that this patient has a ruptured hepatic adenoma.* The key to this diagnosis is knowing anatomy on imaging, and understanding the risk factors for hepatic adenoma such as anabolic steroid use and OCP use.

22 yo woman comes to ED for eval of bite on her rt arm by her neighbor's dog. Neighbors accompany pt with their dog, which is not immunized against rabies but appears healthy. Pt has no other med conditions and takes no meds. Her last tetanus booster was 3 yrs ago. Temp is 98.6 F, BP is 110/70 mmHg, pulse is 76/min, and respirations are 12/min. On her right forearm is a shallow bite wound that is 1 cm long with no erythema or purulent drainage. Wound is cleaned with soap, water, and povidone-iodine solution. Most appropriate next step in management of this pt?

*Observe dog for 10 days for signs of rabies; no immediate prophylaxis for pt* Rabies - motor weakness, paresthesias, encephalitis --> progresses to coma & death - developing countries: dogs >90% - US: wild animals (bats, raccoons, foxes, skunks) Postexposure prophylaxis (PEP) - series of rabies immunizations + rabies immune globulin High-risk wild animals - PEP for rabies if animals unavailable for testing - animal available for testing --> PEP can be withheld until animal euthanized --> brain can be tested for rabies Domestic animals (pets) in US - don't require PEP if pet available for testing - incubation period for rabies = 1-3 mos - animals that are contagious (rabies in saliva) --> symptomatic 5-10 days after becoming contagious --> pets available for quarantine can be observed for 10 days for signs of rabies - if pet unavailable for quarantine (or symptomatic) --> PEP administered Low-risk animals (squirrels, chipmunks) --> don't need PEP

Involves pattern of preoccupation with orderliness, perfectionism, and control. Lack of true obsessions and compulsions. Ego-syntonic nature of symptoms.

*Obsessive-compulsive personality disorder (OCPD)* - to maintain sense of control, such individuals become so preoccupied with details and rigid rules that the major point of the activity is lost - their perfectionism often interferes with task completion as they perseverate & repeatedly check for possible mistakes

29 yo man comes to ED due to week of progressive bilateral LE weakness. He has no ho trauma or back pain. Pt had trigeminal neuralgia 3 mos ago & self-limited URI 2 wks ago. His temp is 98.6 F, BP is 122/76 mmHg, and pulse is 82/min. PE shows increased resistance to passive flexion and extension of lower limbs. Deep tendon reflexes are 3+ and plantar reflexes are upgoing bilaterally. There is decreased vibratory and positional sensation in his left upper extremity but no other sensory loss. Lumbar puncture performed. Most likely found on CSF in this pt?

*Oligoclonal bands* Multiple sclerosis - neuro deficits disseminated in space & time (trigeminal neuralgia, spastic lower limb paralysis, left upper limb sensory loss) - diagnosis suspected but clinical exam or MRI not classic --> do LP for CSF = oligoclonal bands: 85-95% - opening pressure, protein, cell count normal

A 44-year-old male is rushed to the emergency department after he began complaining of blurry vision in both eyes that developed over the last 45 minutes. He has also experienced excessive sleepiness, having fallen asleep mid-sentence repeatedly over the last several hours. The patient has a past history of alcoholism and reports consuming 1 quart of locally-produced moonshine 12 hours ago. Which of the following sets of signs, symptoms, and findings would be expected upon further evaluation of this patient?

*Optic disk hyperemia, anion gap metabolic acidosis, increased osmolar gap* Methanol poisoning - CNS depression, visual changes (blurry vision progressing to blindness), anion gap metabolic acidosis, and an elevated osmolar gap. - metabolized by alcohol dehydrogenase (ADH) to formaldehyde, which is then converted to formic acid by aldehyde dehydrogenase. Formic acid is toxic to the optic nerve, leading to visual changes and blindness with as little as 30 mL ingested. - Early formation of formic acid leads to metabolic acidosis; however, formic acid also binds to cytochrome oxidase and inhibits oxidative phosphorylation, leading to a later formation of lactic acidosis. Metabolized to oxalic acid -Forms calcium oxalate crystals in renal tubules -Renal failure (oliguria, anorexia, flank pain) -Anion-gap metabolic acidosis -Balooning and vacuolar degenration of PCT in kidney -Calcium oxalate stones are the most common type -Radioopaque

A 25-year-old obese woman with bipolar disorder and schizophrenia presents to the emergency room stating that she is pregnant. She says that she has been pregnant since she was 20 years old and is expecting a baby now that she is breathing much harder and feeling more faint. Her hospital medical record shows multiple negative pregnancy tests over the past five years, most recently 5 months ago. Her temperature is 98°F (37°C), blood pressure is 100/60 mmHg, pulse is 100/min, respirations are 28/min, and oxygen saturation is 85% on room air. Her fingerstick glucose is 100 mg/dL. She has a large abdominal pannus which is soft and nontender. Her legs are both edematous, but the left calf is tender to palpation. Oxygen is provided via nasal cannula. Psychiatry is consulted for possible pseudocyesis. Her urine pregnancy test comes back positive. What is the next best step in diagnosis?

*Order ventilation perfusion scintigraphy of the chest* There is reasonable suspicion that this tachypneic, hypoxemic patient is pregnant with need for evaluation for pulmonary embolism. Ventilation-perfusion scintigraphy is warranted. Pregnant women - three times more likely to develop venous thromboembolism compared to non-pregnant women. Deep vein thrombosis during pregnancy - occurs more commonly on the left side of the body (85%) secondary to compression of the left common iliac vein by the right common iliac artery as well as compression by the gravid uterus.

27 yo woman comes to office for follow-up. She was last seen 2 mos ago for generalized musculoskeletal pain & several mos of fatigue. Eval revealed multiple soft-tissue tender point, negative inflammatory and serological markers, and normal thyroid nodules. Pt was diagnosed with fibromyalgia and prescribed low-impact exercise program & pharmacotherapy. Her pain & fatigue have subsequently improved, but she now notes intermittent episodes of dizziness. Most episodes short-lived, but she occasionally needs to brace herself against wall or sit until dizziness resolves. There have been no associated falls or loss of consciousness. She also notes worsening lethargy and frequent dry mouth. Other mex hx includes migraines and IBS. Pt currently takes daily amitriptyline for fibromyalgia, occasional acetaminophen for muscular pains, and polyethylene glycol as needed for constipation. She doesn't use tobacco, alcohol, or illicit drugs and has no known drug allergies. Temp is 98.2 F, BP is 110/70 mmHg, pulse is 70/min, & respirations are 14/min. BMI 21 kg/m^2. Mucous membranes pink & moist & there is no JVD. Cardiopulm auscultation normal. Abdomen soft & nontender. Neuro exam reveals intact cranial nerves & normal muscle strength in all extremities. What would be most helpful in determining cause of this pt's dizziness?

*Orthostatic BP measurements* - common even at low therapeutic levels - subtle = dizziness, lightheadedness with sudden postural change - ho bracing themselves against wall or sitting until episode resolves - systolic BP decline >20 mmHg or diastolic BP decline >10 mmHg Fibromyalgia treated with amitriptyline - inhibits reuptake of NE & serotonin Side effects that often limit treatment & result in med discontinuation - M1 receptors --> anticholinergic symptoms - Histamine receptors --> lethargy - Alpha adrenergic receptors --> orthostatic hypotension

47 yo man brought to ED following single-car motor vehicle accident. His physical injuries are relatively minor but he appears intoxicated. Pt has known ho polysubstance abuse. Temp is 97 F, BP is 116/79 mmHg, pulse is 56/min, & respirations are 9/min. Pt is sedated and slow to respond to questions. Abdomen soft & nontender, and bowel sounds decreased. Extraocular movements intact and pupils 1 mm. Physician suspects intoxication and orders standard urine drugs screen, but screen is negative. Pt most likely using...

*Oxycodone* sedation + mild resp depression + decreased bowel sounds + miosis = *opioid intoxication* UDS - standardly, performed by immunoassay --> measures morphine = breakdown product of all natural opioids (heroin, codeine) *Semisynthetic (OXYCODONE, hydrocodone, hydromorphone) and synthetic (fentanyl, meperidine, methadone, tramadol) opioids don't trigger positive result on standard UDS)*

Mrs. Jones is a 56-year-old female that is admitted to a general medicine floor of the local university hospital. She is noted to have new onset ascites of unknown origin. Peritoneal fluid was sent for cell count (<50 leukocytes), albumin level (4.8), culture (pending), total protein (2.0), and Gram stain (pending). Serum albumin was noted to be 4.0. Of the options below, what is the most likely cause of this patient's ascites?

*Pancreatitis* *Serum albumin ascites gradient (SAAG) of -0.8 g/dL which puts it into the category of non-portal hypertensive ascites* In patients with new-onset ascites of unknown origin, peritoneal fluid should be sent for evaluation. - *The SAAG is a great test for differentiating ascites of portal hypertensive causes (SAAG >1.1 g/dL) from non-portal hypertensive (SAAG < 1.1 g/dL) causes* - Portal hypertensive causes of ascites include cirrhosis, alcoholic hepatitis, CHF, large hepatic metastases, vascular occlusions, fatty liver disease of pregnancy, and myxedema - *Non-portal hypertensive causes of ascites include peritoneal carcinomatosis, peritoneal TB, pancreatitis, serositis, nephrotic syndrome, and bowel obstruction*

A 21-year-old female college student presents to the clinic complaining of intermittent diarrhea, abdominal pain, and bloating. She has had a feeling of general weakness and fatigue over the last month. She states that her stool has recently seemed to be "grease coated". She does not have any significant past medical history. Vital signs are T 98.9 F, HR 103, BP 110/75, RR 16, and SpO2 99%. Laboratory tests show an iron deficiency anemia. On physical exam, what skin condition is most likely associated with her presentation?

*Papulovesicular eruptions, usually distributed symmetrically on extensor surfaces* Celiac disease - *Dermatitis herpetiformis is a chronic blistering, pruritic, papulovesicular skin condition commonly associated with celiac disease* - autoimmune disease of the small intestine caused by an intolerance to gluten, which can present with nonspecific symptoms such as general weakness/fatigue, iron-deficiency anemia, joint pains, muscle cramps, and irritability - Dermatitis herpetiformis is intensely pruritic and chronic, characterized by papulovesicles and urticarial wheals on the extensor surfaces in a grouped or herpetiform, symmetric distribution. *Illustration A shows dermatitis herpetiformis on the arms; these papulovesicular eruptions are commonly seen on extensor surfaces*

A 30-year-old woman presents to the emergency department claiming that she is on a divine mission to rid the world of overweight children by spreading the good word of vegetables. The patient is energetic and exhibits pressured speech and is easily distracted when being questioned. Upon obtaining further history she states that she has not slept for the past week as she has been working non-stop and has spent over $10,000 on kale. Upon reviewing the patients record, it is notable that she was admitted in the past for a suicide attempt and episode of major depression. The patient is treated appropriately and discharged on new medications with instructions to follow up in 1 week. The patient returns 1 week later complaining of increased urinary frequency. She states she has been using the bathroom constantly and it is quite perturbing. Otherwise she states that she has been sleeping well and was able to get a job at a local farmer's market. Whilst telling you this story you note that she is drinking large volumes of water from a personal gallon. She complains of occasional headaches but is overall grateful for your care. Which of the following is the next step in management for this patient?

*Perform a water deprivation test* Nephrogenic diabetes insipidus (DI) - intense thirst and frequent urination, is a well-known side effect of lithium which is used to treat this patient's episode of bipolar I Lithium inhibits ADH (vasopressin) receptors in the collecting ducts and aquaporin insertion - *Prior to making a diagnosis of lithium induced nephrogenic DI, the workup must be commenced with a water deprivation test* - Daily urine volume should be monitored annually in this group - Treatment involves discontinuation of lithium and supportive care, though DI may not fully resolve *suspect DI -> water deprivation test -> psychogenic or nephrogenic*

4th year med student working in ICU helping to care for 75 yo man with end-stage lung cancer. Pt suffers cardiac arrest and, despite intensive resuscitation and emergency management, cannot recover and is declared dead 15 minutes later. Team members provide emotional support to his fam. Supervising doc reviews details of the pt's management with the student. Doc is aware of the student's interest in critical care and asks if she would like to practice performing procedures, including pericardiocentesis and intubation, on the deceased pt. Most appropriate action by the student?

*Perform the procedures only if permission of fam is obtained?* - ethical for students to perform procedures for training purposes as long as doc obtains permission from fam (or pt prior to death) before student performs procedures - training must occur as part of structured training sequence and be performed under close supervision

25 yo AAF comes to doc with recent onset of diffuse joint pain and rash on her face. She is not taking any meds. Pt found to have low-range proteinuria and abnormal urinary sediment. Her creatinine is 1.3 mg/dL. Renal biopsy findings consistent with focal proliferative glomerulonephritis. Her CBC results are: Hgb 10.8 g/dL Platelets 60,000/uL Leukocyte 2,500/uL Most likely cause of this thrombocytopenia in this pt?

*Peripheral destruction* SLE - focal proliferative glomerulonephritis = subset of class III SLE nephritis - this pt = pancytopenia - can cause pancytopenia via concurrent peripheral immune-mediated destruction of all 3 cells lines

38 yo man who recently migrated from Eastern Europe comes to doc with 2 mo ho exertional SOB and easy fatigability. He has been taking isoniazid for latent TB for last 2 mos. He also became vegan 5 mos ago. PE shows severe conjunctival pallor & areas of depigmentation on his arms suggestive of vitiligo. His tongue appears shiny. Spleen is not palpable. Lab results: Hgb 6.5 g/dL MCV 110 fL WBCs 4,100/mm^3 Platelets 135,000/mm^3 Most likely diagnosis?

*Pernicious anemia* - easy fatigability & SOB - likely secondary to underlying macrocytic or megaloblastic anemia - *most common cause of vit B12 def in whites of northern European background* - frequently have other associated autoimmune dzes, including autoimmune thyroid dz & *vitiligo* - PE = *shiny tongue* via atrophic position & vibration sense - peripheral blood smear = macroovalocytes, megaloblasts, hypersegmented neutrophils - B12 def in PA is due to presence of autoAbs against gastric intrinsic factor

80 yo woman with multiple med probz is admitted to hospital for evaluation of syncope. Her heart rate is found to be 36/min. She was admitted to another hospital recently, then discharged to acute rehab facility, and then sent to nursing home. Review of her history shows that pt is taking high-dose diltiazem, metoprolol, and atenolol. What med reconciliation interventions is most likely to result in decrease in adverse drug events and health care utilization?

*Pharmacist-direct interventions* Transitions of care - interventions that target pharmacy personnel and high-risk pts = most effective in improving quality of pt care

36 yo man comes to office with 6 mo ho fatigue, mild headache, and decreased libido. His past med hx is notable for T2DM and bipolar disorder. His meds include metformin and risperidone. Pt smokes pck of cigz a day and drinks 2 or 3 cans of beer daily. BP is 126/73 mmHg and pulse is 86/min. His BMI is 31 kg/m^2. PE shows decreased testicular volume but is otherwise normal. Lab results: LH 0.5 mIU/mL Testosterone, total 100 ng/dL (normal: 240 - 950 ng/dL) TSH 0.05 uU/mL Thyroxine (T4), free 0.45 ng/dL (normal: 0.9 - 1.7 ng/dL) Prolactin 35 ng/mL Most likely diagnosis?

*Pituitary adenoma* Hypopituitarism: Central hypogonadism - low LH & testosterone Central hypothyroidism - low TSH & thyroxine Mildly elevated prolactin - via anatomic disruption of dopaminergic neural pathways that normally suppress prolactin secretion *Nonfunctioning pituitary adenoma* - via gonadotropin-secreting cells (gonadotrophs) in pituitary glands - dysfunctional cells of gonadotroph adenomas --> secrete common *alpha-subunit*

A 47-year-old man presents to your outpatient clinic for a routine checkup. His only medical problem is mild hypertension, for which he takes hydrochlorothiazide. He works as an accountant and lives with his girlfriend; they are mutually monogamous. He drinks a glass of Scotch each evening at dinner, smokes one pack of cigarettes daily, and denies ever using street drugs. Review of his medical records shows that he received a tetanus-diptheria (Td) booster 6 years ago, and the influenza vaccine 3 months previously at a sick visit for an upper respiratory infection. He also received 2 doses each of the measles-mumps-rubella (MMR) previously and had chickenpox as a child. Which of the following vaccines is indicated at this visit?

*Pneumococcal (polysaccharide)* - indicated in this patient because he smokes cigarettes - typically administered to adults aged 65 and older. *However, smokers, residents of nursing homes or other long-term care facilities, and patients with a number of chronic medical conditions should be given the pneumococcal vaccine even if they have not yet reached age 65*; they then require a second dose after the age of 65, with at least 5 years between doses. Typically, only one dose is necessary after the age of 65, but for those individuals, who may be predisposed to infection because of a weakened immune system, a second dose after five years of receiving the first dose is recommended. For those age 19-64, a booster 5 years after the first dose may be recommended if renal failure, abnormal immune system or functional/anatomical asplenia.

A 60-year-old man with a history of chronic back pain poorly controlled by high-dose over-the-counter pain reliever use presents to the emergency department with abdominal pain of 2 hours duration. On physical exam, he is febrile and tachycardic and lying still on his back moaning in pain. He refuses to let you palpate his abdomen. Which of the following would you be most likely to observe on abdominal radiograph?

*Pneumoperitoneum* Heavy over-the-counter pain relievers (likely non-steroidal anti-inflammatory drugs) and peritoneal signs on physical exam are consistent with a *perforated peptic ulcer.* This results in *pneumoperitoneum,* often seen on abdominal or chest x-ray. Perforated peptic ulcer - most common cause of bowel perforation - *If there is high clinical suspicion, an abdominal or chest radiograph should be ordered to look for pneumoperitoneum, which is free air in the abdomen.* - If present, diagnosis is confirmed and the patient should be taken to the operating room emergently. - Emergent exploratory laparotomy with surgical repair is indicated for all patients with bowel perforation. - Note that the patient's fever, tachycardia, and guarding are all signs of peritonitis secondary to the perforation. - bleeding is the most common indication for surgery in PUD, although proton pump inhibitors (PPIs) and endoscopic therapy can control most bleeds. - Perforation and gastric outlet obstruction are two rare but serious complications. *Illustration shows pneumoperitoneum on abdominal radiograph. Note the presence of air (black) under the diaphragm.*

A seven-year-old girl presents to general pediatrics condition with periorbital swelling. She has never had symptoms like this before. She has no significant past medical history. Her vital signs are stable. Her physical examination is notable for 2+ pitting edema of the bilateral lower extremities. The remainder of her physical examination is within normal limits. She has no costovertebral angle tenderness. A dipstick urinalysis is significant for 3+ proteinuria. On renal biopsy and on microscopic urinalysis, what would one likely expect to find in this patient?

*Podocyte effacement and fatty casts* The most common cause of nephrotic syndrome in children is *minimal change disease*. - *Light microscopy and immunoflouresence reveal no changes in kidney architecture, but podocyte effacement can be seen on electron microscopy.* - *Fatty casts may be seen in the urine.* - Treatment with steroids is initiated often even without a biopsy. - the most common form of nephrotic syndrome in children - present with generalized edema due to a loss of oncotic pressure from large albumin loss - laboratory criteria include hypoalbuminemia and proteinuria > 3.5 g/day as well as hyperlipidemia and lipiduria - the disease may be triggered by an infection and responds very well to corticosteroids - on light microscopy, glomeruli appear normal, but on electron microscopy, one notes effacement of the foot processes. *Image A depicts periorbital edema seen in minimal change disease*

18 yo AAM comes to office with 3 day ho progressively worsening fatigue and exertional dyspnea. 2 wks ago, he was diagnosed with upper respiratory infections that was treated with amoxicillin. Pt has no known med probz and does not take any meds. PE shows soft & nontender abdomen. Spleen enlarged. Mild scleral icterus is present Remainder of PE unremarkable. Lab test results: Hgb 7.8 g/dL MCV 90 um^3 Mean corpuscular hemoglobin concentration 32 g/dL Reticulocytes 10% Platelets 176,000/mm^3 Leukocytes 7,800/mm^3 Most likely to be seen in this pt?

*Positive direct Coombs test* Autoimmune hemolytic anemia - fatigue & dyspnea following resp infection treated with amoxicillin - splenomegaly & anemia with reticulocytosis Warm agglutinin autoimmune hemolytic anemia (AIHA) - possible via antibiotic therapy - etiologies = lymphoproliferative disorders (CLL), viral infections (HIV), autoimmune disorders (SLE), drugs (penicillin) - diagnosis via direct antiglobulin (Coombs) test = autoAbs (anti-IgG) or complement components (anti-C3) - treatment = high-dose steroids (prednisone) --> decreases autoAbs production

60 yo woman comes to doc with several mos of LE swelling. Her past med hx is significant for HT, T2DM, & hep C infection She was treated for TB 10 yrs ago. Her BP is 120/80 mmHg and pulse is 90/min. PE shows symmetric pitting edema of LEs. Liver palpated 4 cm below costal margin, and ascites is present. Tip of spleen palpated on deep inspiration. Abdominal pressure shows persistent distention of jugular veins (positive hepatojugular reflux). Lungs clear on auscultation. What findings is most suggestive of cardiac cause for this pt's edema?

*Positive hepatojugular reflux* Hepatojugular/Abdominojugular reflux - elicited by applying firm & sustained pressure for 10-15 seconds over upper abdomen - positive response = sustained elevation of JVP >3 cm during continued abdominal compression - not specific to particular disorder but rather a reflection of a failing right ventricle that cannot accommodate increase in venous return with abdominal compression - constrictive pericarditis, right ventricular infarction, restrictive cardiomyopathy = most common causes of positive hepatojugular reflux - this pt's ho TB = suggestive of constrictive pericarditis Constrictive pericarditis - major cause of right heart failure --> elevated jugular venous pressure, peripheral edema, ascites, hepatic congestion with hepatomegaly, eventual progression to cirrhosis (cardiac cirrhosis) - PE for constrictive pericarditis = elevated JVP with hepatojugular reflux, Kussmaul's sign (lack of decrease or increase in JVP on inspiration), pericardial knock, pericardial calcifications on chest radiograph

A 16-year-old female presents to her orthopedic surgeon complaining of foot pain. She states that yesterday, while at cheerleading practice, she was dropped after being thrown in the air. She states that she landed on her right foot and she immediately had pain. Today, she has noticed significant swelling of her right foot behind her ankle. On physical examination, she has excruciating posterolateral pain in her right ankle with plantar flexion and flexion of her great toe. She also has decreased range of motion in her subtalar joint. Ankle radiographs are shown in Figure A (flip for image). What is the most likely diagnosis?

*Posterior lateral talar tubercle fractures* - seen in athletes who experience axial loading after landing hard following a jump or fall - often confused with an ankle sprain and the diagnosis is often very difficult - Active flexion of great toe reproduces pain as the flexor hallucis longus (FHL) moves over the fracture site - If the fracture is displaced, open reduction and internal fixation (ORIF) is the best treatment option - If the fracture is completely non-displaced, then immobilization and non-weightbearing is the preferred treatment. - can present with tenderness to palpation anterior to the Achilles tendon over the posterolateral talus - Plantar flexion and dorsiflexion of the great toe typically make the pain worse since it causes compression of the fracture fragment by the flexor hallucis longus tendon as it passes between the medial and lateral tubercles *Figure is a sagittal CT scan depicting a displaced posterior lateral talar bone fragment*

62 yo woman comes to office for follow-up of diabetes treatment. She has had T2DM for last 10 yrs and is managed with oral metformin twice daily and insulin glargine at bedtime. Pt has no chest pain, dizziness, sweating, polyuria, or palpitations. She checks her blood glucose level every morning and has had values of 75-132 mg/dL over last mo. Med hx notable for HT and mild intermittent asthma, for which she takes lisinopril, amlodipine, and albuterol as needed. BP is 128/72 mmHg and pulse is 79/min. BMI is 41 kg/m^2. Lab results show random glucose level of 298 mg/dL & hemoglobin A1c of 8.1%. What best explains suboptimal glycemic control in this pt?

*Postprandial hyperglycemia* This pt... - morning/fasting glucose readings = within her target range (80-130 mg/dL is suggested target) - her random glucose level (298 mg/dL) is very high - hgb A1c is above goal (target <7% in most pts) - *both fasting/premeal and postprandial glucose levels contribute to hgb A1c* --> likely this pt is experiencing frequent postprandial hyperglycemia = impaired overall glycemic control Goal: control postprandial glucose excursions without causing fasting hypoglycemia --> combined regimen = long-acting basal insulin (insulin glargine) to control baseline and rapid-acting mealtime insulin (insulin aspart) to control postprandial glucose excursions

A 29-year-old veteran of the Iraq War presents to the behavioural health department for help controlling his anger. He reports that he can "go from zero to sixty in a few seconds," and he often finds himself getting angry at strangers at the grocery store, on the road, and at restaurants. He also notes frequent flashbacks to a particular battle, in which his tank was ambushed and several of his comrades were killed, and nightmares that wake him up 3-4 nights per week. Which of the following medication combinations is the most appropriate initial therapy?

*Prazosin and fluoxetine* PTSD - can be treated with an alpha receptor antagonist, such as prazosin, and a selective serotonin uptake inhibitor (SSRI), such as fluoxetine - diagnosed based on the presence of avoidance, re-experiencing, and hyperarousal after a trauma - symptoms typically appear within six months of the trauma, however delayed-onset PTSD can occur after this time period. *Typical first-line treatment includes an SSRI for mood symptoms, as well as prazosin (an alpha-blocker) for treatment of nightmares. Talk-therapy is also known to aid in treatment.*

A 16-year-old woman presents to her primary care physician complaining of "staring spells." For the past month, she has experienced episodes in which she suddenly halts activity and is unresponsive to auditory or tactile stimulation. These episodes occur approximately 5 times per day and last approximately 10 seconds. She has no significant past medical history and takes no medications except for estrogen-progesterone combined oral contraceptives. Family history is significant only for pelvic inflammatory disease in her mother. Menarche occurred at age 13. She is sexually active with 1 male partner. Use of ethosuximide to treat the patient's symptoms increases the risk of which of the following?

*Pregnancy* Absence seizures: Ethosuximide - *CYP450 inducer that decreases the effectiveness of oral contraceptive pills, increasing the risk of pregnancy* Oral contraceptive pills (OCPs) - metabolized via the CYP450 pathway in the liver and are susceptible to the effects of medications that induce or inhibit that pathway CYP450 inducers include: barbiturates, griseofulvin, St. John's wort, phenytoin, carbamazepine, rifampin, ethosuximide, and topiramate - increase the rate of metabolism of OCPs, decreasing their efficacy and increasing the risk of pregnancy

45 yo man comes to ED complaining of dyspnea, fatigue, poor appetite, & weight gain over past 2 wks. His symptoms started with worsening SOB with exertion. He has been waking up at night breathless for past few days. He also finds it difficult to open his eyes in morn due to facial edema. Pt has no other med probz and takes no meds. His BP is 200/120 mmHg & pulse is 100/min. PE shows anasarca with distended jugular veins while sitting upright. Lung auscultation shows bibasilar crackles. Urinalysis shows 1+ protein, no nitrite, trace leukocyte esterase, >50 RBCs, RBC casts, and occasional neutrophils. Most likely mechanism of this pt's edema?

*Primary glomerular drainage* anasarca + pulmonary & facial edema + HT + abnormal urinalysis with proteinuria & microscopic hematuria = *acute nephritic syndrome with fluid overload* - primary glomerular damage --> decreased glomerular filtration rate + eventual dev't of significant volume overload (pulmonary edema, distended neck veins, anasarca) - abnormal urinary sediment (RBCs, RBC casts) & variable degrees of proteinuria = present on urinalysis - serum creatinine can be elevated - increased volume --> HT - significant proteinuria (3+) --> hypoalbuminemia --> contributes to edema

55 yo man comes to clinic for follow-up of HT. Pt was diagnosed with HT 3 mos ago after episode of transient vision loss in right eye. He has occasional headaches and general fatigue but otherwise feels well. His current meds include aspirin, lisinopril, and low-dose HCTZ. His BP is 157/95 mmhg and pulse is 69/min. BMI 26 kg/m^2. Cardiac exam shows fourth heart sound on auscultation. No bruits in carotids or abdomen. Pulses 2+ in all extremities. Lab results: Potassium 3.1 mEq/L Creatinine 0.9 mg/dL Plasma renin activity undetectable What best explains this pt's lab findings?

*Primary hyperaldosteronism* - HT = undetectable plasma renin activity - most common cause of secondary HT - usually via adrenal adenoma or bilateral adrenal hyperplasia - HT + mild hypernatremia + hypokalemia + metabolic alkalosis - plasma renin activity low due to feedback inhibition - most pts don't have edema or clinically significant hypernatremia via *aldosterone escape*

33 yo woman brought to ED after episode of generalized tonic-clonic seizures. She is confused and unable to provide further hx. Review of med record indicates that she has schizophrenia but stopped taking all her psychiatric meds 3 wks ago. Temp is 98 F, BP is 130/76 mmHg, pulse is 80/min, & respirations are 14/min. Exam shows normal pupils. Pt's lungs clear to auscultation and her heart sounds normal. Her abdomen is soft. Extremities exam shows no edema. Lab results: Serum sodium 118 mEq/L Serum potassium 3.4 mEq/L Serum creatinine 0.8 mg/dL Serum calcium 8.4 mg/dL Serum glucose 98 mg/dL Serum osmolality 252 mOsm/kg Urine osmolality 78 mOsm/kg Urine specific gravity 1.002 Most likely cause of this pt's electrolyte abnormalities?

*Primary polydipsia* - most common in pts with psychiatric conditions (schizophrenia), possibly via central defect in thirst regulation --> continue to drink water despite decreased serum osmolality that should normally inhibit thirst reflex - seizures via hyponatremia Serum osmolality <290 mOsm/kg --> Urine osmolality <100 mOsm/kg = primary polydipsia or malnutrition (beer potomania)

A 58-year-old female presents to her family physician for an annual check-up. During the visit, the patient asks her physician for help quitting smoking cigarettes. She has unsuccessfully tried going 'cold-turkey' several times previously and has also failed prior attempts with nicotine replacement therapy. The physician prescribes an agent that acts as a partial agonist of the nicotinic receptor. Which of the following would be a contraindication to initiating this medication in this patient?

*Prior suicide attempt* Varenicline - nicotinic receptor partial agonist, smoking cessation agent that should be avoided in patients with unstable psychiatric disease or suicidal ideation, as this medication may worsen these symptoms. - should be initiated 1 week prior to the patient's anticipated quit date and continued for 4 months after. - Nicotine replacement therapy can increase the efficacy of varenicline. - Side effects of the medication can include headache, nausea, and insomnia.

34 yo man comes to ED with palpitations for past 4 hours. He has no associated chest pain, SOB, or dizziness. His med hx is significant for Wolff-Parkinson-White syndrome with 3 prior episodes of SVT. Pt doesn't use tobacco or illicit drugs and drinks alcohol only on social occasions. He had 5 cans of beer at party the previous night. On exam, his BP is 120/80 mmHg and pulse is irregularly irregular. ECG shows atrial fibrillation with rate of 160/min. Best next step in management of this pt?

*Procainamide* Wolff-Parkinson-White syndrome - A fib = 10-30% of pts = potentially life-threatening --> can bypass usual rate-limiting function of AV node = rapid ventricular response rates --> can deteriorate into ventricular fibrillation Acute treatment of AF in pts with WPW --> aimed at prompt control of ventricular response & termination of AF - hemodynamically unstable pts = immediate electrical cardioversion - stable pts = *rhythm control with anti-arrhythmic drugs such as IV ibutilide or procainamide*

34 yo woman comes to office with uncomfortable sensation of rapid heartbeat for last 2 wks. She has had no associated chest pain or SOB. Pt reports that her anxiety level has been high recently due to stress at work, and she has lost close to 4.4 lb during this time. Her LMP was 5 days ago. Med hx unremarkable, and she takes no meds. On exam, thyroid gland diffusely enlarged & nontender. Lab results as follows: CBC - Hgb 12.9 g/dL - Hct 39% - Platelets 200,000/mm^3 - Leukocytes 8500/mm^3 Serum chemistries - Sodium 139 mEq/L - Potassium 4.2 mEq/L - Chloride 100 mEq/L - Bicarb 25 mEq/L - Urea nitrogen 10 mg/dL - Creatinine 9.1 mg/dL - Glucose 102 mg/dL TSH <0.001 uU/mL Free T4 4.6 ng/dL (0.9-2.4 ng/dL) ECG shows sinus tachycardia at 120/min. Radioactive iodine uptake is <5%, and anti-thyroid peroxidase Ab are present in high titers. Most appropriate next step in management of this pt?

*Propranolol* Primary hyperthyroidism - elevated T4 + suppressed TSH - signs of thyrotoxicosis, anxiety, wt loss - via overproduction of thyroid hormone (Graves, toxic nodular goiter) or release of preformed hormone (painless thyroiditis, subacute thyroiditis) painless thyroiditis + thyroid peroxidase autoAbs = similar to chronic lymphocytic (Hashimoto) thyroiditis - *painless thyroiditis doesn't require specific therapy, but as hyperthyroidism causes adrenergic overstimulation, beta blocker may be used to control symptoms*

39 yo woman comes to ED due to 3 days of fever, dysuria, and left flank pain. Pt has ho multiple UTIs and had a ureteropelvic junction calculus removed 2 yrs ago. Last mo, she was treated for vaginal candidiasis. Pt doesn't use tobacco, alcohol, or illicit drugs. Her mom has thyroid cancer. Temp is 100.9 F, BP is 120/70 mmHg, and pulse is 110/min. Left costovertebral angle tenderness present. Lab results: Leukocytes 13,000/mm^3 Urinalysis - pH 8.5 - Blood positive - Leukocyte esterase positive - Bacteria many - WBCs 100+/mm^3 Imaging reveals enlarged left kidney with perinephric fat stranding and nonobstructive renal calculus. What organism is most likely to be seen in this pt's urine culture?

*Proteus mirabilis* - urinary alkalization (pH >8) = urease-producing bacterium = proteus mirabilis (most commonly) or klebsiella pneumoniae - high urine pH --> reduces solubility of phosphate --> struvite stones = magnesium ammonium phosphate young women + acute fever + dysuria + flank pain + costovertebral angle tenderness + leukocytosis + pyuria + perinephric fat stranding = *acute pyelonephritis*

65 yo man comes to doc for fatigue, poor appetite, and lower-leg swelling. Past med hx significant for HT, T2DM, chronic cough, chest infections, & LE peripheral artery dz. Pt has 40 pck yr smoking hx. He doesn't use alcohol or illicit drugs. PE shows barrel-shaped chest, with bilateral scattered wheezing on auscultation. Pt's abdomen distended, and liver edge palpated 4 cm below right costal margin. There is 2+ pitting edema and dilated, tortuous superficial veins over both LEs. Manual pressure on abdomen causes persistent distention of jugular veins. His serum sodium level is 135 mEq/L, and creatinine is 1.2 mg/dL. Abnormality of what is most likely the cause of this pt's edema?

*Pulmonary artery systolic pressure* long-term tobacco + chronic cough + barrel-shaped chest + elevated JVP + enlarged liver = *COPD + cor pulmonale* - impaired function of right ventricle via *pulmonary HT* - exertional symptoms, anorexia or abdominal pain, peripheral edema - loud P2 - right ventricular 3rd heart sound - tricuspid regurgitation murmur = holosystolic at left lower sternal border Echo - increased right heart pressures Right heart cath - pulmonary systolic pressure >25 mmHg

56 yo woman brought to ED with SOB which she says began suddenly 2 hrs ago while she was enjoying her fave TV show. She also reports sharp, left-sided chest pain that worsens with cough. Her previous med hx includes DM for past 10 yrs and HT for past 6 yrs. Fam hx significant for heart disease in her father, who died at age 40. Pt has 30-pck yr smoking hx but doesn't smoke currently. On PE, she is in acute distress and diaphoretic. Temp is 100.5 F, BP is 110/60 mmHg, pulse is 140/min & irregular, and RR is 30/min. Oxygen sat is 84% on RA by pulse oximetry. BMI is 38 kg/m^2. JVP slightly elevated. Lungs clear to auscultation. Lab results: Hct 40% Leukocytes 11,600/mm^3 Platelets 190,000/mm^3 Chest x-ray unremarkable. ECG reveals irregular RR intervals with no definite P waves & narrow QRS complexes. Most likely cause of pt's current symptoms?

*Pulmonary embolism* - acute onset dyspnea + pleuritic chest pain = most common symptoms - tachypnea - ECG findings only minority of cases - atrial fibrillation associated - low O2 sat & atrial fibrillation = poor prognsois

50 yo woman brought to ED after suddenly developing right upper- and lower-extremity weakness while jogging on treadmill. Her weakness gradually worsened over next hour, and she started to experience severe headache, nausea, vomiting. Pt has ho chronic HT but stopped taking antiHTs several mos ago as she is "tired of taking medicine" and trying to "cure" herself with exercise. She doesn't use tobacco but drinks 1 or 2 glasses of wine on weekends. Her BP is 174/102 mmHg and pulse is 76/min and regular. Neuro exam shows right hemiplegia, right hemisensory loss, and leftward deviation of eyes. Most likely diagnosis?

*Putaminal hemorrhage* - almost always involve adjacent internal capsule --> contralateral hemiparesis & hemianesthesia (via disruption of corticospinal & somatosensory fibers in posterior limb) & conjugate gaze deviation toward side of lesion (via damage of frontal eye field efferents in anterior limb) Intraparenchymal hemorrhage: - sudden focal neuro deficits --> gradually worsen over minutes to hours - symptoms often occur during routine activity but may be ppted by exertion - as hemorrhage expands --> headache, vomiting, seizures (if lobar), altered mental status

Patellofemoral pain syndrome is common cause of anterior knee pain in young women. It is usually due to chronic overuse or malalignment. Patellofemoral compression test can reproduce pain. Initial management ...

*Quadriceps strengthening exercises*, activity modification, NSAIDs, strengthening and stretching exercises

A 32-year-old woman presents to your office with a one month history of heat intolerance, racing heart, unintentional 4 pound weight loss, and sweating. On physical examination there is a non-tender enlarged thyroid without evidence of proptosis or exophthalmos. Lab tests return with increased total T4, decreased TSH, and increased free T4. A radioactive iodine uptake exam is ordered and the imaging is displayed in Figure A. What is the most appropriate definitive treatment option for this patient?

*Radioactive iodine ablation alone* Graves' disease - *In the US, the definitive treatment for Graves' with mild or no ophthalmopathy is radioactive iodine ablation* Radioactive iodine uptake scans will show diffuse uptake. Indications for radioactive iodine over antithyroid agents include a large thyroid gland, multiple symptoms of thyrotoxicosis, high levels of thyroxine, and high titers of TSI. Figure demonstrates an image of a radioactive iodine uptake scan with the characteristic pattern of diffuse uptake seen in patients with Grave's disease.

A 14-year-old boy presents to the emergency department with hand pain after falling from his skateboard one day ago. He reports that he lost his balance while attempting a new trick and fell on his outstretched hands. He has been icing his hand and taking several tablets of ibuprofen every few hours, but the pain and swelling have not improved. The patient reports that he has not been able to use the hand to take notes in school. His past medical history is significant for infectious mononucleosis last year and type 1 diabetes mellitus for which he has an insulin pump. On physical exam, there is mild swelling over the dorsal aspect of the hand and wrist. He has tenderness in the region between the extensor pollicus longus and the extensor pollicus brevis of the right hand. Which of the following is the best next step in management?

*Radiograph of the wrist* Pain in the anatomic snuffbox after falling on outstretched hands, which suggests a diagnosis of *scaphoid fracture* - The next best step in management is radiograph of the wrist. - area on the dorsal aspect of the wrist bounded medially by the extensor pollicus longus and laterally by the abductor pollicus longus and extensor pollicus brevis - radiograph helps rule out both a distal radius fracture as well as a displaced scaphoid fracture that would require surgical management - compressed or nondisplaced fractures may not be visible on radiograph for 7-10 days following the injury. For that reason, if the radiograph is negative, the patient's hand and wrist should be placed in a thumb spica cast for 7-10 days before repeating the imaging. - Alternatively, a MRI of the wrist would be the test of choice for a scaphoid fracture if an immediate diagnosis is needed.

A 45-year-old woman presents to the ED with sharp, severe, colicky right flank pain radiating to the groin that she reports started suddenly several hours ago. She also reports discolored urine when she last voided. Vital signs are within normal limits. On exam, the patient is visibly in pain and shifts positions every few minutes. Costovertebral tenderness is elicited on percussion. Past medical history is significant for type II diabetes mellitus, fibromyalgia, gout, and depression. What additional information about the patient's history would likely explain the radiographic findings?

*Recent diagnosis of acute myeloid leukemia* Nephrolithiasis - Radiolucent stones, such as uric acid stones, and cystine stones to some extent, are not visible on radiographs - *Risk factors for uric acid stones include gout (which this patient has), myelo- and lymphoproliferative malignancy, chemotherapy, and xanthine oxidase deficiency* - commonly presents as acute abdominal pain that waxes and wanes, with radiation to the groin - may be associated with nausea and vomiting, as well as gross or microscopic hematuria - diagnosis can be confirmed by abdominal plain film for radiopaque stones (such as calcium), ultrasonography, or non-contrast helical CT scan, which has become the gold standard - initial conservative treatment involves hydration and analgesia - smaller stones are likely to spontaneously pass, while larger stones may require intervention.

47 yo man comes to office in mid-January due to persistent high-grade fever, headache, and severe myalgias that began abruptly 4 days ago. Pt also has sore throat, nonproductive cough, and mild nasal congestion but no SOB or chest pain. It has been >5 yrs since his last appt, and he is not up to date on health maintenance or immunizations. Pt has no chronic med probz, takes no meds, and has no known drug allergies. Temp is 101.3 F, BP is 135/80 mmHg, pulse is 88/min, & respirations are 16/min. Oxygen sat is 98% on RA. PE shows mildly erythematous oropharynx with normal tonsils. No significant cervical LAD present. Breath sounds are clear & equal bilaterally with no dullness to percussion or egophony. Heart sounds normal. Most appropriate next step in management of this pt?

*Recommend symptomatic treatment* Influenza - seasonal resp infection with peak prevalence in Jan and Feb - 1-5 days after inoculation --> abrupt systemic (fever, malaise, headache) & upper/lower resp (rhinorrhea, sore throat, nonproductive cough) symptoms - PE = normal but may show pharyngeal erythema (without exudates) and mild alterations in leukocyte count (low early, high late) - *most adults without risk factors (age >65 yo, chronic med conditions) for influenza = don't require testing --> treated symptomatically*

A 42-year-old man presents to his physician with dark urine and intermittent flank pain. He has no significant past medical history. Vital signs are as follows: Temp 37C, HR 78, BP 180/105, RR 13, and O2 saturation 99% on room air. Physical examination is significant for bilateral palpable flank masses. Urinary analysis is positive for hemoglobin. Which of the following diagnostic modalities should be used to screen members of this patient's family to assess if they are affected by the same condition?

*Renal US* Adult polycystic kidney disease (ADPKD) - *Ultrasound is the best method to screen family members of patients with ADPKD to see if they are affected* - autosomal dominant condition characterized by normal kidneys at birth that subsequently develop multiple cysts bilaterally - often leads to kidney failure requiring dialysis and transplantation after the 3rd or 4th decade Beyond the complications secondary to renal disease, associations with ADPKD include: polycystic liver disease, berry aneurysm, male infertility, mitral valve prolapse, bicuspid aortic valve, aortic dissection, abdominal hernia, or diverticulosis.

64 yo man comes to ED with worsening SOB. For past 3 days, pt has developed rhinorrhea & cough and thinks he might have contracted something from his grandchildren. He has 40 pck yr ho smoking with baseline SOB with exertion. Pt also reports bilateral ankle swelling. PE shows mildly overwt man in mild resp distress. Lung auscultation indicates bilateral wheezes & prolonged expiratory phase. WBC count is 14,500/mm^3, & hemoglobin level is 16 g/dL. Arterial blood gas analysis shows: pH 7.34 PaO2 68 mmHg PaCO2 65 mmHg Absence of marked acidosis in this pt is best explained by ...

*Renal tubular compensation* COPD --> worsening dyspnea & significant hypercapnia PaCO2 65 mmHg - pts with chronic hypoventilation via COPD, obesity ventilation syndrome, or neuromuscular causes = increased PaCO2 --> chronic respiratory acidosis --> to compensate, *kidneys increase bicarbonate retention --> secondary metabolic alkalosis* --> with acute exacerbation of COPD, PaCO2 can increase further - *this pt's near-normal pH of 7.34 suggests large component of chronic hypercapnia for which the kidney shave had time to compensate*

67 yo man brought to ED due to right-sided weakness, numbness, and slurred speech. Pt was in his usual state of health last nt but awoke this morn with these symptoms. He has hx of HT, hyperlipidemia, T2DM, and peripheral arterial disease. Pt doesn't drink alcohol regularly but has 45-pck-yr smoking hx. PE shows right hemiplegia, hemisensory loss, homonymous hemianopsia, and aphasia. CT scan of head reveals large left hemispheric infarction due to occluded middle cerebral artery. Pt hospitalized but is not treated with fibrinolytic therapy due to unclear timing of onset. 48 hrs later, he comes progressively obtunded. Temp 101 F, BP is 154/86 mmHg, and pulse is 64/min. PE now shows dense right hemiplegia with deviation of pt's eyes and head to left. Best next step in management of this pt?

*Repeat CT scan of brain* Hemorrhagic transformation - deterioration of neurologic status 2 days after ischemic stroke - *emergent noncontrast CT of head required to evaluate for HT as urgent surgical decompression is often needed* - usually within 48 hrs of stroke - manifests with deteriorating mental status

67 yo woman with PMHx of HT, hypercholesterolemia, & T2DM calls 911 for severe dyspnea. Her symptoms started 2 hrs ago with chest pain and progressed rapidly to orthopnea and dyspnea. Her BP is 170/100 mmHg and pulse is 120/min and regular. Third heart sound present. Bilateral crackles heard on chest auscultation. Her O2 sat is 78% with 40% inspired oxygen. She is intubated in field by paramedics for progressive respiratory failure and treated with nitrates and diuretics. After initial mesures, breath sounds on left side are markedly decreased. Her repeat BP is 168/96 mmHg. Most likely to restore breath sounds to the left hemithorax?

*Repositioning of endotracheal tube* acute pulmonary edema Pt was intubated for adequate oxygenation & further airway security - ideal location of distal tip of ETT is 2-6 cm above carina - *bc right mainstem bronchus diverges from trachea t relatively non-acute angle, an ETT advanced too far --> preferentially enters into right main bronchus = overinflation of right lung, underventilation of left lung, asymmetric chest expansion* --> auscultation = decreased or absent breath sounds --> chest x-ray confirms diagnosis --> *repositioning ETT by pulling back slightly will move tip between carina and vocal cords & solve prob*

53 yo woman comes to office due to severe insomnia, hot flashes, and mood swings for past several mos. Her LMP was a year ago. Pt had recent wrist fracture that didn't require surgery. She has hx of hypothyroidism diagnosed 7 yrs ago for which she takes levothyroxine, and dose has been stable for several years. Pt's last TSH level was 3.5 uU/mL. She has no other chronic med probz. She doesn't use tobacco, alcohol, or illicit drugs. BP is 120/70 mmHg mmHg and pulse is 75/min. BMI 24 kg/m^2. PE shows non-enlarged thyroid without nodules. Pelvic exam reveals erythematous, atrophic vulva and vagina without discharge or bleeding. Uterus is small, and ovaries are not palpable. Initial oral estrogen replacement therapy in pt would most likely result in what?

*Requirement for L-thyroxine would increase* Most pts with hypothyroidism have increased requirement for levothyroxine after starting oral estrogen Oral estrogen formulations --> decreased clearance of TBG --> elevated TBG

85 yo man comes to ED due to acute-onset SOB associated with coughing spells for 24 hrs. He has no chest pain, fevers, or chills. He has no recent URI symptoms. Pt has med hx significant for COPD and has had previous hospitalizations for COPD exacerbation. His other probz include persistent atrial fibrillation, hypothyroidism, RA, & HT. He is ex-smoker with 40-pck-yr hx. Temp is 98.4 F, BP is 140/90 mmHg, pulse is 110/min & irregular, & respirations are 22/min. PE shows bilateral decrease in breath sounds with prolonged expiration and wheezing in both lung fields. No JVD or peripheral edema. X-ray of chest reveals hyperinflation of both lungs. Pt admitted to hospital & treated with bronchodilators and systemic corticosteroids. After 3 days of hospitalization, he is ready to discharge & is being considered for home oxygen therapy. Indication for long-term home oxygen therapy in such pts?

*Resting pulse oxygen sat <88%* Progressive hypoxemia = common in advanced COPD --> survival benefit of long-term oxygen therapy (LTOT) in those with significant chronic hypoxemia Criteria for LTOT in COPD - resting arterial oxygen tension (PaO2) < 55 mmHg or pulse oxygen sat (SaO2) <88% on RA - PaO2 < 59 mmHg or SaO2 <89% in pts with cor pulmonale, evidence of right heart failure, or hct >55% - dose of supplemental oxygen should be titrated so that SaO2 maintained at >90% during sleep, normal walking, and at rest - survival benefits of home oxygen therapy are significant when used for >15 hrs/day

A 26-year-old male is hit by a car while riding his bike to work. On presentation to the emergency room, the trauma team is activated. On physical exam he has abrasions on the face, hands, and bruising over both iliac crests. His blood pressure is 90/55mmHg and pulse rate is 110 bpm. Scrotal and perineal ecchymosis are noted. Two large bore IV cannulas are inserted and he is given 3L of Ringer's lactate and his blood pressure is re-measured: 120/71 (HR 93). A pelvic binder is applied, and orthopedic surgery and trauma surgery are consulted. What is the most appropriate next step?

*Retrograde urethrogram* Pelvic fracture - resulted in urethra trauma, as evidenced by the scrotal and perineal ecchymosis - before inserting a foley catheter it is important to perform a retrograde urethrogram. Symptoms of urethral injury include an inability to urinate, urinary retention, hematuria, and abdominal pain. Findings on physical exam include evidence of blood at the urethral meatus, swelling and bruising of the penis, scrotum, or perineum, and leakage of urine into tissues surrounding the urethra may result in swelling and inflammation. - *A pelvic radiograph with urethrography is important to perform PRIOR to the insertion of a transurethral bladder catheter* *Illustration displays a cystourethrogram in which a urethral injury is present*

19 yo man brought to ED after being stabbed in back Vitals normal. He is alert & oriented, with Glasgow Coma Score of 15. Neuro exam demonstrates absence of motor activity in all muscle groups of right lower extremity as well as decreased muscle tone. Left leg motor function normal. Right patellar reflex, Achilles reflex, and Babinski sign absent. Loss of light touch and proprioception below right costal margin. Pinprick sensation absent below level of umbilicus on left side. Most likely location of this pt's injury?

*Right spinal hemisection at T8* stabbed in back = *Brown-Sequard syndrome* - injury of hemisection of spinal cord ipsilateral hemiparesis - lateral corticospinal tract - at level of cord injury or below ipsilateral diminished proprioception, vibratory sensation, light touch - dorsal columns - at level of cord injury and below contralateral diminished pain & temp - lateral spinothalamic tract - 1-2 levels below cord injury bc LST decussates 1-2 levels above entry point for corresponding sensory neuron This pt... - right spinal hemisection at T8 = left-sided/contralateral loss of pain sensation + temp sensation beginning around T10 = level of umbilicus

76 yo man brought to office by his daughter due to increasing confusion and falls. Pt was diagnosed with dementia with Lewy bodies 2 wks ago and started on carbidopa-levodopa, risperidone, and rivastigmine for symptomatic management. Temp is 99 F & respirations are 14/min. Supine BP and pulse are 129/82 mmHg and 75/min; standing BP and pulse are 101/74 mmHg and 97/min. Pt not oriented to place or time. CNs are intact and upper and lower limb deep tendon reflexes are 2+. Rigidity is present in both upper and lower extremities on passive movement and is notably worse compared with previous exam. Most likely explanation for this pt's symptoms?

*Risperidone* - may be indicated in pts with functionally impairing visual hallucinations or delusions but with caution via *extreme antipsychotic HS of pts with DLB* --> worsening confusion, parkinsonism, autonomic dysfunction Dementia with Lewy bodies - dementia +/= 2 of the following: visual hallucinations, parkinsonism, fluctuating cognition, REM sleep behavior disorder Treatment = carbidopa-levodopa for parkinsonism, cholinesterase inhibitors (rivastigmine) for cognitive impairment, melatonin for REM sleep behavior disorder

Child most likely have acute otitis media (AOM), as evidenced by the irritability and poor feeding and confirmed by otoscopic exam. The most common cause of acute otitis media is

*S pneumoniae*

Patient's clinical presentation is consistent with body dysmorphic disorder, which can be treated with

*SSRIs* (ex. fluoxetine) Whenever the disorder tends to be obsession based, the treatment typically involves SSRI's (usually high dose) as part of the regimen (such as in BDD or OCD, etc.). - In BDD, the treatment is high dose SSRI's. In BDD, typically there is no defect, or the defect the patient complains of is minor, and it results in impaired ability to function in a social/occupational setting.

24 yo man comes to office due to nonproductive cough for past 2 mos. He has used OTC cough syrups without improvement. Pt has no fever, chest pain, or dyspnea but recently began having dull back pain. He has no chronic med issues and takes no other meds. Vitals are within normal limits. Pulm auscultation reveals normal breath sounds. Back exam shows no deformity or tenderness, and a straight leg raise doesn't elicit pain. Bilateral lower extremity motor strength and sensations are equal, symmetric, and normal. Imaging studies reveals several pulmonary nodules and retroperitoneal lymphadenopathy. What would most likely establish the primary source of this pt's current findings?

*Scrotal ultrasonography* Young pt with 2 mos of cough + pulmonary nodules + retroperitoneal lymphadenopathy --> neoplastic process - malignancies = rare in young man = testicular cancer, lymphoma, leukemia Testicular cancer - minority of pts present with symptoms of metastatic dz - spreads primarily through lymphatics --> *retroperitoneal lymph nodes* often initially affected --> may compress adjacent structures (nerve roots, psoas muscle) --> lumbar back pain - lung mets (nodules) --> cough or dyspnea - if pass is present, scrotal US & tumor markers without delay

28 yo man comes to office due to chronic diarrhea. He reports 5-6 nonbloody liquid bowel movements daily, which also sometimes awakens him at nt. Pt has lost 4.4 lb and says he has diarrhea even when not eating. He has occasional bloating sensations but no abdominal pain, nausea, or vomiting. Several yrs ago, pt was robbed at gunpoint and shot in abdomen. He underwent multiple surgeries and developed PTSD. Temp is 98.8 F, BP is 118/70 mmHg, and pulse is 78/min. BMI is 20 kg/m^2. Abdomen is soft, nondistended, and without tenderness. Bowel sounds are active in all 4 quadrants. On rectal exam, brown stool is present and occult blood is negative. Serum electrolytes normal. Gap between measured and calculated stool osmolarity normal. Most likely mechanism of this pt's diarrhea?

*Secretory diarrhea* - chronic nonbloody + weight loss after multiple abdominal surgeries - larger daily stool volumes (>1 L/day) - occurs even during fasting or sleep - most commonly when luminal ion channels are disrupted in GI tract = state of active secretion Stool osmotic gap = plasma osmolality - 2 x (stool sodium + stool potassium) = <50 mOsm/kg in secretory diarrhea

A 78-year-old man is brought in to the emergency department by ambulance after his wife noticed that he began slurring his speech and had developed facial asymmetry during dinner approximately 30 minutes ago. His past medical history is remarkable only for hypertension. On physical exam, vital signs are within normal limits except for a heart rate of 105 bpm. He is noted to have distinct right facial paralysis. Which of the following is the next best step in the management of this patient?

*Send the patient for a non-contrast head CT scan* The patient in this vignette is most likely suffering from an acute left MCA stroke. - *Initial work-up of a patient with concern for stroke is a non-contrast head CT in order rule out a hemorrhagic stroke before administering tPA.* - The reason to get a non-con head CT in suspected stroke is not to distinguish between ischemic and hemorrhagic stroke, but rather to rule out hemorrhage. - An acute ischemic stroke is diagnosed by diffusion weighted MRI or by using the clinical history (i.e. don't delay tPA just to get an MRI if there is strong clinical suspicion and no evidence of bleed). - In fact, in many cases, you may see nothing on a noncon head CT. - If an ischemic stroke suspected clinically and CT is negative for evidence of a hemorrhagic stroke, the recommended treatment is to give IV tPA if the presentation is within 3-4.5 hours. - For embolic disease and hypercoagulable states give warfarin or aspirin only once the hemorrhagic stroke has been ruled out. - Of note, a contraindication to tPA is systolic BP > 185 or diastolic BP > 110 mm Hg. Illustration shows an MR of an acute left MCA stroke. The left is pane is a T2WI image while right is FLAIR.

A 30-year-old male gang member is brought to the emergency room with a gunshot wound to the abdomen. The patient was intubated and taken for exploratory laparotomy, which found peritoneal hemorrhage and injury to the small bowel. Following the operation, the patient was sedated and remained on a ventilator in the surgical intensive care unit (SICU). Total parenteral nutrition (TPN) was started and the patient has mild hyperglycemia that normalizes after two days. Six days later, the patient developed sudden hyperglycemia. What is the likely etiology of the patient's hyperglycemia?

*Sepsis* While hyperglycemia is common at the start of TPN therapy, sudden onset of hyperglycemia in a patient acclimated to TPN and previously normal blood glucose levels suggests sepsis. Hyperglycemia - in the first 1-2 days after TPN initiation is common, and is associated with increased hospital complications and mortality. Sudden hyperglycemia after several days of TPN warrants a thorough search for a source of infection. - Infection of the IV access catheter site is the most common complication of TPN and may occur in up to 15% of patients.

43 yo man comes to doc reporting acute-onset right knee pain. He was diagnosed with DM year ago and takes metformin. He doesn't use tobacco, alcohol, or illicit drugs. Pt is in monogamous relationship. His dad also has diabetes. His temp is 98.6 F, BP is 134/86 mmhg, pulse is 86/min, and respirations are 16/min. PE shows slightly swollen & tender right knee & mild hepatomegaly. Right knee x-ray reveals chondrocalcinosis and moderate effusion. Appropriate analgesic administered for joint pain. Best next step in management of this pt?

*Serum iron studies* monoarticular arthritis with chondrocalcinosis = CPPD/pseudogout --> should be evaluated for secondary cause of hyperparathyroidism, hypothyroidism, hemochromatosis DM + hepatomegaly = *hereditary hemochromatosis* - HH-induced iron deposition in synovial fluid --> promotes CPPD - diabetes via loss of insulin secretion - often requires injectable insulin - initial evaluation = iron studies = increased levels of serum iron, ferritin, and transferrin saturation - diagnosis can be confirmed with genetic testing for HFE

An 81-year-old man with a long history of type II diabetes presents with altered mental status. He is accompanied with his wife who reports that he has become confused over the past twelve hours and has been complaining of thirst. She also notes that he has hardly been urinating. Initial assessment in the ER reveals blood pressure of 90/62 and heart rate of 124 but no fever. The patient is lethargic but has no other pertinent findings on his physical examination. Initial labs reveal a serum glucose of 720 mg/dL. What would be an additional cardinal feature of this clinical syndrome?

*Serum osmolarity > 320 mOsm/L* This type II diabetic presents with signs and symptoms of *hyperglycemic hyperosmolar nonketotic coma* - A feature of this condition is serum osmolarity > 320 mOsm/L. - hyperosmolar hyperglycemic nonketotic syndrome (HHNS) / hyperosmolar nonketotic coma (HHNC) / hyperosmolar nonacidotic diabetes - clinical syndrome refers to a state of severe hyperglycemia, hyperosmolarity, and dehydration, most commonly affecting an elderly type II diabetic - low insulin levels cause hyperglycemia resulting in osmotic diuresis which leads to dehydration - ketogenesis is minimal because a small amount of endogenous insulin is present, thus preventing significant acidosis *Diagnostic criteria include hyperglycemia with serum glucose greater than 600 mg/dL and hyperosmolarity with serum osmolarity > 320 mOsm/L, along with no acidosis.* Treatment includes fluid and electrolyte replacement and insulin.

A 33-year-old presents to your clinic for a 28-week prenatal visit. She denies any contractions, vaginal bleeding, fluid leakage, or illness. On physical exam you find her to be 4 cm dilated with membranes intact. What condition is most likely present in this patient's past gynecologic/obstetric history?

*Severe cervical dysplasia* Cervical incompetence = *painless dilation of the cervix* - *risk factor for this condition is prior cervical surgery, including loop electrosurgical excision procedure (LEEP) and conization procedures performed in the diagnosis and treatment of cervical dysplasia* - can result when the structural integrity of the cervix is compromised following an excisional procedure for dysplasia - morbidity of incompetence is high, leading most commonly to preterm delivery - diagnosis can be made by vaginal examination, as in this case, or with sonography which shows a short or funneled cervix. A cerclage (stitches place in and/or around the cervix) can be placed to externally reinforce the cervix, followed by a cesarian delivery later in the pregnancy

A woman comes to your office for a prescription for an oral contraceptive. Which of the following characteristics would be an absolute contraindication for prescribing a combined oral contraceptive in this patient?

*She is 37-years-old and has occasional migraines with aura* Patients who are older than 35 years who have coincident migraine with aura are absolutely contraindicated for Combined Oral Contraceptive Pills (COCPs). Studies have shown that this patient population is at increased risk of stroke. Combined oral contraceptive pills (COCPs) are a widely used method of contraception. - The combination of estrogen and progesterone functions to prevent ovulation and thicken the cervical mucous to prevent sperm penetration into the uterus. Absolute contraindications of COCPs include cardiovascular disease (e.g. history of coronary heart disease, stroke, and pulmonary emboli) because of increased coagulability. - Other absolute contraindications include estrogen sensitive malignancies (e.g. breast, endometrial) and hepatic disease. - Relative contraindications include diabetes, sickle cell disease, chronic hypertension, hyperlipidemia and depression. Edlow and Bartz emphasize that the American College of Obstetricians and Gynecologists outline specifically that COCPs are contraindicated in those with migraine with aura but are not contraindicated in patients with non-migraine headaches.

A 28-year-old woman presents to the emergency department with the complaint of double vision. She experienced a similar episode one year ago that has since resolved. Two years ago she had transient gait instability that also resolved. In between these episodes, she has maintained near-normal functioning. She sees a neurologist who prescribes interferon-beta in an effort to decrease the frequency of these events. Which of the following should this patient be counseled of regarding this medication?

*She should be on contraceptives if she is currently or will be sexually active, as this drug is teratogenic* This patient's history of neurologic deficits separated by both time and space with a return to baseline in between is consistent with relapsing-remitting multiple sclerosis (RRMS). Interferon-beta - used in the treatment of RRMS - known teratogen and women should be started on contraceptives if prescribed this medication American Academy of Neurology's report on disease-modifying therapies for MS: summarize the available evidence for IFN-beta in the treatment of RRMS - Class I evidence that treatment with IFN-beta delays the development of clinically definite MS by delaying the time between attacks 2 and 3 or between attacks 3 and 4 - They caution though that studies do not show that early treatment affects long-term disability outcome.

A 28-year-old woman is following up with her primary care physician for treatment of depression. Three months ago she was diagnosed with a first episode of major depressive disorder and was started on sertraline at that time. She now reports significant improvement in her symptoms for the last month. She denies any difficulty sleeping or difficulty tolerating the medication. She is exercising regularly and enjoying her work as an attorney. She wants to know how much longer she should continue treatment with sertraline. Which of the following is the appropriate course of management?

*She should continue treatment for an additional 5 months.* Antidepressant therapy following an initial episode of major depressive disorder (MDD) should be continued for 6 months following response to treatment. This woman's symptoms resolved 1 month ago, therefore she should continue treatment for another 5 months.

45 yo woman brought to ER by ambulance after neighbor found her to be in somewhat obtunded state with empty bottle of med next to her. While en route to ER she suffered a tonic clonic seizure. Her PMHx is significant for major depression and chronic back pain. Her temp is 100.9 F, BP is 90/70 mmHg, pulse is 120/min, & respirations are 16/min. She is able to respond to painful stimuli. Pupils are 8 mm in size bilaterally. Skin is warm and flushed. Lungs clear to auscultation. Abdomen soft & non-tender. Bowel sounds decreased. No neck stiffness. EKG shows QRS duration of 130 msec, a change from her previous EKG mo ago. Most appropriate next step in management?

*Sodium bicarbonate* TCA overdose - ho depression - hyperthermia - anticholinergic effects = dilated pupils, intestinal ileus - can cause seizures - decreased myocardial velocity --> QRS prolongation, risk of ventricular arrhythmia - major cause of mortality = TCA-induced hypotension - first priority = secure airway, breathing, circulation - *hypotension, QRS prolongation, ventricular arrhythmia --> administer sodium bicarb* --> improves systolic BP, narrows QRS complex, decreases incidence ventricular arrhythmia

28 yo woman comes to office due to episode of lightheadedness followed by syncopal episode. She has had several days of headaches, malaise, and myalgia but no vomiting or diarrhea. Pt has no ho similar symptoms. She was diagnosed with chlamydia cervicitis 3 wks ago and received single dose of azithromycin.. Pt has no chronic med probz or fam hx of sudden cardiac death. She lives in rural Massachusetts and has not traveled recently. Temp is 98.6 F, BP ia 120/80 mmHg, pulse is 43/min. No orthostatic BP changes are present. Cardiopulm auscultation reveals no heart murmurs. Right thigh has 2 erythematous skin lesions. ECG shows sinus rhythm with 2:1 AV block. Most likely cause of this pt's current condition?

*Spirochete infection* Lyme dz = spirochetal infection = via Borrelia burgdorferi via Ixodes scapularis tick Lyme carditis - within first wks or mos of B burgdorferi infection - most common manifestation = AV conduction block - symptoms = lightheadedness, syncope, dyspnea, chest pain, and/or palpitations *Young person who lives in Lyme endemic region has cardiac (lightheadedness, syncope, bradycardia, AV block), systemic (headache, malaise, myalgia), and skin (likely EM) manifestations = early disseminated dz

18 yo man brought to ED for possible injury during football practice. While running for touchdown, pt was tackled from front and fell to ground with another player landing on top of him. Immediately after he started to experience abdominal discomfort, nausea, and left-sided chest and shoulder pain. Pt has no prior med issues other than mild sore throat and fever several weeks ago. He doesn't use tobacco, alcohol, or illicit drugs. BP is 106/67 mmHg, pulse is 107/min, and respirations are 24/min. Pt is diaphoretic and appears anxious. Heart sounds normal without murmur. Bilateral breath sounds clear & equal. He has left chest wall tenderness. Abdomen mildly distended and tender with decreased breath sounds. ROM of left shoulder full and not painful. Further eval of this pt most likely to reveal what?

*Splenic laceration* - most likely in setting of abdominal pain, tachycardia, left chest wall & should pain without evidence of abnormalities of shoulder (likely referred pain via phrenic nerve irritation from splenic hemorrhage) - *pt's recent fever & sore throat may = IM = associated with splenomegaly = risk of splenic injury* - next step in eval = focused assessment with sonography of trauma (FAST) exam --> identify signs of hemorrhage Blunt abdominal trauma (abdominal tenderness) + Blunt thoracic trauma (chest wall tenderness) following football injury - tachycardia (or hypotension) concerning for hemorrhage via solid organ or vascular injury - *most common intra-abdominal organ injuries due to BAT = hepatic & splenic lacerations*

Should be suspected in *any* pt with cirrhosis and ascites who presents with low-grade fever, abdominal discomfort, or altered mental status. Reitan trail test, a timed connect-the-numbers test, helps detect subtle mental status changes sometimes present in pts with this.

*Spontaneous Bacterial Peritonitis* (SBP) - intestinal bacterial translocation directly into ascitic fluid or hematogenous spread to liver & ascitic fluid (via other bacterial infections) - pts with cirrhosis are often relatively hypothermic --> any temp >100 F warrants investigation Reitan trail test - timed connect-the-#s test similar to children's connect-the-dots pictures - detects subtle mental status changes sometimes in pts with cirrhosis & SBP

Cardiac auscultation in pts with MVP typically shows systolic click and/or mid to late systolic murmurs of mitral regurgitation. What decreases the intensity of the murmur?

*Squatting* from standing position --> increases preload & left ventricular volume --> decreases intensity of murmur

A 23-year-old woman with no significant past medical history presents to the Emergency Department with 2 days of worsening fever, vomiting, and myalgias. Physical exam is remarkable for a red, macular rash diffusely present across her chest and abdomen. Vital signs are as follows: BP 90/54 mmHg, HR 110 bpm, T 102.7F. Her last menstrual period was 5 days prior. Urine pregnancy test is negative. She denies respiratory or urinary symptoms. Which of the following organisms is most likely implicated in this patient's presentation?

*Staphylococcus aureus* Toxic Shock Syndrome, caused by pre-formed Staphylococcus aureus toxin. - associated with prolonged tampon use. - caused by the release of a pre-formed S. aureus exotoxin, TSST-1. This toxin acts as a superantigen, which activates T cells leading to massive cytokine release - presents with fever, vomiting, diarrhea, and the development of a diffuse macular erythematous rash - complications of TSS include acute respiratory distress syndrome (ARDS), hypotension, disseminated intravascular coagulation (DIC), and hemorrhage - The diffuse erythematous rash seen in TSS often resembles a sunburn but is associated with oral and genital hyperemia, as well as conjunctival injection. Desquamation of palms and soles may occur during recovery. - Treatment necessitates intensive care unit admission and primarily consists of supportive care (fluid resuscitation, monitoring of urine output, supplemental oxygen or intubation) per Surviving Sepsis Campaign guidelines - The tampon, if present, must be removed. Antibiotic coverage for Staphylococcus, such as nafcillin (for MSSA) or vancomycin (for MRSA), should be started promptly after diagnosis. Adding clindamycin to the antibiotic regimen is thought to decrease TSST-1 production

19 yo man brought to ED by his college roommate due to day of confusion, severe dyspnea, and cough productive of yellowish sputum streaked with blood. According to his roommate, pt visited an urgent care clinic last wk for fever, rhinorrhea, and malaise. He was diagnosed with "viral infection" and given a prescription med that seemed to improve his illness. He has no other med issues, takes no other meds, and doesn't smoke cigz or drink alcohol. Temp is 104 F, BP is 124/75 mmHg, pulse is 125/min, & respirations are 32/min. Lung auscultation shows bilateral crackles in lung midfields. 2/6 ejection systolic murmur heard at left upper sternal border. Extremities warm with bounding peripheral pulses. Chest x-ray reveals alveolar infiltrates in midlung fields bilaterally as well as several thin-walled cavities. Most likely cause of this pt's condition?

*Staphylococcus aureus* - preferentially attacks young pts with influenza - CA-MRSA --> severe, necrotizing pneumonia that is rapidly progressive and fatal - high fever + productive cough with hemoptysis + leukopenia + mutilobar cavitary infiltrates Pt given prescription med for viral URI, like influenza --> several days later after symptomatic improvement --> high fever, dyspnea, cough = characteristic chronology for secondary bacterial pneumonia - most pts require: admission to ICU, broad-spectrum abx, empiric abx, including vanc or linezolid

65 yo woman comes to doc for evaluation of deteriorating memory. SHe used to pride herself on her sharp memory and is very concerned about its loss, saying, "Over last several mos I have been forgetting most trivial things." She describes having become "extremely inefficient" in focusing as well. Pt also complains of easy fatigability, poor appetite, and frequent nighttime awakenings. She feels worthless and has lost interest in gardening, her fave hobby. On coughing or laughing, she loses urine involuntarily, contributing to her misery. Pt lives with her husband, to whom she has delegated tasks such as household chores and cooking. Her med hx is significant for high BP and hypercholesterolemia. She discontinued hormone replacement therapy last yr. PE normal. On cognitive testing with MMSE, she scores 25/30 with difficulties in delayed recall and attention. Lab studies, including thyroid function, are unremarkable. CT scan of head normal. Best next step in management of this pt?

*Start selective serotonin reuptake inhibitor therapy* severe, especially in older adults, may present with features similar to dementia = *pseudodementia / reversible cognitive impairment* - antidepressants = treatment of choice --> should reverse cognitive deficits

A 68-year-old man with hypertension, coronary artery disease, and hyperlipidemia presents to his primary care physician for an annual check-up. He reports that he has recently been having repeated episodes of dizziness and light-headedness on a daily basis. As a result, he has significantly reduced his physical activities. Currently, he feels well. His vital signs are HR 52 bpm, BP 130/80 mmHg, RR 11 rpm, T 98.2F. Cardiac exam reveals a bradycardic, regular rhythm with no extra sounds or murmurs. His medications include atorvastatin, aspirin, hydrochlorothiazide, lisinopril, and metoprolol. An ECG is presented in Figure A. What is the most appropriate next step in the management of this patient?

*Stop pt's metoprolol* sick sinus syndrome - specifically persistent severe sinus bradycardia. - all medications that may worsen bradycardia should be avoided. Symptomatic bradycardia may be a result of the patient's beta blocker or a result of sinus node dysfunction. - The first step in management should be to stop offending medications and then monitor for continued symptoms or arrhythmias. Atropine is a appropriate treatment for HEMODYNAMICALLY UNSTABLE patients, but not just solely SYMPTOMATIC patients. In this question stem, the patient has episodic symptoms, but based on vital signs is completely hemodynamically stable. Patients with sick sinus syndrome are rarely hemodynamically unstable for a prolonged period; however, those who are unstable should be urgently treated using the Advanced Cardiac Life Support (ACLS) protocol with atropine, dopamine, or epinephrine as well as temporary cardiac pacing. For symptomatic patients who are hemodynamically stable, one of the first steps is to look at the patient's medication list to see what medications could be exacerbating the symptoms.

HIV-positive patient has community-acquired pneumonia. The most common cause of pneumonia in HIV-positive patients is

*Streptococcus pneumonia* Pneumococcal pneumonia and meningitis are the leading causes of morbidity and mortality in HIV-positive patients. Pneumococcal vaccination should be administered in all HIV-positive patients regardless of CD4 count, although the effectiveness of the vaccine will wane with greater levels of immunosuppression. Prophylaxis with TMP-SMX or other appropriate antibiotics also reduces the risk of pneumococcal pneumonia in HIV patients.

54 yo woman brought to ED due to fever & lethargy. Pt has had several days of productive cough & yesterday developed severe headache, nausea, & vomiting. This morn, she became weak and confused and could not get out of bed by herself. This morn, she became weak and confused and could not get out of bed by herself. She has ho T2DM. Pt drinks 1 or 2 glasses of wine daily. Temp is 102.4 F, BP is 110/70 mmHg, pulse is 108/min. Pt is somnolent and requires constant physical stimuli to remain awake. She is oriented to person only. Marked neck rigidity is present. Pulm auscultation reveals crackles & bronchial breath sounds in right lower lung field. No skin rash. Lab testing shows leukocytosis. CT scan of head normal. LP performed. What organisms is most likely to be seen in the spinal fluid of this pt?

*Streptococcus pneumoniae* - most common cause of community-acquired bacterial meningitis Bacterial pneumonia: >2 .... - headache - fever (100.4) - nuchal rigidity - altered mental status - concurrent pneumococcal pneumonia may or may not be present

24 yo woman comes to urgent care due to increasing pain in her rt leg. She initially felt dull, aching pain at anterior aspect of lower leg 2 wks ago. Pt is avid dancer and often practices 4-5 hours a day; pain has worsened and is now interfering with her dance sessions. Her med hx is remarkable for irregular, light menstrual periods, for which she is being evaluated by her gynecologist. Pt drinks alcohol only on social occasions and does not smoke or use illicit drugs. Temp is 98 F, BP is 122/78 mmHg, and pulse is 70/min. BMI is 16 kg/m^2. Exam shows point tenderness over anterior aspect of right shin. No skin abnormalities, and rest of physical exam is unremarkable. X-ray of lower leg reveals no abnormalities. Most likely diagnosis?

*Stress fracture* Tibial stress fracture - subacute leg pain - most commonly occurs in athletes (runners, dancers) or individuals who suddenly increase their activity level - via repeated tension or compression without adequate rest - women with low bone density + low caloric intake & hypomenorrhea/amenorrhea (female athlete triad) - localized activity-related pain + swelling + point tenderness on palpation - x-rays frequently normal (especially in first few wks) but may reveals periosteal reaction at site of fracture

A 45-year-old woman presents to your office with a 2-week history of a painful mass in her neck after having a sore throat and fever for 3 days. The patient reports the mass has slowly been enlarging over that time span and has become more painful to the touch. She also reports feeling hot, even when her coworkers feel cold, and reports loose stools over the past week. The patient's vital signs are T 37 C, BP 140/90, Pulse 110 bpm, SpO2 100%. On exam, you note a diffusely enlarged thyroid that is painful to the touch. Thyroid stimulating hormone is decreased, T4/T3 is elevated, and radioactive iodine uptake and scan at 24 hours reveals an uptake of 3% (normal 8-25%). What is the most likely diagnosis?

*Subacute thyroiditis* Post-infectious painful enlargement of the thyroid + hyperthyroidism = *subacute granulomatous thyroiditis /de Quervain's thyroiditis* - most common cause of thyroid pain and has a greater incidence in women - etiology is often post-infectious and viral in origin, usually following symptoms such as fever, myalgia, and pharyngitis - Early in the course of disease, the patient may be hyperthyroid as follicular cells are damaged and release large amounts of T3/T4 --> This is often followed by a period of hypothyroidism as T3/T4 is depleted, and eventually euthyroidism within 6-12 months. - Radioactive iodine (RAI) scan reveals diminished uptake.

68 yo man comes to ED due to severe dizziness while playing tennis an hour ago. He says that he had spinning sensation accompanied by nausea & vomiting; symptoms have now subsided. Pt has had brief episodes of dizziness in past, especially when performing vigorous work with his arms. He has also experienced heaviness and fatigue of left arm with exertion. Pt has ho HT, hyperlipidemia, and T2DM. BP is 140/90 mmHg on right arm & 100/74 mmHg on left, and pulse is 82/min and regular. Systolic bruit present at base of neck just above clavicle on left side. Cardiac auscultation discloses fourth heart sound. No extremity weakness or sensory loss. Most likely cause of this pt's symptoms?

*Subclavian artery occlusion* Subclavian steal syndrome - via severe atherosclerosis of left subclavian artery proximal to origin of vertebral artery --> decreased pressure in distal subclavian artery & reversal ("steal") of blood flow in ipsilateral vertebral artery - when symptomatic = ischemia if affected upper extremity (pain, fatigue, paresthesias) - PE = significantly lower brachial systolic BP (>15 mmHg) in affected arm & systolic bruit in supraclavicular fossa on affected side - fourth sound may be present due to left ventricular hypertrophy from systemic HT - diagnosis via Doppler US or MRA - treatment = lifestyle management = lipid-lowering interventions, smoking cessation; sometimes stent

72 yo man undergoes CABG for severe CAD. He is extubated on 2nd postop day. His temp is 97.9 F, BP is 120/70 mmHg, pulse is 80/min, & respirations are 12/min. On 3rd postop day, he complains of dyspnea and worsening retrosternal pain despite continuous analgesia with morphine. His temp is 101.5 F, BP is 112/52 mmHg, pulse is 125/min & irregularly irregular, and respirations are 28/min. Exam shows normal heart sounds. Small amount of cloudy fluid present in sternal wound drain. His ECG shows atrial fibrillation with rapid ventricular response. X-ray film of chest shows widening of mediastinum. Echo reveals small amount of pericardial fluid. Lab studies: Hgb 8.9 mg/dL WBC count 16,300/mm^3 Platelet count 512,000/mm^3 Creatinine 1.7 mg/dL CPK 430 U/L What will this pt most likely require?

*Surgical debridement and antibiotic therapy* Acute mediastinitis - possible complication of cardiac surgery - usually due to intraoperative wound contamination - can complicate up to 5% of sternotomies - typically present post-operatively within 14 days = fever, tachycardia, chest pain, leukocytosis, sternal wound drainage or purulent discharge

A 15-year-old boy presents to the emergency room with severe lower abdominal pain that awoke him from sleep about 3 hours ago. The pain is sharp and radiates to the left thigh. While in the emergency room, the patient experiences one episode of vomiting. He denies any fever, dysuria or chills. Physical examination reveals normal vitals with blood pressure 100/60 and temperature of 98.6F. Abdominal examination is relatively benign. Scrotal examination reveals an elevated left testis that is diffusely tender. The cremasteric reflex is absent. What is the next step in the management of this patient?

*Surgical exploration* Left testicular torsion - surgical emergency - patient should be taken directly to the operating room without delay in order to save the testicle - acute scrotal pain in prepubescent and young adult males is testicular torsion until proven otherwise - on physical exam, the most accurate diagnostic sign is an absent ipsilateral cremasteric reflex - treatment should restore blood flow as quickly as possible, ideally, in less than six hours after the onset of symptoms - it is possible to manually detorse the testicle by external rotation but restoration of blood flow must be confirmed - surgical exploration is definitive treatment of choice for the affected testis - following detorsion, both testes should undergo orchiopexy as prophylaxis

65 yo man found to have lung nodule on chest x-ray performed for evaluation of dry cough. Cough has since resolved. He currently feels well and says that he is full of energy. Pt has 23 pck yr smoking hx but quit 2 yrs ago. He has no fam hx of cancer. CT scan of chest reveals 2-cm, peripheral, round lesion in right lower lung lobe that is completely surrounded by lung parenchyma. Borders are irregular, and there is no calcification. No other lung lesions or lymph node enlargement. Best next step in management of this pt?

*Surgical resection* Solitary pulmonary nodule (SPN) - rounded opacity - <3 cm - completely surrounded by pulmonary parenchyma - no associated lymph node enlargement - first step = determine if nodule has low, intermediate, or high probability for malignancy This pt... > 60 yo + 2-cm nodule with irregular margins + recently quit smoking (<5 yo) = high risk of malignancy - surgically excise! Surgical excision also recommended for pts with... - lesion growth on serial imaging - positive 18-fluorodeoxyglucose positron emission tomography (FDG-PET) scans - suspected malignancy on tissue pathology

A 36-year-old woman with a long history of a heart murmur presents with one week of increasing fatigue and low-grade fevers. She had a dental cleaning two weeks ago. She denies any hematuria, neurological symptoms, or changes in the appearance of her hands and fingernails. Her past medical history is otherwise insignificant. On physical exam, her temperature is 38.1 C (100.6 F), heart rate is 92/min, blood pressure is 118/67, and respiratory rate is 16/min. She appears fatigued but in no acute distress. Cardiac auscultation reveals a III/VI holosystolic murmur best heart at the cardiac apex in the left lateral decubitus position. Pulmonary, abdominal, and extremity exams are within normal limits. An echocardiogram is performed and is shown in Figure A. The patient is started on empiric IV vancomycin and gentamicin. Three days later, 4/4 blood cultures grow Streptococcus viridans that is highly sensitive to penicillin. What is the next best step in management?

*Switch to IV ceftriaxone* *Subacute bacterial endocarditis (SBE) with Streptococcus viridans highly susceptible to penicillin should be treated with IV penicillin G or IV ceftriaxone*, based on American Heart Association recommendations. Transient bacteremia from dental work can cause seeding of a pre-existing abnormal valve. - Abnormal valves cause turbulent flow, which erodes the endocardium, thereby predisposing the valve to bacterial colonization and endocarditis. - Patients with SBE present with prolonged intermittent low-grade fever and progressive fatigue. - *Streptococcus viridans is the most common microbe involved in native valvular infections and should be treated with intravenous penicillin or ceftriaxone for four weeks when susceptible.* Figure shows an echocardiogram of a mitral valve with subacute bacterial endocarditis.

A 45-year-old female presents to your office with a 2-week history of a painful mass in her neck after having a sore throat and fever for 3 days. The patient reports the mass has slowly been enlarging over that time span, and has become more painful to the touch. She also reports feeling hot even when her coworkers feel cold, and also reports loose stools over the past week. The patient's vital signs are T 37 C, BP 140/90, P 110 bpm, O2 100%. On exam, you note a goiter that is painful to the touch. TSH is decreased, T4/T3 is elevated, and radioactive iodine uptake and scan at 24 hours reveals an uptake of 3%. What is the next step in treatment?

*Symptomatic relief* Subacute granulomatous thyroiditis (also known as de Quervain's thyroiditis) - *First-line treatment involves symptomatic relief, including NSAIDs and beta blockers.* - typically presents as pain in the thyroid region due to transient inflammation and thyroid enlargement, and is typically preceded by a viral infection - natural course of subacute granulomatous thyroiditis involves destruction of follicular cells and the subsequent release of T3/T4 stores, causing transient hyperthyroidism lasting approximately 2-8 weeks. Once depleted, the patient will often experience a period of hypothyroidism, which naturally resolves over a period of 6-12 months.

36 yo woman, gravida 0 para 0, comes to office with frequent headaches and amenorrhea for last 2 mos. Pt has also had wt loss, intermittent palpitations, and vague visual disturbances. Her med hx is unremarkable, and she takes no meds. Pt experienced menarche at age 12 and has had normal menstrual cycles all her life. She doesn't smoke cigz or drink alcohol. BP is 130/60 mmHg, and pulse is 103/min and regular. Wt is 152 lb; a yr ago it was 168 lb. Exam shows symmetrically enlarged, nontender thyroid gland. Auscultation of chest shows tachycardia with regular heart rhythm. Find hand tremor noted. Remainder of PE unremarkable. Lab results: Serum T3 222 ng/dL Serum T4 13.9 ug/dL Serum TSH 6 uU/mL Most likely diagnosis in this pt?

*TSH-secreting pituitary adenoma* Central hyperthyroidism - elevation of both TSH & thyroid hormones - associated with tumor mass effects = headache, visual symptoms, impaired production of other pituitary hormones - uncommon - mostly macroadenomas - goiter via effect of TSH on growth of thyroid follicles - high concentration of circulating thyroid hormone + elevated or inappropriately normal TSH

85 yo male nursing home resident with PMHx of stroke has developed pressure ulcer over right greater trochanter. Ulcer is 2 cm in size & is noted to be shallow with reddish-pink woundbase. No secondary infection. What would be best for cleansing wound?

*Tap water* Stage 2 pressure ulcer - pressure ulcer treatment = either saline or tap water and covered with hydrocolloid, foam, or another nonadherent dressing that promotes moist env't

45 yo woman comes to office due to 3 mos of excessive hair growth over her face & body. Her LMP was 4 mos ago. Pt's menstrual cycle had previously been regular, and she has had 2 successful pregnancies in the past. Med hx includes hypothyroidism, for which she takes stable dose of levothyroxine. Pt has smoked half pack of cigz daily for 25 yrs. BP is 124/82 mmHg and pulse is 72/min & regular. BMI 24 kg/m^2. Large amount of coarse terminal hair noted on face, back, chest, and lower abdomen. Remainder of PE normal. Lab results: Sodium 136 mEq/L Potassium 4 mEq/L Chloride 102 mEq/L Bicarbonate 22 mEq/L BUN 10 mg/dL Creatinine 1.2 mg/dL Glucose 80 mg/dL TSH level normal, and hemoglobin 16 g/dL (1 yr ago it was 14 g/dL). Most appropriate next step in management of this pt?

*Testosterone and DHEAS levels* Women with suspected androgen-producing tumors should be evaluated with serum testosterone and DHEAS levels - elevated testosterone levels + normal DHEAS = ovarian source (more common) - elevated DHEAS = adrenal tumor (less common)

67 yo woman comes to office for follow-up. Four wks ago, she experienced sudden-onset right side numbness, and eval revealed ischemic stroke. Pt feels like some of her sensation is returning but experiences transient burning pain in right upper and lower limbs that can be induced even by light touch. Her med hx is significant for HT & T2DM. She smoked half-pack of cigz daily for 30 yrs but stopped after her stroke. BP is 125/70 mmHg, and pulse is 74/min & regular. Neuro exam shows right-side of body demonstrated by exaggerated pain on light touch. Motor strength is normal in all 4 extremities. Most probable location of stroke experienced 4 wks ago by this pt?

*Thalamus* Lacunar stroke of posterolateral thalamus - typically via atherothrombotic occlusion of small, penetrating (thalamogeniculate) branches of posterior cerebral artery - ventral posterolateral and ventral posteromedial nuclei of thalamus --> transmit sensory info from contralateral side of body and face --> *sudden-onset contralateral sensory loss* involving all sensory modalities (pure sensory stroke) - can be accompanied by transient hemiparesis, athetosis, or ballistic movements via disruption of neighboring basal ganglia structures and corticospinal fibers in posterior limb of internal capsule Several weeks to month following stroke, sensory deficits can improve --> however, some pts develop *thalamic pain syndrome / Dejerine-Roussy syndrome* - severe paroxysmal burning pain over affected area - classically exacerbated by light touch (allodynia)

A 65-year-old farmer presents to the ER with abdominal pain. He is grasping his right upper quadrant. The patient reports that he has had the pain for several months and it has been worsening. He has no past medical history. Vital signs are stable. His physical exam is notable for hepatomegaly. An MRI is performed which reveals the following (flip for image). If this patient's symptoms are due to an infectious agent, what would be the appropriate management?

*The cysts should be surgically resected after preinjection with ethanol and the patient treated with mebendazole* Hydatid cysts with large cysts enclosing daughter cysts. - *Treatment is with surgical resection and mebendazole. If cyst contents are spilled, there is a risk for anaphylaxis, which can be minimized by injection of ethanol, which kills daughter cysts, prior to resection.* - caused by infection from the tapeworm Echinococcus granulosus or multilocularis. - Contact with sheep is a risk factor and may have contributed to the infection in this farmer. - Most often cysts are found in the right lobe of the liver. *Image depicts the cystic appearance of the liver on MRI which is classic for hydatid disease.*

A 60-year-old woman presents with diffuse colicky abdominal pain, nausea and vomiting. The patient's past medical history is significant for hypertension and perforated appendicitis at age 25 that was managed surgically. Her physical exam reveals hyperactive bowel sounds, diffuse abdominal tenderness and distention. An abdominal x-ray is shown (flip for image). Which of the following additional features would result in the patient being managed surgically?

*The patient has rebound tenderness* Small bowel obstruction - *Signs of perforation (peritonitis, fever, leukocytosis, tachycardia, metabolic acidosis) are an absolute indication for immediate surgery* - most commonly caused by intra-abdominal adhesions, masses, or intestinal herniation - with presentation and diagnostic confirmation (x-ray or CT), the appropriate initial management involves making the patient NPO, administering IV fluids, and nasogastric decompression with NG tube placement - Surgery is indicated if the obstruction is refractory to such interventions, or if signs of perforation or vascular compromise are present - Adjunctively, water-soluble contrast studies (i.e. Gastrograffin) should be utilized in patients whose obstruction are refractory to conservative treatment after 48 to 72 hours. Image A shows a "step-ladder" appearance of air-dilated proximal small bowel with little to no gas distal to the obstruction.

A 19-year-old girl presents to clinic with a chief complaint of anxiety. She reports that she has an intense fear of public speaking. She has been enrolled in a new government class in college that requires a presentation in front of the class and the thought of it has induced panic. She reports often being embarrassed in social situations. She notes that she cannot use public restrooms with others present. The patient wants to succeed in her new class and overcome her fear. What is the principle diagnosis and a potential therapeutic strategy for this patient?

*The patient has social phobia and a possible therapy would be paroxetine* Social phobia - exaggerated fear of embarrassment in social situations such as public speaking and performance situations - cluster C personality disorders have a genetic association with anxiety disorders such as social phobia - *Treatment for social phobia includes behavioral therapy in conjunction with SSRIs such as fluoxetine, paroxetine, sertraline, and citalopram*

Two studies were conducted on different samples from same population to assess relationship between oral contraceptive use and risk of DVT. Study A showed increased risk of DVT among oral contraceptive users, with relative risk of 2.0 and 95% confidence interval of 1.2-2.8. Study B showed relative risk of 2.01 and 95% confidence interval of 0.8-3.1. What statement is most likely to be true regarding these 2 studies?

*The sample size is likely smaller in study B than study A* If RR is 1.0 (null value), there is no association between exposure and outcome (or dz) RR > 1 indicates that exposure is associated with increased risk of dz RR < 1 means exposure is associated with decreased risk of dz RR in studies A & B were >1 --> increased DVT risk among OC users *For result to be considered statistically significant, its corresponding confidence interval (CI) must NOT contain null value* - 95% CI does NOT contain null value --> corresponding p-value <0.05 & association between exposure and outcome considered statistically significant --> p-value <0.05 = low probability that results was due to chance alone Power of a study = ability to detect difference between 2 groups (exposed vs unexposed) when there is truly a difference - increasing sample size --> increases power of study = narrows the confidence interval surrounding the point estimate

Recognize possible fetal heart rate tracing in *placental abruption* "A 36-year-old G3P2002 at 34 weeks gestation presents to the obstetric triage unit with acute abdominal pain that started this afternoon at an outdoor festival where she ate tacos from a food truck. She describes the pain as diffuse and nearly constant but denies vaginal bleeding and loss of fluid. Her husband who is accompanying her, states that he had one episode of diarrhea after arriving in the triage waiting room. The patient has no past medical history. She had one spontaneous vaginal delivery five years ago and a low transverse C-section three years ago for arrest of descent. Her fetal heart rate tracing is shown in Figure A. Temperature is 98.8°F (37.1°C), blood pressure is 104/58 mmHg, pulse is 99/min, and respirations are 15/min. The patient has dry skin that appears to be mildly sunburned. Cervical exam is fingertip dilated, long, and high. Which of the following is the most likely etiology of this patient's condition?"

*There are frequent, low-amplitude contractions, fetal tachycardia, minimal variability, and 1 late deceleration* - constant (as opposed to episodic) abdominal pain is suggestive of abruption, even without vaginal bleeding - classically presents with painful vaginal bleeding - some women have concealed abruptions, where the blood is trapped between the fetal membranes and decidua This patient's fetal heart rate tracing is also very concerning for abruption. Possible findings include frequent low-amplitude contractions, decreased variability, late decelerations, and fetal bradycardia or tachycardia. - If abruption is suspected, further confirmation of diagnosis can be achieved with fibrinogen level < 200 mg/dL and ultrasound, which may show a retroplacental hematoma - Complications include maternal hemorrhage, DIC, and fetal distress from uteroplacental insufficiency.

You are conducting a study comparing the efficacy of two different statin medications. Two groups are placed on different statin medications, statin A and statin B. Baseline LDL levels are drawn for each group and are subsequently measured every 3 months for 1 year. Average baseline LDL levels for each group were identical. The group receiving statin A exhibited an 11 mg/dL greater reduction in LDL in comparison to the statin B group. Your statistical analysis reports a p-value of 0.052. Which of the following best describes the meaning of this p-value?

*There is a 5% chance that A is more effective than B due to chance rather than an actual difference.* The p-value states whether the difference is due to random chance. The smaller the p-value the more likely there is a real difference. In this study, we see that the group taking statin A does better than the other group, but we do not know if that difference is due to the drugs or whether the difference is due to random chance. Therefore, to decide whether there is a real difference in the drugs, we ask "What is the likelihood that we would have gotten this result just by random chance if the two drugs are the same (the null hypothesis)?". The answer to this question is known as the p-value, which is defined as the likelihood of getting this result or a more extreme result if the null hypothesis is true. The smaller this number, the less likely it is that the result was due to chance alone. Therefore in this case a p value of 0.052 means that there is only a 5.2% chance that this reduction of 11 or a greater reduction would be seen even if the drugs are the same.

Health officials are considering a change be made to the interpretation of the tuberculin skin test that will change the cut-off for a positive purified protein derivative (PPD) from 10 mm to 5 mm for healthcare workers. Which of the following is a true statement regarding this potential change?

*This change will decrease the positive predictive value of the test* This change - increase the sensitivity of the test and, assuming the prevalence of disease in healthcare workers has not changed, will decrease the positive predictive value (PPV) The PPV of a test - probability that an individual will have the disease if they receive a positive test result In the case of changing the threshold for a positive PPD - lower cut-off will result in more positive test results--both true and false positives. - increase in false positives will be disproportionately larger than the increase in true positives when the incidence of disease is unchanged - As a result, the PPV will decrease.

A 23-year-old female with no past medical history presents to general medical clinic. She is accompanied by her mother. Her mother reports that over the past several months, her daughter has not "been herself." She graduated with highest honors as a history major and has no history of behavioral issues. In general, she has always been quite shy. However, recently, she has been striking up long conversations with strangers. Her friends have noticed that they can't cut her off and that she'll keep speaking for an hour without giving them a turn. She has been staying up all night reading, cleaning the house, and running on the treadmill. She has been driving recklessly and received two tickets for speeding. Her mother has tried to approach her about her behavior but she insists nothing is wrong and gets very angry when her mother confronts her. Which of the following is true regarding this patient's illness?

*This patient may be treated for one year with lithium then tapered off* In patients with bipolar disorder who have experienced a first manic episode, lithium treatment may be continued for a year and then tapered off. Patients with multiple recurrences (three or more) should be on life long therapy. Bipolar disorder is defined by the presence of at least 1 manic or hypomanic episode. Depressive symptoms may not initially be apparent, but almost always occur eventually. Standard of care is with a mood stabilizer such as lithium, valproic acid, or carbamazepine. This patient's symptoms are characteristic of a manic episode which is marked by at least 3 of the following criteria (4 if mood is only irritable): distractibility, irresponsibility, grandiosity, flight of ideas, psychomotor agitation, a decreased need for sleep, and pressured speech.

34 yo woman comes to office due to fatigue for last several mos. She becomes tired easily after walking short distances. She also has difficulty combing her hair due to probz holding her hands over her head for long periods. Pt reports anxiety, irritability, and unintentional weight loss of 8 lb over last 2 mos. She has no fever or loss of appetite. Pt doesn't use tobacco or alcohol. Her dad died of a stroke at age 54 and her mom has DM. On exam, pt is afebrile with pulse of 115/min. Cardiac exam reveals regular tachycardia with no murmur. Lungs are clear to auscultation. Pt's gait is normal, but she awkwardly drops into chair when asked to slowly sit. Pt appears to have decreased muscle mass in her shoulders, but her muscles are nontender. Deep tendon reflexes are normal. Most likely cause of this pt's symptoms?

*Thyroid disease* proximal muscle weakness = myopathy - can be via drugs, CT, endocrine, neuromuscular Hyperthyroid symptoms - anxiety, tachycardia, weight loss - fatigue = common via chronic sympathetic hyperstimulation Acute thyrotoxic myopathy - severe distal or proximal muscle weakness - usually without bulbar or respiratory muscle inv't - chronic hyperthyroid myopathy = proximal muscle weakness wks to months after onset of hyperthyroidism

28 yo woman develops tachycardia in postop recovery room. She was admitted for right femur fracture following motor vehicle accident and has open reduction & internal fixation of fracture. While in recovery area, she began experiencing nausea & vomiting, and became anxious and agitated. Pt has no known med probz. She drinks 1 or 2 glasses of wine on weekend. Her temp is 103 F, BP is 160/90 mmHg, pulse is 148/min, and respirations are 24/min. Pulse oximetry shows 98% on RA. On exam, pt is delirious and has fine tremor. Mild lid lag present. SHe has no muscle rigidity and deep-tendon-reflexes are 2+ in bilateral extremities. Lab results: Hct 30% Sodium 132 mEq/L Potassium 4.5 mEq/L Bicarbonate 24 mEq/L BUN 32 mg/dL Creatinine 1.2 mg/dL Glucose 120 mg/dL Creatinine kinase, serum 176 U/L Best nex step in management of this pt?

*Thyroid function tests & propranolol* Thyroid storm - life-threatening thyrotoxicosis - triggered by: surgery, trauma, infection in pts with undiagnosed or inadequately treated hyperthyroidism - *tachycardia, HT, cardiac arrhythmias/a fib, fever up to 104-106 F* - anxiety, altered mentation, seizure, severe nausea, vomiting, hepatic dysfunction, tremor, lid lag, goiter

32 yo woman with 13 yr ho T1DM comes to hospital with palpitations and SOB. She has had these symptoms for last several mos, but they have worsened recently. Pt also has experienced anxiety, poor sleep, and unintentional wt loss that she attributes to drinking too much coffee at work. She drinks 2 8-oz cups of coffee daily and glass of red wine on weekends. Pt's diabetes managed with regimen of insulin glargine and insulin lispro, and she has had good glycemic control in past. However, in last few mos she has had rising home glucose readings and has required increasing insulin doses to maintain control. Pt has also had irregular menstrual periods, for which she was started on combo estrogen-progesterone oral contraceptive pill 2 mos ago. CT scan of chest with contrast performed in ED shows no evidence of PE, and pt is admitted for additional eval. Following admission, she develops nausea and vomiting. Six hrs later, pt is found to be confused, diaphoretic, and agitated. Her temp is 102 F, BP is 145/65 mmHg, and pulse is 154/min. Chest is clear to auscultation. Exam of skin and abdomen unremarkable. Lab results: Serum chemistry Sodium 136 mEq/L Potassium 4 mEq/L Chloride 106 mEq/L Bicarbonate 22 mEq/L BUN 28 mg/dL Creatinine 1.2 mg/dL Calcium 9.6 mg/dL Glucose 320 mg/dL Liver function studies Albumin 3.7 g/dL Total bilirubin 2.3 mg/dL Direct bilirubin 1.3 mg/dL AST 62 U/L ALT 74 U/L Most likely diagnosis?

*Thyrotoxicosis crisis* Hyperthyroidism - palpitations + wt loss + irregular menses + impaired glycemic control Few hrs following her initial eval & hospitalization... - pyrexia, tachycardia, delirium, vomiting = *thyroid storm* = likely triggered by acute iodine load from undergoing CT scan with iodinated contrast agent - can also be triggered by sx, trauma, infection, or childbirth Thyroid storm... = cardiac arrhythmias, CHF, seizures, hypotension, shock - diagnosis based on clinical evaluation

This patient is suffering from an ST-elevation myocardial infarction (STEMI). What is the strongest predictor of long-term prognosis for these patients?

*Time to restoration of coronary blood flow following a myocardial infarction* - *Rapid re-perfusion, whether via fibrinolysis or percutaneous coronary intervention (PCI), decreases the severity of myocardial damage as well as limits the size of the infarct. Both of these benefits lead to improved morbidity and mortality.* - *PCI is the preferred re-perfusion strategy, with a 'door-to-balloon' time goal of 90 minutes.* - Subacute management of MI, after coronary blood flow has been restored: Pharmacologic therapies may include aspirin or other antiplatelet agents, ACE inhibitors or ARBs, beta blockers, and/or statins.

A 62-year-old man and his wife come to visit a family member in the hospital, when the man suddenly collapses in the parking lot. His wife calls for help, and a nearby internal medicine resident immediately assesses the man, finding no pulse. The wife explains that her husband had been feeling vague chest discomfort the previous day. Which of the following is the most critical determinant of survival for this patient?

*Time to rhythm analysis and defibrillation* Ventricular arrhythmia - the time to defibrillation is critically important in unstable ventricular rhythms, as it improves survival - leading cause of cardiac arrest in non-hospitalized individuals is ventricular arrhythmia - either ventricular tachycardia (VT) or ventricular fibrillation (VF).

23 yo woman comes to doc with nasal breathing, stuffy nose, and occasional dry cough for more than a year. Her symptoms fluctuate in intensity without any obvious inciting factors. She has no eye or ear symptoms, itching, wheezing, or skin rash. She reports no food allergies. Pt tried OTC oral loratadine without significant improvement. Her PMHx is unremarkable. On inspection, nasal mucosa appears boggy and erythematous. Lungs clear to auscultation. Best next step in management of this pt?

*Topical intranasal glucocorticoids* Nonallergic rhinitis (NAR) / Vasomotor rhinitis - predominant nasal congestions or stuffiness - postnasal drip (dry cough) - no specific identifiable triggers - clinical diagnosis - best managed with either topical intranasal antihistamine spray (azelastine, olopatadine) or intranasal glucocorticoids

A 26-year-old G2P1001 presents for a routine obstetric visit at 13 weeks gestation complaining of increased vaginal discharge for one week. The discharge is "white and clumpy" with accompanying severe vulvar itching. The patient has no other complaints. She has no past medical history, and her previous delivery was a spontaneous vaginal delivery. She states that she has been using panty liners for the last month or so due to mild stress incontinence, and she has no new sexual partners. At this visit, her temperature is 98.7 °F (35.1 °C), blood pressure is 119/74 mmHg, pulse is 82/min, and respirations are 14/min. The patient has no evidence of oral thrush. On pelvic exam, the vulva is erythematous with multiple excoriations. Which of the following is the best treatment for this patient?

*Topical miconazole* This pregnant patient most likely has candidial vaginitis which should be treated with topic miconazole. - In addition, vaginal pH is typically normal (4-4.5) in candida and elevated in the other two conditions. - *In early pregnancy, oral fluconazole (the first-line treatment for nonpregnant patients) is contraindicated due to an association with miscarriage and craniofacial abnormalities. Topical miconazole should be used instead.*

75 yo man with known T2DM brought to ED with weakness and blurred vision. Over last several days, he has had hacking cough, sore throat, and poor appetite. His temp is 100.5 F, BP is 100/60 mmHg, pulse is 112/min and regular, and respirations are 18/min. Mucous membranes are dry. Lungs clear to auscultation. No heart murmur. Abdomen soft, nontender, and nondistended. No skin rashes present. Mild generalized weakness on neuro exam. Lab results: Serum chemistry - Sodium 134 mEq/L - Potassium 5.9 mEq/L - Chloride 101 mEq/L - Bicarbonate 22 mEq/L - BUN 52 mg/dL - Creatinine 1.5 mg/dL - Calcium 9.1 mg/dL - Glucose 1070 mg/dL Liver function studies: AST 17 U/L ALT 15 U/L Most likely present in this pt?

*Total body potassium depletion* Hyperosmolar hyperglycemic state (HHS) - most likely ppted by recent URI - severe hyperglycemia (>1000 mg/dL) - increased serum osmolality (>320 mOsm/kg) - little or no ketonemia or acidosis - most pts = pH > 7.3 - serum bicarb > 20 mEq/L - neurologic symptoms = focal signs, lethargy, blurry vision, obtundation = via severe hyperglycemia & elevated serum osmolality - aggressive insulin therapy for HHS can lower serum potassium levels further = severe hypokalemia

A 32-year-old female presents to her gynecologist noting that she missed her menstrual period two weeks ago. She denies pelvic pain, but notes that she has a very regular cycle and typically does not miss periods. She is concerned and states she would like to have an abortion if she is pregnant. Her blood pressure is 135/81 mmHg, heart rate is 82 beats per minute, and she is afebrile. A pelvic ultrasound is performed, which shows no sign of a gestational sac in the uterus, and her beta-human chorionic gonadotropin (HCG) level is found to be 2600 mIU/mL. She tested negative for Rh and the father's Rh status is unknown. Her liver and renal function tests are normal. Which of the following is the next best step?

*Transvaginal US* Ectopic pregnancy (EP) that is less than 6 weeks old (and therefore less than 3 cm), with a risk for Rh sensitization. - *With a negative transabdominal pelvic ultrasound and a beta-HCG level over 2000 mIU/mL (the discriminatory level at which a uterine pregnancy should be visualized by pelvic ultrasound), this patient should undergo a transvaginal ultrasound to confirm an ectopic pregnancy before being treated with methotrexate and RhoGAM for a medical abortion* - Treatment with methotrexate should be used in patients with a beta-HCG <5000 mIU/mL and a fetus size of <3 cm after confirmation with a transvaginal ultrasound - In addition, patients who are Rh-negative, regardless of the Rh-status of the father, should be treated with RhoGAM.

57 yo homeless man with known HIV infections comes to ED complaining of SOB & nonproductive cough over past wk. He has ho T2DM with poor follow-up and med compliance. His last CD4 count was 120 cells/uL 3 mos ago. His temp is 102 F, pulse is 110/min, respirations are 24/min, and BP is 120/80 mmHg. Oxygen sat is 86% on RA. Lung exam notable for diffuse rales bilaterally. Abdomen soft and nontender. Lab results: Creatinine 1.2 mg/dL Potassium 4.2 mEq/L Chest x-ray shows bilateral patchy opacities in lower lungs. Pt treated with azithromycin, ceftriaxone, trimethoprim-sulfamethoxazole, prednisone, and albuterol. He is also given insulin before meals for his DM. On third day of hospitalization, his serum potassium is 5.9 mEq/L and his creatinine is 1.5 mg/dL. What med is most likely responsible for these lab changes?

*Trimethoprim* - can cause hyperkalemia by blocking epithelial sodium channel in collecting tubule = occurs more often in HIV_infected pts who are treated with high doses - requires serial monitoring of potassium to avoid serious complications - also competitively inhibits renal tubular creatinine secretion = artificial increase in serum creatinine (GFR unchanged tho) TMP-SMX - used for treating uncomplicated UTIs (gram-negative rods), pneumocystis jirovecii pneumonia, community-acquired MRSA skin infections, nocardiosis Meds that can cause hyperkalemia - nonselective beta blockers, ACEIs, ARBs, K spring diuretics, digitalis, cyclosporine, heparin, NSAIDs, succinylcholine

54 yo man comes to clinic with fever, exertional dyspnea, and nonproductive cough for one wk. He was diagnosed with HIV infection 3 yrs ago but has been asymptomatic since. He has not been taking his med consistently. His temp is 102 F, BP is 120/80 mmHg, pulse is 100/min, & respirations are 28/min. Pt's pulse oximetry shows 80% on room air. With use of 100% nonrebreather mask, his oxygen sat increases to 92%. Lab results: CBC Hgb 9.6 g/dL Platelets 120,000/mm^3 Leukocytes 8,000/mm^3 (no bands) Arterial blood gases on room air pH 7.45 PaO2 54 mmHg PaCO2 44 mmHg Chest x-ray shows diffuse bilateral interstitial infiltrates. His CD4 count is 190/mm^3 and lactate dehydrogenase is 400 U/L. Most appropriate next step in management of this pt?

*Trimethoprim-sulfamethoxazole and corticosteroids* - corticosteroids have been shown to decrease mortality in cases of severe PCP (possibly by reducing inflammation due to dying organisms) - indications for steroid use = partial pressure of oxygen (PaO2) <70 mmHg or alveolar-arterial (A-a) gradient >35 mmHg on room air PCP - via fungal organism - AIDS-defining illness - when CD4 < 200/mm^3 - pts with HIV + dry cough + exertional dyspnea + fever - chest x-ray = bilateral interstitial infiltrates - hypoxia out of proportion to radiographic findings - serum LDH elevated - diagnosis confirmed by demonstration of organism in sputum or bronchoalveolar lavage aspirate

An obese, 32-year-old G3P2A0L2 woman presents to her obstetrician for follow-up at week 26 of her pregnancy. She previously had a random plasma glucose during her pregnancy of 140 mg/dL. One hour after a 75 g oral glucose load, the patient's plasma glucose level is 200 mg/dL. Which of the following is a potential complication of her diagnosis?

*Type II diabetes after completion of her pregnancy* Gestational diabetes (GDM) - increases the risk of developing type 2 diabetes (T2DM) after completion of the pregnancy. - women diagnosed with GDM have insulin secretory defects from their pancreatic beta cells before and after pregnancy - While hPL may partially contribute to insulin resistance, more and more evidence suggests that GDM represents chronic beta-cell dysfunction that has been exacerbated due to pregnancy and its increased insulin requirements, and continues to worsen post-pregnancy with re-identification as T2DM. - Diagnosis increases during pregnancy because most women never get oral glucose tests until they establish routine prenatal care.

A 71-year-old female is brought to the emergency department by EMS following a syncopal episode. Earlier in the day, the patient had multiple bowel movements that filled the toilet bowl with copious amounts of bright red blood. Minutes later, she felt dizzy and lightheaded and collapsed into her daughter's arms in the kitchen at which point her daughter called 911. The patient has a medical history of diabetes mellitus and hypertension. Upon arrival in the ED, the patient is alert and oriented and complaining that she feels fine and wants to go home. Vitals signs are T 36.8, HR 119, BP 100/70, RR 18, SaO2 97%. The patient is stable and there is no evidence of acute bleed at the time of presentation. Two large bore IVs are placed and 1L normal saline is given. Which of the following is the most appropriate next step in the treatment of this patient?

*Type and cross* GI bleed, as evidenced by her hematochezia, or bright red blood per rectum. - *A patient presenting with a GI bleed should undergo a type and cross to allow for prompt blood transfusion if it becomes necessary.* The initial management of a patient with a GI bleed is as follows: 1. Assess severity: tachycardia 10% volume loss, orthostatic hypotension 20% volume loss, shock 30% volume loss. 2. Resuscitation: place 2 large-bore IV lines and give NS or LR 3. Transfuse: type and cross, transfuse 2-8 units 4. Reverse coagulopathy: FFP and vitamin K to normalize PT 5. Triage: consider ICU if unstable Once this initial management has been completed, the patient may undergo diagnostic studies such as esophagogastroduodenoscopy (EGD) or colonoscopy. EGD is indicated for an upper GI bleed (characterized by hematemesis and melena) while colonoscopy is indicated for a lower GI bleed (characterized by hematochezia). Therefore, a colonoscopy would be indicated in this patient.

A 32-year-old woman presents to her primary care physician for a general wellness appointment. The patient has no complaints currently and just wants to be sure that she is in good health. The patient has a past medical history of asthma, hypertension, and anxiety. Her current medications include albuterol, fluticasone, hydrochlorothiazide, lisinopril, and fexofenadine. Her temperature is 99.5°F (37.5°C), blood pressure is 165/95 mmHg, pulse is 70/min, respirations are 15/min, and oxygen saturation is 98% on room air. On exam, you note a healthy young woman with a lean habitus. Cardiac exam reveals a S1 and S2 heart sound with a normal rate. Pulmonary exam is clear to auscultation bilaterally with good air movement. Abdominal exam reveals a bruit, normoactive bowel sounds, and an audible borborygmus. Neurological exam reveals cranial nerves II-XII as grossly intact with normal strength and reflexes in the upper and lower extremities. Which of the following is the best next step in management?

*Ultrasound* In renal artery ultrasound, one is looking for several things = assessment of the diameter of the renal artery = whether there is an specific areas of renal artery narrowing that may be causing the problem = DOPPLER flow of blood through the renal arteries - In a stenotic artery, the FLOW VELOCITY of blood is increased as the diameter of the lumen is decreased, so increased FLOW VELOCITY is a marker of renal artery stenosis Young and lean + abdominal bruit + hypertension that is refractory to multiple medications = *renal artery stenosis* - *best initial step in management is to confirm the diagnosis with an ultrasound* - can occur in older patients due to atherosclerosis or in younger patients due to fibromuscular dysplasia - present with abnormally high blood pressure that does not respond to multiple medications - physical exam = abdominal bruit = suggests turbulent flow through stenotic renal blood vessels - best initial test is an ultrasound - definitive therapy is angioplasty and stenting.

A 62-year-old male with a past medical history of back pain, headaches, and bipolar disorder presents to his primary care doctor with wrist pain. He reports that his pain has become significantly worse since he has been employed as a transportation assistant at a local hospital. He brings along a form for workers compensation, which he would like the physician to sign. He reports that he has already gone to a neurologist, a geriatrician, and to the emergency room in an attempt to have the form signed. Which of the following is most likely true about this patient?

*Unlikely to return for care if the form is signed* Malingering - Patients who are malingering are likely to stop presenting to care once their *secondary* gain is fulfilled. - conscious faking of medical illnesses for a secondary purpose - likely to stop complaining when their secondary gain is fulfilled, in contrast to patients with factitious disorder

A 29-year-old female presents to general medical clinic with dysphagia. Her symptoms began several months ago. She has trouble swallowing solids and liquids though liquids seem to make her choke and sputter the most; therefore, she has been unable to eat and has thus experienced significant weight loss. She has no significant past medical history apart from a 20-pack-year smoking history. She denies any recent travel. Vital signs are stable. Physical examination is within normal limits. A barium esophagram shows what seems to be achalasia. What is the next best step in management?

*Upper endoscopy* *In diagnosing achalasia, one must first rule out malignancy with an endoscopic evaluation.* After a barium swallow and esophageal manometry suggest achalasia, then further management and definitive treatment can be started. - motor disorder of the distal esophagus resulting from degeneration of Aurbach's plexus - most common motility disorder and is often found in patients under 50 - lower esophageal sphincter fails to relax during swallowing. As a consequence, natural peristalsis is disrupted and the patient experiences dysphagia to solids and liquids, with liquids often being most problematic. - A barium esophagram is helpful in making the diagnosis and should reveal the classic bird's beak tapering at the esophageal sphincter. This is the first step in management. Subsequently, diagnosis may be confirmed with esophageal manometry. Once endoscopy is completed, palliative treatment may begin. - Treatment includes medical management consisting of calcium channel blockers, botulinum toxin injections, and surgical therapy may include endoscopic balloon dilation of the lower esophageal sphincter or a more invasive option, myotomy with fundoplication.

A 56-year-old female with a history of diabetes, peptic ulcer disease, and hypertension presents to the emergency department extremely concerned because she had a bowel movement 1 hour ago that was accompanied by large amounts of bright red blood. She states she has also been having abdominal pain and lightheadedness ever since the bloody bowel movement. On exam, her vitals are: T: 37 deg C, HR: 110 bpm, BP: 82/55, RR: 10, SaO2: 100%. IV access is obtained and volume resuscitation is initiated. What should be the first diagnostic step following volume resuscitation?

*Upper endoscopy* *In this question stem, the patient has hemodynamically instability, hematochezia, and HISTORY OF PEPTIC ULCER DISEASE. Thus, suspicion for upper GI source is high, and upper endoscopy should be performed first after stabilization of the patient. * Bright red blood per rectum (BRBPR) with hemodynamic compromise - *The first diagnostic step is upper endoscopy to rule out a brisk upper GI bleed* Lower GI bleeding is classified as GI bleeding distal to the ligament of Treitz - It often presents as hematochezia - bright red blood per rectum as compared to upper GI bleeds which typically present with hematemesis or melena - There are multiple causes of a lower GI bleed including hemorrhoids, diverticulosis, angiodysplasia, malignancy, inflammatory bowel disease, and infection conditions - *Additionally, a very brisk upper GI bleed can present as hematochezia and a lower GI bleed* - *A brisk upper GI bleed is often caused by a peptic ulcer that has invaded an artery such as the gastroduodenal artery. This will often present with BRBPR and hemodynamic compromise.* - *Any patient with lower GI bleeding and hemodynamic compromise must first receive an upper endoscopy to rule out a brisk upper GI bleed.* - Following upper GI endoscopy, further diagnostic procedure such as colonoscopy, Tc-TBC scan, and angiography can be performed *For hemodynamically unstable patients or patients with severe bleeding, resuscitation is the initial step. After the resuscitation has been performed and the patient is now hemodynamically stable, the first step is to rule out an upper GI source with upper endoscopy, before proceeding to colonoscopy.* *NOT ALL FRESH BLOOD IN RECTUM IS LOWER GI BLEEDING, as MASSIVE UPPER GI BLEED can also cause hematochezia, and especially if it is a massive bleed, it will lead to a hemodynamically unstable patient.*

78 yo woman brought to hospital for urinary incontinence. Pt has ho Alzheimer dementia and has been living in nursing home for past 3 yrs. She requires assistance for most of her daily activities. Pt can indicate to nursing home staff when she needs to use bathroom. She has been less active for past wk and sleeps most of time. She also has developed urinary incontinence, which she has never had before. Her temp is 98 F, BP is 130/70 mmHg, and pulse is 76/min. On exam, pt is somnolent but responsive to voice. She is disoriented but has no focal neuro deficits. Abdomen soft and nontender. GU exam shows thin and pale vaginal mucosa with ill-defined minora. Best next step in managing this pt's urinary incontinence?

*Urinalysis and culture* Acute urinary incontinence: *Older adults often lack typical signs or symptoms of UTI --> urinalysis with culture should be obtained routinely*

60 yo man comes to ED due to 2 wks of progressive exertional dyspnea & fatigue. He has difficulty falling asleep and can hardly walk to bathroom without becoming SOB. Pt has no chest pain, syncope, cough, or exertional dyspnea. 2 mos ago, he was hospitalized with anterior wall MI but was no revascularized due to late presentation. He was discharged home on aspirin, clopidogrel, metoprolol, lisinopril, and atorvastatin. His temp is 97.8 F, BP is 100/67 mmHg, and pulse is 67/min and regular. Exam shows bilateral crackles in lower lung fields. Apical impulse displaced to left. Faint systolic murmur heard over apex. ECG shows normal sinus rhythm and deep Q waves in leads I, aVL, and V2-V5 with 2-mm ST-segment elevation; ECG tracings unchanged compared to those at discharge 2 mos ago. Most likely underlying cause of this pt's symptoms?

*Ventricular aneurysm* Progressive decompensated CHF due to left ventricular aneurysm - post-MI complications = 5 days - 3 mos post-MI - thin & scarred or fibrotic myocardium in remodeled areas affected by ST-segment elevation or transmural MI - VAs present with ECG findings of persistent ST-segment elevation after recent MI & deep Q waves in same leads - Large VAs --> progressive left ventricle enlargement = HF, refractory angina, ventricular arrhythmias, mural thrombus with systemic arterial embolization, or mitral annular dilation with mitral regurgitation = murmur - diagnosis confirmed by echocardiography = thinned, dyskinetic LV portion in area of prior MI

35 yo woman comes to ED with 3 days of burning substernal chest pain & "excruciating" pain with swallowing. She could not eat or drink anything today due to severe pain. She has not had nausea, vomiting, difficulty initiating swallowing, or sensation of food getting stuck in her throat. Pt has ho HIV, asthma, uterine fibroids. Her current meds include atazanavir-ritonavir, tenofovir, emtricitabine, trimethoprim-sulfamethoxazole, azithromycin, and albuterol. SHe has ho med noncompliance but says that she is currently taking all her med as prescribed. Her most recent CD4 count was 30/mm^3. Her temp is 100.2 F, BP is 108/59 mmHg, and pulse is 103/min. Pt is thin and somewhat pale. She has significant dental caries, but oral exam is otherwise normal. Cardiopulm exam is unremarkable. Abdomen soft & nontender. Empiric therapy initiated, and upper GI endoscopy scheduled. Mos likely cause of this pt's symptoms?

*Viral esophagitis* - most commonly implicated = HSV CMV - diagnosis via upper GI endoscopy with biopsy HSV - circular or ovoid vesicular and ulcerated lesions CMV - large, linear distal esophageal ulcers Infectious esophagitis common in advanced HIV, especially with CD4 count < 100/mm^3 --> most common cause = Candida (>60%), but viral esophagitis more likely than candidal esophagitis in pts with... - *severe odynophagia* - *no dysphagia (difficulty swallowing)* - *no thrush*

45 yo man comes to office due to 2 mo ho painless, nonpruritic, purple lesions on his legs. He has also had progressive fatigue, weakness, and fleeting joint pains in his knees & elbows for past several mos. Pt has had no fever, wt loss, diarrhea, or abdominal pain. Temp is 98 F, BP is 147/91 mmHg, & pulse is 84/min. Cardiac exam reveals no murmurs or additional sounds. There are multiple purpuric, palpable lesions on both lower extremities that don't blanch with pressure. Lab results: Hct 39% Platelets 160,000/mm^3 Leukocytes 3,000/mm^3 Serum creatinine 1.9 mg/dL AST 78 U/L ALT 99 U/L C3 low-normal C4 very low Rheumatoid factor positive Anti-nuclear Ab negative HIV-1 Ab negative Urinalysis shows 3+ blood, 3+ protein, & few dysmorphic RBCs. What would be most helpful in establishing pt's diagnosis?

*Viral hepatitis etiology* Mixed cryoglobulinemia syndrome (MCS) - via immune complex deposition in small to medium vessels = endothelial injury & end-organ damage - fetigue - nonblanching, palplable purpura, arthralgias, renal disease, peripheral neuropathies - HT often - liver inv't common

21 yo man brought to ED by his fam after a witnessed generalized tonic-clonic seizure. His sis says that the pt has never had previous seizure but has had progressive headaches, fever, nasal congestion, and rhinorrhea over last 3 wks. He has no chronic med probz and doesn't use alcohol, tobacco, or illicit drugs. Temp is 102 F, BP is 130/85 mmHg, and pulse is 96/min. Pt is drowsy and postictal. CT scan of head with IV contrast reveals 3-cm ring-enhancing lesion in left frontal lobe. There are air-fluid levels & mucosal edema in paranasal sinuses. Rapid HIV negative. Tissue biopsy of brain lesion is most likely to demonstrate....

*Viridans streptococci* Single brain abscess - via direct extension from adjacent tissue infection (otitis media, sinusitis, dental infection) - 2 most commonly isolated organisms = viridans streptococci, staph aureus - headaches = severe, unilateral, resistant to analgesics - if diagnosis delayed, focal neuro deficits or *seizures* may emerge - brain CT or MRI --> may reveal cerebritis initially --> within 1-2 wks = consolidates to ring-enhancing lesion with central necrosis - diagnosis requires CT-guided aspiration or surgical biopsy to obtain tissue for Gram stain and culture - most pts treated with empiric, IV abx (metronidazole, ceftriaxone, and vanc) & aspiration of lesion

A 33-year-old woman comes to the emergency department after noting persistent weakness that has progressed over the last few days. Three days ago, the patient noticed her lower legs felt "rubbery." Now, she has significant tingling throughout her legs and difficulty climbing stairs. Her medical history is unremarkable other than a mild, transient gastroenteritis several weeks ago. Physical exam reveals +1 patellar reflexes and 2+ triceps reflexes bilaterally. Strength is 1/5 in knee flexion/extension, and 3/5 in hip flexion/extension. The patient is hospitalized and supportive treatment is initiated. What is the best method for monitoring respiratory function in this patient?

*Vital capacity* Guillain-Barre Syndrome (GBS) - *Since respiratory failure is common in GBS patients, vigilance is essential. Respiratory monitoring is best done with bedside vital capacity.* - Clinical diagnosis can be confirmed with a lumbar puncture, which will show increased protein but normal white blood cell count (albumino-cytologic dissociation). - plasma exchange therapy can improve outcomes and IV Ig can hasten recovery in patients with GBS - up to 1/3 of patients develop severe distress requiring mechanical ventilation

Bowing of legs. During the well exam of an otherwise healthy two-year-old African-American male, you note this deformity. Lab results are as follows: Serum Calcium = 9.1 mg/dL (N=8.4-10.2), Phosphorus = 1.1 mg/dL (N=3.8-6.2), Alkaline Phosphatase = 462 U/L (N=68-217), PTH = 23 pg/mL (N=10-55), 1,25-dihydroxyvitamin D = 22 pmol/L (N=15-30). What is the most likely cause of his deformity?

*Vitamin D Resistant Rickets* (aka Familial Hypophospatemic Rickets). - the most common form of rickets - often caused by an X-linked dominant mutation in the PHEX gene = inability of the renal tubules to absorb phosphate (GFR remains normal) = inadequate mineralization of bone - commonly presents as bilateral tibial bowing between the ages of 6 months and 2 years Key lab values for the diagnosis are: - low serum phosphorous - elevated alkaline phosphatase - serum calcium is usually normal or low normal High dose vitamin D3 and corrective surgery is currently the standard of care for hypophosphatemic rickets

64 yo man undergoes emergency colonic resection for extensive ischemic colitis. He lives alone and consumes large amount of alcohol. His fam hx is noncontributory. Pt is extubated on 4th postop day. After extubation, he has episodes of confusion and agitation treated with lorazepam and haloperidol. He is also receiving piperacillin and tazobactam. Pt has had nothing by mouth since surgery. On postop day 7, nurse notices bleeding from venipuncture site. His temp is 98 F, BP is 121/76 mmHg, pulse is 80/min, & respirations are 16/min. Lab results: Hgb 11.5 g/dL MCV 88 fL Platelets 160,000/uL Leukocytes 7,500 - Segmented neutrophils 68% - Bands 1% - Eosinophils 1% - Lymphocytes 24% - Monocytes 6% Prothrombin time 24 sec (INR 2.2) Partial thromboplastin time 44 sec (normal 25-40) Most likely cause of this pt's current condition?

*Vitamin K deficiency* - acquired bleeding disorder - via inadequate dietary intake, intestinal malabsorption, or hepatocellular dz - *acutely ill person with underlying dz can become vit K def in as little as 7-10 days* This pt... - received no enteral nutrition postoperatively & was given broad-spectrum antibiotic --> both natural sources of vitamin K compromised - likely has alcoholic liver dz --> further limits vit K stores - increased PT, followed by increased PTT Admin of vit K rapidly replenishes stores in 8-10 hours - FFP may be used for management of acute hemorrhage in interim

A 42-year-old male presents to his primary care physician at the urging of his wife who thinks he is becoming increasingly "hard of hearing." He denies any sudden change in hearing, and says that he feels like it has been a gradual process and seems to be worse in his right ear. He denies headaches, vertigo, or imbalance. His past medical history is significant for hypertension, which is well controlled on a thiazide. As part of the physical exam, Weber and Rinne tests are performed. If the patient had otosclerosis of the right ear, which of the following would be the expected result of these two tests?

*Weber lateralizing to right with bone conduction > air conduction on right* Otosclerosis - conductive hearing loss presenting as gradual hearing loss in adulthood - would manifest with a Weber lateralizing to the diseased side with bone conduction greater than air conduction - caused by fusion of otic bones, resulting in conductive hearing loss Conduction deafness = Rinne: BC > AC (in damaged ear) = Weber: lateralization to damaged ear

24 yo woman comes to office after 6 mos of burning pain in her rt hand. Pain is mild and intermittent but worse at nt, and is felt mainly over first, second, and third digits of hand. Shaking hands or letting them hang down briefly relieves the pain. No associated neck for forearm pain. Pt attempted treatment with OTC analgesics and topical "arthritis cream" without relief. Her med hx is notable for ESRD, for which pt underwent living-donor transplant 2 yrs ago. SHe does not use tobacco, alcohol, or illicit drugs. VItals normal. On exam, there is no thenar or hypothenar atrophy, & thumb opposition is normal. Percussion over volar aspect of wrist reproduces pain. Most appropriate next step in management of this pt?

*Wrist splinting* Carpal tunnel syndrome - reproduction of pain on percussion over median nerve at wrist = Tinel sign - holding wrist in full flexion with dorsum of hands pressed together = Phalen test - most pts respond to conservative measures, including nocturnal wrist splinting --> holds wrist in neutral position & prevents excessive flexion during sleep

50 yo woman comes to office due to right shoulder pain that radiates to her hand. Pt was diagnosed with RA 10 yrs ago and usually has pain in her joints. However, she believes that current pain is not due to her arthritis. Pt has had cough for several mos and feels more tired than usual. Her meds include methotrexate and naproxen for RA. She has 25 pck yr smoking hx and drinks 1 or 2 beers on weekends. VItals are within normal limits. PE shows bilateral hand joint deformities with mild swelling & tenderness. Right pupil is constricted and there is drooping of eyelid. ROM over rt shoulder is normal. Serum creatinine level & liver function tests are normal. ESR is 55 mm/hr and C-reactive protein is 35 mg/L (normal <8 mg/L). Best next step in management of this pt?

*X-ray of chest* shoulder pain + Horner syndrome (ipsilateral ptosis & miosis) + smoking hx = *superior pulmonary sulcus (SPS) tumor / Pancoast tumor* - malignant lung neoplasm = most commonly squamous cell or adenocarcinoma - *tumors located in SPS often present with shoulder pain as initial symptoms due to invasion of brachial plexus or adjacent structures* - pain may also radiate up head & neck or down ipsilateral arm in ulnar nerve distribution - weakness and atrophy of medial hand muscles may occur - Horner syndrome common = via tumor invasion of paravertebral sympathetic chain & inferior cervical ganglion - initial eval = chest imaging = to evaluate lung mass

72 yo man presents to clinic for routine health maintenance. He has been feeling well recently, and has no specific complaints. His med hx is significant for HT & hyperlipidemia. His current meds are HCTZ, felodipine, and hyperlipidemia. He has 40 pck yr smoking hx, but quit 5 yrs ago. He drinks alcohol occasionally. His bro was recently diagnosed with bladder cancer. PE reveals normal findings. Pt wants to know whether he should be screened for bladder cancer. Most appropriate statement for this pt?

*You don't require any screening for bladder cancer* Bladder cancer - 2nd most common urologic cancer - more common in men - average age = 65 yo USPSTF: recommends against screening for bladder cancer via relative low incidence & poor positive predictive value of current screening tests

A 24-year-old male is brought to the emergency room after fainting while lifting weights. Several needles are found in his belongings, along with an unmarked, empty vial. His blood pressure is 103/68 mmHg, heart rate is 112 beats per minute, and temperature is 98.5 degrees Fahrenheit. On exam, he is noted to have extremely large biceps, very defined abdominal muscles, and small testicles. After hemodynamic stabilization, which of the following tests is most likely to reveal the cause of this patient's hypotension?

*abdominal CT* Hypotension due to anabolic androgenic steroid intoxication, which can be caused by *rupture of a hepatic adenoma* Testosterone and anabolic androgenic steroid toxicity - associated with masculinization in females, testicular atrophy in males, premature closure of epiphyseal plates in adolescents, worsening lipid profile, and hepatic adenomas - *high testosterone exposure and acute hypotension --> should be evaluated for a ruptured hepatic adenoma, which is best characterized on abdominal ultrasound* Patients with questionable hemodynamic instability should undergo an abdominal CT, which is also sensitive for hepatic adenomas. Of note, most hepatic adenomas occur in women aged 20-40 who are treated with oral contraceptives, as estrogen likewise increases the risk of hepatic adenomas. The adenoma is characterized by normal hepatocytes in a disorganized formation lacking classical lobular architecture.

A 63-year-old man with a long history of constipation presents with steady left lower quadrant pain. Physical examination reveals a low grade fever, mild abdominal distention, and left lower quadrant tenderness. Stool guaiac is negative. An absolute neutrophilic leukocytosis and a shift to the left are noted on the CBC. He is placed on broad spectrum oral antibiotics, bowel rest, and pentazocine for pain. His symptoms do not seem to improve over the 72 hours and he is started on IV piperacillin/tazobactam. After 24 hours there is still no improvement and it is decided to proceed to sigmoid colectomy with primary resection and anastamosis. The procedure is uncomplicated and he is discharged four days later. Five days following discharge, he presents to the emergency room with a temperature of 101.5 F. Which of the following is the most likely reason for this patient's fever?

*abdominal abscess* A fever on post-operative day 9 is most likely caused by an abdominal abscess Fever - common in surgical patients, though only approximately 40% of fever episodes are caused by infection Atelectasis = post-op day 1. Urinary tract infection = post-op day 3. Surgical site infections = post-op days 5-7. Deep vein thromboses = post-op days 5-7.

Woman with a history of hypertension and painful antepartum hemorrhage with a normal ultrasound is most likely experiencing

*abruptio placentae* - can be classified as a revealed or concealed abruption - 78% of patients with abruptio placentae presented with vaginal bleeding, 66% with uterine or back pain, 60% with fetal distress and only 17% with uterine contractions or hypertonus - management of abruptio placentae is primarily supportive and entails both aggressive hydration and monitoring of maternal and fetal well-being

A 62-year-old patient presents with severe dysphagia, odynophagia, and difficulty speaking. If this patient is diagnosed with a retropharyngeal abscess, which of the following is a potential sequelae of this condition?

*acute necrotizing mediastinitis* The most dreaded complication of a retropharyngeal abscess involves spread of the infection into the mediastinum. This can result in acute necrotizing mediastinitis. - the retropharyngeal space is continuous with the mediastinum --> communication results in the potential for infection of this space to spread into deeper areas (including the mediastinum) with disastrous consequences - emergent exploration and drainage is required when this diagnosis is made.

Patient's presentation is consistent with shingles, which is treated with

*acyclovir* Shingles - vesicular, dermatomal, and painful rash - caused by reactivation of the varicella zoster virus (VZV), and nearly all cases occur in patients with a prior history of chicken pox - treatment should ideally begin within 72 hours of the appearance of the rash and continue for 7 days, and the efficacy of initiating treatment after the rash has crusted over is unknown

A 68-year-old woman is brought to the emergency room by her husband following confusion and lethargy of a week's duration. Two weeks ago she was diagnosed with small cell cancer of the lung but has not yet begun any chemotherapy regimens. She has a 30-pack year history of smoking. She does not take medications. Physical examination reveals decreased deep tendon reflexes symmetrically. Jugular venous pressure is normal, mucous membranes are moist, and no edema is present in the patient's extremities. CT scan of the brain reveals no abnormalities. Laboratory studies reveal the following: Sodium: 112 mEq/L Potassium: 4.0 mEq/L Chloride: 95 mEq/L Bicarbonate: 24 mEq/L BUN: 6 mg/dL Creatinine: 0.65 mg/dL Serum osmolality is 220 mOsm/kg and urine osmolality is 400 mOsm/kg. Which of the following is the most appropriate next step in management of this patient:

*administer hypertonic saline* Neurological symptoms and low sodium in the setting of recent small cell lung cancer diagnosis consistent with hyponatremia due to syndrome of inappropriate anti-diuretic hormone secretion (SIADH) that requires the administration of hypertonic saline. SIADH - common paraneoplastic syndrome of small cell lung cancer and is due to ectopic ADH production - patients present with euvolemia or mild hypervolemia and inappropriately elevated urine osmolality and normal urine sodium content - *treatment involves free water restriction and treatment of the underlying cause of the SIADH. Hypertonic saline is indicated if symptoms of hyponatremia (such as confusion and lethargy in this patient) are present or if sodium fails to respond to free water restriction* - Loop diuretics, salt tabs, vasopressin receptor antagonists, and demeclocycline may be used in the treatment of chronic SIADH refractive to fluid restriction. SIADH is a diagnosis of exclusion and should be suspected when hyponatremia is accompanied by urine that is hyperosmolar compared with the plasma. This situation implies the presence of a low plasma osmolality with an inappropriately high urine osmolality, although the urine osmolality does not necessarily have to exceed the normal range.

Recognize *whipple's disease*, in which the treatment is...

*antibiotics* Whipple's Disease - systemic illness consisting of arthralgia, weight loss, fever, diarrhea, and abdominal pain - diagnosis is made by detecting PAS-positive material in the lamina propria of the small intestine on biopsy and treatment is with antibiotics - infection with the gram positive bacterium Tropheryma whipplei may be detected by PAS positive foamy macrophages in the intestinal lamina propria and mesenteric lymph nodes - while Whipple's disease primarily affects the gastrointestinal tract - resulting in a malabsorptive diarrhea - it may affect almost any organ - there are a number of symptoms including cardiac manifestations, arthralgias and neurologic symptoms (including the brain and eye) - the classic clinical presentation will be an older man with weight loss, diarrhea, joint pain, and arthritis - further, the disease is more common in farmers and those exposed to soil and animals - *treatment involves antibiotics: penicillin, ampicillin, tetracycline, doxycycline, hydroxychloroquine, or co-tirmoxazole for several years* - If not treated, the disease may be fatal. *Image depicts the biopsy of a patient with Whipple's disease. Note the PAS positive foamy macrophages.*

A 17-year-old previously healthy, athletic male suddenly falls unconscious while playing soccer. His athletic trainer comes to his aid and notes that he is pulseless. He begins performing CPR on the patient until the ambulance arrives and pronounces the teenager dead. What is the mechanism behind the most likely cause of death?

*arrhythmia* This case of sudden death is most likely caused by a ventricular arrhythmia that arises from hypertrophic cardiomyopathy. - defined as a left ventricular or right ventricular hypertrophy disproportionate to the hemodynamic load - autosomal dominant mutation in cardiac sarcomere genes that results in myocardial fiber disarray with hypertrophy --> results in a subaortic outflow obstruction, mitral regurgitation, diastolic dysfunction, ischemia, syncopy, and arrhythmia

A 37-year-old woman with a history of glomerulonephritis secondary to systemic lupus erythematosis presents with worsening malaise. She states that she is currently on medication to "suppress her immune system," but she cannot remember the name. Her physical exam is unremarkable. Several blood tests are sent, of which an AST and ALT are markedly elevated. Which of the following drugs was this patient most likely taking?

*azathioprine* Hepatotoxicity as a side effect of azathioprine azathioprine - dose-related diarrhea, leukopenia, and hepatotoxicity

37-year-old man presents to general medical clinic with dysphagia. He notes that his symptoms began several weeks ago and have worsened over time. He now has trouble swallowing solids and liquids, though liquids have always given him the most trouble. He denies any other symptoms. He has no significant past medical history. Travel history reveals a recent trip to South America but no other travel outside the United States. Vital signs are stable. Physical examination is within normal limits. He has no palpable masses. What is the next step in management?

*barium esophagram* Achalasia, possibly due to Chagas disease. - barium esophagram is the next step in management and should precede endoscopy in patients with dysphagia and a broad differential diagnosis - motor disorder of the distal esophagus resulting from degeneration of Auerbach's plexus where lower esophageal sphincter fails to relax during swallowing - natural peristalsis is disrupted and the patient experiences dysphagia to solids and liquids, with liquids often being most problematic - most common motility disorder and is often found in patients under 50 - associated with Chagas disease, where the parasitic amastigotes destroy ganglion cells If the question says: -What is the best initial step in management = barium swallow -What is the diagnostic test of choice, most accurate test or test that would confirm the diagnosis = manometry -Treatment of choice: surgery, CCB or botulinum toxin if patient can't handle surgery

INR of 2.5-3.5 is recommended for patients with

*bileaflet mechanical aortic valve and atrial fibrillation* The American College of Chest Physicians recommendations for anticoagulation in patients with prosthetic heart valves are as follows: INR 2-3: bileaflet mechanical valve in aortic position *INR 2.5-3.5: mechanical valve in mitral position OR bileaflet mechanical aortic valve with atrial fibrillation* INR 2.5-3.5 + ASA: mechanical valve and history of systemic embolization Atrial fibrillation - most common cardiac arrhythmia. It impairs cardiac function and increases the risk of stroke - Rate control is the preferred management option in most patients Use of rapid anticoagulation - Rapid anticoagulation can be achieved either with unfractionated heparin or with low-molecular weight heparin (LMWH) - Heparin is initiated with an intravenous bolus of 80 U/kg bodyweight, and an infusion of 18 U/kg/h - The activated thromboplastin time should be 60-80s - An alternative to intravenous heparin is subcutaneous LMWH, which is prescribed in a mg/kg dose

A 31-year-old G6P6 woman with a history of fibroids gives birth to twins via vaginal delivery. Her pregnancy was uneventful, and she reported having good prenatal care with all of her children. Both placentas are delivered immediately after the birth, however the patient continues to bleed significantly over the next 8 hours. Total blood loss is estimated at 850 cc. Assuming the most common cause of this patient's condition, what is the most appropriate treatment?

*bimanual uterine massage* Postpartum hemorrhage caused by uterine atony in 90% of cases. - The first-line treatment for this condition is bimanual uterine massage. - defined by blood loss totaling >500 cc in the first 24 hours after a vaginal birth, or >1000 cc with a Caesarian section - vast majority of postpartum hemorrhage is caused by uterine atony, which is characterized by a boggy uterus that is not able to effectively compress the spiral arteries - Risk factors for uterine atony include a history of fibroids, multiparity, and multiple gestations - Bimanual uterine massage, along with administration of oxytocin and prostaglandins, can be used to stimulate uterine contraction, and surgical treatment can be used if medical management fails

In acromegaly, the most common cause of death is...

*cardiovascular dz* Acromegaly - excess of growth hormone - most commonly occurs in the setting of a pituitary adenoma - excess growth hormone in acromegaly results in significant organomegaly - *in the case of the cardiovascular system, patients can develop hypertension, left ventricular hypertrophy, asymmetric septal hypertrophy, diastolic heart failure, and even cardiomyopathy* - treatment of acromegaly involves resection of the pituitary adenoma when possible and medical management of growth hormone excess with somatostatin analogues

You would like to conduct a study investigating potential risk factors that predispose patients to develop cirrhosis. Using a registry of admitted patients over the last 10 years at your local hospital, you isolate all patients who have been diagnosed with cirrhosis. Subsequently, you contact this group of patients, asking them to complete a survey assessing their prior exposure to alcohol use, intravenous drug abuse, blood transfusions, personal history of cancer, and other medical comorbidities. An identical survey is given to an equal number of patients in the registry who do not carry a prior diagnosis of cirrhosis. Which of the following best describes the type of study you are attempting to conduct?

*case control study* Retrospective case-control study - start with a disease (the sick patients) and look for prior exposures to certain risk factors and calculate an odds ratio to analyze the data Case-control studies - observational studies (no treatment group) and are almost always retrospective in nature - answer the question; what are the odds that given exposure to a certain risk factor you will have the disease Odds ratio - typical calculated result from such investigations - ratio of the odds that a patient has a disease - what are the odds that I have a disease given exposure to a risk factor divided by the odds that I don't have a disease when exposed to a risk factor. An example is given below: The number of subjects with: Alcohol exposure + cirrhosis = A Alcohol exposure + no cirrhosis = B No alcohol exposure no cirrhosis = C No alcohol exposure + no cirrhosis = D The odds of having alcohol exposure in patients with cirrhosis is A/C The odds of having alcohol exposure in patients with no cirrhosis is B/D The ratio of these two odds or the odds ratio is (A/C)/(B/D) which simplifies to AD/BC.

This patient has scarlet fever, for which the treatment of choice is penicillin. However, given the patient's previous reaction to penicillin administration, he should be treated with

*cephalexin (first-gen cephalosporin)* Scarlet fever - caused by infection with Group A Streptococcus (GAS) strains that produce erythrogenic exotoxins - typically present with fever, an exudative pharyngitis, an erythematous sandpaper-like rash that classically starts on the neck and spreads to the trunk and face (with circumoral pallor), and strawberry tongue Untreated scarlet fever may progress to cause purulent sinusitis, otitis media, or retropharyngeal or peritonsillar abscesses. Post infectious sequelae include acute rheumatic fever and post-streptococcal glomerulonephritis. *Patients with severe penicillin allergies (i.e. anaphylaxis or respiratory symptoms) should be treated with erythromycin or clindamycin to avoid the 10% cross-reactivity that occurs between penicillin and cephalosporins.* Overall, I would say this patient's minor allergic reaction coupled with the lower cross reactivity would make this an appropriate clinical choice.

A 72-year-old male, former smoker (50 year pack history), with a history of poorly controlled diabetes and hypertension five years status post three-vessel CABG and 20 years status post cholecystectomy is brought to your clinic by his son, who states that his father has lost 18 lbs in the past year. The patient states that he no longer feels like eating. He endorses a dull, crampy pain in his epigastrium when he does eat. He denies alcohol use. His home medications include lisinopril, hydrochlorothiazide, metformin, and aspirin. Chest and abdominal non-contrast CT show no abnormalities, and upper endoscopy demonstrates mild gastritis. What is the most likely diagnosis?

*chronic mesenteric ischemia* = causing food aversion, leading to his significant weight loss. = also known as "abdominal angina" = caused by mesenteric atherosclerotic disease --> insufficient splanchnic blood flow immediately following meals - typically found in patients with a history of smoking, peripheral vascular disease, or coronary artery disease = often present with chronic weight loss, food aversion, and dull, crampy abdominal pain within the first hour after a meal - diagnosis can be made with duplex ultrasonography, or CT- or MR-angiography

A 27-year-old female presents to your office for a check up. The patient is currently on paroxetine, though she denies depressive symptoms for the last 6 months. She has had 2 episodes of major depression in the past. Both episodes culminated in a suicide attempt that required hospitalization. How do you proceed with this patient?

*continue paroxetine indefinitely* In remission on SSRI therapy - with a history of 2 (or greater) major depressive episodes, therapy should be continued indefinitely *According to the APA, the criteria for indefinite treatment is 2 episodes within few years or greater than 3 in a lifetime* - Such patients may be discouraged by the prospect of treatment that lasts "forever." For these patients, it is important to emphasize the long-term nature of the relationship between the clinician and patient, and that the need for maintenance treatment will be reevaluated periodically in light of the patient's progress in maintaining symptomatic and functional stability, and discontinuation may sometimes be attempted despite the guideline recommendations for indefinite therapy would consider the 2 suicide attempts to make this a more severe presentation that would raise concern for a third attempt. Considering the very high risks associated with a suicide attempt it may be best to keep the patient on SSRI's indefinitely. This may be a finer clinical discretion, however, it is always best to be on the safe side and to not discontinue a medication such as this.

Treatment for ITP

*corticosteroids* - also: rituximab, IV immunoglobulin

Prolonged infusion of sodium nitroprusside at high rates can lead to...

*cyanide toxicity* - especially in pts with CKD - typical findings = headache, confusion, arrhythmias, flushing, respiratory depression - neuro changes = headache, confusion, *hyperreflexia* via thiocyanate accumulation - treatment = sodium thiosulfate

A 40-year-old man is brought to the emergency department following a motor vehicle accident. He complains of severe pain and is found to have a comminuted pelvic fracture. Following physical examination and imaging studies, the physician suspects traumatic injury to the femoral nerve. Assuming the diagnosis is correct, which of the following findings would likely be present in this patient:

*decreased knee jerk* Femoral nerve injury --> will have a decreased knee jerk (patellar) reflex Traumatic femoral nerve injury - uncommon but can occur with injury to the pelvis or anterior thigh - *Important signs of femoral nerve injury include an absent or decreased knee jerk, an inability to extend the knee against resistance, and sensory loss over the anterior and medial thigh, medial shin, and foot* - Knee jerk (or patellar reflex) examines the motor function of L2-L4, the spinal roots of the femoral nerve.

Hypocalcemia in nephrotic-range proteinuria most likely due to ...

*decreased serum albumin* / hypoalbuminemia - corrected calcium = (measured total calciu) + 0.8 (4 g/dL - serum albumin in g/dL) *Pts with hypoalbuminemia can have decreased total serum calcium, but ionized calcium (physiological active form) is hormonally regulated and remains stable*

Newborn with increased work of breathing and fluid-filled fissures has transient tachypnea of the newborn (TTN), a condition caused by...

*delayed resorption and clearance of alveolar fluid* - normally: mature fetal lungs begin to reabsorb liquid in late gestation in response to increased hormonal (catecholamine) signals --> resorption mechanism increases during labor --> pts born prematurely or by c-section = increased risk of TTN TTN - excess pulm fluid = respiratory distress = tachypnea, grunting, retractions, hypoxia - few hours after delivery - breath sounds often clear bc fluid in interstitial space not alveoli - chest x-ray = hyperinflation, fluid in interloba fissures

A 52-year-old woman is brought to the emergency department by fire and rescue after being involved in a motor vehicle accident. The paramedics report that the patient's car slipped off the road during a rainstorm and rolled into a ditch. The patient was restrained and the airbags deployed during the crash. The patient has a past medical history of hypertension, hyperlipidemia, hypothyroidism, and gout. Her home medications include hydrochlorothiazide, simvastatin, levothyroxine, and allopurinol. The patient is alert on the examination table. Her temperature is 98.2°F (36.8°C), blood pressure is 83/62 mmHg, pulse is 131/min, respirations are 14/min, and SpO2 is 96%. She has equal breath sounds in all fields bilaterally. Her skin is cool with diffuse bruising over her abdomen and superficial lacerations, and her abdomen is diffusely tender to palpation. She is moving all four extremities equally. The patient's FAST exam is equivocal. She is given several liters of intravenous fluid during her trauma evaluation but her blood pressure does not improve. Which of the following is the best next step?

*diagnostic peritoneal lavage* Hemodynamic instability in the setting of blunt abdominal trauma. In light of this equivocal FAST exam, the next best step in management to evaluate for abdominal injury is diagnostic peritoneal lavage. - number of signs pointing to abdominal injury, including the history of being a restrained passenger, bruising over the abdomen, and tenderness to palpation - the first step in management of this patient would be fluid resuscitation - she should then undergo evaluation to determine if she requires surgical management - because she is hemodynamically unstable, the best first test would be a Focused Abdominal Sonography for Trauma (FAST) exam - *since this patient's FAST exam is equivocal, the next best step is diagnostic peritoneal lavage (DPL), which can be performed rapidly* - If either the FAST exam or DPL shows signs of intraperitoneal hemorrhage, this patient would need an emergency laparotomy to repair the injury Experts agree that the aspiration of 10 mL of gross blood confirms the presence of a significant intra-abdominal wound that warrants emergent laparotomy.

A 61-year-old female presents with several episodes of feeling "dizzy." She reports that these episodes were associated with a "ringing" in her right ear as well as sense of hearing loss. She also describes a sensation of increased "fullness" on the affected side. Past medical history is significant for osteoporosis for which she takes ibandronate and hyperlipidemia for which she takes atorvastatin. Which of the following is the first-line treatment for this patient?

*dietary changes* vertigo, hearing loss and tinnitus = Meniere's disease - first-line treatment is dietary modifications, including a low salt diet, with avoidance of alcohol, nicotine, and caffeine - result of an abnormal accumulation of endolymph in the inner ear. Treatment first involves non-pharmacologic treatment including a low salt diet and the avoidance of alcohol, nicotine, and caffeine. - If symptoms persist, pharmacologic treatment may be indicated with thiazides, anticholinergics, or antihistamines. - Refractory cases can be treated with surgical decompression, intraaural gentamicin injections and labyrinthectomy. Illustration depicts an hydropic inner ear on histology, characteristic of Meniere's disease.

A 57-year-old female presents to her primary care physician with complaints of nausea, vomiting, abdominal pain, and bloating that have increased in severity over the past several months. She reports that she occasionally vomits after eating; the emesis contains undigested food particles. Additionally, the patient states that she often is satiated after only a few bites of food at meals. Her medical history is significant for hypertension and type II diabetes mellitus that was first diagnosed 10 years ago. Gastric emptying scintigraphy is conducted and shows gastric retention of 80% at 2 hours and 40% at 4 hours. Which of the following is the best first step in management of this patient's condition.

*dietary modification* Gastroparesis/delayed gastric emptying - initial treatment for this condition is diet modification to frequent, small meals that are low in fiber and fat - presenting symptoms of gastroparesis include nausea, vomiting, early satiety, postprandial fullness, as well as abdominal pain and bloating *Management of this condition should first include: stopping medications that exacerbate gastric stasis, improving blood glucose control, increasing liquids in the patient's diet, transitioning to smaller more frequent meals throughout the day, stopping the use of tobacco and alcohol, as well as decreasing the amount of insoluble fiber and fat in the patient's diet.* - If these preliminary medication treatment options fail, pharmacologic management can include prokinetic agents such as metoclopramide and erythromycin. Illustration is a gastric emptying scintigraphy study in a patient with gastroparesis; note how the tracer does not disappear from the stomach and very little tracer appears distally in the small intestine.

Patient has acute onset of congestive heart failure (CHF) following a recent upper respiratory tract infection, suggesting dilated cardiomyopathy secondary to acute viral myocarditis. Echocardiography would reveal

*dilated ventricles with diffuse hypokinesia* Dilated cardiomyopathy - diagnosis is made by echocardiogram, which shows dilated ventricles with diffuse hypokinesia resulting in a low ejection fraction - cardiomyopathy in viral myocarditis can result from direct viral damage or from the immune response to persistent viral infection. - A displaced cardiac apex, an S3 gallop, and venous congestion or interstitial edema on chest x-ray are useful in identifying CHF. - Echocardiography is the gold standard to confirm systolic or diastolic heart failure through assessment of left ventricular ejection fraction - Elevated B-type natriuretic peptide (BNP) levels indicate systolic cardiac dysfunction.

A 63-year-old man presents to your office for a routine check-up. He currently has no physical complaints. At his last visit three months ago, he was started on therapeutic lifestyle changes to decrease his blood pressure, which was measured in the 150s/90s on two separate visits. Today, his blood pressure is 154/93. You and the patient agree to start on an antihypertensive medication. The medication that you prescribe for him decreases aldosterone production and serum angiotensin I concentration and increases natriuresis. To which of the following drug classes does this medication belong?

*direct renin inhibitor* In the renin-angiotensin-aldosterone (RAA) system, renin catalyzes the hydrolysis of angiotensinogen to angiotensin I. Of the medications listed, *only a direct renin inhibitor would decrease the serum concentration of angiotensin I.*

All antidepressants are associated with risk of inducing mania in susceptible pts. First step is ....

*discontinuation of offending medication* Antidepressant-induced mania - if manic symptoms persist despite discontinuing antidepressant, treatment with mood stabilizer (lithium, valproate) or antipsychotic should be considered....

Ehrlichiosis should be suspected in pts from endemic region with ho tick bite, febrile illness with systemic symptoms, leukopenia and/or thrombocytopenia, and elevated transaminases. Rash is uncommon. Drug of choice is...

*doxycycline* - often initiated empirically when diagnosis suspected while confirmatory test pending Ehrlichosis - via Ehrlichia chaffeensis & E ewingii - lone star tick (Amblyomma americanum) - southeastern and south central US - principal reservoir = white tail deer = acute febrile illness + malaise + altered mental status - "RMSF without spots" Diagnosis - often clinical - definitive diagnosis can be made through visualization of intracytoplasmic morulae in WBCs or through PCR testing

A 67-year-old man presents to general medical clinic with a chief complaint of fatigue. He also reports poor concentration and a general sense of hopelessness. He denies any other symptoms. He feels as if these symptoms will never go away as they have been present for the past three years. His vital signs are within normal limits and a physical exam is unremarkable. A laboratory workup reveals that he is euthyroid. What is the most likely diagnosis?

*dysthymia* - milder form of depression that lasts for at least two years in adults or one year in children or adolescents For the diagnosis of dysthymia, patients must have at least two of the following: - poor concentration or difficulty making decisions - hopelessness - poor appetite or overeating - insomnia or hypersomnia - fatigue - low self-esteem for two years During the two year period, patients must not be without symptoms for more than two months at a time and must not have symptoms that meet the threshold for major depressive disorder. As in major depressive disorder, it is important to rule out medical causes such as hypothyroidism and Cushing's.

A 27-year-old G2P1 female is diagnosed with an HIV infection after undergoing routine prenatal blood work testing. Her estimated gestational age by first-trimester ultrasound is 12 weeks. Her CD4 count is 550 cells/mm^3 and her viral load is 26,000 copies/mL. She denies experiencing any symptoms of HIV infection. Which of the following highly active antiretroviral therapy (HAART) medications should be avoided in this patient?

*efavirenz*, delavirdine - HAART medications with teratogenic effects - should be avoided when treating pregnant patients, especially those in the first trimester There are two goals of treating HIV infected pregnant patients: \ 1) primary treatment of the HIV-infected mother 2) reduction of the risk of mother-to-child transmission of the disease. If the viral load is high and the membranes have not yet ruptured, elective c-section at 38 weeks may reduce the risk of mother-to-child transmission. Breast feeding should be avoided in women with HIV infection. Although monotherapy with zidovudine was used historically in pregnant patients, more recent research suggests HAART combination therapy is more effective for this population. Nonnucleoside reverse transcriptase inhibitors (NNRTI) have been associated with hepatotoxicity (nevirapine) and teratogenicity (efavirenz).

An obese 64-year-old female is being worked up for daytime somnolence. She reports that for the last several months she has had difficulty staying awake and alert during her work as a librarian. Her husband says she's always snored frequently throughout the night. She also complains of morning headaches. On exam, you note an obese female who seems to drift to sleep several times during the exam. Her vitals are as follows: blood pressure is 155/85 mmHg, pulse is 95/min, respirations are 11/min, and T 98.9F. Basic laboratory results reveal a hemoglobin of 15.8 mg/dL and blood HCO3 of 34 mmol/L with a pH of 7.35. An EKG is notable for right axis deviation. Which of the following additional findings might you expect to see in this patient?

*elevated jugular venous distention* Obesity hypoventilation syndrome (OHS) - associated with chronic hypercapnic and hypoxic respiratory failure resulting in a secondary erythrocytosis, pulmonary hypertension, and cor pulmonale. - Cor pulmonale, or right-sided heart failure due to pulmonary hypertension, can be evidenced by elevated jugular venous distension. - impaired respiratory drive, or an inability to maintain adequate respiratory rates or tidal volumes to appropriately oxygenate and ventilate their blood - chronic hypoxia is thought to lead to pulmonary vasoconstriction (to compensate for the shunting physiology), which results in elevated pulmonary pressures and excessive strain on the right side of the heart. - Continuous positive airway pressure (CPAP) is the most effective treatment for significant obstructive sleep apnea. OHS, historically called Pickwickian syndrome - consists of the triad of obesity, sleep disordered breathing, and chronic hypercapnia during wakefulness in the absence of other known causes of hypercapnia

Patient's clinical presentation is consistent with polycystic ovarian syndrome (PCOS). Patients with this condition have an increased risk of ...

*endometrial cancer* PCOS - abnormally elevated luteinizing hormone (LH) The Rotterdam criteria are considered the major diagnostic criteria, and require 2 of three criteria: - menstrual irregularity - hyperandrogenism - polycystic ovaries Endometrial hyperplasia occurs frequently in this population as a result of anovulation, and leads to an increased risk of endometrial cancer

well-demarcated, nonpruritic rash in right axilla. fine-scaled with cig-paper appearance. rash has coral-red *fluorescence under wood's light*. diagnosis?

*erythrasma* - gram-positive via corynebacterium minitissimum - soldiers & hospitalized pts - humidity - trichomonas axillaris = associated condition - toe webs = most common location - clinda/erythromycin; fusidic acid; imidazole antifungals

Recognize erythematous pharynx and bilateral white exudates typical of GAS pharyngitis in 7 yo with non-anaphylaxis allergy to penicillin, which would be treated with...

*erythromycin* Group A Streptococcus (S. pyogenes) (GAS) pharyngitis and a non-anaphylaxis allergy to penicillin. Reasonable alternative treatments of GAS pharyngitis for patients with penicillin allergies include erythromycin or first-generation cephalosporins, such as cefalexin. GAS pharyngitis - commonly presents with sore throat, temperature greater than 100.4 deg F, tonsillar exudates, and cervical adenopathy - must be treated promptly to avoid the development of rheumatic fever and other post-streptococcal sequelae. In patients without penicillin allergy, penicillin V oral for 10 days or amoxicillin oral for 10 days is first-line treatment. In penicillin-allergic patients, erythromycin and first-generation cephalosporins are options. Figure depicts the erythematous pharynx and bilateral white exudates typical of GAS pharyngitis. This question is a classic strep throat question. It will come up in practice that you must know reasonable alternatives for treatment given allergies to certain antibiotics. Many students recall their own pediatrician prescribing erythromycin for them as a child because of a GI aversion to penicillin.

This patient's clinical presentation is consistent with endometriosis. Patients with endometriosis should AVOID:

*estrogen-only contraceptive pills* - they can promote endometrial growth Endometriosis - most common cause of infertility and is characterized by the presence of endometrial tissue outside of the uterus. Hormonal treatment is aimed at reducing endometrial proliferation and management of pain symptoms. Gonadotropin releasing hormone (GnRH) agonists overstimulate GnRH receptors, leading to receptor downregulation within approximately 10 days of constant use. - Decreased GnRH signaling then causes decreased secretion of luteinizing hormone (LH) and follicle stimulating hormone (FSH) from the anterior pituitary gland, leading to decreased endometrial proliferation. Progesterone-containing oral contraceptive pills and danazol are also effective via reduction of endometrial tissue bulk. Illustration shows the classic, dark-purple appearance of endometriosis on laparoscopy.

Recognize chalazion, which presents with painful swelling of the eyelid that progresses to a nodular rubbery lesion. Proper work-up includes treating with hot compresses to see if it resolves. If ineffective, further work-up includes to

*excise the lesion* - histopathologic examination via excision to rule out malignancy, especially when there is unilateral meibomian gland dysfunction or loss of eyelashes around the lesion Chalazion - result of meibomian gland obstruction and if persistent or recurrent may be a sign of meibomian gland carcinoma or sebaceous cell carcinoma - nodule forms on the conjunctival surface of the eyelid and is the result of granulomatous inflammation Failure to respond to conservative treatment (e.g., hot compresses), and thus having a persistent, or recurrent chalazia should prompt you to perform histopathological testing to rule out malignancy. Lack of eyelashes around the lesion is a very concerning physical exam finding for malignancy as well.

A 48-year-old male with type I diabetes mellitus and end-stage renal disease currently on hemodialysis presents to the emergency department with dyspnea, cough and chest pain. He describes the pain as worse during inspiration and when he is lying on his back. A basic metabolic panel is remarkable for a BUN > 60 mg/dL. Which of the following would be expected during the evaluation of this patient?

*fever* Uremic pericarditis, as a result of end-stage renal disease (ESRD) - *Fever is commonly present with this condition* - classically presents with dyspnea, cough, fever, and pleuritic chest pain, worse when the patient is supine and during inspiration and alleviated when the patient leans forward - Physical exam is remarkable for pericardial friction rub best heard with the patient leaning forward - An elevated JVP and pulsus paradoxus may also be present - Pericarditis occurs in 6-10% of renal failure patients (BUN > 60mg/dL) - The most important therapy is hemodialysis (HD) - Other treatment modalities commonly used for pericarditis from other causes (antinflammatory meds - NSAIDS, colchicine) are 2nd line agents after HD.

A 37-year-old female homemaker is rushed to the ED after having been found unresponsive by her 15-year-old son. In the ED she opens her eyes and mumbles incoherently in response to noxious stimuli. She also strikes the ED resident's hand during this examination, but does not follow commands. Her vital signs are: T 98.3 deg F; BP 110/85 mm Hg; RR 15 breaths/min; SpO2 99% on room air. Her medical history is significant for a remote history of depression, anxiety disorder for which she is prescribed alprazolam, and irritable bowel syndrome. Per her son, she takes no other medications. In addition to overdose of her home medication, her toxicology screen is positive for ethanol only. Her EKG is shown in Figure A. Which of the following medications could be considered as an antidote to this patient's overdose?

*flumazenil* CNS depression in the setting of normal vitals signs consistent with her toxicology screen positive for benzodiazepine overdose. - *Flumazenil is a GABA-A receptor partial agonist that functions as a competitive inhibitor of benzodiazepines* Benzodiazepine overdose - characterized by CNS depression in the setting of normal vital signs - Most patients will exhibit ataxia and slurred speech, but are able to respond appropriately to questions and provide an accurate history unless an additional CNS depressant (such as alcohol) is co-ingested. - *As opposed to tricyclic antidepressant overdose, the EKG is expected to be normal.* - Flumazenil can be used to reverse CNS depression in severe cases; however supportive therapy is often preferred when possible because of the risk of precipitating seizures in the habituated patient. Figure A shows a normal EKG that would be expected in a patient with benzodiazepine overdose. *In almost all situations, you will NOT use flumazenil as an antidote. Rather, you can just let the patient sleep off the benzos. You do not often know the patient's history or if they abuse alcohol - giving flumazenil could precipitate a seizure. Rather, monitor their airway and intubate if necessary. At times, a touch of flumazenil can be given if you know the patient's history, you know they are not dependent, and you are perhaps the one that gave too many benzos. This question said "could be considered," used to treat the overdose which dodges this principle. In practice, it would not be used though!*

The patient in this vignette suffers from bipolar disorder. With a history of 3 manic episodes, how long should he remain on mood stabilizing therapy?

*for the remainder of the pt's life* Bipolar disorder - disease characterized by manic episodes lasting greater than 1 week and may or may not be associated with intermittent depressive episodes - important that these episodes not be caused by substance abuse, which can present similarly to organic mania - Effective treatment requires mood stabilizing therapy, including lithium, valproic acid or carbamazepine

Diabetic nephropathy is characterized by proteinuria and progressive decline in GFR. Pathologic hallmark responsible for persistent urinalysis abnormality...

*glomerular basement membrane changes* = nodular glomerulosclerosis and/or diffuse glomerulosclerosis - podocyte injury or decreased glomerular filtration barrier integrity - light microscopy = segmental scarring - treat underlying etiology in secondary causes - steroid therapy Can be secondary to ... HIV sickle cell disease heroin abuse interferon treatment

This patient has adult polycystic kidney disease (ADPKD). The most common extra-renal complication of ADPKD is

*hepatic cysts* - US imaging shown Adult polycystic kidney disease (ADPKD) - autosomal dominant disease that causes small, saccular fluid pockets to develop in the kidneys. - Of note, patients will not demonstrate renal cysts at birth. Rather, they are usually noted later in life when they become large enough to affect kidney function. Patients may initially present in their 30s-40s with abdominal discomfort, hypertension, hematuria, and progressive renal impairment. - Hepatic cysts are noted to be found in approximately 30% of patients diagnosed with ADPKD.* - Polycystic liver disease (PLD) is the most common extra-renal manifestation of ADPKD; this is likely due to increased life expectancy, improved renal survival, reduced cardiovascular mortality, and renal replacement therapy.

Patient presents with evidence of portal hypertension secondary to hepatitis C cirrhosis, including severe ascites, pitting edema, and recent upper GI tract bleeding from esophageal varices. His poor urine output and elevated serum creatinine unresponsive to fluid challenge indicate ...

*hepatorenal syndrome (HRS)* - progressive renal failure that can affect patients with cirrhosis, alcoholic hepatitis, or fulminant hepatic failure - affected kidneys are morphologically normal but function poorly and do not respond to fluids - clinical features include azotemia, oliguria, hyponatremia, and low urine sodium - often follows acute injury that causes liver function to deteriorate rapidly, such as infection, upper GI tract bleeding, or overuse of diuretic medications - affected kidneys function normalizes following liver transplant, which is the only known cure for HRS - believed to be a result of under-filling of the arterial system due to arterial vasodilation in the splanchnic circulation HRS is divided into two types. - Type 1 is defined as acute renal failure and rapid functional deterioration of other organs. - Renal failure in Type 2 progresses more slowly and patients experience ascites that is resistant to treatment with diuretics. - Renal function in both cases may improve with the administration of vasoconstrictors. Vaptans and nonpeptide vasopressin receptor antagonists help resolve the hyponatremia and ascites.

Clinical presentation is consistent with Wilson's disease, which is associated with what important lab values?

*high 24-hour urinary copper excretion*, low serum ceruloplasmin level, low total serum copper level

Recognize femoral neck fracture, which is treated with... "A 69-year-old male presents to the emergency department for pain in his right leg. The patient was reaching for a jar of preserves when he fell from a small foot stool. He experienced immense pain in his right leg and presented to the emergency department shortly thereafter. The patient has a past medical history of diabetes mellitus managed with insulin and metformin. On physical exam, you note a shortened, externally-rotated right lower extremity. Radiography is ordered (Figure A). Which of the following is the best next step in management?"

*hip prosthesis* Femoral neck fractures - commonly occur in elderly patients after experiencing trauma such as a fall and presenting with a shortened, externally rotated lower extremity - *Once the diagnosis has been confirmed, the best next step in management is placement of a hip prosthesis as surgical repair of the injury is not possible* *A femoral neck fracture classically presents with pain after trauma and a shortened, externally rotated lower extremity; the best treatment is placement of a hip prosthesis.*

A 64-year-old man presents for a routine physical. He states that he is not doing very well and, in fact, has not had a bowel movement in over a week. Upon further questioning, the patient notes that for about 4 months he has experienced significant back pain and finds that he is easily fatigued. His wife has remarked that he is unusually "pale." In addition to his lack of a bowel movement for the past week, the patient has also had a loss of appetite, has been urinating more than usual, and has had a depressed mood. Which of the following is the most likely cause of this patient's constipation?

*hypercalcemia* Multiple myeloma with resultant hypercalcemia = electrolyte disturbance that can cause severe constipation. - have plasma cells which produce large quantities of ineffective monoclonal antibodies leading to a variety of clinical manifestations Most common mnemonic for the symptoms of multiple myeloma is CRAB (elevated Calcium, Renal failure, Anemia, Bone pain [often back pain]). The elevated calcium in multiple myeloma - typically a result of excess bone resorption by osteoclasts leading to lytic bone lesions. Clinical symptoms associated with hypercalcemia in general include fractures, nephrolithiasis, vomiting, constipation and altered mental status. - These symptoms are typically summarized as "bones (fractures), stones (nephrolithiasis), groans (vomiting, constipation) and psychic overtones (altered mental status)." Patients with hypercalcemic crisis need to be treated with aggressive intravenous rehydration along with anti-resorptive agents (e.g. calcitonin, bisphosphonates).

A 40-year-old woman presents to her primary care physician for an annual check-up. She complains of intermittent fatigue. Laboratory tests are notable for the following: TSH, serum: 11.2 uU/mL (normal 0.5-5.0 uU/mL) Thyroxine (T4), serum: 9.8 ug/dL (normal 5-12 ug/dL) Which of the following other conditions, if present, would merit treatment with thyroxine?

*hypercholesterolemia* The patient presents with elevated TSH (normal range 0.5-5.0 uU/mL) and T4 levels within the normal reference range (5-12 ug/dL) consistent with subclinical hypothyroidism. *Patients with subclinical hypothyroidism should be treated with thyroxine if hypercholesterolemia is present.*

A 62-year-old man presents to the emergency department with worsening shortness of breath. Two days ago, he developed fever, rhinorrhea, and worsening of his chronic cough. His past medical history is significant for chronic obstructive pulmonary disease (COPD). His medications include ipratropium, but his prescription ran out a few days ago. He does not take supplemental oxygen at home. His temperature is 37.8 C (100.0 F), blood pressure 136/78 mm Hg, heart rate 98/min, respiratory rate 24/min, and oxygen saturation is 87 percent on room air. On physical exam, he is in acute respiratory distress, using accessory muscles for breathing. He has increased anteroposterior diameter of his chest, as well as diffuse wheezing on auscultation. His cardiac and abdominal exams are normal. There is no evidence of clubbing or cyanosis. After an hour of noninvasive positive pressure ventilation, the patient's respiratory distress continues to worsen, requiring endotracheal intubation. Which of the following is a contraindication to the use of succinylcholine in rapid-sequence intubation?

*hyperkalemia* Succinylcholine - can cause life-threatening hyperkalemia and therefore should not be used in patients with or at high risk for hyperkalemia - depolarizing neuromuscular blocker often used during rapid-sequence intubation due to its rapid onset (45-60 seconds) and offset (8-10 minutes) - can cause significant potassium release from cells, resulting in hyperkalemia - patients with crush or burn injuries more than 8 hours old (risk of rhabdomyolysis), demyelinating syndromes like Guillain-Barre, and tumor lysis syndrome should receive alternative drugs, such as non-depolarizing agents like vecuronium

Rare but serious side effect of fluphenazine

*hypothermia* Fluphenazine - high-potency typical antipsychotic used in the treatment of schizophrenia, manic-phase bipolar disorder, and other psychoses. - majority of its more common side effects are attributed to its dopamine-blocking action. - the drug carries a black-box warning of increased risk of death in patients with dementia-related psychosis.

Patient presents with a likely COPD exacerbation and is found to have *multifocal atrial tachycardia*, which is most commonly due to...

*hypoxia, electrolyte disturbances, and medication effects* Multifocal atrial tachycardia (MAT) - an arrhythmia most closely associated with severe underlying lung disease such as COPD or asthma - characterized on ECG by a fast rate, typically greater than 100 bpm, and at least three distinct P wave morphologies preceding the QRS complexe *Other etiologies for MAT include electrolyte disturbances, especially hypokalemia and hypomagnesemia, and medication effects, such as from isoproterenol and the phosphodiesterase inhibitors theophylline and aminophylline*

A 29-year-old G4P0 presents following a spontaneous pregnancy loss in the 18th week. This is her fourth loss in the second trimester and she is concerned about her inability to have a successful pregnancy. Which of the following is the best diagnostic test to evaluate the cause of recurrent miscarriages in this patient?

*hysterosalpingogram* Repeated second trimester pregnancy losses = most likely a result of maternal anatomic factors (e.g. uterine septa, uterine duplication or submucous leiomyomas) There is an important distinction between 1st and 2nd trimester losses in that *1st trimester miscarriages are typically the result of fetal anomalies (e.g. aneuploidies) that impair fetal development*. Conversely, *2nd trimester miscarriages are typically a result of maternal factors which allow for normal embryonic development through the 1st trimester, but, as the fetus and placenta increase in size, cannot support the pregnancy resulting in termination* - hysterosalpingogram (HSG) is indicated in women with a 2nd trimester loss to assess uterine anatomy and allow for surgical correction if a septum or submucous leiomyoma is present.

A 43-year-old male presents to his primary care physician for a routine health maintenance visit. Two months ago, the patient developed left thigh pain that was due to a deep venous thrombosis (DVT) in his left femoral vein following a total knee replacement. The patient has no history of venous thromboembolism (VTE) or malignancy. The patient was discharged on warfarin. The patient is currently asymptomatic, but irritated with the frequent blood draws to check his INR. His INR today is 2.7. Ultrasound imaging 1 week prior showed regression of the thrombus. When is the soonest that he can discontinue warfarin?

*in one month* *Patient with a provoked DVT associated with a transient risk factor (i.e., surgery), the minimum recommended anticoagulation duration is three months. Given that two months have passed, the soonest that anticoagulation can be discontinued is in one month.* The duration of warfarin therapy for DVT depends on the patient's risk of developing a subsequent DVT. - *For a first DVT and for DVTs associated with transient risk factors, the recommended anticoagulation duration is specifically 3 months.* - If not, treatment period is considered "extended" (i.e. indefinite). - memorization of the 3 month duration as well as understanding the factors that led to the VTE can garner an easy point - A unilateral erythematous and swollen leg with >3cm of asymmetry should raise concern for a DVT. *Illustration B shows an ultrasound of the femoral vein with thrombus occluding the lumen.*

A 62-year-old male is evaluated in the emergency department for an episode of painless bright red blood per rectum for 2 hours. The patient is known to have sigmoid diverticulosis, noted on colonoscopy 1 month ago. The patient's bleed most likely arose from a distal branch of which of the following vessels?

*inferior mesenteric artery* Diverticular bleeds - most commonly arterial, due to erosion of an artery by a diverticulum - *As the sigmoid colon is supplied by the inferior mesenteric artery (IMA), a distal branch of this artery is most likely responsible for the arterial bleed* - the most common cause of lower GI hemorrhage - typical presentation includes a painless flow of blood per rectum - while only about 15% of patients with diverticular disease have bleeding, one third of the bleeds will be massive - *Erosion of an artery is the most common source of a diverticular bleed* - diverticular hemorrhage resolves spontaneously in approximately 80 percent of patients - the most appropriate initial diagnostic test is colonoscopy

A 50-year-old patient with a longstanding history of hypertension presents to the hospital with back pain. His only medication is warfarin for intermittent atrial fibrillation. His heart rate is 110 bpm and regular, blood pressure is 90/60 mmHg, and respiratory rate is 12 rpm. On exam he is found to have significant bruising over his flanks. CT imaging demonstrates a large retroperitoneal hematoma, and subsequent lab testing is normal except for an INR of 3.5 and a low hemoglobin. Which of the following is the next best treatment to halt further bleeding?

*infusion of fresh frozen plasma* This patient on warfarin has serious bleeding with vital signs suggesting hypovolemia and an elevated INR. The best treatment for his coagulation abnormality is an infusion of fresh frozen plasma (FFP), which rapidly reverses the action of warfarin. Retroperitoneal bleeds - can be one site of bleeding in patients on anticoagulant therapy - life threatening bleed --> infusion of FFP, which contains all of the clotting factors, rapidly reverses the anticoagulant effect of warfarin and stops further bleeding. The effects of FFP are transient, so while it is important to immediately initiate FFP transfusion, vitamin K should also be given to counteract the long-term effects of warfarin.

Patient presents with fever, abdominal pain and cervical motion tenderness, vaginal discharge, and an adnexal mass, suggestive of pelvic inflammatory disease (PID) and possible tubo-ovarian abscess (TOA). Given her nausea and vomiting and the concern for TOA, the next best step to treat this condition is

*inpatient admission with intravenous cefotetan and doxycycline*

Nonalcoholic fatty liver dz resembles alcohol-associated liver dz but occurs in pts with minimal or no alcohol history. Severity can range from bland steatosis to steatohepatitis to fibrosis and cirrhosis. It is associated with...

*insulin resistance*

When a bone marrow transplant patient develops a respiratory tract infection, what should be suspected?

*invasive aspergillosis should be suspected* - amphotericin B = gold standard treatment

Patient with a positive PPD skin test and a negative chest radiograph is suffering from latent tuberculosis (TB). Treatment for latent tuberculosis is...

*isoniazid and vitamin B6 for 9 months* Although a 9-month regimen of isoniazid and vitamin B6 is preferable due to greater efficacy, a 6-month regimen for adults is acceptable as well; this shorter course may be more cost-effective and have improved patient compliance. Latent tuberculosis is an asymptomatic and non-transmissible infection that has a 5-10% chance over a lifetime of progression to active TB. Latent TB is most commonly seen in immigrants from high-incidence countries, health care professionals, institutionalized patients, and homeless individuals.

chronic stable angina + bradycardia --> treatment =

*isosorbide mononitrate* nitrates (e.g., nitroglycerin, isosorbide mononitrate, and dinitrate) mechanism: venodilation reduces preload to the heart, which reduces myocardial demand recall that a reduction in preload reduces ventricular wall stress coronary artery dilation improves blood flow to the myocardium recall that venodilation >> arterial dilation ↑ cGMP adverse events: nitroprusside causes cyanide toxicity development of tolerance for the vasodilating action during the work week a headache may result from vasodilation of cerebral arteries can results in hypotension when combined with other blood pressure lowering medications

Recognize classic rash of lymphocutaneous sporotrichosis--a reddish nodule that later ulcerates at the site of a thorn prick or skin injury, then spreads along lymphatics, resulting in subcutaneous nodules and ulcers. Treatment is...

*itraconazole*

A 32-year-old female presents to her primary care provider with pelvic pain. She reports that for the last several years, she has had chronic pain that is worst just before her menstrual period. Over the past two months, she has also had worsening pain during intercourse. She denies dysuria, vaginal discharge, or vaginal pruritus. The patient has never been pregnant and previously used a copper intrauterine device (IUD) for contraception, but she had the IUD removed a year ago because it worsened her menorrhagia. She has now been using combined oral contraceptive pills (OCPs) for nearly a year. The patient reports improvement in her menorrhagia on the OCPs but denies any improvement in her pain. Her past medical history is otherwise unremarkable. Her temperature is 98.0°F (36.7°C), blood pressure is 124/73 mmHg, pulse is 68/min, and respirations are 12/min. The patient has tenderness to palpation during vaginal exam with lateral displacement of the cervix. A pelvic ultrasound shows no abnormalities, and a urine pregnancy test is negative. Which of the following is the best next step in management to confirm the diagnosis?

*laparoscopy* Chronic pelvic pain that is worse before her menstrual period, vaginal tenderness on physical exam, and lateral displacement of the cervix = *endometriosis* - *Since this patient has failed therapy with oral contraceptive pills (OCPs), she should undergo laparoscopy.* Endometriosis - classically presents with chronic pelvic pain and the "three D's" of dysmenorrhea, dyspareunia, and dyschezia. - Since laparoscopy with biopsy is required for definitive diagnosis, patients with suspected endometriosis may be empirically managed with OCPs and NSAIDs.

Post-spinal surgery. Last normal void was 12 hrs ago. Acute retention via mild BPH exacerbated by pain med and lack of activity. Tx?

*leaving indwelling cath in place for 48-72 hrs* - can teach self-cath too

To decrease pt with COPD's use a SABA, they can use...

*long-acting anticholinergic inhaler* - maintenance Ipratropium/tiotropium: Anti-cholinergic - COPD - Asthma - Dry mouth - Sedation

Recognize S4 heart sound, which is best explained by ...

*long-standing hypertension* S4 - additional diastolic sound just prior to S1 - "TEN-nes-see" - indicator of still left ventricle in adults - restrictive cardiomyopathy or left ventricular hypertrophy from prolonged HT - corresponds with atrial contraction - via sound of blood striking stiffened left ventricle exertional dyspnea + S4 = diastolic heart failure = sequela of left ventricular hypertrophy from prolonged HT

The first stage of labor is complete following

*maximal cervical dilation and full effacement of the cervix* First stage of labor - begins at the onset of labor - typically lasts 10-12 hours in a nulliparous patient and 6-8 hours in a multiparous patient - divided into latent and active phases, with the latent phase characterized by slow cervical dilation and slow cervical change - latent phase ends when the cervix is dilated roughly 6 cm - active phase follows the latent phase and is marked by more rapid cervical change and dilation - maximal cervical dilation usually occurs around 10 cm The definition of protracted dilation is <1.2 cm/hr for nulliparous patients and <1.5 cm/hr for multiparous patients. Arrest of dilation is defined as >2 hours with no cervical change for either group.

Most commonly occurs after cardiac or esophageal surgery/perforation. It can also arise from contiguous spread of a retropharyngeal infection. The cause is typically polymicrobial. Most common microorganisms causing mediastinitis include MSSA, MRSA, gram-negative bacilli, coagulase-negative staph, and streptococci.

*mediastinitis* - purulent discharge from mediastinal area - CT hallmarks of mediastinitis: localized mediastinal fluids; pneumomediastinum - Treatment: surgical debridement and antibiotics

For patients without a history of coronary artery disease, what should be done before undergoing a cardiac stress test?

*medications including beta-blockers, calcium channel blockers (amlodipine), and nitrates should be stopped 48 hours before undergoing cardiac stress test* - as these agents affect the accuracy of stress testing - such medications may cause a false negative by masking ischemic changes on the electrocardiogram or stress test imaging - they may cause a false positive result by creating artificial ischemic changes on the electrocardiogram - stopping these agents 48 hours prior to the stress test allows for 4-5 half-lives to pass, thereby reversing the hemodynamic effects of these medications - Indications for stress testing can include pre-operative assessment, identification of the extent/existence of coronary artery disease, evaluation of the efficacy of current cardiac treatment, and identification of appropriate and safe levels of exercise

A 52-year-old woman comes to your clinic for her annual physical exam. She is obese, does not exercise, and regularly eats fried foods. A random blood glucose is 249 mg/dL. Her hemoglobin A1C is 9.5. Which of the following treatments would be weight neutral or cause weight loss in this patient?

*metformin* Type II diabetes - *Of the given treatments, only metformin is weight neutral in the majority of cases (i.e. it does not cause significant weight gain/loss)* Metformin - first-line treatment for type II DM in most patients. - Although the exact mechanism is unknown, it appears to decrease gluconeogenesis and increase insulin sensitivity - *There is no risk of hypoglycemia or weight gain (though some patients even lose weight)* - *The most high yield side effect involves lactic acidosis, particularly in patients with renal insufficiency*

A 31-year-old man is brought to the ED via ambulance status-post a grand mal seizure, the first of his life. His wife states that earlier in the day he was complaining of a terrible headache, but attributed it to too much time in the sun at the beach. He vomited once prior to the seizure. His past medical history is unremarkable. Vitals are BP 160/90 mmHg, temperature 100.9 degrees F, pulse 110/min, respirations 20/min. On exam, he is lethargic and not oriented, has nuchal rigidity and a positive Kernig's sign. Non-contrast brain CT appears as seen in Figure A. What is an important component of treatment for this patient?

*nimodipine* Vasospam - secondary to the inflammatory response - important complication of ruptured aneurysms, as seen in this patient *As a result, nimodipine, a calcium channel blocker, is an important part of therapy to prevent this complication.* Berry aneurysms form as a result of congenital weaknesses in the blood vessels. They present with a sentinel bleed (subarachnoid hemorrhage or SAH) that is felt as the "worst headache" of the patient's life. - Vomiting, *signs of meningismus*, cranial nerve palsies, and seizures can follow. - Rebleed and vasospasm are common complications over the next few days. *Figure shows a subarachnoid hemorrhage (SAH) as seen on CT (arrow points to the blood, which appears white on the image).*

A 74-year-old woman was admitted to the hospital for a CHF exacerbation and is started on the appropriate treatment for CHF exacerbation. Routine admission workup includes urinalysis, which shows >100,000 colonies/mL of E. coli. The patient endorses some shortness of breath and lower extremity swelling, but denies chest pain, nausea/vomiting, dysuria, hematuria, or changes in urinary frequency. She has no known drug allergies. Which of the following is the most appropriate treatment for this patient for this finding?

*no treatment* Asymptomatic bacteriuria - usually resolves spontaneously in elderly patients without antibiotic therapy - defined as >100,000 colony-forming units of bacteria without symptoms of a UTI. The following patients do NOT require treatment for asymptomatic bacteriuria: - The elderly - Diabetics - Nonpregnant, premenopausal women - Spinal cord injury patients - Patients with chronic, indwelling urinary catheters These patients DO require treatment for asymptomatic bacteriuria: - Pregnant - Urologic intervention - Hip arthroplasty

This patient's clinical presentation is consistent with attention deficit hyperactivity disorder (ADHD). The two first-line agents for treatment of this disorder are methylphenidate (Ritalin) and dextroamphetamine (Adderall), which increase

*norepinephrine and dopamine release.* - Both medications increase levels of synaptic norepinepherine and dopamine in the prefrontal cortex, nucleus accumbens, and locus ceruleus.

Recognize spontaneous subconjunctival hemorrhage, in which the treatment is

*observation* - benign - may be due to simple trauma, rubbing eyes vigorously, violent coughing spells, hypertensive episodes, or coagulopathy

A 33-year-old patient presents to the clinic with a chief complaint of a dry cough that has started recently. The patient complains of shortness of breath in addition to his current symptoms. The patient has a history of HIV and medical non-compliance. In addition, he has an allergy to sulfa medications, for which he had been incubated in the past. His medical history is also significant for a blood transfusion secondary to primaquine administration. Notable laboratory findings are CD4 count of 175, an elevated LDH, and a pO2 of 55 mmHg. A chest radiograph is performed and demonstrates the findings in Figure A. The diagnosis is confirmed with bronchoalveolar lavage. The physician is concerned with this patient's toxic appearance and severe symptoms. Which of the following is the most appropriate next step in management?

*pentamidine* History of HIV and a CD4 count < 200. His clinical findings are suggestive of *pneumocystis jirovecci pneumonia (PJP)* - *In the presence of a sulfa allergy that caused anaphylaxis, the best therapy is IV pentamidine in addition to prednisone (in severe cases) as it has been shown to reduce mortality in PJP* PJP commonly presents in patients with the following characteristics: 1. HIV patients with CD4 count < 200 cells 2. Pulmonary symptoms such as cough and shortness of breath 3. Increased LDH Given these classic symptoms, the best initial test is to perform a chest radiograph followed by confirmation with bronchoalveolar lavage. The treatment of choice in a case of severe PJP with a history of anaphylaxis to sulfa drugs is IV pentamidine and prednisone. Mycobacterium avium-intracellulare could be considered in this patient, however, this patient's CD4 count is > 50. Figure demonstrates a chest radiograph with increased bilateral interstitial markings.

A 53-year-old man from Guatemala presents with a 6-month history of progressively worsening dyspnea on exertion and cough, which has led to marked limitation of his physical activity. As a child, he had a febrile illness characterized by arthritis affecting multiple joints and abnormal movements. On cardiac exam, he has a low-pitched diastolic rumble that is most prominent at the apex. Subsequent workup with echocardiogram confirms the diagnosis of a valvular abnormality. What is the best treatment option?

*percutaneous mitral balloon valvotomy* Rheumatic heart disease and severe mitral stenosis with New York Heart Association (NYHA) class III symptoms. *With favorable valvular anatomy, percutaneous mitral balloon valvotomy would be appropriate treatment.* Mitral stenosis often caused by rheumatic involvement of the mitral valve, but other causes include congenital abnormalities and calcifications. - Symptoms of mitral stenosis range from dyspnea due to elevated left atrial pressures and pulmonary congestion, hemoptysis, arial fibrillation due to left atrial enlargement, and right-sided heart failure, among other possibilities. - The low-pitched diastolic rumble, loudest at the apex PMBV - currently the preferred treatment for rheumatic mitral stenosis if the valve morphology is favorable and the patient does not have left atrial thrombus or moderate-to-severe mitral regurgitation NYHA Classes of CV disability- Class 1 - cardiac disease only (no limitation of physical activity). Class 2 - cardiac disease + some limitation of physical activity. Class 3 - cardiac disease + limitation of physical activity, relieved by rest. Class 4 - cardiac disease + severe limitation or complete impairment of physical activity, possibly even present at rest.

A 40-year-old man presents to the emergency department with altered mental status. He has a history of cirrhosis of the liver secondary to alcoholism. Laboratory studies show Na 140, K 2.9, Cl 100, HCO3 36, BUN 27, Cr 1.0. His home medications are lactulose and furosemide. Physical exam reveals shifting dullness consistent with ascites. Which of the following is the most appropriate treatment for this patient?

*potassium* Hepatic encephalopathy secondary to hypokalemia --> should be treated with potassium. - decline in mental status secondary to liver failure - occurs when the liver is unable to clear the blood of toxic metabolites, especially ammonia *There are multiple factors that may cause a patient with liver failure to become encephalopathic -- one important one is hypokalemia* - Loop diuretics, such as this patient's furosemide, predispose to hypokalemia. - Potassium must be repleted immediately when hypokalemia precipitates hepatic encephalopathy.- Recall that hypokalemia can also cause ventilatory failure, EKG changes, and arrhythmias.

A 24-year-old female is referred to a psychiatrist by her employer, who notes that she has had trouble finishing projects in a timely manner. After a lengthy interview, the psychiatrist writes that he believes the patient has obsessive-compulsive personality disorder. Which of the following is characteristic of obsessive-compulsive personality disorder?

*preoccupation with order and control* Preoccupation with order and control is characteristic of obsessive-compulsive personality disorder (OCPD). - Cluster C personality disorder - need for perfection, over-attention to detail, and insistence on following rules, often to the point that such behavior has a negative functional impact - display ego syntonic behavior and believe their behavior is useful, while patients with obsessive compulsive disorder (OCD) understand their behavior is irrational (ego dystonic behavior). - traits of OCPD can sometimes be adaptive - can be treated with SSRIs to relieve anxiety; brief inpatient stays have shown benefit as well. - Cluster C personality disorders predispose patients to major depression and are likely to exacerbate major depressive symptoms in affected patients

Beta-blockers such as propranolol are the first-line treatment for essential tremor; if a patient cannot tolerate propranolol due to bradycardia, hypotension, or lung disease such as asthma or COPD, the patient should be switched to

*primidone*

A 20-year-old college student presents to her primary care physician's office for an annual check-up. She is otherwise healthy, but when asked about how classes are going, she expresses serious concern about her upcoming oral presentation for a public-speaking course. She says that she has never been a good speaker in front of other people, and even refrains from asking questions in class because of her nerves. What is the most appropriate treatment to alleviate these symptoms for her upcoming final?

*propanolol* Glossophobia / fear of public speaking - *for acute treatment, beta-blockers such as propranolol or atenolol are likely to be most effective* - one example of the broader social anxiety disorder, which is a characterized by an unreasonable and persistent fear of scrutiny in social performance situations - it usually begins in childhood or adolescence and is diagnosed in the early 20s - this particular phobia might also be coupled with performance anxiety of any sort (i.e. during sports matches, music recitals, etc) - cognitive behavioral therapy and pharmacotherapy are both effective for social phobias - *the use of beta-blockers has also been shown to be effective in the acute situational treatment of speech anxiety* If you are looking for a non-pharmacologic therapy, then the best initial therapy would be CBT and relaxation exercises. However, since this patient has an upcoming presentation, treatment with a more aggressive medical therapy would be useful for the acute episode. CBT is likely not to take effect if the patient's presentation is very soon. For this reason, the best therapy for this patient's acute symptoms is a beta-blocker. After this acute event, she can likely receive CBT which should help her deal with her anxiety.

A 62-year-old female presents to her primary care physician with a chief complaint of constipation that has become more bothersome over the four months. She states that she had prior issues with constipation complicated by hemorrhoids 10 years ago. She denies any pain with defecation or blood in her stool. The patient reports having just had a colonoscopy 3 months ago with normal results. Her vital signs at this visit are within normal limits. Physical examination, including a rectal exam, does not reveal any abnormalities. The physician instructs the patient to increase her fluid and dietary fiber intake. In addition to these recommendations, which of the following agents would be most appropriate for the patient's constipation at this time?

*psyllium* Uncomplicated constipation. - *Appropriate initial treatment for constipation includes recommendations to increase fluid and fiber intake in the diet as well as initiation of a bulk-forming laxative such as psyllium, methylcellulose, or over-the-counter fiber supplements.* In patients who do not respond to bulk-forming laxatives and dietary changes, additional classes of medications can be tried, including osmotic laxatives, stimulant laxatives, or emollient laxatives. Osmotic laxatives increase the amount of water in the colon and small intestine, thereby improving the pliability and passage of the stool; examples include milk of magnesia, magnesium citrate, polyethylene glycol. Chronic constipation is traditionally defined as 3 or fewer bowel movements per week for at least 3 months. - Constipation risk factors include female gender, low activity level, increased age, low-fiber diet, low caloric intake, low fluid intake, and low socioeconomic status.

This patient presents with fever, conjunctival pallor, and dactylitis, which are classic findings seen in sickle cell disease (SCD). The peripheral blood smear shows Howell-Jolly bodies in red blood cells (RBCs) caused by

*reduced function of reticuloendothelial system* Via pt's functional hyposplenism. - The spleen is a part of the reticuloendothelial system. Howell-Jolly bodies - basophilic nuclear remnants found in RBCs normally removed by splenic macrophages. In patients with sickle cell disease, sickled RBCs cause microvascular occlusion that lead to splenic infarction. - Functional hyposplenism develops early in childhood, with patients becoming functionally asplenic by 2-4 years of age. - Functional asplenia puts patients at higher risk of infection by encapsulated organisms, including S. pneumoniae, N. meningitidis, and H. influenzae. Patients should be vaccinated against these bacteria, and children should receive penicillin as prophylaxis until the age of five.

The patient is experiencing a pulmonary embolism (PE). Appearance on EKG?

*right bundle branch block* PE results in right ventricular dysfunction --> may appear as right bundle branch block (RBBB) on electrocardiogram (EKG)

In patient with a conjugated hyperbilirubinemia and elevated alkaline phosphatase, what is the diagnostic modality of choice in distinguishing between intra- and extrahepatic biliary obstruction.

*right upper quadrant ultrasound* Biliary obstruction - right upper quadrant ultrasound is used to determine where this obstruction may lie - *Ultrasound can be diagnostic for one of the major extrahepatic causes of obstruction: choledocholithiasis* - This woman's history of scleroderma raises concern for the possibility of primary biliary cirrhosis (PBC), an intrahepatic cause of biliary obstruction that typically presents in middle-aged women - Fatigue and pruritus are common initial complaints, while jaundice is usually a later finding *Illustration is an ultrasound of a patient with intrahepatic biliary obstruction. Note the diffusely dilated bile ducts throughout the liver parenchyma. This would be seen in isolation in the setting of PBC, but may be seen in addition to dilation of the extrahepatic biliary tree in choledocolithiasis*

isolated systolic hypertension, an important cause of hypertension in elderly pts, is caused by

*rigidity of arterial wall* - increased stiffness or decreased elasticity of arterial wall - associated with: increased in CV morbidity and mortality - management = lifestyle modification & pharm

A 65-year-old male with a history of hypertension presents to the emergency with severe back pain. He states that he was straining to pick up a box in the hospital storeroom in which he works when it started. His vitals at the emergency room shortly thereafter are T 98.7, HR 110, BP 90/65, and RR 20. Which of the following is the most likely diagnosis?

*ruptured aortic aneurysm* Even though the patient was lifting when the pain started, *the presentation of acute onset back pain and severe hypotension is a ruptured aortic aneurysm until proven otherwise* - generally asymptomatic and can be discovered incidentally on exam or imaging study - As it enlarges, an AAA may cause a pulsatile sensation, back pain, and vague epigastric pain - Upon rupture, AAAs cause severe, tearing abdominal pain radiating to the back and hypotension - Emergent surgery is required

primary biliary cholangitis / primary biliary cirrhosis is a chronic liver dz characterized by intrahepatic cholestasis via autoimmune destruction of small bile ducts. it presents in middle-aged women with fatigue, pruritus, hepatomegaly, and elevated alk phos. diagnosis confirmed with...

*serum anti-mitochondrial antibody titers*

2 yo white male + not walking yet + normal orthopedic eval + normal & other dev'tal milestones. Next test?

*serum creatinine kinase level* Duchenne - massive CK elevation --> genetic counseling regarding future pregnancy

Confirms diagnosis of septic shock

*serum lactate level of 2 (N 0.5 - 1)* / hyperlactatemia or hypotension refractory to IV fluids

HIV-associated diarrhea has many potential causes. CD4 count, chronicity, and presence of absence of symptoms of colitis help narrow the differential diagnosis. Work-up typically involves...

*stool exam studies including ova & parasites*, acid-fast stain, C difficile antigen HIV + acute onset, nonbloody diarrhea with low-grade fevers = infectious etiology --> common causes = cryptococcus sporidium, mycobacterium avium complex (CD4 <50/mm^3), microsporidia, Giardia, Isospora belli

A 42-year-old gentleman is brought to the emergency department following a motorcycle accident. Upon entrance into the trauma bay, the patient was found to be unconscious and unresponsive. An airway was established and fluids were started through two large bore IV lines. The patient was not responding to several liters of crystalloid fluids and the FAST examination revealed fluid around his spleen. The trauma surgeon deduced that the patient had a lacerated or ruptured spleen and emergent surgical intervention was necessary. A splenectomy was preformed. Four days post-operatively, the patient was found to have a fever, a white blood cell count of 14,000, pain on inspiration, and left shoulder pain. What is the most likely cause of this patient's constellation of post-operative symptoms?

*subphrenic abscess* - can present as a complication of a splenectomy with elevated white blood cells, fevers, pleuritic pain, and left shoulder pain. - can present as complications from acute pancreatitis, trauma, post-splenectomy, and other abdominal surgeries. - these abscesses are essentially infected fluid accumulations between the diaphragm, liver, and spleen. - the postoperative fevers seen with a subphrenic abscess usually present *3-5 days after the operation.* - unique on presentation in that they have *left shoulder pain, which is referred pain from phrenic nerve irritation.* - other common presenting signs are cough, tachypnea, diminished breath sounds, dullness to percussion, and fevers. - CT was found to be an irreplaceable tool for rapid and easy diagnosis, along with follow-up surveillance of subphrenic abscesses. The CT scan was also used for guided drainage of the abscesses (first-line treatment).

A 61-year-old woman presents to your office complaining of mild fatigue and weight loss over the past 6 months. Physical exam reveals splenomegaly. A complete blood count with differential shows the following: Hb 8.1 Platelet count 80,000 Leukocyte count 59,000 Segmented neutrophils 68% Lymphocytes 10% Monocytes 8% Bands 7% Basophils 10% Metamyelocytes 6% Blasts 2% Based on the results of cytogenetic analysis, imatinib treatment is initiated. Which of the following chromosomal abnormalities is most likely present in this patient?

*t(9;22)* CML - The presence of a t(9;22) translocation is required for the diagnosis of CML. - The first line treatment for CML is a tyrosine kinase inhibitor such as imatinib, dasatinib, and nilotinib. - myeloproliferative neoplasm characterized by clonal overproduction of hematopoietic myeloid stem cells that can differentiate. 1) Chronic phase: fatigue, splenomegaly, malaise, weight loss, night sweats, abdominal fullness(as seen in the patient in the question stem). 2) Accelerated phase: refractory leukocytosis and worsening symptoms including bone pain, bleeding, infections, pruritis. 3) Blastic phase: WBC >100,000 with severe constitutional symptoms.

Pt taking low-dose beclomethasone inhaler daily and inhaled albuterol as needed. Most likely complication of chronic beclomethasone use...

*thrush* Local delivery of corticosteroids to airways by inhalation = cornerstone of chronic therapy for asthma via low incidence of adverse effects - most common adverse effect = oropharyngeal candidiasis / oral thrush High doses of inhaled corticosteroids - increased systemic absorption --> *adrenal suppression, cataract formation, decreased growth in children, interference with bone metabolism, purpura*

A 7-year-old boy is brought in to your clinic by his mother, who is complaining of her son's "dry scalp and hair loss." She reports a scaling spot on his scalp developed around 2 months ago, soon after moving to a new school, which then became a larger bald spot. She denies that anyone in the household is exhibiting similar symptoms, but does recall a note that came home from school warning of a lice outbreak. Thus far, she has only tried daily baths with little improvement. The patient denies pain or itchiness, but the mother reports she does notice him scratching his scalp. The mother also reports that since her son has started at the new school, she has noticed an increase in his tics including excessive blinking and clearing of his throat. The patient's medical history is significant for asthma, allergic rhinitis, and attention deficient hyperactivity disorder. His current medications include fluticasone, salmeterol, loratadine, and methylphenidate. Family history includes obsessive compulsive disorder in his older sister. The mother also reports her father had a debilitating skin and joint disorder. On examination, you note swollen lymph glands at the back of the patient's ears and neck, and there are residual black dots within a patch of alopecia. Which of the following is the most likely diagnosis?

*tinea capitis* - dry patch on his scalp that developed into an area of alopecia with a black dot pattern - most frequently occurs in children between 3 to 8 years old. Clinically, it presents as a scaly patch and patchy alopecia - often have an erythematous "active border" and within the patch the hairs will break off at the scalp surface, causing a residual black dot - may also appear as favus, or yellow crusts and matted hair with smooth areas of hair loss - postauricular lymphadenopathy can be present - best initial diagnostic test is a potassium hydroxide (KOH) test - positive Wood's lamp test is helpful if tinea capitis is suspected, but this is not diagnostic - most accurate diagnostic test is a fungal culture, but this is not commonly used - treatment is oral terbinafine or itraconazole for 4 to 6 weeks - oral griseofulvin can also be used, but requires a longer duration of therapy.

Patient is presenting with deafness and long QT syndrome that progressed to torsades de pointes, suggesting the diagnosis of Jervell and Lange-Nielsen syndrome. The next best step in management is

*to replete potassium and magnesium and then start a BETA-BLOCKER* Jervell and Lange-Nielsen syndrome - autosomal recessive disorder caused by defective potassium ion channels --> results in abnormal functions of the inner ear and cardiac muscle associated with congenital sensorineural deafness and long QT syndrome --> Prolonged QT increases the risk of ventricular arrhythmias, especially torsades de pointes, which may manifest as seizures, syncope, or sudden death usually stimulated by exercise or being startled - *Management of symptomatic congenital long QT or torsades de pointes should involve the repletion of potassium and magnesium, and then the initiation of a beta-blocker for long-term management. If symptoms are refractory to beta-blockers, a pacemaker can be implanted to avoid cardiac arrest.*

Recognize rosacea, in which the best course of action is

*topical metronidazole* Rosacea - very common cause of a red appearing face in Caucasians and people of Mediterranean descent - most often develops between the ages of 30 and 50, although females tend to present earlier than males - patients should avoid items that are known triggers - several topical metronidazole preparations may be used, and are significantly more effective in the papulopustular subtype of rosacea - best treatment for persistent flushing is brimonidine (an alpha-2 agonist and vasoconstrictor)

Recognize variable decelerations, which are associated with...

*umbilical cord compression* The electronic fetal heart monitoring tracing in this vignette depicts variable decelerations, which are abrupt departures from baseline at intervals unrelated to the timing of uterine contractions. Variable decelerations are directly caused by compression of the umbilical cord. Umbilical cord compression = direct cause of variable decelerations - can occur more frequently in the setting of oligohydramnios --> can occur due to rupture of membranes or fetal congenital anomaly such as renal agenesis or another urinary tract abnormality. - Initial treatment include shifting maternal position or, in cases of ruptured membranes, amnioinfusion

Following hemoglobin breakdown, majority (95%) of bilirubin is in what form?

*unconjugated* = tightly bound to albumin = highly insoluble --> cannot be excreted in urine --> undergoes hepatic conjugation --> goes into intestines --> predominately deconjugated & recycled (enterohepatic circulation) to be reconjugated in liver - some of metabolism products excreted in feces as urobilinogen --> 1% of urobilinogen renally excreted Excess unconjugated bilirubin (hemolysis) --> higher than normal amount of urobilinogen excreted in feces or urine --> positive urine urobilinogen assay

Heparin-induced thrombocytopenia should be suspected in pts receiving heparin anticoagulation who have thrombocytopenia, thrombosis, or >50% drop in platelet count from baseline 5-10 days after initiation of treatment. This is suggestive of use of...

*unfractionated heparin* Type 1 HIT - nonimmune direct effect of heparin on platelet activation - within first 2 days of heparin exposure Type 2 HIT - more serious immune-mediated disorder - via Abs to platelet factor 4 (PF4) complexed with heparin - nadir of 30,000 - 60,000/uL - within 5-10 days - life-threatening consequences = limb ischemia, stroke

The combination of an anal fissure in an aberrant location with apthous ulcers in the oral mucosa is very suspicious for Crohns disease. The diagnosis is made by noting focal, asymmetric, transmural, or granulomatous features on endoscopy, radiography, and pathology, which can be acquired by

*upper and lower endoscopy* Crohns disease - chronic inflammatory disorder affecting the entire length of the gastrointestinal tract - frequently complicated by oral and perianal manifestations - can present with exclusively perianal or oral disease, making the diagnosis challenging Image depicts an apthous ulcer. Apthous ulcers of the oral mucosa are the most common manifestation of oral Crohns disease.

A 40-year-old South Asian male presents to a primary care provider complaining of a chronic cough that is worse at night. Through a translator, he reports that he has had the cough for several years but that it has been getting worse over the last few months. He recently moved to the United States to work in construction. He attributes some weight loss of ten pounds in the last three months along with darker stools to difficulties adjusting to a Western diet. He denies any difficulty swallowing or feeling of food getting stuck in his throat. He drinks alcohol once or twice per week and has never smoked. He denies any family history of cancer. On physical exam, his lungs are clear to auscultation bilaterally without wheezing. His abdomen is soft and non-distended. He has no tenderness to palpation, and bowel sounds are present. He expresses concern that he will be fired from work if he misses a day and requests medication to treat his cough. Which of the following is the best next step in management?

*upper endoscopy* Chronic cough that is worse at night = typical gastroesophageal reflux disease (GERD) - *In light of this patient's new weight loss and melena, the next best step in management would be upper endoscopy to rule out cancer* - In addition to nighttime cough, typical GERD may present as substernal or epigastric pain, regurgitation, nausea, or bitter taste in the mouth - Symptoms are typically worse after eating or lying down - *Patients with typical GERD should undergo upper endoscopy if they have concerning symptoms, including dysphagia, odynophagia, vomiting, weight loss, gastrointestinal bleeding or melena* - *They should also undergo upper endoscopy if they are men over the age of 50 with chronic symptoms (>5 years) and cancer risk factors, such as history of tobacco use or family history of cancer*

A 53-year-old gentleman presents to your office with a two-month history of abdominal pain. Of note, the patient is a long-time patient of yours that you have been treating for uncontrolled diabetes. In the office today, his blood sugar is 322 mg/dL. Otherwise, the patient appears non-toxic and his vital signs are stable. Upon further questioning, the patient endorses daily nausea, occasional vomiting, and a feeling of post-prandial fullness. What is the most appropriate next step in the patient's care?

*upper endoscopy* Diabetic gastroparesis - *initial work-up includes ruling out mechanical obstruction via an upper endoscopy study* - pathologic state resulting in delayed gastric emptying - can be due to injury to the vagus nerve leading to delayed muscular contraction of the stomach and ultimately delayed emptying - diagnosis is made on the basis of a gastric emptying scan - prior to making the diagnosis, ruling out mechanical obstruction with an upper endoscopy must be undertaken

A 31-year-old G3P2 presents for a first-trimester prenatal visit. You perform the standard first-visit tests including blood type and screen, Rh type, CBC, immunization status, urinalysis for protein, and Pap smear. Which of the following first-visit tests will you also administer in the third trimester?

*urinalysis* *The only study required on 1st and 3rd visits is urinalysis/urine dipstick.* Serial urinalysis --> screens for asymptomatic bacteria and preeclampsia (by measuring urine protein). If a women is at risk for diabetes (e.g. obesity, polycystic ovary syndrome, previous history of gestational diabetes, first degree relative with diabetes, etc), then screening for diabetes can also be done at the 1st trimester visit in addition to routine screening at 24 to 28 weeks.

Recognize postoperative infectious endophthalmitis, the most common cause of endophthalmitis, in which the next step would be...

*vitreous tap and injection of intravitreal antibiotics* In patients who present with vision better than light perception, the appropriate next step in management is vitreous tap and injection of antibiotics. Infectious endophthalmitis - dreaded complication of cataract surgery - patients may present up to 6 weeks following surgery - most common pathogens are gram positive agents such as strep and staph - in a patient who has a traumatic eye injury from a tree branch, be aware of fungal pathogens - clinically, patients present with a red, painful eye, and on slit lamp examination, a collection of pus may be seen filling the anterior chamber --> this finding is known as a hypopyon - patients also have decreased visual acuity. - management involves a vitreous tap for culture and injection of intravitreal antibiotics Figure shows a hypopyon, an accumulation of pus in the anterior chamber

Adrenal insufficiency (Addison dz) secondary to TB consistent with... - ACTH = - Aldosterone = - Na = - K = - Glucose = - Acid Base =

- *ACTH = elevated* - *Aldosterone = decreased* - Na = low - *K = high* - *Glucose = low* - Acid Base = metabolic acidosis - eosinophilia

Recognize *glaucoma*

- 70% cupping/abnormal enlargement of light-colored area around central vascular area in disc

Recognize microscopic appearance of Mucor spp., which is associated with *diabetes*

- Note the irregular, septate hyphae branching at wide angles Rhinocerebral mucormycosis - most commonly occurs secondary to diabetes, more specifically, diabetic ketoacidosis (DKA) Mucormycosis - invasive fungal infection - most often caused by Mucor or Rhizopus species - most commonly occurs in immunocompromised patients (particularly patients with diabetes, hematologic malignancies, and increased levels of serum iron) - signs and symptoms include sinus pain, headache, and even proptosis or impaired extraocular movements from involvement of contiguous structures - treatment is prompt surgical debridement and intravenous amphotericin B - prognosis is poor

Recognize *Ichthyosis vulgaris*

- Progressively dry, scaly skin, a history of asthma and keratosis pilaris and a family history of a similar condition - autosomal dominant disorder that causes progressively dry, scaly skin with horny plates over the extensor surfaces of the limbs - generally appears sometime between childhood and young adulthood - other problems associated with the disorder include a failure to sweat, decreased sensation and joint motion, ectropion, and skin infections - caused by mutations in the filaggrin gene (FLG): filaggrin protein binds to keratin in epithelial cells. this gene mutation also makes individuals more at risk for other dermatological conditions such as eczema and contact dermatitis - prevalence of this disorder is higher in lighter-skinned populations

Recognize *diffuse esophageal spasm*

- Sudden chest pain following ingestion of cold water and relieved with nitroglycerin is classic - If performed during an episode, upper GI contrast study will show the "corkscrew" esophagus - painful uncoordinated, non-peristatlic contraction of the esophagus with normal lower esophageal sphincter tone - often precipitated by the ingestion of hot or cold liquids - associated with a history of gastric-esophageal reflux disease (GERD) - associated symptoms include dysphagia, odynophagia, and chest pain radiating to the back, neck, and jaw which is unrelated to exertion, but relieved with nitroglycerin Upper GI study will show corkscrew esophagus Manometry will show high-amplitude, simultaneous contractions Endoscopy will be normal Medical treatment includes symptomatic relief with nitrates or calcium-channel blockers, and long esophagomyotomy may be indicated in refractory cases

Recognize subarachnoid hemorrhage via *ruptured berry aneurysm*

- acute severe thunderclap headache - symptoms of meningeal irritation - CT scan without contrast = preferred initial imaging = high-density signal surrounding brainstem and basal cisterns

Recognize *Pseudoxanthoma elasticum*

- autosomal recessive disease - appears as small, yellow plaques that are often found in the neck, axillae, and antecubital and popliteal fossae

Recognize liver hematoma

- blunt abdominal trauma + worsening abdominal pain + signs of peritoneal inflammation (guarding) + abdominal CT scan: liver hematoma + perisplenic fluid + free intraperitoneal air = *exploratory laparotomy*

Typical CSF findings of bacterial meningitis include:

- elevated CSF opening pressure above 200 mmHg - WBC count > 1000 cells/mm^3, - polymorphonuclear leukocyte predominance > 80% - *glucose < 45 mg/dL* - protein > 250 mg/dL.

Recognize abdominal radiograph consistent with *CF* = dilated loops of bowel on abdominal radiograph characteristic of meconium ileus.

- ground glass appearance Cystic fibrosis (CF) - commonly presents with meconium ileus, characterized by bilious vomiting and failure to pass meconium - the most common lethal genetic mutation in the United States - commonly present with failure to thrive, respiratory compromise, and meconium ileus - meconium ileus = abnormally thick meconium results in bowel obstruction, perforation, or volvulus

Individuals who received a blood transfusion before 1992 should be screened for - Individuals who received transfusions before 1986 should be screened for -

- hepatitis C - hepatitis B According to the United States Center for Disease Control (CDC), screening blood donors for HBsAg began in 1969 and became mandatory in 1972, though the modern screening methodologies were not implemented until after 1986. Screening blood donors for Hepatitis C was not available until July 1992.

Ottawa ankle rules

- malleolar region pain or point tenderness of tip of malleolus or unable to bear weight at time of injury - *unable to take 4 steps* at time of evaluation and injury

lithium mechanism

- negative regulator of GSK-3 Beta - positive regulator of Akt. - acts on signaling pathways that result in both neuroplasticity and neuroprotection

Recognize *pyoderma gangrenosum*

- neutrophilic ulcerative skin disease - starts as inflammatory papule, pustule, vesicle, or nodule --> progresses to form expanding ulcer with purulent base & ragged violaceous borders - single or multiple - usually on trunk or lower extremities - 30% triggered by local trauma (pathergy) - 50% hae underlying systemic disorder - treatment = local or systemic corticosteroids

Recognize *osteoarthritis*

- noninflammatory degeneration of articular cartilage - subchondral cysts and bony outgrowths (osteophytes) at joint margins - DIPs = Heberden nodes - PIPs = Bouchard nodes

Recognize histological appearance of *candida*

- septate pseudohyphae in a branching pattern - most common cause of esophagitis in advanced AIDS (CD4 count < 50) - in HIV, symptoms of esophagitis should be treated empirically with oral fluconazole for presumed Candidal infection --> if treatment fails, esophagoscopy is indicated for further workup - randomized controlled trial found that fluconazole was superior to ketoconazole in the treatment of candidal esophagitis in AIDS

The patient is at high risk for rhabdomyolysis from his crush injury. Classic findings in rhabdomyolysis include: - serum CK: - serum potassium: - LDH: - serum calcium:

- serum CK: increased - serum potassium: increased - LDH: increased - serum calcium: decreased - Low serum calcium results from binding of free calcium to the damaged myocytes

Recognize chest radiograph of *Tuberculosis*

- subacute symptoms = fatigue, fever, wt loss, cough in setting of IV drug use & recent incarceration - *diffuse reticulonodular pattern* (millet seed) = *miliary tuberculosis (TB)* Miliary TB - via hematogenous spread of M tuberculosis - may arise during primary infection or with reactivation

Recognize *dermatofibroma* (DF)

- via fibroblast proliferation = isolated or multiple lesions - most commonly on lower extremities - etiology unknown - some pts may develop after trauma or insect bites - nontender - discrete, firm, hyperpigmented nodules - <1 cm in diameter - fibrous component that may cause dimpling in center when area is pinched ("dimple" or "buttonhole" sign - treatment = cryosurgery or shave excision = usually not required unless symptomatic

Recognize *toxic adenoma,* in which the process responsible for this pt's elevated thyroid hormone is *autonomous thyroid hormone production*

- via hyperplastic thyroid follicular cells - radioactive iodine uptake scan --> uptake only in hyperactive nodule with suppression of uptake in rest of gland

Recognize vitiligo, which is *destruction of melanocytes* "65 yo man comes to office due to skin condition. For last 6 yrs, pt has had slowly progressive, patchy depigmentation involving face, posterior scalp, hands, genital area, and feet. There is no associated itching, pain, or scaling. Med hx notable for HT, stable CAD, RA, BPH. Pt quit smoking 20 yrs ago and doesn't use alcohol, or drugs. Temp is 98.4 F, BP is 136/88 mmHg, and pulse is 76/min. SKin exam shows widespread areas of depigmentation. PE other wise unremarkable."

- well-demarcated depigmented macules - via destruction of melanocytes - most likely via autoimmune etiology - when treatment desired = topical or systemic corticosteroids

Systemic inflammatory response syndrome (SIRS) can be diagnosed when 2 or more of the following 4 criteria are present:

1) Body temperature less than 36 C or greater than 38 C 2) HR greater than 90 bpm 3) respiratory rate (RR) greater than 20 breaths per minute 4) WBC count less than 4,000 or greater than 12,000 or greater than 10% bands. Sepsis is present when a patient fulfills SIRS criteria and exhibits evidence of an infection. SIRS can be caused by either infectious or noninfectious etiologies - noninfectious etiologies include entities such as trauma, hemorrhage, burns, hemorrhage, or ischemia

Pts newly diagnosed with HIV should be screened for latent TB with PPD or interferon-gamma release assay. PPD test with induration >5 mm is considered positive in pts with HIV. In absence of active TB manifestations (positive chest x-ray, fever, night sweats, cough), treatment of latent TB =

9 mos of *isoniazide & pyridoxine* required

Annual failure rate of combined OCPs

9% - 18% for male condoms

Sepsis --> max blood glucose goal?

< *180 mg/dL*

Transvaginal ultrasound is the study of choice for evaluating cervical incompetence or insufficiency. What is concerning for potential cervical insufficiency and warrants further investigation?

A *cervical length appearing less than 2.5 cm* on pelvic exam at 23-28 weeks gestation Cervical length at or around 24 weeks gestation is an indicator of the potential risk of preterm birth, with shorter cervical lengths demonstrating an association with increasing frequency of spontaneous preterm delivery. - The cervical length is expected to shorten as pregnancy progresses - Short cervix is defined as cervical length less than the 10th percentile for gestational age. - Normal cervical length outside of pregnancy is 4 to 5 cm. - Past history of prior cervical dilation or cervical excision procedures (such as LEEP or conization) increases the likelihood of cervical incompetence and therefore of preterm birth. While routine screening is not done exclusively to see the cervix, when a transvaginal ultrasound is performed, the cervix can be assessed at that time. However, if you have a patient that has a concerning history for cervical insufficiency then you may want to assess. You would make the diagnosis with the following: - 1-2 prior second-trimester pregnancy losses/preterm births + cervical length < 25mm on TVU - Advanced cervical changes on physical exam prior to 24 weeks In terms of when to place the cerclage it is at 14-16 weeks, and they are removed at 36-37 weeks!

This patient's presentation of tinnitus in the setting of excessive use of over-the-counter pain medications is suggestive of aspirin toxicity. What acid-base status is expected?

A mixed metabolic acidosis and respiratory alkalosis is expected. Ex. *pH: 7.41, PaCO2: 26 mmHg, HCO3-: 16 mmol/L, PaO2: 92 mmHg* First, there will be stimulation of the respiratory centers without enough time for uncoupling to be significant. This will present with respiratory alkalosis that is not compensated. As time continues, respiratory alkalosis will continue but it will be overwhelmed by a metabolic acidosis. So, you will see a low CO2 and a low pH. The boards could present either of these situations and it is important to be aware of them!

Recognize hysterosalpingogram of *Asherman syndrome* "A 32-year-old G4P0 with a history of 4 surgical abortions presents to her gynecologist after 1 year of unprotected sex with her partner that did not result in pregnancy. Her partner's semen is found to be normal, and the patient's follicle stimulating hormone (FSH) and luteinizing hormone (LH) are within normal limits. She undergoes a hysterosalpingogram, which is shown in Figure A. What is the most likely cause of this patient's infertility?"

Asherman's syndrome. - endometrial fibrosis and adhesions, usually as a result of prior intrauterine instrumentation - at risk for scarring when vigorous curettage is performed during low-estrogen states - risk of Asherman's syndrome is roughly 16% after a single curettage, but increases to >30% after 3 procedures - patients with mild to moderate disease may become fertile after removal of adhesions, but pregnancy in the presence of adhesions increases the risk of placental abnormalities. - most common cause of outflow obstruction in secondary amenorrhea. - Hysterosalpingography, sonohysterography, and hysteroscopy can all be useful for establishing the diagnosis. - miniature scissors should be used for lysis of uterine adhesions, and a balloon stent can be placed in the uterus immediately after the procedure. In addition, estrogen therapy in the post-operative period promotes regrowth of the endometrium. *Figure shows a hysterosalpingogram of a patient with Asherman's syndrome, with noted uterine filling defects*

Recognize *pseudomonal keratitis*

Contact lens-associated keratitis - painful, red eye - opacification & ulceration of cornea - mostly via gram-negative organisms, such as Pseudomonas & Serratia - medical emergency - can lead to corneal perforation, scarring, permanent vision loss

Recognize NST of *fetal sleep cycle*

Fetal sleep cycle NST - lasts 20 minutes - after 32 weeks, a reactive (i.e. "normal") NST requires two 15-beat-per-minute accelerations lasting for at least 15 seconds - this patient's NST shows no accelerations and is therefore considered nonreactive - *common cause of a nonreactive NST is the fetal sleep cycle, and it is reasonable to continue the test for an additional 20 minutes so as to increase the change of monitoring the fetus while awake* - otherwise, a nonreactive NST should prompt vibroacoustic stimulation or further testing such as a biophysical profile (BPP) Figure shows a representative portion of a nonreactive NST with no accelerations

Recognize "paget's disease of bone"

Figure demonstrates a CT scan of an individual with Paget's bone disease. Note the thicker trabeculae and cortex in addition to the heterogeneous density of the bone. heterogeneous density of the bone noted on CT + elevated alkaline phosphatase with normal calcium = Paget's disease of the bone. - metabolic disorder of abnormal bone remodeling - increased osteoclastic bone resorption and increased osteoblastic bone formation often leading to skull deformities and aching pelvic and joint pain. If one suspects Paget's bone disease as a diagnosis, alkaline phosphatase & calcium levels should be ordered. In this pathology they would be elevated and normal, respectively. Paget's often occurs in three stages, with the initial phase consisting of osteoclastic activity and bone resorption. Next, the osteolytic-osteoblastic phase takes over and osteoblasts produce woven bone with ineffective mineralization. Finally, dense cortical and trabecular bone is deposited in a disorganized manner, making it weaker than normal bone. Bisphosphonates are the mainstay of medical therapy. They work to decrease bone resorption by inhibiting osteoclast resorption.

Recognize contrast-enhanced axial CT section of the upper abdomen showing signs of *acute pancreatitis*. Note the peripancreatic and retroperitoneal edema (large arrows), fat stranding, and relatively normal pancreas (small arrow). "A 47-year-old female presents to the emergency department with 4 hours of abdominal pain that radiates to her back, nausea, and vomiting. She is married, has 3 children, and works as a waitress. She denies smoking, consuming alcohol, or taking recreational drugs. Vital signs are as follows: T 39.1 C (102.4 F), HR 104, RR 18, BP 130/80. Image A shows a contrast-enhanced axial CT section of the upper abdomen. In reviewing her past medical history, you come across Image B, a right-upper quadrant ultrasound from the previous year, when she was evaluated for mild, intermittent abdominal pain. Unfortunately, she was lost to follow-up after the diagnosis. Following supportive care with intravenous fluids and pain control for two days, she has complete resolution of symptoms. Which of the following is the most appropriate next step in the management of this patient?"

Gallstone pancreatitis, after which she should be scheduled for elective *laparoscopic cholecystectomy* Acute pancreatitis - presents with mild to severe epigastric pain that can radiate to the back, flank, or both. - Nausea and vomiting are present in the vast majority of patients. - Evidence of pancreatitis on abdominal CT includes peripancreatic and retroperitoneal edema (large arrows in Image A) as well as inflammatory changes, such as fat stranding. The pancreas itself (small arrow in Image A) can appear relatively normal.

Exertional dyspnea + orthopnea (choking sensation/dyspnea when lying flat) + bibasilar rales + LE edema + normal ejection fraction on echo is consistent with....

HF with preserved ejection fraction (HFpEF) / *diastolic dysfunction* - impaired myocardial relaxation and/or increased LV wall stiffness --> increase in LV end-diastolic pressure

32 yo man comes to office to "check up on my liver." He was seen about 2 yrs ago due to fatigue, anorexia, and jaundice, which developed after sharing needles with another injection drug user. Viral hepatitis serology testing at that time was positive for hepatitis B surface antigen but negative for all other antigens and Abs, including hepatitis C. Pt has not sought med care since then and has been asymptomatic. He stopped using injection drugs & is now sober. He doesn't use tobacco or alcohol and is in monogamous relationship with his gf. PE unremarkable. Pt's hep B virus serology testing is most likely to show what? Hepatitis B surface antigen (HBsAg) = Hepatitis B surface antibody (HBsAb) = Hepatitis B core antibody (HBcAb), total =

Hepatitis B surface antigen (HBsAg) = - Hepatitis B surface antibody (HBsAb) = + Hepatitis B core antibody (HBcAb), total = + Acute hepatitis B virus - pts infected during perinatal period almost universally progress to chronic HBV - children age 1-5 yo = intermediate response --> 50-80% achieve spontaneous remission - *among healthy adults (this pt), acute HBV is self-limited in >95%* = positive HBsAg, hep B e antigen, IgM hep B core Ab, HBV DNA *For majority of pts who contracted HBV as adult and CLEAR the infection, serology typically demonstrates hep B surface Ab (immunity), IgG anti-HBc, and hep B e antigen antibody (anti-HBe) = recovery phase*

Recognize herpetic whitlow, which can be seen in a *dentist* "A 48-year-old man presents to his primary care doctor to discuss a painful lesion on his hand. He reports that a week ago his index finger was tingling. A day later he noticed several small painful vesicles on the skin. The appearance of his lesion today is shown in Image A. Which of the following is most likely this patient's profession?"

Herpetic whitlow - Individuals who come in contact with orotracheal secretions (health care workers) are at high risk of developing this infection - viral infection of the hand caused by either type 1 or type 2 herpes simplex virus (HSV) - classic presentation is a painful, ulcerative lesion on one or more fingers - lesion is often preceded by a prodrome of numbness or tingling then appears as a cluster of small vesicles that progress to ulcers

Recognize abdominal film of *malignancy* resulting in a mechanical large bowel obstruction. "n 86-year-old man with hypertension and hyperlipidemia presents to the emergency department complaining of abdominal pain that has persisted for the past three days. He denies ever having had abdominal surgery. His wife reports that he has not been able to keep any food down and has not had anything to eat or drink in more than 24 hours. An abdominal film is obtained (Figure A). What is the most likely diagnosis?"

In an otherwise healthy older male, who has never had intraabdominal surgery that might have resulted in adhesions that can cause obstruction, clinical suspicion for a mass must be high. *Image A is an abdominal radiograph that shows multiple dilated loops of large bowel indicative of large bowel obstruction. Of note, there is no free air to suggest bowel perforation.*

Recognize late decelerations which indicate *Compression of maternal vessels traversing the uterine wall*

Late decelerations on fetal monitoring - indicative of uteroplacental insufficiency (UPI) - due to compression of maternal vessels traversing the uterine wall when contractions occur Late decelerations most commonly occur in the setting of uterine contractions that are too close together (such as when oxytocin is used for augmentation of labor) or when maternal hypotension (for example, after an epidural is placed) adds additional compromise to fetal oxygenation. Immediate measures include placing the mother on her side, administering oxygen by mask, and correcting any hypotension with intravenous fluids. If tachysystole, or tetanic contractions, are noted, terbutaline may be used to slow the frequency and allow for fetal oxygenation. Late decelerations are a gradual decrease in fetal heart rate, with a nadir that falls after the peak of the corresponding contraction.

Common symptoms of opioid withdrawal include nausea, cramps, diarrhea, restlessness, rhinorrhea, lacrimation, myalgias, arthralgias. Exam can show HT, tachycardia, mydriasis, piloerection, and hyperactive bowel sounds. Treat of choice?

Low-dose methadone + adjunctive meds

Recognize *left bundle branch block* "A 60-year-old male presents to the emergency room with sharp chest pain of sudden onset. He said his symptoms began one hour ago while he was working at his computer in his office. He complains of general malaise, including headaches, myalgias, and dry cough, since quitting smoking two weeks ago. The patient is afebrile. Blood pressure is 140/80 mmHg, pulse is 110/min, and respirations are 23/min. On physical examination he is diaphoretic and pale. An electrocardiogram (ECG), shown in Figure A, shows marked changes from one taken 1 month ago and read as normal, and prompts the attending physician to send the patient for urgent coronary angiography. The patient's ECG shows which of the following:"

New onset left bundle branch block (LBBB) - heart rhythm that is supraventricular in origin - QRS duration of greater than 120 ms - QS or rS complex in lead V1 - RsR' wave in lead V6. - common presentation of left anterior descending artery occlusion and may mask typical ECG findings associated with myocardial infarction - combination of chest pain and new onset LBBB is an indication for urgent coronary angiography - a 12 lead ECG should be the first test when the physician wants to determine if there are any ST segment changes, new-onset LBBB, presence of Q waves, and new-onset T wave inversions

Recognize oral leukoplakia, which is an example of squamous *hyperplasia*

Oral leukoplakia - associated with an increased risk of oral squamous cell cancer - most commonly seen at the vermilion border of lower lip but can be seen anywhere in the mouth - can be caused by alcohol and tobacco use (as in this patient), infection (EBV), and chronic irritation (e.g. poor-fitting dentures) - cannot be scrapped off with gentle scrapping. - gold standard diagnosis being histopathological examination of suspicious tissue

Associated with no following diet for diagnosis of celiac sprue (or vitamin D deficiency, decreased intestinal calcium and phosphorous absorption with results secondary hyperparathyroidism). Typical lab findings include hypophosphatemia, hypocalcemia, elevated alkaline phosphatase.

Osteomalacia / *Impaired osteoid matrix mineralization* Causes - malabsorption, intestinal bypass sx, celiac sprue, chronic liver dz, CKD - increased alk phos & PTH - decreased serum calcium & phosphorous, urinary calcium, 25 OH-D levels - x-rays = thinning of cortex with reduced bone density - bilateral & symmetric pseudofractures (looser zones)

Recognize *retinal angiomas as seen in von Hippel-Lindau*

PTH-related peptide - associated with squamous cell carcinoma of the lung, renal cell carcinoma, and breast carcinoma - causes hypercalcemia of malignancy Patients with von Hippel-Lindau disease, are at high risk for renal cell carcinoma --> can secrete PTH-related peptide causing hypercalcemia - marked by hemangioblastomas of the retina, cerebellum, and medulla and a high risk of bilateral renal cell carcinoma - associated with the deletion of the VHL tumor suppressor on chromosome 3p - causes constitutive expression of hypoxia inducible factor (HIF) and activation of angiogenic factors

Recognize EKG of shortened QT interval as seen in hypercalcemia

PTH-related peptide - associated with squamous cell carcinoma of the lung, renal cell carcinoma, and breast carcinoma - causes hypercalcemia of malignancy Patients with von Hippel-Lindau disease, are at high risk for renal cell carcinoma --> can secrete PTH-related peptide causing hypercalcemia - marked by hemangioblastomas of the retina, cerebellum, and medulla and a high risk of bilateral renal cell carcinoma - associated with the deletion of the VHL tumor suppressor on chromosome 3p - causes constitutive expression of hypoxia inducible factor (HIF) and activation of angiogenic factors

Recogniz *empyema*

Pneumonias --> often cause pleural effusions - if bacteria persistently cross into pleural space --> complicated parapneumonic effusions or empyemas URI --> progressed into pleuritic chest pain, productive cough, fever + chest x-ray showing large right-sided pleural effusion - *immunocompromised (HIV, post-transplant) = greater risk of complications of pneumonia, such as EMPYEMA* = frank pus or bacteria (by Gram stain) in pleural space - all empyemas require prolonged abx (2-4 wks) & drainage (usually with chest tube)

Recognize classic audiogram of *presbycusis*, in which there is a decrease in high frequency hearing

Presbycusis - bilateral sensorineural hearing loss with predominant of high frequency hearing loss on audiogram - common cause of hearing loss in the older individual (presby = old) - classic reason why older males often struggle to hear female voices as they age, as women speak with a higher voice frequency On exam - Weber test is non-localizing, as the problem is most often bilateral - Because this condition is a form of sensorineural hearing loss, the Rinne reveals better air conduction compared to bone (the inner ear has impaired ability to transmit auditory signals so both air and bone conduction reduced equally, maintaining the normal balance of air conduction over bone conduction). Most common causes of sensorineural hearing loss include inherited disorders, noise exposure, and presbycusis

Should be suspected in patients who present with clinical features of adrenal insufficiency and have bilateral calcification of the adrenal glands on CT scan.

Primary adrenal insufficiency secondary to *M. tuberculosis* - Patients with extrapulmonary tuberculosis like this one are unable to contain the bacteria in the primary or secondary stage and experience dissemination throughout the body - Extrapulmonary tuberculosis is more often found in patients with *HIV* - Sites of extrapulmonary tuberculosis include the lymph nodes, pleura, genitourinary tract, spine (Pott's disease), intestine, meninges, eye, and *adrenal glands*

A 25-year-old female with a history of childhood asthma presents to clinic complaining of a three month history of frequent, loose stools. She currently has three to four bowel movements per day, and she believes that these episodes have been getting worse and are associated with mild abdominal pain. She also endorses seeing red blood on the toilet tissue. On further questioning, she also endorses occasional palpitations over the past few months. She denies fevers, chills, headache, blurry vision, cough, shortness of breath, wheezing, nausea, or vomiting. She describes her mood as slightly irritable and she has been sleeping poorly. A review of her medical chart reveals a six pound weight loss since her visit six months ago, but she says her appetite has been normal. The patient denies any recent illness or travel. She is a non-smoker. Her only current medication is an oral contraceptive pill. Her temperature is 37°C (98.6°F), pulse is 104/min, blood pressure is 95/65 mmHg, respirations are 16/min, and oxygen saturation is 99% on room air. On physical exam, the physician notes that her thyroid gland appears symmetrically enlarged but is non-tender to palpation. Upon auscultation there is an audible thyroid bruit. Her cranial nerve and ocular exam are normal. Her abdomen is soft and non-tender to palpation. Deep tendon reflexes are 3+ throughout. Lab results are as follows: Serum: Na+: 140 mEq/L K+: 4.1 mEq/L Cl-: 104 mEq/L HCO3-: 26 mEql/L BUN: 18 mg/dL Creatinine 0.9 mg/dL Hemoglobin: 14.0 g/dL Leukocyte count: 7,400/mm^3 Platelet count 450,000/mm^3 TSH & Free T4: pending A pregnancy test is negative. The patient is started on propranolol for symptomatic relief. What is the most likely best next step in management for this patient?

Propylthiouracil The patient's diagnosis is most consistent with Graves' disease. The most common first-line treatment for Graves' disease is a thionamide such as propylthiouracil or methimazole. The patient presents with symptoms of loose stools, palpitations, and weight loss, as well as associated exam findings of tachycardia and a symmetrically enlarged, non-tender thyroid with audible bruit. These findings are associated with a suppressed serum TSH level and elevated serum free T4. The lack of tenderness or a focal area of enlargement makes thyroiditis or a solitary hyperfunctioning thyroid nodule less likely. Short-term symptomatic relief in Graves' disease is achieved with a course of therapy with a ß-adrenergic blocker, such as propranolol. For long-term treatment, a thionamide, such as propylthiouracil or methimazole, should be administered. Patients who receive either of these drugs must be cautioned regarding the potential side effects of rash, joint pain, hepatitis, and agranulocytosis.

A 5-month-old male infant presents to your office in January. His mother complains that his cheeks are "red and scaly." She's concerned that they are uncomfortable, since she's noticed him scratching them. The rash has worsened since winter started. Otherwise, he has no medical history. Family history is relevant for asthma and some seasonal allergies. On exam, his cheeks bilaterally are erythematous with erosion, scaling and excoriated papules and plaques. Small, similar appearing patches are also present on his chest. What is the most likely outcome for this condition?

Recurrence, regardless of treatment This infant with red, scaly cheeks that have worsened in the winter, with a family history of asthma and allergies, likely has atopic dermatitis. This condition usually recurs, regardless of treatment. Atopic dermatitis (eczema) - common, chronic pruritic inflammatory disorder of the skin that is most common in children.

Jaundice and icterus with positive urine bilirubin assay consistent with buildup on conjugated bilirubin. Normal transaminases and normal alkaline phosphatase.

Rotor's syndrome = rare, benign hereditary condition = chronic or fluctuating conjugated hyperbilirubinemia due to defect in *hepatic secretion of conjugated bilirubin* into biliary system

Most common complication of influenza and should be suspected in any pt who develops worsening fever or pulmonary symptoms after initial improvement.

Secondary bacterial pneumonia - Streptococcus pneumoniae and *Staphylococcus aureus* = most common etiologic organisms - S aureus = rapid-onset, severe, necrotizing pneumonia with high risk of death

Characterized by systemic symptoms (fever, malaise), widespread lymphadenopathy (particularly epitrochlear), and diffuse maculopapular rash that *begins on trunk and extends to extremities*, including palms and soles.

Secondary syphilis - diagnosis via serology (using both treponemal and nontreponemal test) - 1 IM dose of penicillin G benzathine = standard treatment

Recognize *lichen planus* "A 45-year-old man with a history of chronic hepatitis C infection, hypertension, type II diabetes mellitus, and congestive heart failure presents to his primary care physician with complaints of a new rash. He states that he first noticed the rash two weeks prior, and endorses intermittent itchiness. The patient works as a butcher at a local meat processing shop and is sexually active with several partners. He does not use barrier protection. The patient's temperature is 98.6°F (37.0°C), pulse is 90/min, blood pressure is 155/95 mmHg, and respirations are 12/min. Physical exam is notable for Figure A. Fine, lace-like white lines are also noted on the patient's buccal mucosa. What is the most likely diagnosis?"

Several purple, polygonal, pruritic papules and plaques = *lichen planus* - inflammatory skin disorder most commonly characterized by an eruption of Purple, Polygonal, Planar, Pruritic, Papules and Plaques (the 6 P's), often on extensor surfaces such as the wrist - development of fine white lines on the surfaces of skin lesions, as well as on the buccal mucosa (Wickham's striae), are characteristic - concurrent *hepatitis C* infection is a risk factor for lichen planus. *Figure shows multiple purple, polygonal, papules and plaques on the wrist, characteristic of lichen planus*

Patient with symptoms similar to biliary colic, a history of cholecystectomy, and common bile duct (CBD) dilation in the setting of absent pancreatic masses likely has...

Sphincter of Oddi dysfunction (SOD) - *manometry of the sphincter* is the best diagnostic modality for this disease. - The sphincter of Oddi connects the CBD to the duodenum; stenosis or dyskinesia of this sphincter can cause symptoms similar to choledocholithiasis in the absence of stones - Rare disease and is often confused with cholelithiasis *until the patient's symptoms persist despite cholecystectomy.* - Recurrent idiopathic pancreatitis can also occur - *The best diagnostic tool is sphincter manometry, which should show increased pressure in the sphincter* - Symptoms typically resolve with sphincterotomy, performed via ERCP.

Recognize rash consistent with *Parvovirus B19* "A 35-year-old female presents to her family physician with a complaint of painful joints for the past 2 weeks. She reports symmetric bilateral joint pain in her hands, knees, and ankles. Additionally, the patient states that she experienced a cold-like illness 3 weeks ago that has since resolved. Physical examination is significant for a rash on her hands and feet that is shown in Figure A. Radiographs of the bilateral hands do not show any notable abnormalities. Which of the following is the most likely diagnosis as the cause of this patient's joint pains?"

Symmetric arthritis of the hands, knees, and ankles in the context of a preceding cold-like illness and papular, purpuric rash in a stocking-glove distribution is consistent with a *parvovirus B19 infection* - Arthritis is a relatively uncommon manifestation of parvovirus B19 infection in children; however, the majority of adults with the infection develop arthropathy. - Parvovirus B19 arthropathy is also more common in women. - The distribution of arthritis is symmetric, most commonly involving PIP and MCP joints (similar to RA) in addition to knees, wrists, and ankles. - The arthritis is often self-resolving within weeks but may last for months to years in some instances. - In addition to acute or persistent arthropathy, parvovirus B19 also causes a papular, purpuric rash on the hands and feet ("stocking glove distribution"). - Infection can also cause failure of red blood cell production, leading to potential aplastic crisis. *Figure A is a photograph of the papular purpuric rash that occurs in parvovirus B19 infection; note, the distribution on the distal extremities (hands in this photograph).*

68 yo man comes to office due to progressive dyspnea on exertion and nonproductive cough over last yr. Pt now has dyspnea after performing common household chores. He had asthma during childhood but otherwise had no respiratory symptoms until year ago. He has HT & OA. Meds include lisinopril and acetaminophen as needed. Pt has never smoked and drinks alcohol on social occasions. He travelled overseas extensively during his long career in navy but has been retired for 8 yrs. He has 2 dogs. Temp is 98 F, BP is 143/76 mmHg, pulse is 93/min, & respirations are 20/min. Oxygen sat 94% on ambient air. BMI 27 kg/m^2. Neck veins flat. Lung auscultation reveals fine inspiratory crackles at bases, and heart sounds are normal without murmur. Abdomen nontender without hepatosplenomegaly. No extremity edema present. What sets of findings are most likely to be observed on pulmonary function testing of this pt? Total lung capacity = FEV1/FVC ratio = Diffusion capacity for carbon monoxide =

TLC = decreased FEV1/FVC = Normal Diffusion capacity for CO = decreased slowly progressive dyspnea + dry cough + fine crackles in absence of smoking hx = interstitial lung dz via *idiopathic pulmonary fibrosis* - Restrictive profile = decreased TLC & normal FEV/FVC ratio - Reduced DLCO = via decreased pulmonary capillary erythrocyte volume and alveolar-capillary surface area - 50-70 yo - end-inspiratory squeaks, digital clubbing, abnormal S2 (loud P2, fixed, split S2) via pulmonary HT - pt's long hx in navy = increased risk due to asbesosis exposure

Recognize large pericardial effusion, in which *nonpalpable point of maximal impulse* would most likely be an additional finding.

URI + dyspnea + elevated JVP + clear lung fields + increased cardiac silhouette on chest x-ray = suggestive of *early cardiac tamponade due to large pericardial effusion* Large pericardial effusion - chest x-ray = enlarged and globular cardiac silhouette / "water bottle" heart shape + clear lung fields

A 63-year-old female presents to the emergency room with left-sided back pain. Her past medical history includes persistent urinary retention, for which she has a foley catheter in place 24 hours/day. Her blood pressure is 117/75 mmHg, pulse is 103/min, and temperature is 99.0 deg F (37.2 deg C). Urinalysis shows too numerous to count white blood cells, 5-10 red blood cells per high-powered field, positive leukesterase, and a pH of 7.31 (nl 5-6). Which of the following images is most likely to be associated with this patient's symptoms?

Urinary tract infection (UTI) and kidney stones - an alkaline pH should raise suspicion for Proteus infection, which is associated with the *coffin-lid shaped crystals composed of struvite.* UTIs - extremely common in patients with persistent foley catheters - Although 80% of UTIs are caused by Escherischia coli, an alkaline urine pH (6.5-8) should raise suspicion for Proteus species - The urine becomes alkaline as a result of Proteus production of urease, which is an enzyme that breaks down urea into ammonia and carbon dioxide. - Treatment for struvite kidney stones includes antibiotic treatment of the underlying infection (typically with ampicillin or cephalosporins), as well as hydration. A pH = 7 is considered the neutral point for pH. A pH < 7 is considered acidic and > 7 is considered basic (alkaline). In this question, a pH of 7.31 would be considered basic.

Recognize flank ecchymosis or Grey Turner's sign which can be observed in retroperitoneal bleeds such as AAA rupture. Any elderly patient with a significant smoking history who presents with chest/abdominal/back pain and hypotension should immediately receive

an *abdominal* emergency medicine *bedside ultrasound (EMBU)* - to assess for ruptured abdominal aortic aneurysm (AAA) AAA - local enlargement of the abdominal aorta >3 cm that is commonly observed in elderly patients - Risk factors include smoking, high blood pressure, and other heart or blood vessel diseases - AAA rupture is a true emergency with mortality exceeding 90% - The majority of patients (~70%) do not make it to the hospital alive - Symptoms include vague back, abdominal, or chest pain and are often accompanied by hypotension and hypovolemic shock - If the AAA ruptures in the intraabdominal cavity, death usually results. - Retroperitoneal rupture can result in temporary tamponade which can increase survival and allow enough time for surgical intervention to be performed - *In cases of retroperitoneal rupture, flank ecchymosis (Grey Turner's sign) may be observed* - *Diagnosis is usually made via ultrasound or contrast enhanced abdominal CT scan* - Treatment is immediate volume resuscitation and emergency operative repair

In an elderly patient presenting with new-onset atrial fibrillation (flip for EKG) and hypoactive altered mental status, suspect

apathetic thyrotoxicosis - atypical presentation of hyperthyroidism - *thyroid stimulating hormone level (TSH)* is the most appropriate screening test for hyperthyroidism Diagnosis of hyperthyroidism in the elderly patient requires a high index of suspicion, as the disease does not always present with the classic symptoms of hyperthyroidism, such as heat intolerance, weight loss, and hyperactivity. - *In elderly patients, the most common presenting symptoms are cardiac, specifically atrial fibrillation, congestive heart failure, and angina.* - *Apathetic thyrotoxicosis manifests with apathy, weakness, depression, and lethargy.*

Acute spinal cord compression can present with loss of motor and sensory function, loss of rectal tone, and urinary retention. Management includes...

emergency *surgical consultation*, neuroimaging, possibly IV glucocorticoids

Amebiasis should be considered in pts with liver abscess who have lived or traveled in an endemic area for Entamoeba histolytica (ex. Mexico). Initial evaluation includes imaging, serologic testing, and...

empiric treatment with *oral metronidazole* (to treat abscess) and luminal agent (to eradicate intestinal colonization) - >90% cure with oral therapy - luminal agent = paromomycin = required to eradicate intestinal colonization - cyst drainage *NOT* recommended routinely

EKG shows more than three morphologically different P-waves, varying P-R intervals, tachycardia, and narrow QRS complexes, all consistent with multifocal atrial tachycardia (MAT), which is most commonly associated with..

hypoxia (via *smoking*) and COPD - most commonly due to *hypoxia, electrolyte disturbances, and medication effects* = atrial arrhythmia characterized by irregular and rapid rhythm = associated with pulmonary pathologies - potentially methylxanthine toxicity - at least 3 different P wave forms - avoid beta-blockers - usually oxygen first, followed by supplemental meds (non-dihydropyridine CCBs)

gestational thrombocytopenia treatment

no treatment - resolves postnatally In general, it is often helpful to separate causes of low platelets in pregnancy based on the severity of the thrombocytopenia and the time course. For gestation age of 20 weeks of less: - Platelet count 80,000-149,000 and asymptomatic: Most likely gestational thrombocytopenia (GT), but can not rule out immune thrombocytopenic purpura (ITP). - Platelet count <80,000: Not GT, but may be ITP For gestation age of greater than 20 weeks: - Platelet count 80.000-149,000 and asymptomatic: Almost aways GT, but ITP cannot be excluded. - Platelet count <80,000 and asymptomatic: Rarely GT. ITP is likely. - Platelet count <80,000 and hypertension: Consider pre-eclampsia. - Platelet count <80,000, systemic symptoms, and/or microangiopathic hemolytic anemia: If LFTs are increased, think acute fatty liver/HELLP. Acute fatty liver differentiated from HELLP by renal failure, hypoglycemia, hyperbilirubinemia, and coagulopathy. If coagulation abnormalities, consider DIC/sepsis. If transient focal neurologic abnormalities, consider thrombotic thrombocytopenic purpura

Recognize *paralytic ileus*

paralytic/adynamic ileus - most likely diagnosis in pt who has *abdominal pain* following *traumatic injury* & abdominal x-ray revealing gas-filled loops of both small and large intestines + gastric dilation - ileus commonly due to abdominal surgery but can be seen in retroperitoneal/abdominal hemorrhage or inflammation, intestinal ischemia, & electrolyte abnormalities - N/V, abdominal distention, obstipation, hypoactive bowel sounds

The patient has signs and symptoms of right-sided heart failure in the setting of an acute myocardial infarction. Hemodynamic collapse after nitroglycerin suggest injury to

right ventricle --> may manifest as changes in the inferior EKG leads: *II, III and aVF* Right ventricular infarctions - usually occur secondary to an occlusion of the right coronary artery, which supplies the inferior wall - telltale signs of right heart failure (hypotension, JVD, peripheral edema, hepatic congestion) as well as findings of inferior wall ischemia on EKG with changes in II, III, and aVF - Right ventricular infarction may also lead to bradycardia, AV block, and atrial fibrillation - These patients are preload-dependent and may quickly decompensate if their preload is significantly reduced; thus nitrates and diuretics should generally be avoided. - Hemodynamic effects of this type of myocardial infarction result in systemic hypotension. Management is directed towards reperfusion, rate and rhythm control, and volume loading with careful attention not to make these patients intravascularly depleted as they depend heavily on preload

Patient presents with symptoms of hyperthyroidism in the setting of a recent flu-like illness, low radioactive iodine uptake, and a non-painful thyroid gland consistent with subacute lymphocytic thyroiditis or silent thyroiditis. These patients initially spill thyroid hormone in a hyperthyroid state but eventually become euthyroid so ...

treatment = *observation and follow-up* Subacute lymphocytic thyroiditis / painless thyroiditis / silent thyroiditis - typically presents with a transient thyrotoxic phase of 2-5 months that may be followed by a self-limited hypothyroid disease phase During the thyrotoxic phase TSH levels are low due to suppression from elevated T4 and T3 levels - symptoms are similar to subacute granulomatous thyroiditis (de Quervain's thyroiditis, viral thyroiditis - given the return to a euthyroid state with time there is no need to treat this patient's current hyperthyroidism Figure shows scintigraphy results in subacute lymphocytic thyroiditis. Radioactive iodine uptake is low diffusely with no specific localization to different areas *These cases depict a clear inciting event (viral illness) followed by hyperthyroid symptoms, followed by hypothyroid symptoms within a short time frame which suggests silent thyroiditis.*


Kaugnay na mga set ng pag-aaral

US Government Lesson 1, 2, 3, and 4

View Set

Ch. 21 Blood Cells and the Hematopoietic System

View Set

Government UNIT 2 EXAM REVIEW Chapter 17-20

View Set

Chapter 11, Chapter 12, Chapter 13

View Set

Chapter 2 business finance (actually ch 2)

View Set

FCA2 ADVANCEMENT LOGIC CIRCUITS CH 1,3

View Set

303 Hinkle PrepU Chapter 47: Management of Patients With Intestinal and Rectal Disorders

View Set